Download as doc, pdf, or txt
Download as doc, pdf, or txt
You are on page 1of 116

INSURANCE Page 1

ABOITIZ SHIPPING CORPORATION V NEW - Petitioner elevated the case to the Court of delegated, still, the shipowner must exercise close
INDIA ASSURANCE COMPANY, LTD. Appeals, which in turn, affirmed in toto the trial supervision over its men.
[CITATION] court’s decision. - In the present case, petitioner has the burden of
QUISUMBING; May 2, 2006 - Petitioner moved for reconsideration but the same showing that it exercised extraordinary diligence in
was denied. the transport of the goods it had on board in order to
NATURE - Hence, this petition for review invoke the limited liability doctrine. Differently put,
Petition for review on certiorari Petitioner’s Claim to limit its liability to the amount of the insurance
> Petitioner contends that respondent’s claim for proceeds, petitioner has the burden of proving that
FACTS damages should only be against the insurance the unseaworthiness of its vessel was not due to its
- Societe Francaise Des Colloides loaded a cargo of proceeds and limited to its pro-rata share in view of fault or negligence.
textiles and auxiliary chemicals from France on board the doctrine of limited liability - Considering the evidence presented and the
a vessel owned by Franco-Belgian Services, Inc. Respondent’s Comments circumstances obtaining in this case, we find that
- The cargo was consigned to General Textile, Inc., > Respondent counters that the doctrine of real and petitioner failed to discharge this burden. Both the
in Manila and insured by respondent New India hypothecary nature of maritime law is not applicable trial and the appellate courts, in this case, found that
Assurance Company, Ltd. in the present case because petitioner was found to the sinking was not due to the typhoon but to its
- While in Hongkong, the cargo was transferred to have been negligent. Hence, according to unseaworthiness. Evidence on record showed that
M/V P. Aboitiz for transshipment to Manila. respondent, petitioner should be held liable for the the weather was moderate when the vessel sank.
- Before departing, the vessel was advised that it total value of the lost cargo These factual findings of the Court of Appeals,
was safe to travel to its destination, but while at sea, affirming those of the trial court are not to be
the vessel received a report of a typhoon moving ISSUE disturbed on appeal, but must be accorded great
within its path. WON the limited liability doctrine applies in this case weight. These findings are conclusive not only on
- To avoid the typhoon, the vessel changed its the parties but on this Court as well.
course. However, it was still at the fringe of the HELD Disposition Petition is denied for lack of merit.
typhoon when its hull leaked. NO
- On October 31, 1980, the vessel sank, but the Ratio Where the shipowner fails to overcome the PHILAMCARE HEALTH SYSTEMS, INC. V CA
captain and his crew were saved. presumption of negligence, the doctrine of limited (TRINOS)
- On November 3, 1980, the captain of M/V P. liability cannot be applied. 379 SCRA 357
Aboitiz filed his “Marine Protest”, stating that the YNARES-SANTIAGO; March 18, 2002
wind force was at 10 to 15 knots at the time the ship Reasoning
foundered and described the weather as “moderate - From the nature of their business and for reasons NATURE
breeze, small waves, becoming longer, fairly of public policy, common carriers are bound to Petition for review of CA decision
frequent white horse observe extraordinary diligence over the goods they
- Petitioner notified the consignee of the total loss of transport according to all the circumstances of each FACTS
the vessel and all of its cargoes. case. In the event of loss, destruction or - Ernani TRINOS, deceased husband of respondent
- General Textile, lodged a claim with respondent for deterioration of the insured goods, common carriers Julita, applied for a health care coverage with
the amount of its loss. are responsible, unless they can prove that the loss, Philamcare Health Systems, Inc. In the standard
- Respondent paid General Textile and was destruction or deterioration was brought about by application form, he answered no to the question:
subrogated to the rights of the latter. the causes specified in Article 1734 1 of the Civil “Have you or any of your family members ever
- Respondent hired a surveyor to investigate, and Code. In all other cases, common carriers are consulted or been treated for high blood pressure,
the same concluded that the cause was the flooding presumed to have been at fault or to have acted heart trouble, diabetes, cancer, liver disease,
of the holds brought about by the vessel’s negligently, unless they prove that they observed asthma or peptic ulcer? (If Yes, give details).”
questionable seaworthiness. extraordinary diligence. Moreover, where the vessel - The application was approved for period of one
- Respondent filed a complaint for damages against is found unseaworthy, the shipowner is also year; upon termination, it was extended for another
petitioner Aboitiz, Franco-Belgian Services and the presumed to be negligent since it is tasked with the 2 years. Amount of coverage was increased to a
latter’s local agent, F.E. Zuellig, Inc. (Zuellig) maintenance of its vessel. Though this duty can be maximum sum of P75T per disability.
- On November 20, 1989, the trial court ruled in - During this period, Ernani suffered a HEART
favor of respondent and held petitioner Aboitiz liable 1 ATTACK and was confined at the Manila Medical
for the total value of the lost cargo plus legal interest Art. 1734. Common carriers are responsible for the loss, destruction, or deterioration of the Center (MMC) for one month. While her husband was
goods, unless the same is due to any of the following causes only:
- The complaint with respect to Franco and Zuellig (1) Flood, storm, earthquake, lightning, or other natural disaster or calamity; in the hospital, Julita tried to claim the
was dismissed (2) Act of the public enemy in war, whether international or civil; hospitalization benefits.
(3) Act of omission of the shipper or owner of the goods;
(4) The character of the goods or defects in the packing or in the containers;
- Petitioner treated the Health Care Agreement
(5) Order or act of competent public authority. (HCA) as void since there was a concealment
INSURANCE Page 2

regarding Ernani’s medical history. Doctors at agreement was in the nature of non-life insurance, - These conditions have not been met. When the
the MMC allegedly discovered at the time of his which is primarily a contract of indemnity. Once the terms of insurance contract contain limitations on
confinement, he was hypertensive, diabetic and member incurs hospital, medical or any other liability, courts should construe them in such a way
asthmatic. Julita then paid the hospitalization expense arising from sickness, injury or other as to preclude insurer from non-compliance of
expenses herself, amounting to about P76T. stipulated contingent, the health care provider must obligation. Being a contract of adhesion, terms of an
- After her husband died, Julita instituted action for pay for the same to the extent agreed upon under insurance contract are to be construed strictly
damages against Philamcare and its Pres. After the contract. against the party which prepared it – the insurer.
trial, the lower court ruled in her favor and ordered Reasoning - Also, Philamcare had 12 months from the date of
Philamcare to reimburse medical and hospital - A contract of insurance3 is an agreement whereby issuance of the Agreement within which to contest
coverage amounting to P76T plus interest, until fully one undertakes for a consideration to indemnify the membership of the patient if he had previous
paid; pay moral damages of P10T; pay exemplary another against loss, damage or liability arising from ailment of asthma, and six months from the issuance
damages of P10T; atty’s fees of P20T. an unknown or contingent event. of the agreement if the patient was sick of diabetes
- CA affirmed the decision of the trial court but - An insurance contract exists where the following or hypertension.
deleted all awards for damages and absolved elements concur: * The health care agreement is in the nature of a
petitioner Reverente. (a) The insured has an insurable interest; contract of indemnity. Hence, payment should be
Petitioner’s Claims (b) The insured is subject to a risk of loss by the made to the party who incurred the expenses. It is
(1) Agreement grants “living benefits” such as happening of the peril; clear that respondent paid all the hospital and
medical check-ups and hospitalization which a (c) The insurer assumes the risk; medical bills; thus, she is entitled to reimbursement.
member may immediately enjoy so long as he is (d) Such assumption of risk is part of a general Disposition Petition DENIED.
alive upon effectivity of the agreement until its scheme to distribute actual losses among a large
expiration. group of persons bearing a similar risk; and PINEDA V CA (INSULAR LIFE INSURANCE
(2) Only medical and hospitalization benefits are (e) In consideration of the insurer’s promise, the COMPANY)
given under the agreement without any insured pays a premium. 226 SCRA 755
indemnification, unlike in an insurance contract 2. NO DAVIDE; September 27, 1993
where the insured is indemnified for his loss. Ratio Where matters of opinion or judgment are
(3) HCAs are only for a period of one year; called for, answers made in good faith and without NATURE
therefore, incontestability clause does not apply, as intent to deceive will not avoid a policy even though Appeal by certiorari for review and set aside the
it requires effectivity period of at least 2 yrs. they are untrue; since in such case the insurer is not Decision of the public respondent Court of Appeals
(4) It is not an insurance company, governed by justified in relying upon such statement, but is and its Resolution denying the petitioners' motion for
Insurance Commission, but a Health Maintenance obligated to make further inquiry. reconsideration
Organization under the authority of DOH. Reasoning
(5) Trinos concealed a material fact in his - The fraudulent intent on the part of the insured FACTS
application. must be established to warrant rescission of the - In 1983, Prime Marine Services, Inc. (PMSI)
(6) Julita was not the legal wife since at the time insurance contract. The right to rescind should be procured a group policy from Insular Life to provide
of their marriage, the deceased was previously exercised previous to the commencement of an life insurance coverage to its sea-based employees
married to another woman who was still alive.* action on the contract. No rescission was made. enrolled under the plan. During the effectivity of the
Besides, the cancellation of health care agreements policy, 6 covered employees perished at sea. They
ISSUES as in insurance policies requires: were survived by complainants-appellees, the
1. WON a health care agreement is an insurance (a) Prior notice of cancellation to insured; beneficiaries under the policy.
contract (If so, “incontestability clause” under the (b) Notice must be based on the occurrence after - complainants-appellees sought to claim death
Insurance Code is applicable) effective date of the policy of one or more of the benefits due them and approached Capt. Roberto
2. WON the HCA can be invalidated on the basis of grounds mentioned; Nuval, President and GM of PMSI, then executed
alleged concealment (c) Must be in writing, mailed or delivered to the special powers of attorney authorizing Capt. Nuval to
insured at the address shown in the policy; “follow up, ask, demand, collect and receive” for
HELD (d) Must state the grounds relied upon provided in their benefit indemnities of sums of money due
YES Section 64 of the Insurance Code and upon request them…”
Ratio Every person has an insurable interest in the of insured, to furnish facts on which cancellation is - Unknown to the complainants, PMSI filed with
life and health of himself2. The health care based. Insular Life claims for and in behalf of them through
Capt. Nuval, even using the 5 special powers of
2 (3) of any person under a legal obligation to him for the payment of money, respecting property or attorney that they executed as documents. Insular
Sec.10. Every person has an insurable interest in the life and health: service, of which death or illness might delay or prevent the performance; and
(1) of himself, of his spouse and of his children; (4) of any person upon whose life any estate or interest vested in him depends. Life then released 6 checks, payable to the order of
(2) of any person on whom he depends wholly or in part for education or support, or in whom he 3 the complainant-appellees, to the treasurer of PMSI
has a pecuniary interest; Section 2 (1) of the Insurance Code
INSURANCE Page 3

(who happened to be Capt. Nuval’s son-in-law). with ordinary prudence and reasonable both the employer and the employees, the insurer
Capt. Nuval then endorsed and deposited these diligence. Obviously, if he knows or has good also enjoys significant advantages from the
checks (which were for the complainants) in his bank reason to believe that the agent is exceeding his arrangement. The reduction in the premium which
account. authority, he cannot claim protection. So if the results from employer-administration permits the
- 3 years after, the complainants-appellees found out suggestions of probable limitations be of such a clear insurer to realize a larger volume of sales, insurer to
that they were entitled, as beneficiaries, to life and reasonable quality, or if the character assumed realize a larger volume of sales, and at the same
insular benefits under a group policy with by the agent is of such a suspicious or unreasonable time the insurer's own administrative costs are
respondent-appellant so they sought to recover nature, or if the authority which he seeks to exercise markedly reduced.
these benefits from Insular Life. Insular Life denied is of such an unusual or improbable character, as - the employee has no knowledge of or control over
the claim, saying that their liability to complainants would suffice to put an ordinarily prudent man upon the employer's actions in handling the policy or its
was already extinguished upon delivery to and his guard, the party dealing with him may not shut administration. An agency relationship is based upon
receipt by PMSI of the 6 checks issued in the his eyes to the real state of the case, but should consent by one person that another shall act in his
complainants’ names. Complainants filed case with either refuse to deal with the agent at all, or should behalf and be subject to his control. It is clear from
the Insurance Commission which decided in their ascertain from the principal the true condition of the evidence regarding procedural techniques here
favor. affairs. that the insurer-employer relationship meets this
- Insurance Commission held that the special powers Reasoning agency test with regard to the administration of the
of attorney executed by complainants in favor of the - The execution by the principals of special powers of policy, whereas that between the employer and its
complainants do not contain in unequivocal and clear attorney, which clearly appeared to be in prepared employees fails to reflect true agency. The insurer
terms authority to Capt. Nuval to obtain, receive, forms and only had to be filled up with their names, directs the performance of the employer's
receipt from respondent company insurance residences, dates of execution, dates of administrative acts, and if these duties are not
proceeds arising from the death of the seaman- acknowledgement and others, excludes any intent to undertaken properly the insurer is in a position to
insured; also, that Insular Life did not convincingly grant a general power of attorney or to constitute a exercise more constricted control over the
refuted the claim of Mrs. Alarcon that neither she nor universal agency. Being special powers of attorney, employer's conduct.
her husband executed a special power of authority in they must be strictly construed. Insular Life knew - ON GROUP INSURANCE: Group insurance is
favor of Capt. Nuval (and therefore, the company that a power of attorney in favor of Capt. Nuval for essentially a single insurance contract that
should have not released the check to Capt. Nuval- the collection and receipt of such proceeds was a provides coverage for many individuals. In its
PMSI); and that it did not observe Sec 180(3), as deviation from its practice with respect to group original and most common form, group insurance
repealed by Art. 225 of the Family Code, when it policies (that the employer-policyholder is the agent provides life or health insurance coverage for
released the benefits due to the minor children of of the insurer). the employees of one employer. The coverage
Ayo and Lontok, when the said complainants did not - The employer acts as a functionary in the collection terms for group insurance are usually stated in a
post a bond as required and payment of premiums and in performing related master agreement or policy that is issued by the
- Insular Life appealed to the CA; CA modified the duties. Likewise falling within the ambit of insurer to a representative of the group or to an
decision of the Insurance Commission, eliminating administration of a group policy is the disbursement administrator of the insurance program, such as an
the award to the Lontoks and Ayo of insurance payments by the employer to the employer. Although the employer may be the titular
employees. Most policies, such as the one in this or named insured, the insurance is actually
ISSUES case, require an employee to pay a portion of the related to the life and health of the employee.
1. WON Insular Life should be liable to the premium, which the employer deducts from wages Indeed, the employee is in the position of a real
complainants when they relied on the special powers while the remainder is paid by the employer. This is party to the master policy, and even in a non-
of attorney, which Capt. Nuval presented as known as a contributory plan as compared to a contributory plan, the payment by the employer of
documents, when they released the checks to the non-contributory plan where the premiums are solely the entire premium is a part of the total
latter paid by the employer. compensation paid for the services of the employee.
2. WON Insular Life should be liable to the - the labor of the employees is the true source 2. YES
complainants when they released the check in favor of the benefits, which are a form of additional Ratio Regardless of the value of the unemancipated
of Ayo and LOntok, even if no bond was posted as compensation to them. common child's property, the father and mother ipso
required - the employer is the agent of the insurer in jure become the legal guardian of the child's
performing the duties of administering group property. However, if the market value of the
HELD insurance policies. It cannot be said that the property or the annual income of the child exceeds
1. YES employer acts entirely for its own benefit or for the P50,000,00, a bond has to be posted by the parents
Ratio Third persons deal with agents at their peril benefit of its employees in undertaking concerned to guarantee the performance of the
and are bound to inquire as to the extent of the administrative functions. While a reduced premium obligations of a general guardian.
power of the agent with whom they contract. The may result if the employer relieves the insurer of Reasoning
person dealing with an agent must also act these tasks, and this, of course, is advantageous to
INSURANCE Page 4

- Sec 180, Insurance Code: 'In the absence of a - Cebu Shipyard and Engineering Works, Inc. 20. The insurance on the vessel should be
judicial guardian, the father, or in the latter's (CSEW) is engaged in the business of dry-docking maintained by the customer and/or owner of the
absence or incapacity, the mother of any minor, who and repairing of marine vessels while the Prudential vessel during the period the contract is in effect.”
is an insured or a beneficiary under a contract of life, Guarantee and Assurance, Inc. (Prudential) is in the - While the M/V Manila City was undergoing dry-
health or accident insurance, may exercise, in behalf non-life insurance business. docking and repairs within the premises of CSEW,
of said minor, any right under the policy, without - William Lines, Inc. is in the shipping business. It the master, officers and crew of M/V Manila City
necessity of court authority or the giving of a bond was the owner of M/V Manila City, a luxury stayed in the vessel, using their cabins as living
where the interest of the minor in the particular act passenger-cargo vessel, which caught fire and sank quarters. Other employees hired by William Lines to
involved does not exceed twenty thousand pesos " on Feb. 16, 1991. At the time of the unfortunate do repairs and maintenance work on the vessel were
- …repealed by Art. 225, Family Code: "ART. 225. occurrence sued upon, subject vessel was insured also present during the dry-docking.
The father and the mother shall jointly exercise legal with Prudential for P45M for hull and machinery. The - On February 16, 1991, after subject vessel was
guardianship over the property of their Hull Policy included an “Additional Perils transferred to the docking quay, it caught fire and
unemancipated common child without the necessity (INCHMAREE)” Clause covering loss of or damage to sank, resulting to its eventual total loss.
of a court appointment. In case of disagreement, the the vessel through the negligence of, among others, - On February 21, 1991, William Lines, Inc. filed a
father's decision shall prevail, unless there is judicial ship repairmen complaint for damages against CSEW, alleging that
order to the contrary. - Petitioner CSEW was also insured by Prudential for the fire which broke out in M/V Manila City was
Where the market value of the property or the third party liability under a Shiprepairer’s Legal caused by CSEW’s negligence and lack of care.
annual income of the child exceeds P50,000, the Liability Insurance Policy. The policy was for P10 - On July 15, 1991 was filed an Amended Complaint
parent concerned shall be required to furnish a bond million only, under the limited liability clause, to wit: impleading Prudential as co-plaintiff, after the latter
in such amount as the court may determine, but not - On Feb. 5, 1991, William Lines, Inc. brought its had paid William Lines, Inc. the value of the hull and
less than ten per centum (10%) of the value of the vessel, M/V Manila City, to the Cebu Shipyard in machinery insurance on the M/V Manila City. As a
property or annual income, to guarantee the Lapulapu City for annual dry-docking and repair. result of such payment Prudential was subrogated to
performance of the obligations prescribed for general - On Feb. 6, 1991, an arrival conference was held the claim of P45 million, representing the value of
guardians." between representatives of William Lines, Inc. and the said insurance it paid.
-"market value of the property or the annual CSEW to discuss the work to be undertaken on the On June 10, 1994, the trial court a quo came out
income of the child": the aggregate of the child's M/V Manila City. The contracts, denominated as with a judgment against CSEW:
property or annual income; if this exceeds Work Orders, were signed thereafter., with the 1. To pay unto plaintiff Prudential Guarantee and
P50,000.00, a bond is required - There is no following stipulations: Assurance, Inc., the subrogee, the amount of P45M,
evidence that the share of each of the minors in the “10. The Contractor shall replace at its own work and with interest at the legal rate until full payment is
proceeds of the group policy in question is the at its own cost any work or material which can be made; the amount of P56,715,000 representing loss
minor's only property. Without such evidence, it shown to be defective and which is communicated in of income of M/V MANILA CITY, with interest at the
would not be safe to conclude that, indeed, that is writing within one (1) month of redelivery of the legal rate until full payment is made;
his only property. vessel or if the vessel was not in the Contractor’s 2. To pay unto plaintiff, William Lines, Inc. the
Disposition the instant petition is GRANTED. The Possession, the withdrawal of the Contractor’s amount of P11M as payment, in addition to what it
Decision of 10 October 1991 and the Resolution of workmen, or at its option to pay a sum equal to the received from the insurance company to fully cover
19 May 1992 of the public respondent in CA-G.R. SP cost of such replacement at its own works. These the injury or loss, in order to replace the M/V
No. 22950 are SET ASIDE and the Decision of the conditions shall apply to any such replacements. MANILA CITY, with interest at the legal rate until full
Insurance Commission in IC Case No. RD-058 is 11. Save as provided in Clause 10, the Contractor payment is made; the sum of P927,039 for the loss
REINSTATED. Costs against the private respondent. shall not be under any liability to the Customer of fuel and lub oil on board the vessel when she was
SO ORDERED. either in contract or for delict or quasi-delict or completely gutted by fire at defendant, Cebu
otherwise except for negligence and such liability Shipyard’s quay, with interest at the legal rate until
CEBU SHIPYARD ENGINEERING WORKS, INC. V shall itself be subject to the following overriding full payment is made; the sum of P3,054,677.95 as
WILLIAM LINES, INC. and PRUDENTIAL limitations and exceptions, namely: payment for the spare parts and materials used in
GUARANTEE and ASSURANCE COMPANY, INC. (a) The total liability of the Contractor to the the M/V MANILA CITY during dry-docking with
[CITATION] Customer (over and above the liability to replace interest at the legal rate until full payment is made;
PURISIMA; May 5, 1999 under Clause 10) or of any sub-contractor shall be P500,000 in moral damages;the amount of P10Min
limited in respect of any defect or event (and a attorney’s fees; and to pay the costs of this suit.
NATURE series of accidents arising out of the same defect or - On September 3, 1997, the Court of Appeals
Petition for review on certiorari event shall constitute one defect or event) to the affirmed the appealed decision of the trial court,
sum of Pesos Philippine Currency One Million only. ordering CSEW to pay Prudential, the subrogee, the
FACTS x x x sum of P45 Million, with interest at the legal rate
until full payment is made.
INSURANCE Page 5

CSEW’s version: being done by CSEW on the vessel, when he saw - The CA and the Cebu RTC are agreed that the fire
On Feb. 13, 1991, the CSEW completed the that some workers of CSEW were cropping out steel which caused the total loss of subject M/V Manila
drydocking of M/V Manila City at its grave dock. It plates on Tank Top No. 12 using acetylene, oxygen City was due to the negligence of the employees and
was then transferred to the docking quay of CSEW and welding torch. He also observed that the rubber workers of CSEW. Both courts found that the M/V
where the remaining repair to be done was the insulation wire coming out of the air-conditioning Manila City was under the custody and control of
replating of the top of Water Ballast Tank No. 12 unit was already burning, prompting him to scold the petitioner CSEW, when the ill-fated vessel caught
which was subcontracted by CSEW to JNB General workers. fire. The decisions of both the lower court and the CA
Services. Tank Top No. 12 was at the rear section of > At 2:45 PM of the same day, witnesses saw smoke set forth clearly the evidence sustaining their finding
the vessel, on level with the flooring of the crew coming from Tank No. 12. The vessel’s reeferman of actionable negligence on the part of CSEW. This
cabins located on the vessel’s second deck. reported such occurence to the Chief Mate who factual finding is accorded great weight and is
At around 7AM of Feb. 16, 1991, the JNB workers immediately assembled the crew members to put conclusive on the parties. The court discerns no basis
trimmed and cleaned the tank top framing which out the fire. When it was too hot for them to stay on for disturbing such finding firmly anchored on
involved minor hotworks (welding/cutting works). board and seeing that the fire cannot be controlled, enough evidence.
The said work was completed at about 10AM. The the vessel’s crew were forced to withdraw from - Furthermore, in petitions for review on certiorari,
JNB workers then proceeded to rig the steel plates, CSEW’s docking quay. only questions of law may be put into issue.
after which they had their lunch break. The rigging - In the morning of Feb. 17, 1991, M/V Manila City Questions of fact cannot be entertained. The finding
was resumed at 1PM sank. As the vessel was insured with Prudential of negligence by the CA is a question which this
While in the process of rigging the second steel Guarantee, William Lines filed a claim for Court cannot look into as it would entail going into
plate, the JNB workers noticed smoke coming from constructive total loss, and after a thorough factual matters on which the finding of negligence
the passageway along the crew cabins. When one of investigation of the surrounding circumstances of the was based. Such an approach cannot be allowed by
the workers, Mr. Casas, proceeded to the tragedy, Prudential found the said insurance claim to this Court in the absence of clear showing that the
passageway to ascertain the origin of the smoke, he be meritorious and issued a check in favor of case falls under any of the exceptions to the well-
noticed that smoke was gathering on the ceiling of William Lines in the amount of P45 million pesos established principle.
the passageway but did not see any fire as the crew representing the total value of M/V Manila City’s hull The finding by the trial court and the Court of
cabins on either side of the passageway were locked. and machinery insurance. Appeals that M/V Manila City caught fire and sank by
He immediately sought out the proprietor of JNB, reason of the negligence of the workers of CSEW,
Mr. Buenavista, and the Safety Officer of CSEW, Mr. ISSUES when the said vessel was under the exclusive
Aves, who sounded the fire alarm. CSEW’s fire 1. WON CSEW had “management and supervisory custody and control of CSEW is accordingly upheld.
brigade immediately responded as well as the other control“ of the m/v manila city at the time the fire 2. YES
fire fighting units in Metro Cebu. However, there broke out - For the doctrine of res ipsa loquitur to apply to a
were no WLI representative, officer or crew to guide 2. WON the doctrine of res ipsa loquitur applies given situation, the following conditions must
the firemen inside the vessel. against the crew concur: (1) the accident was of a kind which does
- Despite the combined efforts of the firemen of the 3. WON CSEW’S expert evidence is admissible or of not ordinarily occur unless someone is negligent;
Lapulapu City Fire Dept., Mandaue Fire Dept., probative value and (2) that the instrumentality or agency which
Cordova Fire Dept. Emergency Rescue Unit 4. WON Prudential has the right of subrogation caused the injury was under the exclusive control of
Foundation, and fire brigade of CSEW, the fire was against its own insured THE CONTRACTUAL 5. 5. 5. the person charged with negligence.
not controlled until 2AM of the following day. 5. WON the provisions limiting csew’s liability for The facts and evidence on record reveal the
- On the early morning of Feb. 17, 1991, gusty winds negligence to a maximum of p1 million are valid concurrence of said conditions in the case under
rekindled the flames on the vessel and fire again scrutiny. First, the fire that occurred and consumed
broke out. Then the huge amounts of water pumped HELD M/V Manila City would not have happened in the
into the vessel, coupled with the strong current, 1. YES ordinary course of things if reasonable care and
caused the vessel to tilt until it capsized and sank - The that factual findings by the CA are conclusive diligence had been exercised. In other words, some
- When M/V Manila City capsized, steel and angle on the parties and are not reviewable by this Court. negligence must have occurred. Second, the agency
bars were noticed to have been newly welded along They are entitled to great weight and respect, even charged with negligence, as found by the trial court
the port side of the hull of the vessel, at the level of finality, especially when, as in this case, the CA and the CA and as shown by the records, is the
the crew cabins. William Lines did not previously affirmed the factual findings arrived at by the trial herein petitioner, CSEW, which had control over
apply for a permit to do hotworks on the said portion court. When supported by sufficient evidence, subject vessel when it was docked for annual repairs.
of the ship as it should have done pursuant to its findings of fact by the CA affirming those of the trial So also, as found by the RTC, “other responsible
work order with CSEW. court, are not to be disturbed on appeal. The causes, including the conduct of the plaintiff, and
Prudential’s version rationale behind this doctrine is that review of the third persons, are sufficiently eliminated by the
> At around 7AM of Feb. 16, 1991, the Chief Mate of findings of fact of the CA is not a function that the evidence.”
M/V Manila City was inspecting the various works Supreme Court normally undertakes.
INSURANCE Page 6

What is more, in the present case the trial court - It is petitioner’s submission that the loss of M/V dry-docking or repair. Concededly, such a
found direct evidence to prove that the workers Manila City or damage thereto is expressly excluded stipulation works to the benefit of CSEW as the
and/or employees of CSEW were remiss in their duty from the coverage of the insurance because the shiprepairer. However, the fact that CSEW benefits
of exercising due diligence in the care of subject same resulted from “want of due diligence by the from the said stipulation does not automatically
vessel. The direct evidence substantiates the Assured, Owners or Managers” which is not included make it as a co-assured of William Lines. The
conclusion that CSEW was really negligent. Thus, in the risks insured against. Again, this theory of intention of the parties to make each other a co-
even without applying the doctrine of res ipsa petitioner is bereft of any factual or legal basis. It assured under an insurance policy is to be gleaned
loquitur, in light of the direct evidence on record, the proceeds from a wrong premise that the fire which principally from the insurance contract or policy itself
ineluctable conclusion is that CSEW was negligent gutted subject vessel was caused by the negligence and not from any other contract or agreement
and consequently liable for damages to the of the employees of William Lines, Inc. To repeat, because the insurance policy denominates the
respondent, William Lines, Inc. the issue of who between the parties was negligent assured and the beneficiaries of the insurance. The
3. NO has already been resolved against CSEW. Upon proof hull and machinery insurance procured by William
- Petitioner maintains that the CA erred in of payment by Prudential to William Lines, Inc., the Lines, Inc. from Prudential named only “William
disregarding the testimonies of the fire experts, former was subrogated to the right of the latter to Lines, Inc.” as the assured. There was no
Messrs. David Grey and Gregory Michael Southeard, indemnification from CSEW. As aptly ruled by the manifestation of any intention of William Lines, Inc.
who testified on the probable origin of the fire in M/V Court of Appeals, the law on the matter is succinct to constitute CSEW as a co-assured under subject
Manila City. Petitioner avers that since the said fire and clear, to wit: policy. It is axiomatic that when the terms of a
experts were one in their opinion that the fire did not - Art. 2207. If the plaintiff’s property has been contract are clear its stipulations control. i] Thus,
originate in the area of Tank Top No. 12 where the insured, and he has received indemnity from the when the insurance policy involved named only
JNB workers were doing hotworks but on the crew insurance company for the injury or loss arising out William Lines, Inc. as the assured thereunder, the
accommodation cabins on the portside No. 2 deck, of the wrong or breach of contract complained of, claim of CSEW that it is a co-assured is unfounded.
the RTC and the CA should have given weight to the insurance company shall be subrogated to the - Then too, in the Additional Perils Clause of the
such finding based on the testimonies of fire experts; rights of the insured against the wrongdoer or the same Marine Insurance Policy, it is provided that:
petitioner argues. person who has violated the contract. If the amount Subject to the conditions of this Policy, this
But courts are not bound by the testimonies of paid by the insurance company does not fully cover insurance also covers loss of or damage to vessel
expert witnesses. Although they may have probative the injury or loss, the aggrieved party shall be directly caused by the following:
value, reception in evidence of expert testimonies is entitled to recover the deficiency from the person xxx
within the discretion of the court, under Section 49, causing the loss or injury. Negligence of Charterers and/or Repairers, provided
Rule 130 of the Revised Rules of Court. It is never - Thus, when Prudential, after due verification of the such Charterers and/or Repairers are not an
mandatory for judges to give substantial weight to merit and validity of the insurance claim of William Assured hereunder.
expert testimonies. If from the facts and evidence on Lines, Inc., paid the latter the total amount covered - As correctly pointed out by respondent Prudential,
record, a conclusion is readily ascertainable, there is by its insurance policy, it was subrogated to the right if CSEW were deemed a co-assured under the policy,
no need for the judge to resort to expert opinion of the latter to recover the insured loss from the it would nullify any claim of William Lines, Inc. from
evidence. In the case under consideration, the liable party, CSEW. Prudential for any loss or damage caused by the
testimonies of the fire experts were not the only - Petitioner theorizes further that there can be no negligence of CSEW. Certainly, no shipowner would
available evidence on the probable cause and origin right of subrogation as it is deemed a co-assured agree to make a shiprepairer a co-assured under
of the fire. There were witnesses who were actually under the subject insurance policy. To buttress its such insurance policy; otherwise, any claim for loss
on board the vessel when the fire occurred. stance that it is a co-assured, petitioner placed or damage under the policy would be invalidated.
Between the testimonies of the fire experts who reliance on Clause 20 of the Work Order which Such result could not have been intended by William
merely based their findings and opinions on states: Lines, Inc.
interviews and the testimonies of those present 20. The insurance on the vessel should be 5. NO
during the fire, the latter are of more probative maintained by the customer and/or owner of the - Although in this jurisdiction, contracts of adhesion
value. vessel during the period the contract is in effect. have been consistently upheld as valid per se; as
- According to petitioner, under the aforecited binding as an ordinary contract, the Court recognizes
clause, William Lines, Inc., agreed to assume the risk instances when reliance on such contracts cannot be
of loss of the vessel while under drydock or repair favored especially where the facts and circumstances
4. YES and to such extent, it is benefited and effectively warrant that subject stipulations be disregarded.
- Petitioner contends that Prudential is not entitled to constituted as a co-assured under the policy. Thus, in ruling on the validity and applicability of the
be subrogated to the rights of William Lines, Inc., - This theory of petitioner is devoid of sustainable stipulation limiting the liability of CSEW for
theorizing that (1) the fire which gutted M/V Manila merit. Clause 20 of the Work Order in question is negligence to P1M only, the facts and circumstances
City was an excluded risk and (2) it is a co-assured clear in the sense that it requires William Lines to vis-a-vis the nature of the provision sought to be
under the Marine Hull Insurance Policy. maintain insurance on the vessel during the period of
INSURANCE Page 7

enforced should be considered, bearing in mind the and Insurance Co. Inc., and Equitable Insurance goods in process and/or inventories only hereby
principles of equity and fair play. Corporation. insured, and unless such notice be given and the
- It is worthy to note that M/V Manila City was - On May 15, 1981, Western Guaranty Corporation particulars of such insurance or insurances be stated
insured with Prudential for P45M. To determine the issued Fire Insurance Policy No. 37201 in the amount therein or endorsed on this policy pursuant to
validity and sustainability of the claim of William of P350,000.00. This policy was renewed on May 13, Section 50 of the Insurance Code, by or on behalf of
Lines, Inc., for a total loss, Prudential conducted its 1982. the Company before the occurrence of any loss or
own inquiry. Upon thorough investigation by its hull - On July 30, 1981, Reliance Surety and Insurance damage, all benefits under this policy shall be
surveyor, M/V Manila City was found to be beyond Co., Inc. issued Fire Insurance Policy No. 69135 in deemed forfeited, provided however, that this
economical salvage and repair. The evaluation of the the amount of P300,000.00 (Renewed under condition shall not apply when the total insurance or
average adjuster also reported a constructive total Renewal Certificate No. 41997). An additional insurances in force at the time of loss or damage is
loss. The said claim of William Lines, Inc., was then insurance was issued by the same company on not more than P200,000.00."
found to be valid and compensable such that November 12, 1981 under Fire Insurance Policy No. Petitioner’s comments
Prudential paid the latter the total value of its 71547 in the amount of P700,000.00. > Petitioners contend that they are not to be blamed
insurance claim. Furthermore, it was ascertained - On February 8, 1982, Equitable Insurance for the omissions, alleging that insurance agent Leon
that the replacement cost of the vessel (the price of Corporation issued Fire Insurance Policy No. 39328 Alvarez (for Western) and Yap Kam Chuan (for
a vessel similar to M/V Manila City), amounts to in the amount of P200,000.00. Reliance and Equitable) knew about the existence of
P55M. - Thus when the building occupied by the New Life the additional insurance coverage and that they were
- Considering the aforestated circumstances, let Enterprises was gutted by fire at about 2:00 o'clock not informed about the requirement that such other
alone the fact that negligence on the part of in the morning of October 19, 1982, the stocks in or additional insurance should be stated in the
petitioner has been sufficiently proven, it would trade inside said building were insured against fire in policy, as they have not even read said policies.
indeed be unfair and inequitable to limit the liability the total amount of P1,550,000.00. According to the
of petitioner to One Million Pesos only. As aptly held certification issued by the Headquarters, Philippine ISSUE
by the trial court, “it is rather unconscionable if not Constabulary/Integrated National Police, Camp WON New Life Enterprises’ claim for payment be
overstrained.” To allow CSEW to limit its liability to Crame, the cause of fire was electrical in nature. denied
P1M notwithstanding the fact that the total loss According to the plaintiffs, the building and the
suffered by the assured and paid for by Prudential stocks inside were burned. After the fire, Julian Sy HELD
amounted to P45M would sanction the exercise of a went to the agent of Reliance Insurance whom he YES
degree of diligence short of what is ordinarily asked to accompany him to the office of the Ratio Furthermore, when the words and language of
required because, then, it would not be difficult for company so that he can file his claim. He averred documents are clear and plain or readily
petitioner to escape liability by the simple expedient that in support of his claim, he submitted the fire understandable by an ordinary reader thereof, there
of paying an amount very much lower than the clearance, the insurance policies and inventory of is absolutely no room for interpretation or
actual damage or loss suffered by William Lines, Inc. stocks. construction anymore. Courts are not allowed to
Disposition Petition is DENIED. Resolution of the He further testified that the three insurance make contracts for the parties; rather, they will
CA is AFFIRMED. companies are sister companies, and as a matter of intervene only when the terms of the policy are
fact when he was following-up his claim with ambiguous, equivocal, or uncertain. The parties
NEW LIFE ENTERPRISES V CA Equitable Insurance, the Claims Manager told him to must abide by the terms of the contract because
207 SCRA 669 go first to Reliance Insurance and if said company such terms constitute the measure of the insurer's
REGALADO; March 31, 1992 agrees to pay, they would also pay. The same liability and compliance therewith is a condition
treatment was given him by the other insurance precedent to the insured's right of recovery from the
NATURE companies. Ultimately, the three insurance insurer.
Appeal by certiorari companies denied plaintiffs' claim for payment. - While it is a cardinal principle of insurance law that
Respondent’s comments a policy or contract of insurance is to be construed
FACTS > Western Guaranty Corporation through Claims liberally in favor of the insured and strictly against
- The antecedents of this case show that Julian Sy Manager Bernard S. Razon told the plaintiff that his the insurer company, yet contracts of insurance, like
and Jose Sy Bang have formed a business claim 'is denied for breach of policy conditions.' other contracts, are to be construed according to the
partnership in the City of Lucena. Under the business Reliance Insurance purveyed the same message as sense and meaning of the terms which the parties
name of New Life Enterprises, the partnership well as Equitable Insurance Corporation. themselves have used. If such terms are clear and
engaged in the sale of construction materials at its - The said policy in question follows: unambiguous, they must be taken and understood in
place of business, a two storey building situated at "The insured shall give notice to the Company of any their plain, ordinary and popular sense. Moreover,
Iyam, Lucena City. The facts show that Julian Sy insurance or insurances already effected, or which obligations arising from contracts have the force of
insured the stocks in trade of New Life Enterprises may subsequently be effected, covering any of the law between the contracting parties and should be
with Western Guaranty Corporation, Reliance Surety property or properties consisting of stocks in trade, complied with in good faith.
INSURANCE Page 8

Reasoning - The plaintiff was brought to the Manila Sanitarium from death or bodily injury at P12,000.00 per
a. The terms of the contract are clear and and Hospital where the doctors performed 2 major passenger and its maximum liability per accident at
unambiguous. The insured is specifically required to surgical operations on plaintiffs right leg. (P50,000.00. Since only one passenger was injured
disclose to the insurer any other insurance and its - Plaintiff was confined at the hospital for (40) days, in the accident, the insurer's liability for the damages
particulars which he may have effected on the same from June 10, 1984 to August 26, 1984. Medical suffered by said passenger is pegged to the amount
subject matter. The knowledge of such insurance by expenses totaled the amount of P69,444.41. of P12,000.00 only.
the insurer's agents, even assuming the acquisition Plaintiff’s medical expenses were advanced by his - The limit of P50,000.00 per accident means that
thereof by the former, is not the "notice" that would employer Maglines but he was required to reimburse the insurer's maximum liability for any single
stop the insurers from denying the claim. Besides, Maglines on a staggered basis by way of salary accident will not exceed P50,000.00 regardless of the
the so-called theory of imputed knowledge, that is, deductions. After his release from the hospital, he number of passengers killed or injured therein.
knowledge of the agent is knowledge of the principal, returned to the hospital for further treatment and The bus company may not recover from the
aside from being of dubious applicability here has checkup. The injuries had left plaintiff with a huge insurance company more than P12,000.00 per
likewise been roundly refuted by respondent court scar on his right leg. Also, the plaintiff incurred lost passenger killed or injured, or (P50,000.00) per
whose factual findings we find acceptable. earning by way of unearned salaries amounting to accident even if under the judgment of the court, the
b. Petitioners should be aware of the fact that a P7,500.00 due to said physical injuries and the erring bus operator will have to pay more than
party is not relieved of the duty to exercise the consequent hospital confinement. P12,000.00 to each injured passenger. The trial
ordinary care and prudence that would be exacted in - Plaintiff filed on June 26, 1985 the complaint court's interpretation of the insurance contract was
relation to other contracts. The conformity of the against DMTC and its driver. Agpalo was later the correct interpretation.
insured to the terms of the policy is implied from his dropped as a party defendant because he could not Disposition petition for review is GRANTED. The
failure to express any disagreement with what is be served with summons. Upon filing its answer, decision promulgated by the CA, ordering the third
provided for. defendant DMTC filed a thirdparty complaint against party defendent, First Quezon City Insurance Co.,
First Quezon City Insurance Co., Inc. September 17, Inc. to indemnify theI private respondent, (DMTC),
FIRST QUEZON CITY INSURANCE CO. v. CA (DE 1985, third-party defendant filed its answer to the the sum of P50,000.00 for the damages of the
DIOS MARIKINA TRANSPORT CO) third-party complaint. passenger, Jose V. Del Rosario, is hereby modified
218 SCRA 526 - TC held DMTC complaint dismissed for lack of merit by reducing the award to 12,000.00 only. Costs
GRINO-AQUINO; February 28, 1993 and as regards the third-party complaint First against the private respondent De Dios Marikina
Quezon City Insurance Co., Inc. was to indemnify Transportation Co., Inc.
NATURE third-party plaintiff DMTC in the sum of P12,000.00
PETITION for review of the decision of the Court of with interest. There being no satisfactory warrant the TY V FIRST NATIONAL SURETY
Appeals. FQCIC seeks to limit to P12000, the amount court dismissed the rest of the claims in the 1 SCRA 1324
specified in the insurance contract, it’s liability to complaint and third-party complaint. LABRADOR; April 29, 1961
indemnify the respomdemt DMTC, for the damages - The bus company appealed to the CA, which
suffered by a passenger, who accidentally fell off the modified the dispositive as regards the third-party FACTS
bug. complaint, that the third-party defendant First - At different times within a period of two months
Quezon City Insurance Co., Inc. be ordered to prior to 24 December 1953, Diosdado C. Ty,
FACTS indemnify third-party plaintiff DMTC the SUM of employed as operator mechanic foreman in the
- After sending off certain seamen at the departure P50,000.00 with legal interest. Insurance company Broadway Cotton Factory insured himself in 18 local
area of MIA, Jose V. del Rosario proceeded to the filed a MFR which was denied. insurance companies, among which being the 8
public utility bus stop. While at the bus stop, the Hence, this petition for review, assailing the above-named defendants, which issued to him
plaintiff saw a DMTC bus. While moving at a crawling appellate courts' interpretation of the provision of personal accident policies. Plaintiff’s beneficiary was
pace, it was taking several passengers, all of whom the insurance contract on the limit of the insurer's his employer, Broadway Cotton Factory, which paid
managed to board the bus while it was already at the liability. the insurance premiums. On 24 December 1953, a
bus stop; plaintiff was the last one to board the bus. fire broke out which totally destroyed the Broadway
While the plaintiff was still on the bus with his hand ISSUE Cotton Factory. Fighting his way out of the factory,
on the bus door, the slowly moving bus sped forward WON the CA erred in the interpretation of the plaintiff was injured on the left hand by a heavy
at a high speed, as a result of which, the plaintiff lost insurance contract on the limit of the insurer’s object. He was brought to the Manila Central
balance and fell from the bus. As plaintiff clung liability University hospital, and after receiving first-aid, he
instinctively to the handle bar, he was dragged by went to the National Orthopedic Hospital for
the bus along the asphalted road. The bus driver, Gil HELD treatment of his injuries (fractures in index, middle,
Agpalo, abruptly stopped the bus. Then fled from the YES fourth, and fifth fingers of left hand). From 26
scene, leaving the bus and the injured plaintiff - The insurance policy clearly placed the maximum December 1953 to 8 February 1954, he underwent
behind. limit of the petitioner's liability for damages arising medical treatment in the hospital. The above-
INSURANCE Page 9

described physical injuries have caused temporary lower court did not exonerate the said appellant for
total disability of plaintiff’s left hand. Plaintiff filed FACTS the excess because the company's absolution would
the corresponding notice of accident and notice of - Misamis Lumber Corporation, under its former render the insurance contract one-sided and that the
claim with all of the above-named defendants to name, Lanao Timber Mills, Inc., insured its Ford said insurer had not shown that the cost of repairs in
recover indemnity. Defendants rejected plaintiff’s Falcon motor car for the amount of P14,000 with the sum of P302.27 is unreasonable, excessive or
claim for indemnity for the reason that there being Capital Insurance & Surety Company, Inc. The padded, nor had it shown that it could have
no severance of amputation of the left hand, the pertinent provisions of the policy provided, as undertaken the repairs itself at less expense.
disability suffered by him was not covered by his follows:
policy. 1. The Company will subject to the Limits of ISSUE
- Plaintiff sued the defendants in the Municipality Liability indemnify the Insured against loss or WON Capital Insurance can be made to pay more
Court of this City, which dismissed his complaints. damage to the Motor Vehicle and its accessories than P150
Thereafter, the plaintiff appealed to the Court of First and spare parts whilst thereon.
Instance Manila, presided by Judge Gregorio S. 2. (a) by accidental collision or overturning or HELD
Narvasa, which absolved the defendants from the collision or overturning consequent when NO
complaints. Hence, the appeal. mechanical breakdown or consequent upon wear - The insurance policy stipulated in paragraph 4 that
and tear. if the insured authorizes the repair the liability of the
ISSUE 3. At its option, the Company may pay in cash the insurer, per its sub-paragraph (a), is limited to
WON Diosdado Ty is entitled to indemnity under the amount of the loss or damage or may repair, P150.00. The literal meaning of this stipulation must
insurance policy for the disability of his left hand reinstate or replace the Motor Vehicle or any part control, it being the actual contract, expressly and
thereof or its accessories or spare parts. The plainly provided for in the policy.
HELD liability of the Company shall not exceed the value - Recourse to legal hermeneutics is not called for
- The agreement contained in the insurance policies of the parts lost or damaged and the reasonable because paragraph 4 of the policy is clear and
is the law between the parties. As the terms of the cost of fitting such parts or the value of the Motor specific and leaves no room for interpretation.
policies are clear, express and specific that only Vehicle at the time of the loss or damage - The option to undertake the repairs is accorded to
amputation of the left hand should be considered as whichever is the loss. The Insured's estimate of the insurance company per paragraph 2. The said
a loss thereof, an interpretation that would include value stated in the schedule shall be the maximum company was deprived of the option because the
the mere fracture or other temporary disability not amount payable by the Company in respect of any insured took it upon itself to have the repairs made,
covered by the policies would certainly be claim for loss or damage. and only notified the insurer when the repairs were
unwarranted. In the case at bar, due to the clarity of 4. The Insured may authorize the repair of the done. As a consequence, paragraph 4, which limits
the stipulation, distinction between “temporary Motor Vehicle necessitated by damage for which the company's liability to P150.00, applies.
disability” and “total disability” need not be made in the Company may be liable under this policy - The insurance contract may be rather onerous
relation to one’s occupation means that the condition provided that: ("one-sided", as the lower court put it), but that in
of the insurance is such that common prudence (a) the estimated cost of such repair does not itself does not justify the abrogation of its express
requires him to desist from transacting his business exceed the authorized Repair Limit. terms, terms which the insured accepted or adhered
or renders him incapable of working. While the Court (b) a detailed estimate of the cost is forwarded to and which is the law between the contracting
sympathizes with the plaintiff or his employer, for to the Company without delay and providing parties.
whose benefit the policies were issued, it can not go also that the authorized repair limit is P150.00. - To require the insurer to prove that the cost of the
beyond the clear and express conditions of the - One night, the insured car, while traveling along in repairs ordered by the insured is unreasonable, when
insurance policies, all of which define partial Aurora Boulevard, passed over a water hole which the insurer was not given an opportunity to inspect
disability as loss of either hand by a amputation the driver did not see because an oncoming car did and assess the damage before the repairs were
through the bones of the wrist.” There was no such not dim its light. The crankcase and flywheel housing made, is contrary to elementary justice and equity.
amputation in the case at bar. of the car broke when it hit a hollow block lying
- The Supreme Court affirmed the appealed decision, alongside the water hole. The car was towed and SUN INSURANCE OFFICE LTD. V CA (TAN)
with costs against the plaintiff-appellant. repaired by Morosi Motors at a total cost of P302.27. 195 SCRA 193
- When the repairs on the car had already been PARAS; March 13, 1991
MISAMIS LUMBER V CAPITAL INSURANCE made, Misamis made a report of the accident to
17 SCRA 288 Capital Insurance. NATURE
REYES; May 20, 1966 - Since Capital refused to pay for the total cost of to Petition for certiorari to review the decision of the CA
wage and repairs, suit was filed in the municipal
NATURE court originally. FACTS
Direct appeal on a point of law from the judgment of - The defendant-appellant admits liability in the - Private respondent Emilio Tan took from the
the Court of First Instance of Manila amount of P150, but not for any excess thereof. The petitioner a Peso 300,000 property insurance policy
INSURANCE Page 10

to cover his interest in the electrical insurance store running of the prescriptive period. The Court noted the Insured whether acting alone or in
of his brother housed in a building in Iloilo City on that the rationale for the one year period is to ensure conjunction with others.
August 15, 1983. Four days after the issuance of the that the evidence as to the origin and cause of the - Fortune opposes the contention of Producers that
policy, the building including the insured store destruction have not yet disappeared. Atiga and Magalong are not its "officer, employee, x
burned. 2. NO x x trustee or authorized representative x x x at the
- On August 20, 1983, Tan filed his claim for fire - The Court clarified its ruling in Eagle Star Insurance time of the robbery
loss. Sun Insurance, on February 29, 1984, wrote Co. vs Chia Yu where it ruled that “the cause of - Trial Court
the private respondent denying the claim. On April 3, action in an insurance contract does not accrue until > On being “EMPLOYEES”
1984, private respondent wrote another letter to the the Insured’s claim is finally rejected by the Insurer” Magalong and Atiga were not employees or
insurance company requesting reconsideration of the by stating the use of the word “finally” cannot be representatives of Producers as their services as
denial. Tan’s lawyer wrote another letter to the construed to mean the rejection of a petition for armored car driver and as security guard having
insurance company inquiring about the April 3 letter reconsideration. What the court referred to in effect been merely offered by PRC Management and by
which sought for a reconsideration of the denial. In is the rejection in the first instance as claimed by Unicorn Security and which latter firms assigned
its reply to the lawyer’s letter, Sun Insurance Sun Insurance them to plaintiff. The wages and salaries of both
reiterated its denial of the claim and enclosed therein Disposition The decision of the CA is reversed and Magalong and Atiga are presumably paid by their
copies of the two previous denials dated February set aside. The case is dismissed respective firms, which alone wields the power to
29, 1984 and May 17, 1985. dismiss them
- On November 20, 1985, Tan filed a civil case with FORTUNE INSURANCE AND SURETY CO. INC.V > On being “AUTHORIZED REPRESENTATIVE”
the RTC. Petition filed a motion to dismiss on the CA (PRODUCERS BANK OF THE PHILIPPINES) They were merely an assigned armored car driver
alleged ground that the action has already prescribed 244 SCRA 308 and security guard for the money transfer. It was
based on Condition 27 of the Insurance Policy which DAVIDE; May 23, 1995 teller Maribeth Alampay who had "custody" of the
stated that the window to file the appropriate action P725,000.00 cash being transferred along a specified
with either the Insurance Commission or in any court NATURE money route
of competent jurisdiction is twelve months from the Petition for Review on certiorari of CA decision - Court of Appeals
rejection of the claim. RTC denied the motion and > affirmed in toto
the subsequent motion for reconsideration. The CA FACTS > A policy or contract of insurance is to be construed
likewise denied the petition of Sun Insurance. - Producers Bank of the Philippines filed a complaint liberally in favor of the insured and strictly against
against Fortune Insurance and Surety Co., Inc. for the insurance company (New Life Enterprises vs.
ISSUE recovery of P725,000.00 under the policy issued by Court of Appeals; Sun Insurance Office, Ltd. vs.
1. WON the court the filing of a motion for Fortune. The sum was allegedly lost on June 29, Court of Appeals). Contracts of insurance, like other
reconsideration interrupts the 12 months prescription 1987 during a robbery of Producer's armored vehicle contracts, are to be construed according to the sense
period to contest the denial of the insurance claim while it was in transit to transfer the money from its and meaning of the terms which the parties
2. WON the rejection of the claim shall be deemed Pasay City Branch to its head office in Makati under themselves have used. If such terms are clear and
final only if it contains words to the effect that the the custody of its teller, Maribeth Alampay. The unambiguous, they must be taken and understood in
denial is final armored car was driven by Benjamin Magalong Y de their plain, ordinary and popular sense (New Life
Vera, escorted by Security Guard Saturnino Atiga Y Enterprises Case; Sun Insurance Office).
Rosete. Driver Magalong was assigned by PRC > The language used by Fortune in the policy is
HELD Management Systems. plain, ordinary and simple. No other interpretation is
1. NO - After an investigation by the Pasay police, driver necessary. The word "employee" should be taken to
- The SC held that Condition 27 of the Insurance Magalong and guard Atiga were charged, together mean in the ordinary sense. The Labor Code is a
policy is very clear and free from any doubt or with Batigue , Aquino and John Doe, with violation of special law specifically dealing with/and specifically
ambiguity. It has to be taken in its plain, ordinary, P.D. 532 (Anti-Highway Robbery Law) designed to protect labor and therefore its definition
and popular sense. The rejection letter of February - Demands were made by the Producers upon the as to employer-employee relationships insofar as the
29, 1984 was clear and plain. The Court noted that Fortune to pay the amount of the loss of application/enforcement of said Code is concerned
the one year period is likewise in accord with Section P725,000.00 but the latter refused to pay as the loss must necessarily be inapplicable to an insurance
23 of the Insurance Code which states that any is excluded from the coverage of the insurance policy contract. Had it intended to apply the Labor Code in
condition which limits the time for commencing an specifically under "General Exceptions" defining what the word "employee" refers to, it
action to a period of less than one year when the > The company shall not be liable under this must/ should have so stated expressly in the
cause of action accrues is void. The right of action, policy in respect of x x x (b) any loss caused by insurance policy. Said driver and security guard
according to the SC, accrues at the time that the any dishonest, fraudulent or criminal act of the cannot be considered as employees of Producers
claim is rejected at the first instance. A request for insured or any officer, employee, partner, bank because it has no power to hire or to dismiss
reconsideration of the denial cannot suspend the director, trustee or authorized representative of said driver and security guard under the contracts
INSURANCE Page 11

except only to ask for their replacements from the is an employer-employee relationship between the from non-compliance with its obligation. It goes
contractors. owner of the project and the employee of the "labor- without saying then that if the terms of the contract
- Fortune’s Contention only" contractor are clear and unambiguous, there is no room for
> when Producers commissioned a guard and a - Producer’s Contention construction and such terms cannot be enlarged or
driver to transfer its funds from one branch to > Magalong and Atiga were not its employees since diminished by judicial construction.
another, they effectively and necessarily became its it had nothing to do with their selection and - An insurance contract is a contract of indemnity
authorized representatives in the care and custody of engagement, the payment of their wages, their upon the terms and conditions specified therein. It is
the money. Assuming that they could not be dismissal, and the control of their conduct. settled that the terms of the policy constitute the
considered authorized representatives, they were, > International Timber Corp. is not applicable to all measure of the insurer's liability. In the absence of
nevertheless, employees of Producers. It asserts that cases but only when it becomes necessary to prevent statutory prohibition to the contrary, insurance
the existence of an employer-employee relationship any violation or circumvention of the Labor Code, a companies have the same rights as individuals to
"is determined by law and being such, it cannot be social legislation whose provisions may set aside limit their liability and to impose whatever conditions
the subject of agreement." Thus, if there was in contracts entered into by parties in order to give they deem best upon their obligations not
reality an employer-employee relationship between protection to the working man. inconsistent with public policy.
Producers, on the one hand, and Magalong and > American President Lines vs. Clave should be Reasoning
Atiga, on the other, the provisions in the contracts of applied which stated - It should be noted that the insurance policy
Producers with PRC Management System for In determining the existence of employer- entered into by the parties is a theft or robbery
Magalong and with Unicorn Security Services for employee relationship, the following elements are insurance policy which is a form of casualty
Atiga which state that Producers is not their generally considered, namely: (1) the selection insurance. Section 174 of the Insurance Code
employer and that it is absolved from any liability as and engagement of the employee; (2) the provides:
an employer, would not obliterate the relationship. payment of wages; (3) the power of dismissal; Sec. 174. Casualty insurance is insurance covering
> an employer-employee relationship depends upon and (4) the power to control the employee's loss or liability arising from accident or mishap,
four standards: conduct. excluding certain types of loss which by law or
(1) the manner of selection and engagement of the - Since under Producers' contract with PRC custom are considered as failing exclusively within
putative employee Management Systems it is the latter which assigned the scope of insurance such as fire or marine. It
(2) the mode of payment of wages Magalong as the driver of Producers' armored car includes, but is not limited to, employer's liability
(3) the presence or absence of a power to dismiss and was responsible for his faithful discharge of his insurance, public liability insurance, motor vehicle
and duties and responsibilities, and since Producers paid liability insurance, plate glass insurance, burglary
(4) the presence and absence of a power to control the monthly compensation of P1,400.00 per driver to and theft insurance, personal accident and health
the putative employee's conduct. PRC Management Systems and not to Magalong, it is insurance as written by non-life insurance
> Of the four, the right-of-control test has been held clear that Magalong was not Producers' employee. As companies, and other substantially similar kinds of
to be the decisive factor. It asserts that the power of to Atiga, Producers relies on the provision of its insurance. (italics supplied)
control over Magalong and Atiga was vested in and contract with Unicorn Security Services which - Except with respect to compulsory motor vehicle
exercised by Producers. Fortune further insists that provides that the guards of the latter "are in no liability insurance, the Insurance Code contains no
PRC Management System and Unicorn Security sense employees of the CLIENT." other provisions applicable to casualty insurance or
Services are but "labor-only" contractors under to robbery insurance in particular. These contracts
Article 106 of the Labor Code which provides: ISSUE are, therefore, governed by the general provisions
Art. 106. Contractor or subcontractor. - There is WON Fortune Insurance and Surety Co. Inc. is liable applicable to all types of insurance. Outside of these,
"labor-only" contracting where the person under the Money, Security, and Payroll Robbery the rights and obligations of the parties must be
supplying workers to an employer does not have policy it issued to Producers Bank of the Philippines determined by the terms of their contract, taking
substantial capital or investment in the form of or WON recovery is precluded under the general into consideration its purpose and always in
tools, equipment, machineries, work premises, exceptions clause of the policy accordance with the general principles of insurance
among others, and the workers recruited and law.
placed by such persons are performing activities HELD - With the foregoing principles in mind, it may now
which are directly related to the principal business NO be asked whether Magalong and Atiga qualify as
of such employer. In such cases, the person or Ratio A contract of insurance is a contract of employees or authorized representatives has been
intermediary shall be considered merely as an adhesion, thus any ambiguity therein should be aptly observed that in burglary, robbery, and theft
agent of the employer who shall be responsible to resolved against the insurer, or it should be insurance, "the opportunity to defraud the insurer -
the workers in the same manner and extent as if construed liberally in favor of the insured and strictly the moral hazard - is so great that insurers have
the latter were directly employed by him. against the insurer. Limitations of liability should be found it necessary to fill up their policies with
> International Timber Corp. vs. NLRC - a "labor- regarded with extreme jealousy and must be countless restrictions, many designed to reduce this
only" contractor is equivalent to a finding that there construed in such a way as to preclude the insurer hazard. Seldom does the insurer assume the risk of
INSURANCE Page 12

all losses due to the hazards insured against." Disposition instant petition is hereby GRANTED. CA - Verendia is found to have concocted the lease
Persons frequently excluded under such provisions decision and RTC Makati decision are REVERSED and contract to deflect responsibility for the fire towards
are those in the insured's service and employment. SET ASIDE. Civil Case is DISMISSED. an alleged lessee, even making it appear that the
The purpose of the exception is to guard against alleged lessee had disappeared, inflated the value of
liability should the theft be committed by one having VERENDIA V CA (FIDELITY & SURETY CO. OF the property, and insured same property with two
unrestricted access to the property. In such cases, THE PHILS) other companies.
the terms specifying the excluded classes are to be 217 SCRA 417 - An insurance contract is the law between the
given their meaning as understood in common MELO; January 22, 1993 parties, its terms and conditions constitute the
speech. The terms "service" and "employment" are measure of the insurer’s liability and compliance
generally associated with the idea of selection, NATURE therewith is a condition precedent to the insured’s
control, and compensation. Petition to review decision of the CA right to recovery from the insurer.
- There is marked disagreement between the parties - As it is also a contract of adhesion, an insurance
on the correct meaning of the terms "employee" FACTS contract should be liberally construed in favor of the
and "authorized representatives." - Fidelity Co. issued a Fire Insurance Policy covering insured and strictly against the insurer company
It is clear to us that insofar as Fortune is concerned, Verendia’s residential building in the amount of which usually prepares it.
it was its intention to exclude and exempt from P385k. Verendia also insured the same building with - Considering, however, the fact that Verendia used
protection and coverage losses arising from two other companies (Country Bankers Insurance for a false lease contract to support his claim, the terms
dishonest, fraudulent, or criminal acts of persons P56k, and Development Insurance for P400k). of the policy should be strictly construed against the
granted or having unrestricted access to Producers' - While all 3 policies were in force, the insured insured. Verendia failed to live by the terms of the
money or payroll. When it used then the term property was completely destroyed by fire. Verendia policy, specifically Section 13 thereof which is
"employee," it must have had in mind any person filed a claim against Fidelity, but the latter refused expressed in terms that are clear and unambiguous,
who qualifies as such as generally and universally payment, thus a complaint was filed in the RTC. that all benefits under the policy shall be forfeited “If
understood, or jurisprudentially established in the Fidelity’s reason for refusal: the policy was avoided the claim be in any respect fraudulent, or if any false
light of the four standards in the determination of by reason of over-insurance, and that Verendia declaration be made or used in support thereof, or if
the employer-employee relationship or as statutorily maliciously represented that the building was under any fraudulent means or devises are used by the
declared even in a limited sense as in the case of lease to a Roberto Garcia, when it was actually a Insured or anyone acting in his behalf to obtain any
Article 106 of the Labor Code which considers the Marcelo Garcia who was the lessee. benefit under the policy”. Verendia, having presented
employees under a "labor-only" contract as - RTC: policy was violated by Verendia when it failed a false declaration to support his claim for benefits in
employees of the party employing them and not of to inform Fidelity of his other insurance coverages, the form of a fraudulent lease contract, he forfeited
the party who supplied them to the employer. thus no need to pay. all benefits therein by virtue of Section 13 of the
- But even granting for the sake of argument that - CA: reversed decision policy in the absence of proof that Fidelity waived
these contracts were not "labor-only" contracts, and such provision. Worse yet, by presenting a false
PRC Management Systems and Unicorn Security ISSUE lease contract, Verendia reprehensibly disregarded
Services were truly independent contractors, we are (There is a procedural issue involved here, but is the principle that insurance contracts are uberrimae
satisfied that Magalong and Atiga were, in respect of irrelevant to our discussion. It concerns the filing of fidae and demand the most abundant good faith.
the transfer of Producer's money from its Pasay City a motion for extension of time to file a motion for Disposition Decision of CA reversed, and that of
branch to its head office in Makati, its "authorized reconsideration, where the court said that although it RTC is reinstated.
representatives" who served as such with its teller now prohibits filing of such motion, the instant FIELDMEN'S INSURANCE CO. INC V VDA. DE
Maribeth Alampay. Howsoever viewed, Producers motion was filed before the effectivity of this rule, SONGCO
entrusted the three with the specific duty to safely thus allowing the adjudication of the case) 25 SCRA 20
transfer the money to its head office, with Alampay WON Fidelity was liable to pay Verendia considering FERNANDO; 1968
to be responsible for its custody in transit; Magalong the circumstances
to drive the armored vehicle which would carry the FACTS
money; and Atiga to provide the needed security for HELD - An insurance firm, petitioner Fieldmen's Insurance
the money, the vehicle, and his two other 1. NO Co., Inc., was not allowed to escape liability under a
companions. In short, for these particular tasks, the Ratio As the insurance contract is the law between common carrier insurance policy on the pretext that
three acted as agents of Producers. A the parties, Verendia is deemed to have forfeited his what was insured, not once but twice, was a private
"representative" is defined as one who represents or right to claim by the misrepresentation he made. vehicle and not a common carrier, the policy being
stands in the place of another; one who represents Reasoning issued upon the insistence of its agent who
others or another in a special capacity, as an agent, - the court reviewed the factual findings of the courts discounted fears of the insured that his privately
and is interchangeable with "agent." below, since it appears that there was a owned vehicle might not fall within its terms, the
misapprehension of the facts by the CA. insured moreover being "a man of scant education,"
INSURANCE Page 13

finishing only the first grade. So it was held in a whenever they believe a vehicle is insurable' ... In under the terms of the policy 5 was inescapable was
decision of the lower court thereafter affirmed by spite of the fact that the present case was filed and set forth in the decision of respondent Court of
respondent Court of Appeals. Petitioner in seeking tried in the CFI of Pampanga, the defendant Appeals. Thus: "Since some of the conditions
the review of the above decision of respondent Court company did not even care to rebut Amor Songco's contained in the policy issued by the defendant-
of Appeals cannot be so sanguine as to entertain the testimony by calling on the witness-stand agent appellant were impossible to comply with under the
belief that a different outcome could be expected. To Benjamin Sambat, its Pampanga Field existing conditions at the time and 'inconsistent with
be more explicit, we sustain the Court of Appeals. Representative." 2 the known facts,' the insurer 'is estopped from
- The facts as found by respondent Court of Appeals, - The plaintiffs in the lower court, likewise asserting breach of such conditions.' From this
binding upon us, follow: "This is a peculiar case. respondents here, were the surviving widow and jurisprudence, we find no valid reason to deviate and
Federico Songco of Floridablanca, Pampanga, a man children of the deceased Federico Songco as well as consequently hold that the decision appealed from
of scant education being only a first grader ..., the injured passenger Jose Manuel. On the above should be affirmed. The injured parties, to wit,
owned a private jeepney with Plate No. 41-289 for facts they prevailed, as had been mentioned, in the Carlos Songco, Angelito Songco and Jose Manuel, for
the year 1960. On September 15, 1960, as such lower court and in the respondent Court of whose hospital and medical expenses the defendant
private vehicle owner, he was induced by Fieldmen's Appeals.1awphîl.nèt company was being made liable, were passengers of
Insurance Company Pampanga agent Benjamin - The basis for the favorable judgment is the the jeepney at the time of the occurrence, and
Sambat to apply for a Common Carrier's Liability doctrine announced in Qua Chee Gan v. Law Union Rodolfo Songco, for whose burial expenses the
Insurance Policy covering his motor vehicle ... Upon and Rock Insurance Co., Ltd., 3 with Justice J. B. L. defendant company was also being made liable was
paying an annual premium of P16.50, defendant Reyes speaking for the Court. It is now beyond the driver of the vehicle in question. Except for the
Fieldmen's Insurance Company, Inc. issued on question that where inequitable conduct is shown by fact, that they were not fare paying passengers,
September 19, 1960, Common Carriers Accident an insurance firm, it is "estopped from enforcing their status as beneficiaries under the policy is
Insurance Policy No. 45-HO- 4254 ... the duration of forfeitures in its favor, in order to forestall fraud or recognized therein." 6
which will be for one (1) year, effective September imposition on the insured." 4 - Even if it be assumed that there was an ambiguity,
15, 1960 to September 15, 1961. On September 22, - As much, if not much more so than the Qua Chee an excerpt from the Qua Chee Gan decision would
1961, the defendant company, upon payment of the Gan decision, this is a case where the doctrine of reveal anew the weakness of petitioner's contention.
corresponding premium, renewed the policy by estoppel undeniably calls for application. After Thus: "Moreover, taking into account the well known
extending the coverage from October 15, 1961 to petitioner Fieldmen's Insurance Co., Inc. had led the rule that ambiguities or obscurities must be strictly
October 15, 1962. This time Federico Songco's insured Federico Songco to believe that he could interpreted against the party that caused them, the
private jeepney carried Plate No. J-68136- qualify under the common carrier liability insurance 'memo of warranty' invoked by appellant bars the
Pampanga-1961. ... On October 29, 1961, during the policy, and to enter into contract of insurance paying latter from questioning the existence of the
effectivity of the renewed policy, the insured vehicle the premiums due, it could not, thereafter, in any appliances called for in the insured premises, since
while being driven by Rodolfo Songco, a duly litigation arising out of such representation, be its initial expression, 'the undernoted appliances for
licensed driver and son of Federico (the vehicle permitted to change its stand to the detriment of the the extinction of fire being kept on the premises
owner) collided with a car in the municipality of heirs of the insured. As estoppel is primarily based insured hereby, ... it is hereby warranted ...,' admits
Calumpit, province of Bulacan, as a result of which on the doctrine of good faith and the avoidance of of interpretation as an admission of the existence of
mishap Federico Songco (father) and Rodolfo Songco harm that will befall the innocent party due to its such appliances which appellant cannot now
(son) died, Carlos Songco (another son), the latter's injurious reliance, the failure to apply it in this case contradict, should the parol evidence rule apply." 7
wife, Angelita Songco, and a family friend by the would result in a gross travesty of justice. - To the same effect is the following citation from the
name of Jose Manuel sustained physical injuries of - That is all that needs be said insofar as the first same leading case: "This rigid application of the rule
varying degree." 1 alleged error of respondent Court of Appeals is on ambiguities has become necessary in view of
- It was further shown according to the decision of concerned, petitioner being adamant in its far-from- current business practices. The courts cannot ignore
respondent Court of Appeals: "Amor Songco, 42- reasonable plea that estoppel could not be invoked that nowadays monopolies, cartels and concentration
year-old son of deceased Federico Songco, testifying by the heirs of the insured as a bar to the alleged of capital, endowed with overwhelming economic
as witness, declared that when insurance agent breach of warranty and condition in the policy. lt power, manage to impose upon parties dealing with
Benjamin Sambat was inducing his father to insure would now rely on the fact that the insured owned a them cunningly prepared 'agreements' that the
his vehicle, he butted in saying: 'That cannot be, Mr. private vehicle, not a common carrier, something weaker party may not change one whit, his
Sambat, because our vehicle is an "owner" private which it knew all along when not once but twice its participation in the 'agreement' being reduced to the
vehicle and not for passengers,' to which agent agent, no doubt without any objection in its part, alternative to 'take it or leave it' labelled since
Sambat replied: 'whether our vehicle was an "owner" exerted the utmost pressure on the insured, a man Raymond Saleilles 'contracts by adherence' (contrats
type or for passengers it could be insured because of scant education, to enter into such a contract. d'adhesion), in contrast to those entered into by
their company is not owned by the Government and - Nor is there any merit to the second alleged error parties bargaining on an equal footing, such
the Government has nothing to do with their of respondent Court that no legal liability was contracts (of which policies of insurance and
company. So they could do what they please incurred under the policy by petitioner. Why liability international bills of lading are prime examples)
INSURANCE Page 14

obviously call for greater strictness and vigilance on Corporation for which it issued two (2) Marine Cargo Respondents Comments
the part of courts of justice with a view to protecting Policies. - petitioner, being the sole author of the policies,
the weaker party from abuses and imposition, and - While the vessel was docked in Durban, South "arrests" should be strictly interpreted against it
prevent their becoming traps for the unwary (New Africa the civil authorities arrested and detained it because the rule is that any ambiguity is to be taken
Civil Code. Article 24; Sent. of Supreme Court of because of a lawsuit on a question of ownership and contra proferentum. Risk policies should be
Spain, 13 Dec. 1934, 27 February 1942)." 8 possession. TKC Marketing notified Malayan of the construed reasonably and in a manner as to make
- The last error assigned which would find fault with arrest of the vessel and made a formal claim for the effective the intentions and expectations of the
the decision of respondent Court of Appeals insofar dollar equivalent on the policies (US$916,886.66) for parties.
as it affirmed the lower court award for exemplary non-delivery of the cargo. It likewise sought the - the policies clearly stipulate that they cover the
damages as well as attorney's fees is, on its face, of assistance of Malayan on what to do with the cargo. risks of non-delivery of an entire package and that it
no persuasive force at all. - Malayan replied that the arrest of the vessel by civil was petitioner itself that invited and granted the
- The conclusion that inescapably emerges from the authority was not a peril covered by the policies. TKC extensions and collected premiums thereon.
above is the correctness of the decision of advised Malayan that it might tranship the cargo and
respondent Court of Appeals sought to be reviewed. requested an extension of the insurance coverage ISSUES
For, to borrow once again from the language of the until actual transhipment, which extension was 1. WON the arrest of the vessel was a risk covered
Qua Chee Gan opinion: "The contract of insurance is approved upon payment of additional premium. The under the subject insurance policies
one of perfect good faith (uberima fides) not for the insurance coverage was extended under the same 2. WON insurance policies should be strictly
insured alone,but equally so for the insurer; in fact, terms and conditions embodied in the original construed against the insurer
it is more so for the latter, since its dominant policies while in the process of making arrangements
bargaining position carries with it stricter for the transhipment of the cargo from Durban to HELD
responsibility." 9 Manila. However the cargo was sold in Durban, 1.YES
- This is merely to stress that while the morality of South Africa, for US$154.40 per metric ton or a total - With the incorporation of subsection 1.1 of Section
the business world is not the morality of institutions of P10,304,231.75 due to its perishable nature which 1 of the Institute War Clauses, "arrest" caused by
of rectitude like the pulpit and the academe, it could no longer stand a voyage of twenty days to ordinary judicial process is deemed included among
cannot descend so low as to be another name for Manila and another twenty days for the discharge the covered risks. This interpretation becomes
guile or deception. Moreover, should it happen thus, thereof. It reduced its claim to US$448,806.09 (or its inevitable when subsection 1.1 of Section 1 of the
no court of justice should allow itself to lend its peso equivalent of P9,879,928.89 at the exchange Institute War Clauses provided that "this insurance
approval and support.1awphîl.nèt rate of P22.0138 per $1.00) representing its loss covers the risks excluded from the Standard Form of
- We have no choice but to recognize the monetary after the proceeds of the sale were deducted from English Marine Policy by the clause 'Warranted free
responsibility of petitioner Fieldmen's Insurance Co., the original claim.Malayan maintained its position of capture, seizure, arrest, etc. x x x'" or the F.C. &
Inc. It did not succeed in its persistent effort to avoid that the arrest of the vessel by civil authorities on a S. Clause. Jurisprudentially, "arrests" caused by
complying with its obligation in the lower court and question of ownership was an excepted risk under ordinary judicial process is also a risk excluded from
the Court of Appeals. Much less should it find any the marine insurance policies. the Standard Form of English Marine Policy by the
receptivity from us for its unwarranted and Petitioners Claim F.C. & S. Clause.
unjustified plea to escape from its liability. - an arrest by civil authority is not compensable - Petitioner cannot adopt the argument that the
since the term "arrest" refers to "political or "arrest" caused by ordinary judicial process is not
MALAYAN INSURANCE CORP. V CA (TKC executive acts" and does not include a loss caused included in the covered risk simply because the F.C.
MARKETING CORP.) by riot or by ordinary judicial process as in this case & S. Clause under the Institute War Clauses can only
270 SCRA 242 - the deletion of the Free from Capture or Seizure be operative in case of hostilities or warlike
ROMERO; March 20, 1997 Clause would leave the assured covered solely for operations on account of its heading "Institute War
the perils specified by the wording of the policy itself Clauses."
NATURE - the rationale for the exclusion of an arrest pursuant 2. YES
Petition for review on certiorari to judicial authorities is to eliminate collusion Ratio Insurance Policies should be construed
between unscrupulous assured and civil authorities. liberally in favor of the insured and strictly against
FACTS - any loss which private respondent may have the insurer.
- TKC Marketing Corp. was the owner/consignee of incurred was in the nature and form of unrecovered Reasoning
some 3,189.171 metric tons of soya bean meal acquisition value brought about by a voluntary - An insurance contract should be so interpreted as
which was loaded on board the ship MV Al Kaziemah sacrifice sale and not by arrest, detention or seizure to carry out the purpose for which the parties
for carriage from the port of Rio del Grande, Brazil, of the ship. entered into the contract which is, to insure against
to the port of Manila. Said cargo was insured against - its act of rejecting the claim was a result of its risks of loss or damage to the goods. Such
the risk of loss by petitioner Malayan Insurance honest belief that the arrest of the vessel was not a interpretation should result from the natural and
compensable risk under the policies issued reasonable meaning of language in the policy. Where
INSURANCE Page 15

restrictive provisions are open to two interpretations, Dios Hospital. Her face was permanently disfigured, NO
that which is most favorable to the insured is causing her serious anxiety and moral distress. Ratio An insurance policy being in the nature of an
adopted. - Respondent bus company was insured with adhesion contract is to be strictly construed against
Indemnity and liability insurance policies are petitioner Western Guaranty Corporation ("Western") the insurer and liberally in favor of the insured.
construed in accordance with the general rule of under its Master Policy which enumerated specific Reasoning
resolving any ambiguity therein in favor of the liabilities of the insurance company and ended with a - Firstly, the Schedule of Indemnities does not
insured, where the contract or policy is prepared by clause to clarify the limitations of the amount which purport to restrict the kinds of damages that may be
the insurer. A contract of insurance, being a contract could be granted as indemnity. awarded against Western once liability has arisen.
of adhesion, par excellence, any ambiguity therein - Respondent Priscilla Rodriguez filed a complaint for Section 1, quoted above, does refer to certain
should be resolved against the insurer.Limitations of damages before the Regional Trial Court of Makati "Limits of Liability" which in the case of the third
liability should be regarded with extreme jealousy against De Dios Transportation Co. and Walter A. party liability section of the Master Policy, is
and must be construed in such a way as to preclude Saga. Respondent De Dios Transportation Co., in apparently P50,000.00 per person per accident.
the insurer from noncompliance with its obligations turn, filed a third-party complaint against its Within this over-all quantitative limit, all kinds of
- It must be borne in mind that such contracts are insurance carrier, petitioner Western. damages allowable by law "actual or compensatory
invariably prepared by the companies and must be - On 6 August 1985, the trial court rendered a damages"; "moral damages"; "nominal damages";
accepted by the insured in the form in which they decision in favor of respondent Priscilla E. Rodriguez, "temperate or moderate damages"; "liquidated
are written. Any construction of a marine policy - On appeal, the Court of Appeals affirmed in toto damages"; and "exemplary damages" may be
rendering it void should be avoided. Such policies the decision of the trial court. Petitioner moved for awarded by a competent court against the insurer
will, therefore, be construed strictly against the the reconsideration of the appellate court's decision. once liability is shown to have arisen, and the
company in order to avoid a forfeiture, unless no In a Resolution dated 10 January 1990, the Court of essential requisites or conditions for grant of each
other result is possible from the language used. Appeals denied the motion for reconsideration for species of damages are present. It appears to us
- If a marine insurance company desires to limit or lack of merit. Petitioner Western is now before us on self-evident that the Schedule of Indemnities was
restrict the operation of the general provisions of its a Petition for Review alleging that the Court of not intended to be an enumeration, much less a
contract by special proviso, exception, or exemption, Appeals erred in holding petitioner liable to pay closed enumeration, of the specific kinds of damages
it should express such limitation in clear and beyond the limits set forth in the Schedule which may be awarded under the Master Policy
unmistakable language. Indemnities and in finding Western liable for loss of Western has issued.
Be that as it may, exceptions to the general earnings, moral damages and attorney's fees. - Secondly, the reading urged by Western of the
coverage are construed most strongly against the Succinctly stated, it is petitioner Western's position Schedule of Indemnities comes too close to working
company. Even an express exception in a policy is to that it cannot be held liable for loss of earnings, fraud upon both the insured and the third party
be construed against the underwriters by whom the moral damages and attorney's fees because these beneficiary of Section 1, quoted above. For
policy is framed, and for whose benefit the exception items are not among those included in the Schedule Western's reading would drastically and without
is introduced. Indemnities set forth in the insurance policy. warning limit the otherwise unlimited (save for the
- Petitioner Western in effect contends before this over-all quantitative limit of liability of P50,000.00
WESTERN GUARANTY CORPORATION V CA Court, as it did before the Court of Appeals, that per person per accident) and comprehensive scope
(RODRIGUEZ, and DE DIOS TRANSPORTATION because the Schedule of Indemnities limits the of liability assumed by the insurer Western under
CO) amount payable for certain kinds of expenses Section 1: "all sums necessary to discharge liability
187 SCRA 652 "hospital room", "surgical expenses", "an of the insured in respect of [bodily injury to a third
FELICIANO; July 20, 1990 aesthesiologists' fee", "operating room" and "medical party]". This result which is not essentially
expenses" that Schedule should be read as different from taking away with the left hand what
FACTS excluding liability for any other type of expense or had been given with the right hand we must avoid
- At around 4:30 in the afternoon of 27 March 1982, damage or loss even though actually sustained or as obviously repugnant to public policy. If what
while crossing Airport Road on a pedestrian lane on incurred by the third party victim. We are not Western now urges is what Western intended to
her way to work, respondent Priscilla E. Rodriguez persuaded by Western's contention. achieve by its Schedule of Indemnities, it was
was struck by a De Dios passenger bus owned by incumbent upon Western to use language far more
respondent De Dios Transportation Co., Inc., then ISSUE specific and precise than that used in fact by
driven by one Walter Saga y Aspero. The bus driver WON the Schedule of indemnities as stated in the Western, so that the insured, and potential
disregarded the stop signal given by a traffic insurance policy should be construed strictly to purchasers of its Master Policy, and the Office of the
policeman to allow pedestrians to cross the road. exclude all others not explicitly stated therein Insurance Commissioner, may be properly informed
Priscilla was thrown to the ground, hitting her and act accordingly.
forehead. She was treated at the Protacio Emergency - Petitioner Western would have us construe the
Hospital and later on hospitalized at the San Juan De Schedule of Indemnities as comprising contractual
HELD limitations of liability which, as already noted, is
INSURANCE Page 16

comprehensively defined in Section 1 "Liability to in 1940 for the crime of arson, it being claimed that 1. NO
the Public" of the Master Policy. It is well-settled, they had set fire to the destroyed warehouses to Ratio It is usually held that where the insurer, at the
however, that contractual limitations of liability found collect the insurance. They were, however, acquitted time of the issuance of a policy of insurance, has
in insurance contracts should be regarded by courts by the trial court. knowledge of existing facts which, if insisted on,
with a jaundiced eye and extreme care and should - the civil suit to collect the insurance money would invalidate the contract from its very inception,
be so construed as to preclude the insurer from proceeded to its trial with the CFI holding that: such knowledge constitutes a waiver of conditions in
evading compliance with its just obligations. judgment is rendered for the plaintiff and against the the contract inconsistent with the facts, and the
- Finally, an insurance contract is a contract of defendant condemning the latter to pay the former insurer is stopped thereafter from asserting the
adhesion. The rule is well entrenched in our — (a) Under the first cause of action, the sum of breach of such conditions. The law is charitable
jurisprudence that the terms of such contract are to P146,394.48; (b) Under the second cause of action, enough to assume, in the absence of any showing to
be construed strictly against the party which the sum of P150,000; (c) Under the third cause of the contrary, that an insurance company intends to
prepared the contract, which in this case happens to action, the sum of P5,000; (d) Under the fourth executed a valid contract in return for the premium
be petitioner Western. cause of action, the sum of P15,000; and (e) Under received; and when the policy contains a condition
the fifth cause of action, the sum of P40,000; all of which renders it voidable at its inception, and this
QUA CHEE GAN V LAW UNION AND ROCK which shall bear interest at the rate of 8% per result is known to the insurer, it will be presumed to
INSURANCE CO., LTD. annum in accordance with Section 91 (b) of the have intended to waive the conditions and to execute
96 PHIL 85 Insurance Act from September 26, 1940, until each a binding contract, rather than to have deceived the
REYES; December 17, 1955 is paid, with costs against the defendant. insured into thinking he is insured when in fact he is
- In its first assignment of error, the insurance not, and to have taken his money without
NATURE company alleges that the trial Court should have consideration.
An appeal by defendant insurance company from the held that the policies were avoided for breach of Reasoning
decision of CFI in favor of the plaintiff warranty, specifically the one appearing on a rider - The appellant is barred estoppel to claim violation
pasted (with other similar riders) on the face of the of the so-called fire hydrants warranty, for the
FACTS policies.4 reason that knowing fully all that the number of
- before the last war, plaintiff-appellee owned 4 - It is argued that since the bodegas insured had an hydrants demanded therein never existed from the
warehouses or bodegas in Tabaco, Albay, used for external wall perimeter of 500 meters or 1,640 feet, very beginning, the appellant neverthless issued the
the storage of stocks of copra and of hemp, baled the appellee should have 11 fire hydrants in the policies in question subject to such warranty, and
and loose, in which the appellee dealt extensively. compound, and that he actually had only 2, with a received the corresponding premiums. The insurance
They had been, with their contents, insured with the further pair nearby, belonging to the municipality of company was aware, even before the policies were
defendant Company since 1937, and the loose made Tabaco. issued, that in the premises insured there were only
payable to the Philippine National Bank as mortgage two fire hydrants installed by Qua Chee Gan and two
of the hemp and crops, to the extent of its interest. ISSUES others nearby, owned by the municipality of Tabaco,
- Fire of undetermined origin that broke out in the 1. WON the defendant-appellant can claim the contrary to the requirements of the warranty in
early morning of July 21, 1940, and lasted almost policies it had issued as void ab initio question
one week, gutted and completely destroyed Bodegas 2. WON the insured violated the "Hemp Warranty" - The plain, human justice of this doctrine is
Nos. 1, 2 and 4, with the merchandise stored provisions of Policy No. 2637165 against the storage perfectly apparent. To allow a company to
therein. Plaintiff-appellee informed the insurer by of gasoline accept one's money for a policy of insurance
telegram on the same date; and on the next day, the 3. WON the insured connived at the loss and which it then knows to be void and of no effect,
fire adjusters engaged by appellant insurance fraudulently inflated the quantity of the insured stock though it knows as it must, that the assured
company arrived and proceeded to examine and in the burnt bodegas believes it to be valid and binding, is so
photograph the premises, pored over the books of contrary to the dictates of honesty and fair
the insured and conducted an extensive HELD dealing, and so closely related to positive
investigation. The plaintiff having submitted the fraud, as to the abhorrent to fair-minded men.
4
corresponding fire claims, totalling P398,562.81 (but Memo. of Warranty . — The undernoted Appliances for the extinction of fire being kept on the It would be to allow the company to treat the
reduced to the full amount of the insurance, premises insured hereby, and it being declared and understood that there is an ample and constant policy as valid long enough to get the premium
water supply with sufficient pressure available at all seasons for the same, it is hereby warr anted
P370,000), the Insurance Company resisted that the said appliances shall be maintained in efficient working order during the currency of this
on it, and leave it at liberty to repudiate it the
payment, claiming violation of warranties and policy, by reason whereof a discount of 2 1/2 per cent is allowed on the premium chargeable under next moment. This cannot be deemed to be the
conditions, filing of fraudulent claims, and that the this policy. real intention of the parties. To hold that a
Hydrants in the compound, not less in number than one for each 150 feet of external wall
fire had been deliberately caused by the insured or measurement of building, protected, with not less than 100 feet of hose piping and nozzles for literal construction of the policy expressed the
by other persons in connivance with him. every two hydrants kept under cover in convenient places, the hydrants being supplied with water true intention of the company would be to
pressure by a pumping engine, or from some other source, capable of discharging at the rate of not
- Que Chee Gan, with his brother, Qua Chee Pao, less than 200 gallons of water per minute into the upper story of the highest building protected, and indict it, for fraudulent purposes and designs
and some employees of his, were indicted and tried a trained brigade of not less than 20 men to work the same.' which we cannot believe it to be guilty of.
INSURANCE Page 17

- The appellant company so worded the policies that their own intentions, the courts must, in unreasonable, considering that such listing was
while exacting the greater number of fire hydrants fairness to those who purchase insurance, superfluous because the insurer was not denied
and appliances, it kept the premium discount at the construe every ambiguity in favor of the access to the records, that the volume of Qua Chee
minimum of 2 1/2%, thereby giving the insurance insured. An insurer should not be allowed, by Gan's business ran into millions, and that the
company a double benefit. Such abnormal treatment the use of obscure phrases and exceptions, to demand was made just after the fire when
of the insured strongly points at an abuse of the defeat the very purpose for which the policy everything was in turmoil. That the representatives
insurance company's selection of the words and was procured. of the insurance company were able to secure all the
terms of the contract, over which it had absolute Reasoning date they needed is proved by the fact that the
control. - Appellee admitted that there were 36 cans of adjuster Alexander Stewart was able to prepare his
- Receipt of Premiums or Assessments after Cause gasoline in the building designed. It However, own balance sheet that did not differ from that
for Forfeiture Other than Nonpayment. — It is a well gasoline is not specifically mentioned among the submitted by the insured except for the valuation of
settled rule of law that an insurer which with prohibited articles listed in the so-called "hemp the merchandise, as expressly found by the Court in
knowledge of facts entitling it to treat a policy as no warranty." The cause relied upon by the insurer the criminal case for arson.
longer in force, receives and accepts a premium on speaks of "oils (animal and/or vegetable and/or 3. NO
the policy, estopped to take advantage of the mineral and/or their liquid products having a flash Ratio Both defenses are predicted on the
forfeiture. It cannot treat the policy as void for the point below 300 Fahrenheit)", and is decidedly assumption that the insured was in financial
purpose of defense to an action to recover for a loss ambiguous and uncertain; for in ordinary parlance, difficulties and set the fire to defraud the insurance
thereafter occurring and at the same time treat it as "Oils" mean "lubricants" and not gasoline or company, presumably in order to pay off the
valid for the purpose of earning and collecting kerosene. And how many insured, it may well be Philippine National Bank, to which most of the
further premiums. wondered, are in a position to understand or insured hemp and copra was pledged. Both defenses
- Moreover, taking into account the well known rule determine "flash point below 300 Fahrenheit. are fatally undermined by the established fact that,
that ambiguities or obscurities must be strictly - If the company intended to rely upon a condition of notwithstanding the insurer's refusal to pay the value
interpreted against the party that caused them, the that character, it ought to have been plainly of the policies the extensive resources of the insured
"memo of warranty" invoked by appellant bars the expressed in the policy. enabled him to pay off the National Bank in a short
latter from questioning the existence of the - The contract of insurance is one of perfect good time; and if he was able to do so, no motive appears
appliances called for in the insured premises faith not for the insured alone, but equally so for the for attempt to defraud the insurer. While the
On the alleged violations of the plaintiff The insurer; in fact, it is mere so for the latter, since its acquittal of the insured in the arson case is not res
alleged violation of the warranty of 100 feet of fire dominant bargaining position carries with it stricter judicata on the present civil action, the insurer's
hose for every two hydrants, must be equally responsibility. evidence, to judge from the decision in the criminal
rejected, since the appellant's argument thereon is - Another point that is in favor of the insured is that case, is practically identical in both cases and must
based on the assumption that the insured was bound the gasoline kept in Bodega No. 2 was only incidental lead to the same result, since the proof to establish
to maintain no less than eleven hydrants, which to his business, being no more than a customary 2 the defense of connivance at the fire in order to
requirement appellant is estopped from enforcing. day's supply for the five or six motor vehicles used defraud the insurer "cannot be materially less
- As to maintenance of a trained fire brigade of 20 for transporting of the stored merchandise. "It is well convincing than that required in order to convict the
men, the record is preponderant that the same was settled that the keeping of inflammable oils on the insured of the crime of arson."
organized, and drilled, from time to give, although premises though prohibited by the policy does not - As to the defense that the burned bodegas could
not maintained as a permanently separate unit, void it if such keeping is incidental to the business." not possibly have contained the quantities of copra
which the warranty did not require. On the submission of books, voucbers, etc. The and hemp stated in the fire claims, the insurer's case
2. NO charge that the insured failed or refused to submit to rests almost exclusively on the estimates, inferences
Ratio Here, again, by reason of the exclusive control the examiners of the insurer the books, vouchers, and conclusions of its adjuster investigator who
of the insurance company over the terms and etc. demanded by them was found unsubstantiated examined the premises during and after the fire. His
phraseology of the contract, the ambiguity must be by the trial Court, and no reason has been shown to testimony, however, was based on inferences from
held strictly against the insurer and liberally in favor alter this finding. The insured gave the insurance the photographs and traces found after the fire, and
of the insured, specially to avoid a forfeiture. examiner all the date he asked for, and the examiner must yield to the contradictory testimony of those
Insurance is, in its nature, complex and even kept and photographed some of the examined who actually saw the contents of the bodegas shortly
difficult for the layman to understand. Policies books in his possession. What does appear to have before the fire, while inspecting them for the
are prepared by experts who know and can been rejected by the insured was the demand that mortgagee Bank.
anticipate the hearing and possible he should submit "a list of all books, vouchers, Disposition We find no reversible error in the
complications of every contingency. So long as receipts and other records", but the refusal of the judgment appealed from, wherefore the same is
insurance companies insist upon the use of insured in this instance was well justified, since the hereby affirmed.
ambiguous, intricate and technical provisions, demand for a list of all the vouchers (which were not
which conceal rather than frankly disclose, in use by the insured) and receipts was positively
INSURANCE Page 18

DEL ROSARIO V EQUITABLE INSURANCE & x x x (h) By drowning except as a in the preparation of the policy, together with the
CASUALTY CO., INC consequence of the wrecking or drafting of its terms and conditions. The
8 SCRA 343 disablement in the Philippine waters of a interpretation of obscure stipulations in a contract
PAREDES; June 29, 1963 passenger steam or motor vessel in should not favor the party who caused the obscurity.
which the Insured is traveling as a - SC agreed with the ruling of the lower court:
NATURE farepaying passenger; x x x x x x death by drowning is a ground for recovery
Appeal from judgment of CFI Rizal - A rider to the Policy contained the following; apart from the bodily injury because death by
IV. DROWNING bodily injury is covered by Part I of the policy
FACTS It is hereby declared and agreed that while death by drowning is covered by Part VI
- Francisco del Rosario was insured by Equitable exemption clause Letter (h) in PART VI of thereof. But while the policy mentions specific
Insurance and Casualty Co. Inc under Personal the policy is hereby waived by the amounts that may be recovered for death for
Accident Policy no. 7136. The Company bound itself company, and to form a part of the bodily injury, yet, there is not specific amount
to pay P1000 to P3000 as indemnity for the death provision covered by the policy. mentioned in the policy for death thru drowning
of the insured. - Feb 24, 1957, Francisco del Rosario while on board although the latter is, under Part VI of the
- Under the policy: the motor launch ISLAMA, with his beneficiary to the policy, a ground for recovery thereunder. Since
Part I. Indemnity for Death policy, Remedios Jayme, were forced to jump off the defendant has bound itself to pay P1000 to
If the insured sustains any bodily injury said launch on account of fire which broke out on P3000 as indemnity for the death of the insured
which is effected solely through violent, said vessel, resulting in the death by drowning of the but the policy does not positively state any
external, visible and accidental means, insured and his beneficiary. definite amount that may be recovered in case
and which shall result, independently of - Simeon del Rosario, the insured’s father, filed a of death by drowning, there is an ambiguity in
all other causes and within sixty days claim for payment with the company. The company this respect in the policy, which ambiguity must
from the occurrence thereof, in the Death paid him P1000 pursuant to section 1 Part I of the be interpreted in favor of the insured and strictly
of the Insured, the Company shall pay the policy. against the insurer so as to allow a greater
amount set opposite such injury: - On the same date, Atty. Francisco wrote to the indemnity. x x x plaintiff is therefore entitled to
company acknowledging receipt by his client of the recover P3000.
Section 1. Injury sustained other than P1000 but informing said company that said amount Disposition Judgment appealed from is affirmed.
those specified below unless excepted was not the correct one. He claimed that the amount
hereinafter P1000 payable should be P1500 under the provision of GEAGONIA v. CA (COUNTRY BANKERS
Section 2. Injury sustained by the Section 2 Part I, based on the rule of pari materia. INSURANCE)
wrecking or disablement of a railroad - The company referred the matter to the Insurance 8 SCRA 343
passenger car or street railway car in or Coomissioner, who was of the opinion that the DAVIDE; February 6 1995
on which the Insured is traveling as a liability of the company was only P1000. thus the
farepaying passenger P1500 company refused to pay more that P1000. Atty. FACTS
Section 3. Injury sustained by the burning Francisco wrote a subsequent letter to company -Geagonia is the owner of Norman's Mart located in
of a church, theatre, public library or asking for p3000, which the company refused to pay. the public market of San Francisco, Agusan del Sur.
municipal administration building while - A complaint for recovery of the balance of P2000 On 22 Dec 1989, he obtained from the private
the Insured is therein at the was instituted with the CFI Rizal, praying for a respondent fire insurance policy for P100,000.00.
commencement of the fire P2000 further sum of P10000 as attorney’s fees, expenses The period of the policy was from 22 Dec 1989 to 22
Section 4. Injury sustained by the of litigation and costs. Dec 1990 and covered the ff: "Stock-in-trade
wrecking or disablement of a regular - CFI ruled in favor of petitioner, ordering the consisting principally of dry goods such as RTW's for
passenger elevator car in which the company to pay P2000 to del Rosario. men and women wear and other usual to assured's
Insured is being conveyed as a passenger business.
(Elevator in mines exluded) P2500 ISSUE -The policy contained the following condition:
Section 5. Injury sustained by a stroke of How much should the indemnity be "3. The insured shall give notice to the Company of
lightning or by a cycloneP3000 any insurance or insurances already effected, or
xxxx xxxx HELD which may subsequently be effected, covering any of
xxxx - All the parties agree that indemnity has to be paid, the property or properties consisting of stocks in
Part VI. Exceptions but the conflict centers on how much it should be. trade, goods in process and/or inventories only
This policy shall not cover disappearance - Where there is ambiguity with respect to the terms hereby insured, and unless notice be given and the
of the Insured nor shall it cover Death, and conditions of the policy, the same will be particulars of such insurance or insurances be stated
Disability, Hospital fees, or Loss of time, resolved against the one responsible thereof. therein or endorsed in this policy pursuant to Section
caused to the insured: Generally, the insured has little, if any, participation 50 of the Insurance Code, by or on behalf of the
INSURANCE Page 19

Company before the occurrence of any loss or time the complaint was filed until fully satisfied plus thereof. It expressly provides that the
damage, all benefits under this policy shall be the amount of P10,000.00 as attorney's fees. condition "shall not apply when the total
deemed forfeited, provided however, that this -CA reversed the decision of the Insurance insurance or insurances in force at the time of
condition shall not apply when the total insurance or Commission because it found that the petitioner the loss or damage is not more than
insurances in force at the time of the loss or damage knew of the existence of the two other policies P200,000.00."
is not more than P200,000.00." issued by the PFIC - Interpretation: It is a cardinal rule on insurance
-On 27 May 1990, fire of accidental origin broke out that a policy or insurance contract is to be
at around 7:30 p.m. at the public market of San ISSUES interpreted liberally in favor of the insured and
Francisco, Agusan del Sur. The petitioner's insured 1. WON the petitioner had prior knowledge of the strictly against the company, the reason being,
stocks-in-trade were completely destroyed two insurance policies issued by the PFIC when he undoubtedly, to afford the greatest protection which
prompting him to file w/ the private respondent a obtained the fire insurance policy from the private the insured was endeavoring to secure when he
claim under the policy. On 28 Dec 1990, the private respondent, thereby, for not disclosing such fact, applied for insurance. It is also a cardinal principle of
respondent denied the claim because it found that at violating Condition 3 of the policy law that forfeitures are not favored and that any
the time of the loss the petitioner's stocks-in-trade 2. if he had, WON he is precluded from recovering construction which would result in the forfeiture of
were likewise covered by two fire insurance policies therefrom the policy benefits for the person claiming
for P100,000.00 each, issued by the Cebu Branch of thereunder, will be avoided, if it is possible to
the Philippines First Insurance Co., Inc. (PFIC). HELD construe the policy in a manner which would permit
-The basis of the private respondent's denial was the 1. YES recovery, as, for example, by finding a waiver for
petitioner's alleged violation of Condition 3 of the - We agree w/ the CA that the petitioner knew of the such forfeiture. Stated differently, provisions,
policy. prior policies issued by the PFIC. His letter of 18 conditions or exceptions in policies which tend
- Geagonia then filed a complaint against the private January 1991 to the private respondent conclusively to work a forfeiture of insurance policies should
respondent w/ the Insurance Commission for the proves this knowledge. His testimony to the contrary be construed most strictly against those for
recovery of P100,000.00 under fire insurance policy, before the Insurance Commissioner and which the whose benefits they are inserted, and most
for attorney's fees, and costs of litigation. He claims latter relied upon cannot prevail over a written favorably toward those against whom they are
that the time he obtained the private respondent's admission made ante litem motam. It was, indeed, intended to operate. The reason for this is that,
fire insurance policy he knew that the two policies incredible that he did not know about the prior except for riders which may later be inserted, the
issued by the PFIC were already in existence; policies since these policies were not new or original. insured sees the contract already in its final form and
however, he had no knowledge of the provision in 2. NO has had no voice in the selection or arrangement of
the private respondent's policy requiring him to - It must, however, be underscored that unlike the the words employed therein. On the other hand, the
inform it of the prior policies; this requirement was "other insurance" clauses involved in General language of the contract was carefully chosen and
not mentioned to him by the private respondent's Insurance and Surety Corp. vs. Ng Hua or in deliberated upon by experts and legal advisers who
agent; and had it been so mentioned, he would not Pioneer Insurance & Surety Corp. vs. Yap, which had acted exclusively in the interest of the insurers
have withheld such information. He further asserted read: and the technical language employed therein is
that the total of the amounts claimed under the "The insured shall give notice to the company of any rarely understood by ordinary laymen.
three policies was below the actual value of his insurance or insurances already effected, or which - With these principles in mind, we are of the
stocks at the time of loss, w/c was P1M. may subsequently be effected covering any of the opinion that Condition 3 of the subject policy is
- The Insurance Commission found that the property hereby insured, and unless such notice be not totally free from ambiguity and must be
petitioner did not violate Condition 3 as he had no given and the particulars of such insurance or meticulously analyzed. Such analysis leads us
knowledge of the existence of the two fire insurance insurances be stated in or endorsed on this Policy by to conclude that (a) the prohibition applies only
policies obtained from the PFIC; that it was Cebu or on behalf of the Company before the occurrence to double insurance, and (b) the nullity of the
Tesing Textiles w/c procured the PFIC policies w/o of any loss or damage, all benefits under this Policy policy shall only be to the extent exceeding
informing him or securing his consent; and that Cebu shall be forfeited." or in the 1930 case of Santa Ana P200,000.00 of the total policies obtained.
Tesing Textile, as his creditor, had insurable interest vs. Commercial Union Assurance Co. which - Furthermore, by stating within Condition 3 itself
on the stocks. These findings were based on the provided "that any outstanding insurance upon the that such condition shall not apply if the total
petitioner's testimony that he came to know of the whole or a portion of the objects thereby assured insurance in force at the time of loss does not
PFIC policies only when he filed his claim with the must be declared by the insured in writing and he exceed P200,000.00, the private respondent was
private respondent and that Cebu Tesing Textile must cause the company to add or insert it in the amenable to assume a co-insurer's liability up to a
obtained them and paid for their premiums w/o policy, without which such policy shall be null and loss not exceeding P200,000.00. What it had in mind
informing him. The Insurance Commission then void, and the insured will not be entitled to was to discourage over-insurance. Indeed, the
ordered the respondent company to pay complainant indemnity in case of loss," Condition 3 in the rationale behind the incorporation of "other
the sum of P100,000.00 with legal interest from the private respondent's policy No. F-14622 does insurance" clause in fire policies is to prevent over-
not absolutely declare void any violation insurance and thus avert the perpetration of fraud.
INSURANCE Page 20

When a property owner obtains insurance policies - There mere act of pointing the gun to his temple - Insurance contracts are, as a rule, supposed to be
from two or more insurers in a total amount that showed that Felix willfully exposed himself to danger interpreted liberally in favor of the assured.
exceeds the property's value, the insured may have because a gun should always be handled with 2. NO, the claim for damages should not be granted
an inducement to destroy the property for the caution. for being unjust.
purpose of collecting the insurance. The public as Respondents’ Comments Ratio A person may be made liable to the payment
well as the insurer is interested in preventing a - Felix believed the gun to be safe because he had of moral damages if his act is wrongful. The adverse
situation in which a fire would be profitable to the removed the magazine. result of an action does not per se make the act
insured. - He repeatedly assured his secretary that the gun wrongful and subject the act or to the payment of
Disposition Petition granted. The decision of the was not loaded. moral damages.
Court of Appeals in CA-G.R. SP No. 31916 is SET Reasoning
ASIDE and the decision of the Insurance Commission ISSUES - Petitioner was acting in good faith when it resisted
in Case No. 3340 is REINSTATED. 1. WON Felix Lim’s death was an accident, thus the private respondent’s claim on the ground that
making his widow Nerissa liable to claim the accident the death of the insured was covered by the
SUN INSURANCE OFFICE, LTD. V CA (LIM) insurance exception.
211 SCRA 554 2. WON the award of damages to Nerissa Lim was - The issue was debatable and was clearly not raised
CRUZ; July 17, 1992 justified only for the purpose of evading a legitimate
obligation.
NATURE HELD
Petition for review from the decision of the Court of 1. YES, Felix Lim’s death was an accident. RIZAL SURETY & INSURANCE COMPANY V CA
Appeals Ratio There is no accident when a deliberate act is (TRANSWORLD KNITTING MILLS, INC.)
performed unless some additional, unexpected, 336 SCRA 12
FACTS independent and unforeseen happening occurs which PURISIMA; July 18, 2000
- Felix Lim was issued a Personal Accident Policy produces or brings bout their injury or death.
insurance with petitioner company with a face value Reasoning NATURE
of P200,000. His beneficiary was his wife Nerissa. - An accident has been defined to be that which Petition for Review on Certiorari under Rule 45 of the
- October 6, 1982 – Felix accidentally shot himself in happens by chance or fortuitously without intention Rules of Court
the head with his own gun.  or design and which is unexpected, unusual and
- He was playing with the handgun after he had unforeseen. It an event that takes pace without FACTS
removed the gun’s magazine (kasi naman…). one’s foresight or expectastion – an event that - Rizal Surety & Insurance Company (Rizal
- He pointed the gun at his secretary and only proceeds from an unknown cause or is an unusual Insurance) issued Fire Insurance Policy No. 45727 in
witness Pilar Nalagon as a joke and assured her effect of a known case and therefore not expected. favor of Transworld Knitting Mills, Inc. (Transworld).
that the gun was not loaded (are you sure…). It happens without any human agency, an event - Pertinent portions of subject policy on the buildings
- He then put the gun to his temple and fired it which, under the circumstances, is unusual to and insured, and location thereof, read:
(haaay, sabi ko na nga ba). not expected by the person to whom it happens. "‘On stocks of finished and/or unfinished
- Both parties are in agreement that there was no - The firing of the gun was deemed to be the products, raw materials and supplies of every
suicide. unexpected and independent and unforeseen kind and description, the properties of the
- Nerissa claimed as Felix’s beneficiary but Sun occurrence that led to the insured person’s death. Insureds and/or held by them in trust, on
Insurance would not grant her claim, saying that her - There was no willful exposure to needless peril for commission or on joint account with others
husband’s death was not an accident. the part of Felix. Suicide and exposure to needless and/or for which they (sic) responsible in case
- Nerissa sued Sun Insurance and won the case. peril are similar in the sense that both signify of loss whilst contained and/or stored during
Sun Insurance was ordered to pay her P200,000 disregard for one’s life. Suicide imparts a positive the currency of this Policy in the premises
representing the face value of the claim along with act of ending one’s life whereas the latter indicates occupied by them forming part of the
moral, exemplary and compensatory damages and recklessness that is almost suicidal in intent. buildings situate (sic) within own Compound
attorney’s fees. The decision was affirmed by the - Accident insurance policies were never meant to at MAGDALO STREET, BARRIO UGONG,
CA. reward the insured for his tendency to show off or PASIG, METRO MANILA, PHILIPPINES, BLOCK
Petitioners’ Claim for his miscalculations. They were intended to NO. 601.’
- Sun Insurance cites one of the four exceptions in provide for contingencies. x
the contract of insurance which includes bodily injury - Lim was unquestionably negligent but it should not xx............
consequent upon the insured person attempting to prevent his widow from recovering from the ...xxx.......
commit suicide or willfully exposing himself to insurance policy he obtained precisely against ........xxx
needless peril in an attempt to save a human life. accident. ‘Said building of four-span lofty one storey in
height with mezzanine portions is constructed
INSURANCE Page 21

of reinforced concrete and hollow blocks amusement machines and spare parts stored at - In the case under consideration, both the trial court
and/or concrete under galvanized iron roof the two-storey building because it (Transworld) and the Court of Appeals found that the so called
and occupied as hosiery mills, garment and had no insurable interest in said goods or items. "annex " was not an annex building but an integral
lingerie factory, transistor-stereo assembly - The Court denied the appeal with finality. and inseparable part of the four-span building
plant, offices, warehouse and caretaker's - Petitioner Rizal Insurance and private respondent described in the policy and consequently, the
quarters. Transworld, interposed a Motion for machines and spare parts stored therein were
'Bounds in front partly by one-storey concrete Reconsideration before the Court of Appeals, which covered by the fire insurance in dispute.
building under galvanized iron roof occupied reconsidered its decision of July 15, 1993, as - Verily, the two-storey building involved, a
as canteen and guardhouse, partly by building regards the imposition of interest. permanent structure which adjoins and
of two and partly one storey constructed of - Undaunted, petitioner Rizal Surety & Insurance intercommunicates with the "first right span of the
concrete below, timber above Company found its way to the Court. lofty storey building", formed part thereof, and
undergalvanized iron roof occupied as garage meets the requisites for compensability under the
and quarters and partly by open space and/or ISSUE fire insurance policy sued upon.
tracking/ packing, beyond which is the WON the fire insurance policy litigated upon - So also, considering that the two-storey building
aforementioned Magdalo Street; on its right protected only the contents of the main building aforementioned was already existing when subject
and left by driveway, thence open spaces, and (four-span), and did not include those stored in the fire insurance policy contract was entered into,
at the rear by open spaces.'" two-storey annex building petitioner should have specifically excluded the said
- The same pieces of property insured with the two-storey building from the coverage of the fire
petitioner were also insured with New India HELD insurance if minded to exclude the same but if did
Assurance Company, Ltd., (New India). NO not, and instead, went on to provide that such fire
- Fire broke out in the compound of Transworld, - Resolution of the issue posited hinges on the insurance policy covers the products, raw materials
razing the middle portion of its four-span building proper interpretation of the stipulation in subject fire and supplies stored within the premises of
and partly gutting the left and right sections insurance policy regarding its coverage, which reads: respondent Transworld which was an integral part of
thereof. A two-storey building (behind said four- "xxx contained and/or stored during the currency of the four-span building occupied by Transworld,
span building) where fun and amusement this Policy in the premises occupied by them forming knowing fully well the existence of such building
machines and spare parts were stored, was also part of the buildings situate (sic) within own adjoining and intercommunicating with the right
destroyed by the fire. Compound xxx" section of the four-span building.
- Transworld filed its insurance claims with Rizal - It can be gleaned unerringly that the fire insurance - Indeed, the stipulation as to the coverage of the
Surety & Insurance Company and New India policy in question did not limit its coverage to what fire insurance policy under controversy has created a
Assurance Company but to no avail. were stored in the four-span building. As opined by doubt regarding the portions of the building insured
- Private respondent brought against the said the trial court of origin, two requirements must thereby. Article 1377 of the New Civil Code provides:
insurance companies an action for collection of concur in order that the said fun and amusement "Art.1377. The interpretation of obscure words
sum of money and damages. machines and spare parts would be deemed or stipulations in a contract shall not favor the
- Petitioner Rizal Insurance countered that its fire protected by the fire insurance policy under scrutiny, party who caused the obscurity"
insurance policy sued upon covered only the to wit: - Conformably, it stands to reason that the doubt
contents of the four-span building, which was "First, said properties must be contained and/or should be resolved against the petitioner, Rizal
partly burned, and not the damage caused by the stored in the areas occupied by Transworld and Surety Insurance Company, whose lawyer or
fire on the two-storey annex building. second, said areas must form part of the building managers drafted the fire insurance policy contract
- The trial court dismissed the case as against The described in the policy xxx" under scrutiny. Citing the aforecited provision of law
New India Assurance Co., Ltd. but ordered - Said building of four-span lofty one storey in height in point, the Court in Landicho vs. Government
defendant Rizal Surety And Insurance Company to with mezzanine portions is constructed of reinforced Service Insurance System, ruled:
pay Transwrold (sic) Knitting Mills, Inc. concrete and hollow blocks and/or concrete under "This is particularly true as regards insurance
- Both the petitioner, Rizal Insurance Company, galvanized iron roof and occupied as hosiery mills, policies, in respect of which it is settled that the
and private respondent, Transworld Knitting Mills, garment and lingerie factory, transistor-stereo 'terms in an insurance policy, which are
Inc., went to the Court of Appeals, which required assembly plant, offices, ware house and caretaker's ambiguous, equivocal, or uncertain x x x are to
New India Assurance Company to pay plaintiff- quarter. be construed strictly and most strongly against
appellant the amount of P1,818,604.19 while the - The Court is mindful of the well-entrenched the insurer, and liberally in favor of the insured
Rizal Surety has to pay the plaintiff-appellant doctrine that factual findings by the Court of Appeals so as to effect the dominant purpose of
P470,328.67. are conclusive on the parties and not reviewable by indemnity or payment to the insured, especially
- New India appealed to the Court theorizing inter this Court, and the same carry even more weight where forfeiture is involved' and the reason for
alia that the private respondent could not be when the Court of Appeals has affirmed the findings this is that the 'insured usually has no voice in
compensated for the loss of the fun and of fact arrived at by the lower court. the selection or arrangement of the words
INSURANCE Page 22

employed and that the language of the contract Disposition Decision, and the Resolution of the CA from the wrongdoer to the extent that the insurer
is selected with great care and deliberation by WERE AFFIRMED in toto. No pronouncement as to has been obligated to pay.
experts and legal advisers employed by, and costs. - General Rule: Payment by the insurer to the
acting exclusively in the interest of, the assured operates as an equitable assignment to the
insurance company.' " PAN MALAYAN INSURANCE CORPORATION vs. former of all remedies which the latter may have
- Equally relevant is the following disquisition of the COURT OF APPEALS (ERLINDA FABIE & HER against the third party whose negligence or wrongful
Court in Fieldmen's Insurance Company, Inc. vs. UNKNOWN DRIVER) act caused the loss. The right of subrogation is not
Vda. De Songco, to wit: 184 SCRA 55; G.R. No. 81026 dependent upon, nor does it grow out of, any privity
"'This rigid application of the rule on ambiguities CORTES; April 3, 1990 of contract or upon written assignment of claim. It
has become necessary in view of current accrues simply upon payment of the insurance claim
business practices. The courts cannot ignore FACTS by the insurer.
that nowadays monopolies, cartels and - December 10, 1985: PANMALAY filed a complaint - Exceptions:
concentration of capital, endowed with for damages with the RTC of Makati against private a.) if the assured by his own act releases the
overwhelming economic power, manage to respondents Erlinda Fabie and her driver. PANMALAY wrongdoer or third party liable for the loss or
impose upon parties dealing with them averred the following: that it insured a Mitsubishi damage, from liability
cunningly prepared 'agreements' that the Colt Lancer car registered in the name of Canlubang b.) where the insurer pays the assured the value of
weaker party may not change one whit, his Automotive Resources Corporation [CANLUBANG]; the lost goods without notifying the carrier who has
participation in the 'agreement' being reduced to that on May 26, 1985, due to the "carelessness, in good faith settled the assured's claim for loss
the alternative to 'take it or leave it' labelled recklessness, and imprudence" of the unknown c.) where the insurer pays the assured for a loss
since Raymond Saleilles 'contracts by driver of a pick-up, the insured car was hit and which is not a risk covered by the policy, thereby
adherence' (contrats [sic] d'adhesion), in suffered damages in the amount of P42,052.00; that effecting "voluntary payment"
contrast to these entered into by parties PANMALAY defrayed the cost of repair of the insured - None of the exceptions are availing in the present
bargaining on an equal footing, such contracts car and, therefore, was subrogated to the rights of case.
(of which policies of insurance and international CANLUBANG against the driver of the pick-up and his - When PANMALAY utilized the phrase "own damage"
bills of lading are prime example) obviously call employer, Erlinda Fabie; and that, despite repeated — a phrase which, incidentally, is not found in the
for greater strictness and vigilance on the part demands, defendants, failed and refused to pay the insurance policy — to define the basis for its
of courts of justice with a view to protecting the claim of PANMALAY. PANMALAY clarified that the settlement of CANLUBANG's claim under the policy, it
weaker party from abuses and imposition, and damage caused to the insured car was settled under simply meant that it had assumed to reimburse the
prevent their becoming traps for the unwary.'" the "own damage", coverage of the insurance policy. costs for repairing the damage to the insured
- The issue of whether or not Transworld has an - Private respondents filed a Motion to Dismiss vehicle.
insurable interest in the fun and amusement alleging that PANMALAY had no cause of action - It is a basic rule in the interpretation of contracts
machines and spare parts, which entitles it to be against them. RTC dismissed PANMALAY's complaint that the terms of a contract are to be construed
indemnified for the loss thereof, had been settled in for no cause of action and denied PANMALAY's according to the sense and meaning of the terms
G.R. No. L-111118, entitled New India Assurance motion for reconsideration. CA affirmed. Hence, this which the parties thereto have used. In the case of
Company, Ltd., vs. Court of Appeals, where the petition for review. property insurance policies, the evident intention of
appeal of New India from the decision of the Court of the contracting parties, i.e., the insurer and the
Appeals under review, was denied with finality by ISSUE assured, determine the import of the various terms
this Court on February 2, 1994. WON the insurer PANMALAY may institute an action and provisions embodied in the policy. It is only
- The rule on conclusiveness of judgment, which to recover the amount it had paid its assured in when the terms of the policy are ambiguous,
obtains under the premises, precludes the settlement of an insurance claim against private equivocal or uncertain, such that the parties
relitigation of a particular fact or issue in another respondents as the parties allegedly responsible for themselves disagree about the meaning of particular
action between the same parties based on a different the damage caused to the insured vehicle provisions, that the courts will intervene. In such an
claim or cause of action. "xxx the judgment in the event, the policy will be construed by the courts
prior action operates as estoppel only as to those HELD liberally in favor of the assured and strictly against
matters in issue or points controverted, upon the YES the insurer.
determination of which the finding or judgment was - Article 2207 of the Civil Code is founded on the - Considering that the very parties to the policy were
rendered. In fine, the previous judgment is well-settled principle of subrogation. If the insured not shown to be in disagreement regarding the
conclusive in the second case, only as those matters property is destroyed or damaged through the fault meaning and coverage of Section III-1, specifically
actually and directly controverted and determined or negligence of a party other than the assured, then sub-paragraph (a) thereof, it was improper for the
and not as to matters merely involved therein." the insurer, upon payment to the assured, will be appellate court to indulge in contract construction, to
subrogated to the rights of the assured to recover apply the ejusdem generis rule, and to ascribe
INSURANCE Page 23

meaning contrary to the clear intention and receipts indicating payment for the full amount of the happened during the preparation of the written
understanding of these parties. premium were issued by the petitioner's agent. contract.
- Although the terms "accident" or "accidental" as - Policy description: - Respondent is not estopped from claiming that the
used in insurance contracts have not acquired a Front: by a driveway thence at 18 meters distance policy description is wrong. Evidence on record
technical meaning, the Court has on several by Bldg. No. 2. reveals that respondent’s operating manager, Mr.
occasions defined these terms to mean that which Right: by an open space thence by Bldg. No. 4. Edison Tantuco, notified the petitioner’s agent with
takes place "without one's foresight or expectation, Left: Adjoining thence an imperfect wall by Bldg. whom respondent negotiated for the contract about
an event that proceeds from an unknown cause, or is No. 4. the inaccurate description in the policy. However,
an unusual effect of a known cause and, therefore, Rear: by an open space thence at 8 meters Mr. Borja assured Mr. Tantuco that the use of the
not expected." The concept "accident" is not distance.” adjective new will distinguish the insured property.
necessarily synonymous with the concept of "no - A fire that broke out in the early morning of - Regarding policy requirements that fire
fault". It may be utilized simply to distinguish September 30,1991 gutted and consumed the new extinguishment appliances should be available and in
intentional or malicious acts from negligent or oil mill. Petitioner rejected respondent’s claim for good working condition, warranty did not require
careless acts of man. the insurance proceeds on the ground that no policy respondent to provide for all the fire extinguishing
- Obiter Dicta: Even if under the above was issued by it covering the burned oil mill. It appliances enumerated therein. Neither did it
circumstances PANMALAY could not be deemed stated that the description of the insured require that the appliances are restricted to those
subrogated to the rights of its assured under Article establishment referred to another building. mentioned in the warranty. (Within the vicinity of the
2207 of the Civil Code, PANMALAY would still have a Petitioner’s Claim new oil mill can be found the following devices:
cause of action against private respondents. The The policies referred to the old mill, as stated in the numerous portable fire extinguishers, two fire hoses,
insurer who may have no rights of subrogation due description contained in the policy. fire hydrant, and an emergency fire engine.)
to "voluntary" payment may nevertheless recover - The object of the court in construing a contract is
from the third party responsible for the damage to ISSUE to ascertain the intent of the parties to the contract
the insured property under Article 1236 of the Civil WON new oil mill is insured by fire insurance policy and to enforce the agreement which the parties have
Code. entered into. In determining what the parties
Disposition Petition is GRANTED. Petitioner's HELD intended, the courts will read and construe the policy
complaint for damages against private respondents YES, new oil mill is insured. as a whole and if possible, give effect to all the parts
is REINSTATED. Case remanded to the lower court Ratio In construing the words used descriptive of a of the contract.
for trial on the merits. building insured, the greatest liberality is shown by Disposition Petition is dismissed.
the courts in giving effect to the insurance. In view
AMERICAN HOME ASSURANCE COMPANY V of the custom of insurance agents to examine PERLA COMPANIA DE SEGUROS, INC. v, CA
TANTUCO ENTERPRISES buildings before writing policies upon them, and (MILAGROS CAYAS)
366 SCRA 740 since a mistake as to the identity and character of 185 SCRA 741
PUNO; October 8, 2001 the building is extremely unlikely, the courts are FERNAN; May 28, 1990
inclined to consider that the policy of insurance
NATURE covers any building which the parties manifestly NATURE
Petition for Review on Certiorari assailing the intended to insure, however inaccurate the Petition for review on certiorari of a decision of the
Decision of the Court of Appeals. description may be. Court of Appeals
Reasoning
FACTS - The parties manifestly intended to insure the new FACTS
- Respondent Tantuco Enterprises, Inc. is engaged in oil mill. “On machineries and equipment with - Private respondent Milagros Cayas was the
the coconut oil milling and refining industry. It owns complete accessories usual to a coconut oil mill registered owner of a Mazda bus, insured with Perla
two oil mills. Both are located at its factory including stocks of copra, copra cake and copra mills Compania de Seguros, Inc. (PCSI) under a policy
compound at Iyam, Lucena City. Respondent whilst contained in the new oil mill building, situate issued on February 3, 1978.
commenced its business operations with only one oil (sic) at UNNO. ALONG NATIONAL HIGH WAY, BO. - On December 17, 1978, the bus figured in an
mill. In 1988, it started operating its second oil mill IYAM, LUCENA CITY UNBLOCKED.” accident in Naic, Cavite injuring several of its
( the new oil mill). - If the parties really intended to protect the first oil passengers.
- The two oil mills were separately covered by fire mill, then there is no need to specify it as new. It - One of them, 19-year old Edgardo Perea, sued
insurance policies issued by petitioner American would be absurd to assume that respondent would Milagros Cayas for damages in the CFI of Cavite,
Home Assurance Co. The first oil mill was insured protect its first oil mill for different amounts and while three others agreed to a settlement of
Policy No. 306-7432324-3 for the period March 1, leave uncovered its second one. P4,000.00 each.
1991 to 1992.The new oil mill was insured under - As may be gleaned from the testimony of the - After trial, the court rendered a decision in favor of
Policy No. 306-7432321-9 for the same term. Official petitioner’s employee, the source of the discrepancy Perea, ordering Cayas to compensate him, with an
INSURANCE Page 24

award of exemplary and moral damages, as well as therewith is a condition precedent to the insured's may be, whenever the essential requisites for their
attorney’s fees. ( P32,000 total) right of recovery from the insurer. validity are present.
- On November 11, 1981, Milagros Cayas filed a - In the case at bar, the insurance policy clearly and - In Pacific Oxygen & Acetylene Co. vs. Central
complaint for a sum of money and damages against categorically placed petitioner's liability for all Bank," it was stated that the first and fundamental
PCSI in the Court of First Instance of Cavite. damages arising out of death or bodily injury duty of the courts is the application of the law
- In view of Milagros Cayas' failure to prosecute the sustained by one person as a result of any one according to its express terms, interpretation being
case, the court motu propio ordered its dismissal accident at P12,000.00. called for only when such literal application is
without prejudice. - Said amount complied with the minimum fixed by impossible.
- Alleging that she had not received a copy of the the law then prevailing, Section 377 of Presidential - We observe that although Milagros Cayas was able
answer to the complaint, and that "out of Decree No. 612, which provided that the liability of to prove a total loss of only P44,000.00, petitioner
sportsmanship", she did not file a motion to hold land transportation vehicle operators for bodily was made liable for the amount of P50,000.00, the
PCSI in default, Milagros Cayas moved for the injuries sustained by a passenger arising out of the maximum liability per accident stipulated in the
reconsideration of the dismissal order. Said motion use of their vehicles shall not be less than P12,000. policy. This is patent error. An insurance indemnity,
for reconsideration was acted upon favorably by the - In other words, under the law, the minimum being merely an assistance or restitution insofar as
court. liability is P12,000 per passenger. Petitioner's liability can be fairly ascertained, cannot be availed of by any
- About two months later, Milagros Cayas filed a under the insurance contract not being less than accident victim or claimant as an instrument of
motion to declare PCSI in default for its failure to file P12,000.00, and therefore not contrary to law, enrichment by reason of an accident.
an answer. morals, good customs, public order or public policy, Disposition Petition granted. The decision of the
- The motion was granted and plaintiff was allowed said stipulation must be upheld as effective, valid Court of Appeals is modified in that petitioner shall
to adduce evidence ex-parte. and binding as between the parties. pay Milagros Cayas the amount of Twelve Thousand
- On July 13, 1982, the court rendered judgment by - In like manner, we rule as valid and binding upon Pesos (P12,000. 00) plus legal interest from the
default ordering PCSI to pay Milagros Cayas P50,000 private respondent the condition requiring her to promulgation of the decision of the lower court until
as compensation for the injured passengers, P5,000 secure the written permission of petitioner before it is fully paid and attorney's fees in the amount of
as moral damages and P5,000 as attorney's fees. effecting any payment in settlement of any claim P5,000.00.
- Said decision was set aside after the PCSI filed a against her.
motion therefor. Trial of the case ensued. - There is nothing unreasonable, arbitrary or POLTAN v. BPI & JOHN DOE
- In due course, the court promulgated a decision objectionable in this stipulation as would warrant its G.R. No. 164307
ordering defendant Perla Compania de Seguros, Inc. nullification. The same was obviously designed to CHICO-NAZARIO; March 5, 2007
to pay plaintiff Milagros Cayas the sum of safeguard the insurer's interest against collusion
P50,000.00 under its maximum liability as provided between the insured and the claimants. NATURE
for in the insurance policy; and the sum of P5,000.00 - It being specifically required that petitioner's Petition for review of CA decision
as reasonable attorney's fee written consent be first secured before any payment
- PCSI appealed to the Court of Appeals, which in settlement of any claim could be made, private FACTS
affirmed in toto the lower court's decision. respondent is precluded from seeking reimbursement - Petitioners POLTAN obtained a loan evidenced by a
- Its motion for reconsideration having been denied of the payments made to the three other passangers promissory note from the MANTRADE Dev’t Corp.
by said appellate court, PCSI filed this petition in view of her failure to comply with the condition This was secured by a chattel mortgage over a 1-unit
contained in the insurance policy. Nissan Sentra vehicle.
ISSUE - Clearly, the fundamental principle that contracts - With notice to petitioners, MANTRADE assigned to
WON, as maintained by petitioner, its liability is are respected as the law between the contracting BPI, by way of a Deed of Assignment, all its rights,
limited only to the payment made by private parties finds application in the present case. title and interest to the promissory note and chattel
respondent to Perea and only up to the amount of - It was error on the part of the trial and appellate mortgage.
P12,000.00 courts to have disregarded the stipulations of the - Petitioners defaulted and so BPI demanded the
parties and to have substituted their own whole balance of P286.5T including accrued interest,
HELD interpretation of the insurance policy. or to return to BPI the possession of the motor
YES - In Phil. American General Insurance Co., Inc vs. vehicle for foreclosure. It is specifically provided in
- The insurance policy involved explicitly limits Mutuc, we ruled that contracts which are the private the promissory note and chattel mortgage that
petitioner's liability to P12,000.00 per person and to laws of the contracting parties should be fulfilled failure to pay any installment when due shall make
P50,000.00 per accident. according to the literal sense of their stipulations, if subsequent installments and the entire balance of
- In Stokes vs. Malayan Insurance Co., Inc., the their terms are clear and leave no room for doubt as the obligation due and payable.
Court held that the terms of the contract constitute to the intention of the contracting parties, for - After they refused to do so, BPI then filed
the measure of the insurer's liability and compliance contracts are obligatory, no matter what form they complaint.
INSURANCE Page 25

- Petitioners claimed that BPI required them to the benefit of BPI, this will result in partial or full - The very nature of the term "all risks" must be
obtain a motor vehicle insurance policy from FGU satisfaction of the obligation only if the insurer pays given a broad and comprehensive meaning as
Insurance Corporation (FGU Insurance). This is a the mortgagee, BPI, or if the insurance proceeds covering any loss other than a willful and fraudulent
sister company of BPI. They had been paying the were paid to BPI. In this case, upon the loss of the act of the insured. 7 This is pursuant to the very
monthly installments on the vehicle until it figured in vehicle due to total wreck, the petitioners filed a purpose of an "all risks" insurance to give protection
an accident where it became a total wreck. Under claim under the insurance policy, collected and to the insured in those cases where difficulties of
the terms of the insurance policy from FGU received the proceeds thereof, but did not settle logical explanation or some mystery surround the
Insurance, the vehicle had to be replaced or its value their obligation with BPI which remained outstanding loss or damage to property.
paid to them. Due to the failure and refusal of FGU despite the loss of the vehicle. - Generally, the burden of proof is upon the insured
Insurance to replace the vehicle or pay its value, Disposition CA decision AFFIRMED with the to show that a loss arose from a covered peril, but
they stopped payment of the monthly installments. modification that the interest rate be reduced to under an "all risks" policy the burden is not on the
- RTC ordered POLTANS to pay BPI the said amount. 12% per annum from 24 May 1994 until fully paid, insured to prove the precise cause of loss or damage
- CA reversed and remanded case to RTC for trial on and the award of attorney’s fees be reduced to P50T. for which it seeks compensation. The insured under
the merits. an "all risks insurance policy" has the initial burden
- RTC again ruled in favor of BPI. CA affirmed. FILIPINO MERCHANTS INS. v. CA (CHOA TIEK of proving that the cargo was in good condition when
SENG) the policy attached and that the cargo was damaged
ISSUES 179 SCRA 638 when unloaded from the vessel; thereafter, the
1. WON contracts presented in evidence by BPI were REGALADO; November 28, 1989 burden then shifts to the insurer to show the
unjust and unacceptable contracts of adhesion exception to the coverage. As we held in Paris-
2. WON the terms and conditions of the NATURE Manila Perfumery Co. vs. Phoenix Assurance Co.,
comprehensive car insurance policy issued by FGU Review of the decision of the CA Ltd. the basic rule is that the insurance company has
should be deemed as having automatically operated the burden of proving that the loss is caused by the
in favor of BPI as the assured mortgagee, and if so, FACTS risk excepted and for want of such proof, the
it should be deemed as resulting in the - Plaintiff insured said shipment with defendant company is liable. In the present case, there being
extinguishment of petitioner’s obligation insurance company under said cargo for the goods no showing that the loss was caused by any of the
described as 600 metric tons of fishmeal in new excepted perils, the insurer is liable under the policy.
HELD gunny bags of 90 kilos each from Bangkok, Thailand 2. YES
1. NO to Manila against all risks under warehouse to - Section 13 of the Insurance Code defines insurable
Ratio A contract of adhesion is one in which one of warehouse terms. interest in property as every interest in property,
the parties imposes a ready-made form of contract, - Some of the goods arrived in bad condition. whether real or personal, or any relation thereto, or
which the other party may accept or reject, but Plaintiff made a claim against Filipino Merchants liability in respect thereof, of such nature that a
which the latter cannot modify. It is just as binding Insurance Company. The latter refused to pay. contemplated peril might directly damnify the
as ordinary contracts. Plaintiff brought an action against them. The insured. In principle, anyone has an insurable
Reasoning defendant insurance company presented a third interest in property who derives a benefit from its
- Petitioners failed to show that they were under party complaint against the vessel and the arrastre existence or would suffer loss from its destruction
duress or forced to sign the loan documents. The contractor. whether he has or has not any title in, or lien upon
natural presumption is that one does not sign a - Judgment was rendered against the insurance or possession of the property y. 16 Insurable interest
document without first informing himself of its company. On the third party complaint, the third in property may consist in (a) an existing interest;
contents and consequences. party defendants were ordered to pay the third party (b) an inchoate interest founded on an existing
- Contracts of adhesion are not entirely prohibited plaintiffs. The CA affirmed, but modified the same interest; or (c) an expectancy, coupled with an
even as the courts remain careful in scrutinizing the with regard to the adjudication of the third-party existing interest in that out of which the expectancy
factual circumstances underlying each case to complaint arises.
determine the respective claims of contending - Respondent’s interest over the goods is based on
parties on their efficacy. ISSUES the perfected contract of sale. The perfected contract
2. NO 1. WON some fortuity, casualty or accidental cause is of sale between him and the shipper of the goods
Reasoning needed to be proved despite the “all risks” policy (as operates to vest in him an equitable title even before
- Petitioners failed to show any provision in the asserted by the insurance company) delivery or before be performed the conditions of the
insurance policy or mortgage contract providing that 2. WON the respondent has an insurable interest sale.
the loss of the mortgaged vehicle extinguishes their - Further, Article 1523 of the Civil Code provides that
principal obligation to BPI. HELD where, in pursuance of a contract of sale, the seller
- While it is true that the proceeds from the 1. NO is authorized or required to send the goods to the
insurance policy over the mortgaged chattel is for buyer, delivery of the goods to a carrier, whether
INSURANCE Page 26

named by the buyer or not, for, the purpose of receivable item from their customers and dealers. x the fire must be borne by petitioner since the
transmission to the buyer is deemed to be a delivery xx proviso contained in the sales invoices is an
of the goods to the buyer, the exceptions to said rule - Petitioner is a customer and dealer of the products exception under Article 1504 (1) of the Civil Code, to
not obtaining in the present case. The Court has of IMC and LSPI. On February 25, 1991, the Gaisano the general rule that if the thing is lost by a
heretofore ruled that the delivery of the goods on Superstore Complex in Cagayan de Oro City, owned fortuitous event, the risk is borne by the owner of
board the carrying vessels partake of the nature of by petitioner, was consumed by fire. Included in the the thing at the time the loss under the principle of
actual delivery since, from that time, the foreign items lost or destroyed in the fire were stocks of res perit domino; that petitioner’s obligation to IMC
buyers assumed the risks of loss of the goods and ready-made clothing materials sold and delivered by and LSPI is not the delivery of the lost goods but the
paid the insurance premium covering them IMC and LSPI. On February 4, 1992, respondent payment of its unpaid account and as such the
- Moreover, the issue of lack of insurable interest filed a complaint for damages against petitioner. It obligation to pay is not extinguished, even if the fire
was not raised in petitioner’s answer. alleges that IMC and LSPI filed with respondent their is considered a fortuitous event; that by subrogation,
Disposition Petition denied claims under their respective fire insurance policies the insurer has the right to go against petitioner;
with book debt endorsements; that as of February that, being a fire insurance with book debt
GAISANO CAGAYAN v. INSURANCE Co. OF 25, 1991, the unpaid accounts of petitioner on the endorsements, what was insured was the vendor’s
NORTH AMERICA sale and delivery of ready-made clothing materials interest as a creditor. Petitioner filed a motion for
490 SCRA 296 with IMC was P2,119,205.00 while with LSPI it was reconsideration but it was denied by the CA in its
Austria-Martinez; June 8, 2006 P535,613.00; that respondent paid the claims of IMC Resolution dated April 11, 2001.
and LSPI and, by virtue thereof, respondent was
NATURE subrogated to their rights against petitioner; that ISSUES
Petition for review on certiorari of the Decision of the respondent made several demands for payment 1. WON the CA erred in construing a fire insurance
Court of Appeals upon petitioner but these went unheeded. In its policy on book debts as one covering the unpaid
Answer with Counter Claim dated July 4, 1995, accounts of IMC and LSPI since such insurance
FACTS petitioner contends that it could not be held liable applies to loss of the ready-made clothing materials
- Intercapitol Marketing Corporation (IMC) is the because the property covered by the insurance sold and delivered to petitioner.
maker of Wrangler Blue Jeans. Levi Strauss (Phils.) policies were destroyed due to fortuities event or 2. WON IMC bears the risk of loss because it
Inc. (LSPI) is the local distributor of products bearing force majeure; that respondent’s right of subrogation expressly reserved ownership of the goods by
trademarks owned by Levi Strauss & Co.. IMC and has no basis inasmuch as there was no breach of stipulating in the sales invoices that “[i]t is further
LSPI separately obtained from respondent fire contract committed by it since the loss was due to agreed that merely for purpose of securing the
insurance policies with book debt endorsements. fire which it could not prevent or foresee; that IMC payment of the purchase price the above described
The insurance policies provide for coverage on “book and LSPI never communicated to it that they insured merchandise remains the property of the vendor
debts in connection with ready-made clothing their properties; that it never consented to paying until the purchase price thereof is fully paid.”
materials which have been sold or delivered to the claim of the insured. 3. WON the petitioner liable for the unpaid accounts
various customers and dealers of the Insured - At the pre-trial conference the parties failed to
anywhere in the Philippines.” The policies defined arrive at an amicable settlement. Thus, trial on the HELD
book debts as the “unpaid account still appearing in merits ensued. On August 31, 1998, the RTC 1. NO
the Book of Account of the Insured 45 days after the rendered its decision dismissing respondent’s - It is well-settled that when the words of a contract
time of the loss covered under this Policy.” The complaint. It held that the fire was purely accidental; are plain and readily understood, there is no room
policies also provide for the following conditions: that the cause of the fire was not attributable to the for construction. In this case, the questioned
1. Warranted that the Company shall not negligence of the petitioner; that it has not been insurance policies provide coverage for “book debts
be liable for any unpaid account in established that petitioner is the debtor of IMC and in connection with ready-made clothing materials
respect of the merchandise sold and LSPI; that since the sales invoices state that “it is which have been sold or delivered to various
delivered by the Insured which are further agreed that merely for purpose of securing customers and dealers of the Insured anywhere in
outstanding at the date of loss for a the payment of purchase price, the above-described the Philippines.”; and defined book debts as the
period in excess of six (6) months from merchandise remains the property of the vendor “unpaid account still appearing in the Book of
the date of the covering invoice or until the purchase price is fully paid”, IMC and LSPI Account of the Insured 45 days after the time of the
actual delivery of the merchandise retained ownership of the delivered goods and must loss covered under this Policy.” Nowhere is it
whichever shall first occur. bear the loss. Dissatisfied, petitioner appealed to the provided in the questioned insurance policies that
2. Warranted that the Insured shall submit to the CA. On October 11, 2000, the CA rendered its the subject of the insurance is the goods sold and
Company within twelve (12) days after the close of decision setting aside the decision of the RTC. The delivered to the customers and dealers of the
every calendar month all amount shown in their CA held that the sales invoices are proofs of sale, insured.
books of accounts as unpaid and thus become being detailed statements of the nature, quantity - Indeed, when the terms of the agreement are clear
and cost of the thing sold; that loss of the goods in and explicit that they do not justify an attempt to
INSURANCE Page 27

read into it any alleged intention of the parties, the interest is requisite to the existence of such an excused by fortuitous loss of any specific
terms are to be understood literally just as they interest, it is sufficient that the insured is so situated property of the debtor.
appear on the face of the contract. Thus, what were with reference to the property that he would be - Thus, whether fire is a fortuitous event or
insured against were the accounts of IMC and LSPI liable to loss should it be injured or destroyed by the petitioner was negligent are matters immaterial to
with petitioner which remained unpaid 45 days after peril against which it is insured. Anyone has an this case. What is relevant here is whether it has
the loss through fire, and not the loss or destruction insurable interest in property who derives a benefit been established that petitioner has outstanding
of the goods delivered. from its existence or would suffer loss from its accounts with IMC and LSPI.
2. NO destruction. Indeed, a vendor or seller retains an - With respect to IMC, the respondent has
- The present case clearly falls under paragraph (1), insurable interest in the property sold so long as he adequately established its claim. Petitioner has an
Article 1504 of the Civil Code: has any interest therein, in other words, so long as outstanding account with IMC in the amount of
ART. 1504. Unless otherwise agreed, the goods he would suffer by its destruction, as where he has a P2,119,205.00, check voucher evidencing payment
remain at the seller’s risk until the ownership vendor’s lien. In this case, the insurable interest of to IMC, subrogation receipt executed by IMC in favor
therein is transferred to the buyer, but when the IMC and LSPI pertain to the unpaid accounts of respondent upon receipt of the insurance
ownership therein is transferred to the buyer the appearing in their Books of Account 45 days after the proceeds. All these documents have been properly
goods are at the buyer’s risk whether actual time of the loss covered by the policies. identified, presented and marked as exhibits in
delivery has been made or not, except that: 3. YES court. The subrogation receipt, by itself, is sufficient
(1) Where delivery of the goods has been made to - Petitioner’s argument that it is not liable because to establish not only the relationship of respondent
the buyer or to a bailee for the buyer, in the fire is a fortuitous event under Article 1174 of as insurer and IMC as the insured, but also the
pursuance of the contract and the ownership in the Civil Code is misplaced. As held earlier, amount paid to settle the insurance claim. The right
the goods has been retained by the seller petitioner bears the loss under Article 1504 (1) of of subrogation accrues simply upon payment by the
merely to secure performance by the buyer of the Civil Code. insurance company of the insurance claim.
his obligations under the contract, the goods - Moreover, it must be stressed that the insurance in Respondent’s action against petitioner is squarely
are at the buyer’s risk from the time of such this case is not for loss of goods by fire but for sanctioned by Article 2207 of the Civil Code which
delivery; (Emphasis supplied) petitioner’s accounts with IMC and LSPI that provides:
- Thus, when the seller retains ownership only to remained unpaid 45 days after the fire. Accordingly, Art. 2207. If the plaintiff’s property has been
insure that the buyer will pay its debt, the risk of petitioner’s obligation is for the payment of money. insured, and he has received indemnity from the
loss is borne by the buyer. Accordingly, petitioner Where the obligation consists in the payment of insurance company for the injury or loss arising
bears the risk of loss of the goods delivered. money, the failure of the debtor to make the out of the wrong or breach of contract complained
- IMC and LSPI did not lose complete interest over payment even by reason of a fortuitous event shall of, the insurance company shall be subrogated to
the goods. They have an insurable interest until full not relieve him of his liability. The rationale for this is the rights of the insured against the wrongdoer or
payment of the value of the delivered goods. Unlike that the rule that an obligor should be held exempt the person who has violated the contract. x x x
the civil law concept of res perit domino, where from liability when the loss occurs thru a fortuitous - Petitioner failed to refute respondent’s evidence.
ownership is the basis for consideration of who bears event only holds true when the obligation consists in - As to LSPI, respondent failed to present sufficient
the risk of loss, in property insurance, one’s the delivery of a determinate thing and there is no evidence to prove its cause of action. No evidentiary
interest is not determined by concept of title, stipulation holding him liable even in case of weight can be given to Exhibit “F Levi Strauss”, a
but whether insured has substantial economic fortuitous event. It does not apply when the letter dated April 23, 1991 from petitioner’s General
interest in the property. obligation is pecuniary in nature. Manager, Stephen S. Gaisano, Jr., since it is not an
- Section 13 of our Insurance Code defines insurable - Under Article 1263 of the Civil Code, “[i]n an admission of petitioner’s unpaid account with LSPI.
interest as “every interest in property, whether real obligation to deliver a generic thing, the loss or It only confirms the loss of Levi’s products in the
or personal, or any relation thereto, or liability in destruction of anything of the same kind does not amount of P535,613.00 in the fire that razed
respect thereof, of such nature that a contemplated extinguish the obligation.” If the obligation is generic petitioner’s building on February 25, 1991.
peril might directly damnify the insured.” in the sense that the object thereof is designated - Moreover, there is no proof of full settlement of the
Parenthetically, under Section 14 of the same Code, merely by its class or genus without any particular insurance claim of LSPI; no subrogation receipt was
an insurable interest in property may consist in: (a) designation or physical segregation from all others of offered in evidence. Thus, there is no evidence that
an existing interest; (b) an inchoate interest founded the same class, the loss or destruction of anything of respondent has been subrogated to any right which
on existing interest; or (c) an expectancy, coupled the same kind even without the debtor’s fault and LSPI may have against petitioner. Failure to
with an existing interest in that out of which the before he has incurred in delay will not have the substantiate the claim of subrogation is fatal to
expectancy arises. effect of extinguishing the obligation. This rule is petitioner’s case for recovery of the amount of
- Therefore, an insurable interest in property does based on the principle that the genus of a thing can P535,613.00.
not necessarily imply a property interest in, or a lien never perish. Genus nunquan perit. An obligation Disposition Petition is partly GRANTED. The
upon, or possession of, the subject matter of the to pay money is generic; therefore, it is not assailed Decision dated October 11, 2000 and
insurance, and neither the title nor a beneficial Resolution dated April 11, 2001 of the Court of
INSURANCE Page 28

Appeals in CA-G.R. CV No. 61848 are AFFIRMED


with the MODIFICATION that the order to pay the ISSUE NATURE
amount of P535,613.00 to respondent is DELETED WON petitioner Tai Tong has insurable interest in the Petition for review on certiorari
for lack of factual basis. said policy
FACTS
TAI TONG CHUACHE & CO v. INSURANCE HELD - Primitivo Perez has been insured with the BF
COMMISSION and TRAVELLERS MULTI- YES Lifeman Insurance Corporation (BF hereafter) since
INDEMNITY CORPORATION - First, respondent insurance commission based its 1980 for Php20,000.
158 SCRA 366 findings on mere inference. Respondent Insurance Sometime in 1987, Rodolfo Lalog (agent of BF)
GANCAYCO; February 29, 1988 Commission absolved respondent insurance company convinced him to apply for additional insurance
from liability on the basis of the certification issued coverage of Php50,000.
NATURE by the then CFI, that in a certain civil action against Perez accomplished the application form and passed
Petition for review on certiorari of the decision of the the Palomos, Arsenio Lopez Chua stands as the the required medical examination. He also paid
Insurance Commission complainant and not Tai Tong Chuache. From said Php2,075 premium) to Lalog.
evidence respondent commission inferred that the - On November 25, 1987, Perez died while riding a
FACTS credit extended by herein petitioner to the Palomos banca which capsized during a storm. During this
- Complainants Palomo acquired a parcel of land and secured by the insured property must have been time his application papers for the additional
a building located in Davao City. They assumed the paid. Such is a glaring error which this Court cannot insurance coverage was still with the Gumaca,
mortgage of the building in favor of SSS, which sanction. Quezon
building was insured with respondent SSS Accredited - Second, it has been held in a long line of cases that office of BF.
Group of Insurers for P25K. when the creditor is in possession of the document of - Without knowing that Perez died on November 25,
- On April 19, 1975, Azucena Palomo obtained a credit, he need not prove non-payment for it is 1987, BF approved Perez's application and issued the
P100K loan from Tai Tong Chuache Inc. (TTCC) and presumed. The validity of the insurance policy taken corresponding policy for the Php50,000 on December
executed a mortgage over the land and the building b petitioner was not assailed by private respondent. 2, 1987.
in favor of Tai Tong Chuache & Co. as security of Moreover, petitioner's claim that the loan extended - Virginia Perez (wife of the deceased) claimed the
payment .On April 25, 1975, Arsenio Chua, to the Palomos has not yet been paid was benefits under the insurance policies of the
representative of TTCC insured the latter's interest corroborated by Azucena Palomo who testified that deceased, but she was only able to receive
with Travellers Multi-Indemnity Corporation they are still indebted to herein petitioner. So at the Php40,000 under the first insurance policy.
(Travellers) for P100K (P70K for bldg and P30K for time of the fire, petitioner as mortgagee still had BF refused to pay the proceeds amounting to
the contents thereof) insurable interest therein. Php150,000 under the additional policy coverage of
- On June 11, 1975, Pedro Palomo secured a Fire - And third, petitioner's declaration that Arsenio Php50,000 because they maintain that such policy
Insurance Policy, covering the building for P50K with Lopez Chua acts as the managing partner of the had not been perfected.
respondent Zenith Insurance Corporation (ZIC). partnership was corroborated by respondent - On September 21, 1990, BF filed a complaint
Another Fire Insurance Policy was later procured insurance company. Thus Chua as the managing against Mrs. Perez seeking recission and declaration
from respondent Philippine British Assurance partner of the partnership may execute all acts of of nullity of the insurance contract in question. Mrs.
Company (PBAC), covering the same building for administration including the right to sue debtors of Perez filed a conterclaim
P50K and contents thereof for P70K. On July 31, the partnership in case of their failure to pay their for the collection of Php150,000 plus damages.
1975, the building and the contents were totally obligations when it became due and demandable. Or
razed by fire. at the least, Chua being a partner of petitioner Tai ISSUE
- Based on the computation of the loss, including the Tong Chuache & Company is an agent of the WON there was a consummated contract of
Travellers, respondents, ZIC, PBAC, and SSS paid partnership. Being an agent, it is understood that he insurance between Perez and BF
their corresponding shares of the loss. Complainants acted for and in behalf of the firm.
were paid the following: P41,546.79 by PBAC, Disposition Appealed decision SET ASIDE and HELD
P11,877.14 by ZIC, and P5,936.57 by SSS. Demand ANOTHER judgment is rendered order private NO
was made from respondent Travellers for its share in respondent Travellers to pay petitioner the face - An essential requisite of a valid contract is consent.
the loss but was refused. Hence, complainants value of Fire Insurance Policy in the amount of Consent must be manifested by the meeting of the
demanded from the other 3 respondents the balance P100K. Costs against said private respondent. offer and the acceptance upon the thing and the
of each share in the loss based on the computation cause which are to constitute the contract.
excluding Travellers Multi-Indemnity in the amount - The offer must be certain and the acceptance
of P30,894.31 (P5,732.79-ZIC: P22,294.62, PBAC: PEREZ v. CA (BF LIFEMAN INSURANCE CORP.) absolute. When Perez filed the application, it was
and P2,866.90, SSS) but was refused, hence, this 323 SCRA 613 subject to the acceptance of BF. The perfection was
action. YNARES-SANTIAGO; January 28, 2000
INSURANCE Page 29

also further conditioned upon (1) the issuance of the Sindayen was at work in the Bureau of Printing. On ISSUE
policy, January 12, he complained of a severe headache and WON the insurance policy is valid
(2) the payment of the premium, and (3) the remained at home. On January 15, he called a
delivery to and acceptance by the applicant in good physician who found that he was suffering from HELD
health. acute nephritis and uremia and on January 19, 1933, YES
- The delivery and acceptance by the applicant was a he died. - There is one line of cases which holds that the
suspensive condition which was not fulfilled - On January 18, 1933, the agent, in accordance stipulation contained in paragraph 3 is in the nature
inasmuch as the applicant was already dead at the with his agreement with the insured, delivered the of a condition precedent, that is to say, that there
time the policy was issued. The non-fulfillment of the policy to Felicidad Estrada upon her payment of the can be no valid delivery to the insured unless he is in
condition resulted balance of the first year's annual premium. The good health at the time; that this condition
in the non-perfection of the contract. agent asked Felicidad Estrada if her nephew was in precedent goes to the very essence of the contract
- An application for insurance is merely an offer good health and she replied that she believed so and cannot be waived by the agent making delivery
which requires the overt act of the insurer for it to because she had no information that he was sick and of the policy
ripen to a contract. Delay in acting on the application he thereupon delivered to her the policy. - On the other hand, a number of American decisions
does not constitute acceptance even though the - On January 20, 1933, the agent learned of the hold that an agent to whom a life insurance policy
insured has forwarded death of Arturo Sindayen and called on Felicidad similar to the one here involved was sent with
his first premium with his application. Delay, in this Estrada and asked her to return the policy. But he instructions to deliver it to the insured has authority
case, does not constitute gross negligence because did not return or offer to return the premium paid. to bind the company by making such delivery,
the application was granted within the normal Felicidad Estrada on his aforesaid statement gave although the insured was not in good health at the
processing time. him the policy. time of delivery, on the theory that the delivery of
Disposition Decision of CA affirmed in so far as it - On February 4, 1933 Insular Life obtained from the the policy being the final act to the consummation of
declared the insurance policy for Php50,000 issued beneficiary, Sindayen’s wife, her signature to a legal the contract, the condition as to the insurer's good
by BF null and void (no recission because it document entitled "ACCORD, SATISFACTION AND health was waived by the company.
presupposes a valid contract) RELEASE" whereby in consideration of the sum of - we are inclined to the view that it is more
P40.06 paid to her by a check of the company, she consonant with the well known practice of life
VDA. DE SINDAYEN v. INSULAR "assigns, releases and forever discharges said Isular insurance companies and the evidence in the present
62 Phil 51 Life Assurance Co., Ltd., its successors and assigns, case to rest our decision on the proposition that
BUTTE; September 4, 1935 of all claims, obligation in or indebtedness. The said Mendoza was authorized by the company to make
check for P40.06 was never cashed but returned to the delivery of the policy when he received the
FACTS the company and appears in the record of this case payment of the first premium and he was satisfied
- Arturo Sindayen, up to the time of his death on as Exhibit D. Thereupon this action was brought to that the insured was in good health. As was well said
January 19, 1933, was employed as a linotype enforce payment of the policy. in the case of MeLaurin vs. Mutual Life Insurance Co.
operator in the Bureau of Printing at Manila and had By the terms of the policy, an annual premium of “It is plain, therefore, that upon the facts it is not
been such for eleven years prior thereto. While there P40.06 is due on the first day of December of each necessarily a case of waiver or of estoppel, but a
he made a written application on December 26, year, the first premium already paid by the insured case where the local agents, in the exercise of the
1932, to the defendant Insular Life Assurance Co., covering the period from December 1, 1932. It is to powers lodged in them, accepted the premium and
Ltd., through its agent, Cristobal Mendoza, for a December 1, 1933. It is to be noted that the policy delivered the policy. That act binds their principal,
policy of insurance on his life in the sum of P1,000 was not issued and the company assumed no actual the defendant.”
and he paid to the agent P15 cash as part of the first risk prior to January 11, 1933.The application which - Mendoza was duly licensed by the Insurance
premium. It was agreed with the agent that the the insured signed in Camiling, Tarlac, on December Commissioner to act as the agent of the defendant
policy, when and if issued, should be delivered to his 26, 1932, contained among others the following insurance company. The well known custom of the
aunt. Felicidad Estrada, with whom Sindayen left the provisions: insurance business and the evidence in this case
sum of P26.06 to complete the payment of the first “3 That the said policy shall not take effect until the prove that Mendoza was not regarded by the
annual premium of P40.06. first premium has been paid and the policy has been company as a mere conduit or automaton for the
- On January 1, 1933, Sindayen, who was then delivered to and accepted by me, while I am in good performance of the physical act of placing the policy
twenty-nine years of age, was examined by the health.” in the hands of the insured
company's doctor who made a favorable report, to -Main defense of the company in this case, namely, - Granted that Mendoza's decision that the condition
the company. On January 11, 1933, The company that the said policy never took effect because of had been met by the insured and that it was proper
accepted the risk and issued policy No. 47710 dated paragraph 3 of the application above quoted, for at to make a delivery of the policy to him is just as
back to December 1, 1932, and mailed the same to the time of its delivery by the agent as aforesaid the binding on the company as if the decision had been
its agent, Cristobal Mendoza, in Camiling, Tarlac, for insured was not in good health made by its board of directors. Granted that
delivery to the insured. -On January 11, 1933, Mendoza made a mistake of judgement because he
INSURANCE Page 30

acted on insufficient evidence as to the state of of the policy by its invokes, by reason of the delivery - The chief clerk of the Manila office of Sun Life
health of the insured. But it is not charged that the of the policy by its agent. It is admitted that if the testified that he prepared the letter and handed it to
mistake was induced by any misconduct or omission delivery of the policy was due to fraud, legally there the local manager, Mr. E. E. White, for signature.
of duty of the insured. could have been no waiver. In view of the facts The local manager, Mr. White, testified to having
- It is the interest not only the applicant but of all established and admitted, there is no doubt, as to received the cablegram accepting the application of
insurance companies as well that there should be the existence of the fraud. -Estrada, as a Mr. Herrer from the home office on November 26,
some act which gives the applicant the definite representative of the insured was not only bound to 1917. He said that on the same day he signed a
assurance that the contract has been consummated. give a truthful information on the state of health of letter notifying Mr. Herrer of this acceptance. The
This sense of security and of peace of mind that the insured, but it was her duty to find out it his true witness further said that letters, after being signed,
one's defendants are provided for without risk either state of health in order to give true and correct were sent to the chief clerk and placed on the
of loss or of litigation is the bedrock of life insurance. information. When she gave Mendoza an incorrect mailing desk for transmission. Mr. Tuason, who was
When the policy is issued and delivered, in the information tending to create the impression that the the chief clerk on November 26, 1917, was not
absence of fraud or other grounds for rescission, it is insured was well when in fact he was seriously ill, called as a witness.
plainly not within the intention of the parties that there is no doubt that she committed fraud and - For the defense, attorney Manuel Torres testified to
there should be any questions held in abeyance or imparted a deceitful information to the defendant having prepared the will of Joaquin Ma. Herrer. That
reserved for future determination that leave the very agent on this occasion, Mr. Herrer mentioned his
existence of the contract in suspense and doubt. application for a life annuity, and that he said that
- It is therefore in the public interest, for the public is ENRIQUEZ v. SUN LIFE OF CANADA the only document relating to the transaction in his
profoundly and generally interested in life insurance, 41 PHIL 269 possession was the provisional receipt. Rafael
as well as in the interest of the insurance companies MALCOLM; November 29, 1920 Enriquez, the administrator of the estate, testified
themselves by giving certainly and security to their that he had gone through the effects of the deceased
policies, that we are constrained to hold, as we, do, NATURE and had found no letter of notification from the
that the delivery of the policy to the insured by an Appeal from judgment of trial court denying insurance company to Mr. Herrer.
agent of the company who is authorized to make plaintiff’s (administrator of the estate of the late
delivery or without delivery is the final act which Joaquin Ma. Herrer) action to recover from the ISSUE
binds the company (and the insured as well) in the defendant life insurance company the sum of pesos WON there exists a contract for life annuity between
absence of fraud or other legal ground for rescission 6,000 paid by the deceased for a life annuity. Herrer and defendant
- The company therefore having decided that all the
conditions precedent to the taking effect of the policy FACTS HELD
had been complied with and having accepted the - On September 24, 1917, Joaquin Herrer made NO
premium and delivered the policy thereafter to the application to the Sun Life Assurance Company of Ratio The law applicable to the case is found to be
insured, the company is now estopped to assert that Canada through its office in Manila for a life annuity. the second paragraph of article 1262 of the Civil
it never intended that the policy should take effect. Two days later he paid the sum of P6,000 to the Code providing that an acceptance made by letter
manager of the company's Manila office and was shall not bind the person making the offer except
SEPARATE OPINION given a receipt. from the time it came to his knowledge.
- The application was immediately forwarded to the Reasoning
IMPERIAL [dissent] head office of the company at Montreal, Canada. On - Until quite recently, all of the provisions concerning
- "A local agent of an insurance company, whose November 26, 1917, the head office gave notice of life insurance in the Philippines were found in the
only power is to solicit applications for insurance, acceptance by cable to Manila. (Whether on the Code of Commerce and the Civil Code. After July 1,
and forward them to the company for approval, same day the cable was received, notice was sent by 1915, there was, however, in force the Insurance
when, if approved to the insured, has no power to the Manila office of Herrera that the application had Act. No. 2427. Chapter IV of this Act concerns life
waive any of the provision of the policy so been accepted, is a disputed point, which will be and health insurance. The Act expressly repealed
delivered." discussed later.) On December 4, 1917, the policy Title VIII of Book II and Section III of Title III of
- It is clear, therefore, that the delivery of the policy was issued at Montreal. On December 18, 1917, Book III of the code of Commerce. The law of
by Mendoza does not bind the defendant, nor is the attorney Aurelio A. Torres wrote to the Manila office insurance is consequently now found in the
defendant estopped from alleging its defense, for the of the company stating that Herrer desired to Insurance Act and the Civil Code.
simple reason that Mendoza was not an agent with withdraw his application. The following day the local - While, as just noticed, the Insurance Act deals with
authority to issue policies or to accept risks in the office replied to Mr. Torres, stating that the policy life insurance, it is silent as to the methods to be
name of his principle. had been issued, and called attention to the followed in order that there may be a contract of
-There is another ground upon which the majority notification of November 26, 1917. This letter was insurance. On the other hand, the Civil Code, in
opinion is based, namely, that the defendant waived received by Mr. Torres on the morning of December article 1802, not only describes a contact of life
the defense it now invokes, by reason of the delivery 21, 1917. Mr. Herrer died on December 20, 1917. annuity markedly similar to the one we are
INSURANCE Page 31

considering, but in two other articles, gives strong FACTS notwithstanding the non-payment of the premium
clues as to the proper disposition of the case. For - Petitioners were plaintiffs in a civil case of which
instance, article 16 of the Civil Code provides that public respondent Hon. Apostol was the judge. HELD
"In matters which are governed by special laws, any - The case was an offshoot of an incident: plaintiffs NO
deficiency of the latter shall be supplied by the were riding in their car, when a taxicab crossed a Ratio Act No. 2427: an insurance policy would be
provisions of this Code." On the supposition, center island in the road and collided with their car. valid and binding notwithstanding the non-payment
therefore, which is incontestable, that the special law Private respondent Maharlika was eventually of the premium if there was a clear agreement to
on the subject of insurance is deficient in enunciating impleaded as a defendant in this case, with an grant to the insured credit extension. Such
the principles governing acceptance, the subject- allegation that the taxicab involved was insured agreement may be express or implied.
matter of the Civil code, if there be any, would be against third party liability for P20,000.00 with Reasoning
controlling. In the Civil Code is found article 1262 private respondent at the time of the accident - Petitioners maintain that in spite of their late
providing that "Consent is shown by the concurrence - Maharlika claimed there was no cause of action payment, the policy is binding because there was an
of offer and acceptance with respect to the thing and against it because at the time of the accident, the implied agreement to grant a credit extension so as
the consideration which are to constitute the alleged insurance policy was not in force due to the to make the policy effective. To them, the
contract. An acceptance made by letter shall not bind non-payment of the premium thereon. Also, even if subsequent acceptance of the premium and delivery
the person making the offer except from the time it the cab had been insured, the complaint would be of the policy estops the respondent company from
came to his knowledge. premature since the policy provides that the insurer asserting that the policy is ineffective.
- According to the provisional receipt, three things would be liable only when the insured becomes The court however sees no proof of any such implied
had to be accomplished by the insurance company legally liable. agreement. The purported nexus between the
before there was a contract: (1) There had to be a - Trial court ruled in favor of the plaintiff, holding the delivery of the policy and the grant of credit
medical examination of the applicant; (2) there had defendants liable for repair of the car, medical extension is too tenuous to support the conclusion
to be approval of the application by the head office expenses, etc. BUT Maharlike was exonerated on the for which petitioners contend.
of the company; and (3) this approval had in some gnd that the policy was not in force.  Parenthetically mention: in the present law,
way to be communicated by the company to the - Petitioners elevated this case to this court, faulting Section 77 of the Insurance Code of 1978 has
applicant. The further admitted facts are that the the respondent judge for considering the defense of deleted the clause "unless there is clear agreement
head office in Montreal did accept the application, did late payment of premium when “the same was to grant the insured credit extension of the premium
cable the Manila office to that effect, did actually waived at the pre-trial”, hence the evidence of late due" which was then involved in this controversy.
issue the policy and did, through its agent in Manila, payment should be disregarded supposedly because Disposition Fnding no reversible error, the
actually write the letter of notification and place it in the private respondent had admitted that such fact judgment appealed from is hereby AFFIRMED.
the usual channels for transmission to the was not in issue.
addressee. - (More pertinent to this class: ) petitioners assert
- The contract for a life annuity in the case at bar that the private respondent had agreed to grant the
was not perfected because it has not been proved then prospective insured a credit extension of the
satisfactorily that the acceptance of the application premium due.
ever came to the knowledge of the applicant. - This controversy arose under the old insurance law, TIBAY v. CA (FORTUNE LIFE & GENERAL
Disposition Judgment is reversed, and the plaintiff Act No. 2427. INSURANCE)
shall have and recover from the defendant the sum - The accident occurred in 1973. The complaint was 257 SCRA 126
of P6,000 with legal interest from November 20, filed on July 20, 1974.  both before the BELLOSILLO; May 24, 1996
1918, until paid, without special finding as to costs in effectivity of Presidential Decree no. 612, the
either instance. subsequent insurance law which repealed its FACTS
predecessor - On 22 January 1987 Fortune Life and General
- The former insurance law, which applies to the Insurance Co., Inc. (FORTUNE) issued Fire Insurance
case here, provided: An insurer is entitled to the Policy No. 136171 in favor of Violeta R. Tibay and/or
payment of premium as soon as the thing insured is Nicolas Roraldo on their two-storey residential
VELASCO and ACOSTA v. APOSTOL and exposed to the peril insured against, unless there is building located at 5855 Zobel Street, Makati City,
MAHARLIKA INSURANCE CO., INC. clear agreement to grant the insured credit together with all their personal effects therein. The
173 SCRA 228 extension of the premium due. No policy issued by insurance was for P600,000 covering the period from
REGALADO.; May 9, 1989 an insurance company is valid and binding unless 23 January 1987 to 23 January 1988. On 23 January
and until the premium thereof has been paid. 1987, of the total premium of P2,983.50, Violeta
NATURE Tibay only paid P600 thus leaving a considerable
Petition for review on certiorari ISSUE balance unpaid.
WON the insurance policy would be valid and binding
INSURANCE Page 32

- On 8 March 1987 the insured building was different were it not so stipulated. Ergo, petitioners MAKATI TUSCANY v. CA ( AMERICAN HOME
completely destroyed by fire. Two days later, Violeta had absolute freedom of choice whether or not to be ASSURANCE CO.)
Tibay paid the balance of the premium. On the same insured by FORTUNE under the terms of its policy 215 SCRA 462
day, she filed with FORTUNE a claim on the fire and they freely opted to adhere thereto. BELLOSILLO; November 6, 1992
insurance policy. Her claim was accordingly referred - Indeed, and far more importantly, the cardinal
to its adjuster, Goodwill Adjustment Services, Inc. polestar in the construction of an insurance contract NATURE
(GASI), which immediately wrote Violeta requesting is the intention of the parties as expressed in the Appeal from decision of the CA
her to furnish it with the necessary documents for policy. Courts have no other function but to enforce
the investigation and processing of her claim. the same. The rule that contracts of insurance will be FACTS
Petitioner forthwith complied. On 28 March 1987 she construed in favor of the insured and most strongly - American Home Assurance Co. (AHAC),
signed a nonwaiver agreement with GASI to the against the insurer should not be permitted to have represented by American International Underwriters
effect that any action taken by the companies shall the effect of making a plain agreement ambiguous (Phils.), Inc., issued in favor of petitioner Makati
not be, or be claimed to be, an admission of liability. and then construe it in favor of the insured. Verily, it Tuscany Condominium Corporation an insurance
- FORTUNE denied the claim of Violeta for violation is elemental law that the payment of premium is policy on the latter's building and premises, for the
of Policy Condition No. 2 ♪ and of Sec. 77 of the requisite to keep the policy of insurance in force. If period 1 March 1982 to1 March 1983. The premium
Insurance Code. Efforts to settle the case before the the premium is not paid in the manner prescribed in was paid on installments all of which were accepted
Insurance Commission proved futile. On 3 March the policy as intended by the parties the policy is by AHAC.
1988 Violeta and the other petitioners sued ineffective. Partial payment even when accepted as a - A second policy was issued to renew the first one,
FORTUNE for damages in the amount of P600,000 partial payment will not keep the policy alive even this time covering the period 1 March 1983 to 1
representing the total coverage of the fire insurance for such fractional part of the year as the part March 1984. This was also pain in installment basis.
policy plus 12% interest per annum, P100,000 payment bears to the whole payment. - A third policy was again issued for the period 1
moral damages, and attorney's fees equivalent to Disposition Petition is DENIED. Decision of the CA March 1984 to 1 March 1985. For this, petitioner
20% of the total claim. The trial court ruled for is AFFIRMED. made two installment payments, both accepted by
petitioners. CA reversed. AHAC. Thereafter, petitioner refused to pay the
balance of the premium. AHAC filed an action to
ISSUE recover the unpaid balance of P314,103.05.
WON a fire insurance policy is valid, binding and SEPARATE OPINION - Petitioner explained that it discontinued the
enforceable upon mere partial payment of premium payment of premiums because the policy did not
VITUG [dissent] contain a credit clause in its favor and the receipts
HELD - The law neither requires, nor measures the for the installment payments covering the policy for
NO strength of the vinculum juris by, any specific 1984-85, as well as the two (2) previous policies,
Ratio Where the insurer and the insured expressly amount of premium payment. It should thus be stated the following reservations:
stipulated that the policy is not in force until the enough that payment on the premium, partly or in 2. Acceptance of this payment shall not waive
premium has been fully paid the payment of partial full, is made by the insured which the insurer any of the company rights to deny liability on any
premium by the assured in this particular instance accepts. In fine, it is either that a juridical tie exists claim under the policy arising before such
should not be considered the payment required by (by such payment) or that it is not extant at all (by payments or after the expiration of the credit
the law and the stipulation of the parties. Rather, it an absence thereof). Once the juridical relation clause of the policy; and
must be taken in the concept of a deposit to be held comes into being, the full efficacy, not merely pro 3. Subject to no loss prior to premium payment.
in trust by the insurer until such time that the full tanto, of the insurance contract naturally follows. If there be any loss such is not covered.
amount has been tendered and duly receipted for. Verily, not only is there an insurance perfected but - Petitioner further claimed that the policy was never
Reasoning also a partially performed contract. In case of loss, binding and valid, and no risk attached to the policy.
- As expressly agreed upon in the contract, full recovery on the basis of the full contract value, less It then pleaded a counterclaim for P152k for the
payment must be made before the risk occurs for the the unpaid premium can accordingly be had; premiums already paid for 1984-85, and in its
policy to be considered effective and in force. Thus, conversely, if no loss occurs, the insurer can demand answer with amended counterclaim, sought the
no vinculum juris whereby the insurer bound itself to the payment of the unpaid balance of the premium. refund of P924,206.10 representing the premium
indemnify the assured according to law ever resulted The insured, on the one hand, cannot avoid the payments for 1982-85.
from the fractional payment of premium. The obligation of paying the balance of the premium - Trial court dismissed the complaint and the
insurance contract itself expressly provided that the while the insurer, upon the other hand, cannot treat counterclaim upon the following findings: (1)
policy would be effective only when the premium the contract as valid only for the purpose of payment of the premiums of the three policies were
was paid in full. It would have been altogether collecting premiums and as invalid for the purpose of made during the term of said policies, hence, it could
indemnity. not be said, inspite of the reservations, that no risk

This policy including any renewal thereof and/or any endorsement thereon is not in force until the attached under the policies; (2) as regards the
premium has been fully paid to and duly receipted by the Company in the manner provided herein.
INSURANCE Page 33

unpaid premiums, in view of the reservation in the premiums in installment, or to consider the contract - It is already in effect because Hardwood has
receipts ordinarily issued by AHAC on premium as valid and binding upon payment of the first already paid the insurance premium.
payments the only plausible conclusion is that AHAC premium. It delivered the check to Victorio Chua before the
has no right to demand their payment after the lapse - The records clearly show that petitioner and private vessel sank, but Victorio Chua was only to deliver
of the term of said policy on March 1, 1985. respondent intended subject insurance policies to be the check to South Sea five days after the vessel
Therefore, Tuscany was justified in refusing to pay binding and effective notwithstanding the staggered sank.
the same. payment of the premiums. Acceptance of payments Appellant argues that Chua was not its broker, but it
- CA modified the decision by ordering Tuscany to speaks loudly of the insurer's intention to honor the was found that Chua was authorized by South Sea to
pay the balance of the premiums due on the third policies it issued to petitioner. receive the premium on its behalf.
policy plus legal interest until fully paid, and - Section 78 of the Insurance Code in effect allows
affirming the denial of the counterclaim. waiver by the insurer of the condition of prepayment
Petitioner’s Claims by making an acknowledgment in the insurance AREOLA v. CA (PRUDENTIAL GUARANTEE AND
Petitioner argues that where the premiums is not policy of receipt of premium as conclusive evidence ASSURANCE, INC.)
actually paid in full, the policy would only be of payment so far as to make the policy binding 236 SCRA 643
effective if there is an acknowledgment in the policy despite the fact that premium is actually unpaid. ROMERO; September 22, 1994
of the receipt of premium pursuant to Sec. 78 of the Section 77 merely precludes the parties from
Insurance Code. The absence of an express stipulating that the policy is valid even if premiums NATURE CERTIORARI
acknowledgment in the policies of such receipt of the are not paid, but does not expressly prohibit an
corresponding premium payments, and petitioner's agreement granting credit extension, and such an FACTS
failure to pay said premiums on or before the agreement is not contrary to morals, good customs, - June 29, 1985- 7 months after the issuance of
effective dates of said policies rendered them invalid. public order or public policy. Santos Areola's Personal Accident Insurance Policy
Petitioner thus concludes that there cannot be a - At the very least, both parties should be deemed in No. PA-20015 (covering a period of one year),
perfected contract of insurance upon mere partial estoppel to question the arrangement they have Prudential unilaterally cancelled the same since
payment of the premiums because under Sec. 77 of voluntarily accepted. company records revealed that Areola failed to pay
the Insurance Code, no contract of insurance is valid Disposition Judgment affirmed. Costs against his premiums.
and binding unless the premium thereof has been petitioner. o Under the terms of the statement of account
paid, notwithstanding any agreement to the issued by Prudential, Areola was supposed to
contrary. SOUTH SEA SURETY AND INSURANCE v. CA pay the total amount of P1,609.65 which
(VALENZUELA HARDWOOD) included the premium of P1,470.00,
ISSUE 244 SCRA 744 documentary stamp of P110.25 and 2%
WON payment by installment of the premiums due VITUG; June 2, 1995 premium tax of P29.40.
on an insurance policy invalidates the contract of o The statement of account stated that it must not
insurance NATURE be considered a receipt as an official receipt will
Petition for review on certiorari be issued upon payment of the account. And if
HELD payment was made to a representative, the
Ratio Where the risk is entire and the contract is FACTS client must demand for a Provisional Receipt and
indivisible, the insured is not entitled to a refund of - Hardwood entered into agreement with Seven Bros if Official Receipts aren’t received within 7 days,
the premiums paid if the insurer was exposed to the Shipping, where latter undertook to load the former’s Prudential should be notified. If payment is
risk insured for any period, however brief or logs on vessel. Hardwood insured the logs with made to their office, clients should demand for
momentary. South Sea Surety which issued Marine Cargo an OR.
Reasoning Insurance Policy. The vessel sank Jan 25, 1984. - August 3, 1985- Prudential offered to reinstate
- The obligation to pay premiums when due is - Hardwood filed claim with South Sea and Seven same policy it had previously cancelled and even
ordinarily as indivisible obligation to pay the entire Bros. Trial Court favored Hardwood. CA decided proposed to extend its lifetime to December 17,
premium. Here, the parties herein agreed to make against South Sea, but absolved Seven Bros. South 1985, upon a finding that the cancellation was
the premiums payable in installments, and there is Sea filed this instant petition. erroneous and that the premiums were paid in full by
no pretense that the parties never envisioned to Areola but were not remitted by Teofilo M. Malapit,
make the insurance contract binding between them. ISSUES Prudential's branch manager.
And the insured never informed the insurer that it WON the insurance contract was already in effect Petitioners’ Claims
was terminating the policy because the terms were when the vessel sank - The fraudulent act of in misappropriating Areola’s
unacceptable. premium payments is the proximate cause of the
- There is nothing in Section 77 which suggests that HELD cancellation of the insurance policy.
the parties may not agree to allow payment of the YES
INSURANCE Page 34

- Areola theorized that Malapit's act of signing and provision of law, particularly under Article 1910 of - Although the erroneous cancellation of the
even sending the notice of cancellation himself, the Civil Code, is bound by the acts of its agent. insurance policy constituted a breach of contract,
notwithstanding his personal knowledge of - Article 1910 thus reads: Prudential within a reasonable time took steps to
petitioner-insured's full payment of premiums, Art. 1910. The principal must comply with all the rectify the wrong committed by reinstating the
further reinforces the allegation of bad faith. obligations which the agent may have contracted insurance policy of petitioner.
- Such fraudulent act committed by Malapit is within the scope of his authority. - Moreover, no actual or substantial damage or
attributable to Prudential. As for any obligation wherein the agent has injury was inflicted on petitioner Areola at the time
- Malapit's actuations are therefore not separate and exceeded his power, the principal is not bound the insurance policy was cancelled.
distinct from that of Prudential’s. It must, therefore, except when he ratifies it expressly or tacitly. - Nominal damages are "recoverable where a legal
bear the consequences of the erroneous cancellation - Malapit's failure to remit the premiums he received right is technically violated and must be vindicated
of subject insurance policy caused by the non- cannot constitute a defense for private respondent against an invasion that has produced no actual
remittance by its own employee of the premiums insurance company; no exoneration from liability present loss of any kind, or where there has been a
paid. could result therefrom. breach of contract and no substantial injury or actual
- Subsequent reinstatement could not possibly - Prudential’s earlier act of reinstating the insurance damages whatsoever have been or can be shown.
absolve respondent insurance company from liability, policy can not obliterate the injury inflicted on Disposition Petition for review on certiorari is
there being an obvious breach of contract. After all petitioner-insured. hereby GRANTED. RTC’ s DECISION is REINSTATED.
damage had already been inflicted on him and no - Respondent company should be reminded that a
amount of rectification could remedy the same. contract of insurance creates reciprocal obligations UCPB GENERAL INSURANCE CO., INC. v.
Respondent’s Argument for both insurer and insured. MASAGANA TELAMART, INC.
- Prudential argues that where reinstatement, the - Reciprocal obligations are those which arise from 308 SCRA 259
equitable relief sought by Areola was granted at an the same cause and in which each party is both a PARDO; June 15, 1999
opportune moment, i.e. prior to the filing of the debtor and a creditor of the other, such that the
complaint, Areola is left without a cause of action on obligation of one is dependent upon the obligation of NATURE
which to predicate his claim for damages. the other. Petition for review on certiorari of a decision of the
- Reinstatement effectively restored Areola to all his - Under the circumstances of instant case, the Court of Appeals.
rights under the policy. relationship as creditor and debtor between the
parties arose from a common cause: i.e., by reason FACTS
ISSUES of their agreement to enter into a contract of - On April 15, 1991, petitioner issued five (5)
1. WON the erroneous act of canceling subject insurance under whose terms, Prudential promised insurance policies covering respondent's various
insurance policy entitle petitioner-insured to to extend protection to Areola against the risk property described therein against fire, for the period
payment of damages insured for a consideration in the form of premiums from May 22, 1991 to May 22, 1992.
2. WON the subsequent act of reinstating the to be paid by the latter. - In March 1992, petitioner evaluated the policies
wrongfully cancelled insurance policy obliterate - Under the law governing reciprocal obligations, and decided not to renew them upon expiration of
whatever liability for damages Prudential has particularly the second paragraph of Article 1191, their terms on May 22, 1992. Petitioner advised
the injured party, Areola in this case, is given a respondent's broker, Zuellig Insurance Brokers, Inc.
HELD choice between fulfillment or rescission of the of its intention not to renew the policies.
1. YES obligation in case one of the obligors, such as - On April 6, 1992, petitioner gave written notice to
2. NO respondent insurance company, fails to comply with respondent of the non-renewal of the policies at the
Reasoning what is incumbent upon him. address stated in the policies.
- Malapit's fraudulent act of misappropriating the - However, said article entitles the injured party to - On June 13, 1992, fire razed respondent's property
premiums paid by petitioner-insured is beyond doubt payment of damages, regardless of whether he covered by three of the insurance policies petitioner
directly imputable to Prudential. demands fulfillment or rescission of the obligation. issued.
- A corporation, such as respondent insurance - Untenable then is reinstatement insurance - On July 13, 1992, respondent presented to
company, acts solely thru its employees. The latter’s company's argument, namely, that reinstatement petitioner's cashier at its head office five (5)
acts are considered as its own for which it can be being equivalent to fulfillment of its obligation, manager's checks in the total amount of
held to account. divests petitioner-insured of a rightful claim for P225,753.95, representing premium for the renewal
- The facts are clear as to the relationship between payment of damages. Such a claim finds no support of the policies from May 22, 1992 to May 22, 1993.
private respondent insurance company and Malapit. in our laws on obligations and contracts. No notice of loss was filed by respondent under the
His act of receiving the premiums collected is well DAMAGES: policies prior to July 14, 1992.
within the province of his authority as manager. - The nature of damages to be awarded, however, - On July 14, 1992, respondent filed with petitioner
Thus, his receipt of said premiums is receipt by would be in the form of nominal damages its formal claim for indemnification of the insured
private respondent insurance company who, by property razed by fire. On the same day, petitioner
INSURANCE Page 35

returned to respondent the five manager's checks portion was deleted, and the award of attorney's fees extended or renewed by an implied credit
that it tendered, and at the same time rejected was reduced to 10% of the total amount due. arrangement though actual payment of premium was
respondent's claim for the reasons (a) that the It held that following previous practice, respondent tendered on a later date and after the occurrence of
policies had expired and were not renewed, and (b) was allowed a 60- to 90-day credit term for the the (fire) risk insured against.” The Court resolved
that the fire occurred on June 13, 1992, before renewal of its policies, and that the acceptance of this issue in the negative in view of Section 77 of the
respondent's tender of premium payment. the late premium payment suggested an Insurance Code and its decisions in Valenzuela v.
- On July, 21, 1992, respondent filed with the understanding that payment could be made later. Court of Appeals; South Sea Surety and Insurance
Regional Trial Court, Branch 58, Makati City, a civil Hence, this appeal. Co., Inc. v. Court of Appeals; and Tibay v. Court of
complaint against petitioner for recovery, of Appeals. Accordingly, it reversed and set aside the
P18.645,000.00, representing the face value of the ISSUE decision of the Court of Appeals.
policies covering respondent's insured property razed WON the fire insurance policies issued by petitioner - Respondent seasonably filed a motion for the
by fire, and for attorney's fees. to the respondent covering the period May 22, 1991 reconsideration of the adverse verdict. It alleges in
- On October 23, 1992, after its motion to dismiss to May 22, 1992, had expired on the latter date or the motion that the SC had made in the decision its
had been denied, petitioner filed an answer to the had been extended or renewed by an implied credit own findings of facts, which are not in accord with
complaint. It alleged that the complaint "fails to arrangement though actual payment of premium was those of the trial court and the Court of Appeals.
state a cause of action"; that petitioner was not tendered on a later date after the occurrence of the The courts below correctly found that no notice of
liable to -respondent for insurance proceeds under risk (fire) insured against non-renewal was made within 45 days before 22 May
the policies because at the time of the loss of 1992, or before the expiration date of the fire
respondent's property due to fire, the policies had HELD insurance policies. Thus, the policies in question
long expired and were not renewed. NO were renewed by operation of law and were effective
After due trial, on March 10, 1993, the Regional Trial - An insurance policy, other than life, issued and valid on 30 June 1992 when the fire occurred,
Court, Branch 58, Makati, rendered decision, the originally or on renewal, is not valid and binding until since the premiums were paid within the 60- to 90-
dispositive portion of which reads: actual payment of the premium. Any agreement to day credit term.
"WHEREFORE, premises considered, judgment is the contrary is void. The parties may not agree - Respondent likewise disagrees with its ruling that
hereby rendered in favor of the plaintiff and against expressly or impliedly on the extension of credit or parties may neither agree expressly or impliedly on
the defendant, as follows. time to pay the premium and consider the policy the extension of credit or time to pay the premium
"(1) Authorizing and allowing the plaintiff to binding before actual payment. nor consider a policy binding before actual payment.
consign/deposit with this Court the sum of Disposition Judgment reversed and set aside It urges the Court to take judicial notice of the fact
P225,753.95 (refused by the defendant) as full that despite the express provision of Section 77 of
payment of the corresponding premiums for the the Insurance Code, extension of credit terms in
replacement-renewal policies for Exhibits A, B, C, D premium payment has been the prevalent practice in
and E; "(2) Declaring plaintiff to have fully complied the insurance industry. Most insurance companies,
with its obligation to pay the premium thereby including Petitioner, extend credit terms because
rendering the replacement-renewal policy of Exhibits Section 77 of the Insurance Code is not a prohibitive
A, B, C, D and E effective and binding for the injunction but is merely designed for the protection
duration May 22, 1992 until May 22, 1993; and, of the parties to an insurance contract. The Code
ordering defendant to deliver forthwith to plaintiff itself, in Section 78, authorizes the validity of a
the said replacement-renewal policies; "(3) Declaring policy notwithstanding non-payment of premiums.
Exhibits A & B, in force from August 22, 1991 up to UCPB GENERAL INSURANCE CO., INC. v. - Respondent also asserts that the principle of
August 23, 1992 and August 9, 1991 to August 9, MASAGANA TELAMART, INC. (EN BANC) estoppel applies to Petitioner. Despite its awareness
1992, respectively; and "(4) Ordering the defendant 356 SCRA 307 of Section 77 Petitioner persuaded and induced
to pay plaintiff the sums of. (a) P18,645,000.00 DAVIDE; April 4, 2001 Respondent to believe that payment of premium on
representing the latter's claim for indemnity under the 60- to 90-day credit term was perfectly alright;
Exhibits A, B & C and/or its replacement-renewal NATURE in fact it accepted payments within 60 to 90 days
policies; (b) 25% of the total amount due as and for Motion for reconsideration of the decision of the after the due dates. By extending credit and
attorney's fees; (c) P25,000.00 as necessary Supreme Court. habitually accepting payments 60 to 90 days from
litigation expenses; and, (d) the costs of suit. the effective dates of the policies, it has implicitly
“xxx ” FACTS agreed to modify the tenor of the insurance policy
- In due time, petitioner appealed to the Court of - In its decision of 15 June 1999, the SC defined the and in effect waived the provision therein that it
Appeals (CA). The CA promulgated its decision main issue to be “whether the fire insurance policies would pay only for the loss or damage in case the
affirming that of the Regional Trial Court with the issued by petitioner to the respondent covering the same occurred after payment of the premium.
modification that item No. 3 of the dispositive period from May 22, 1991 to May 22, 1992 had been - Petitioner filed an opposition to the Respondent’s
INSURANCE Page 36

motion for reconsideration. It argues that both the No. 612 (The Insurance Code) promulgated on 18 they may deem convenient, provided they are not
trial court and the Court of Appeals overlooked the December 1974. In turn, this Section has its source contrary to law, morals, good customs, public
fact that on 6 April 1992 Petitioner sent by ordinary in Section 72 of Act No. 2427 otherwise known as order, or public policy.
mail to Respondent a notice of non-renewal and sent the Insurance Act as amended by R.A. No. 3540, - Finally, it would be unjust and inequitable if
by personal delivery a copy thereof to Respondent’s approved on 21 June 1963, which read: recovery on the policy would not be permitted
broker, Zuellig. Both courts likewise ignored the fact SEC. 72. An insurer is entitled to payment of against Petitioner, which had consistently granted a
that Respondent was fully aware of the notice of premium as soon as the thing insured is exposed 60- to 90-day credit term for the payment of
non-renewal. A reading of Section 66 of the to the peril insured against, unless there is clear premiums despite its full awareness of Section 77.
Insurance Code readily shows that in order for an agreement to grant the insured credit extension of Estoppel bars it from taking refuge under said
insured to be entitled to a renewal of a non-life the premium due. No policy issued by an Section since Respondent relied in good faith on such
policy, payment of the premium due on the effective insurance company is valid and binding unless and practice. Estoppel then is the fifth exception to
date of renewal should first be made. Respondent’s until the premium thereof has been paid. Section 77.
argument that Section 77 is not a prohibitive (Underscoring supplied) Disposition Judgment reconsidered and set aside,
provision finds no authoritative support. - It can be seen at once that Section 77 does not that of the Court of Appeals affirmed in toto.
- The following facts, as found by the trial court and restate the portion of Section 72 expressly
the Court of Appeals, are indeed duly established: permitting an agreement to extend the period to pay SEPARATE OPINION
1. For years, Petitioner had been issuing fire the premium. But there are exceptions to Section
policies to the Respondent, and these policies 77. VITUG
were annually renewed. The first exception is provided by Section 77 itself, - An essential characteristic of an insurance is its
2. Petitioner had been granting Respondent a and that is, in case of a life or industrial life policy being synallagmatic, a highly reciprocal contract
60- to 90-day credit term within which to pay whenever the grace period provision applies. where the rights and obligations of the parties
the premiums on the renewed policies. The second is that covered by Section 78 of the correlate and mutually correspond.
3. There was no valid notice of non-renewal of Insurance Code, which provides: - By weight of authority, estoppel cannot create a
the policies in question, as there is no proof at SEC. 78. Any acknowledgment in a policy or contract of insurance, neither can it be successfully
all that the notice sent by ordinary mail was contract of insurance of the receipt of premium is invoked to create a primary liability, nor can it give
received by Respondent, and the copy thereof conclusive evidence of its payment, so far as to validity to what the law so procribes as a matter of
allegedly sent to Zuellig was ever transmitted make the policy binding, notwithstanding any public policy.
to Respondent. stipulation therein that it shall not be binding until
4. The premiums for the policies in question in premium is actually paid. PARDO [dissent]
the aggregate amount of P225,753.95 were - A third exception was laid down in Makati Tuscany - An assured’s failure to give notice of the fire
paid by Respondent within the 60- to 90-day Condominium Corporation vs. Court of Appeals, immediately upon its occurrence blatantly showed
credit term and were duly accepted and wherein we ruled that Section 77 may not apply if the fraudulent character of its claims. Respondent is
received by Petitioner’s cashier. the parties have agreed to the payment in required by law and by express terms of the policy to
installments of the premium and partial payment has give immediate written notice of loss. This must be
ISSUE been made at the time of loss. Tuscany has complied with in the utmost good faith.
WON Sec. 77 of the Insurance Code of 1978 must be provided a fourth exception to Section 77, namely, - Assuming arguendo that the 60- to 90-day credit
strictly applied to Petitioner’s advantage despite its that the insurer may grant credit extension for the has been agreed between the parties, respondent
practice of granting a 60- to 90-day credit term for payment of the premium. This simply means that if could not still invoke estoppel to back up its claim.
the payment of premiums the insurer has granted the insured a credit term for Estoppel cannot give validity to an act that is
HELD the payment of the premium and loss occurs before prohibited by law or against public policy. The actual
NO the expiration of the term, recovery on the policy payment of premiums is a condition precedent to the
- Section 77 of the Insurance Code of 1978 provides: should be allowed even though the premium is paid validity of an insurance contract other than life
SEC. 77. An insurer is entitled to payment of the after the loss but within the credit term. insurance policy. Any agreement to the contrary is
premium as soon as the thing insured is exposed Moreover, there is nothing in Section 77 which void as against law and public policy.
to the peril insured against. Notwithstanding any prohibits the parties in an insurance contract to
agreement to the contrary, no policy or contract of provide a credit term within which to pay the ACME SHOE RUBBER & PLASTIC CORP. v. CA
insurance issued by an insurance company is valid premiums. That agreement is not against the law, (DOMESTIC INSURANCE COMPANY OF THE
and binding unless and until the premium thereof morals, good customs, public order or public policy. PHILS.)
has been paid, except in the case of a life or an The agreement binds the parties. Article 1306 of the 134 SCRA 155
industrial life policy whenever the grace period Civil Code provides: MELENCIO-HERRERA; January 17, 1985.
provision applies. ART. 1306. The contracting parties may establish
- This Section is a reproduction of Section 77 of P.D. such stipulations clauses, terms and conditions as NATURE
INSURANCE Page 37

Petition for Review on Certiorari of the Decision of - TC found INSURER liable for P200k and opined that - What became automatically cancelled by R.A. No.
the then Court of Appeals (CA-G. R. No. 58917-R), there was a clear intention on the INSURER's part to 3540 was the 1964-1965 policy for ACME's failure to
denying recovery on an insurance policy, thereby grant ACME a credit extension for the payment of the pay the premium within the 90-day extension
reversing the judgment of the Court of First Instance premium due; and that to allow the INSURER to granted, and in accordance with the express terms of
of Rizal, Branch XII, at Caloocan City, which had apply the premium ACME paid on January 8, 1964. the Promissory Note that it had signed.
allowed such recovery. CA reversed TC and dismissed the suit on the ground Disposition The judgment under review is hereby
that, as of the moment of loss, ACME's properties affirmed. Without pronouncement as to costs.
FACTS were not insured and the INSURER could not be held
- ACME Shoe Rubber and Plastic Corporation (ACME) liable for any indemnity as a result of the loss. PEDRO ARCE v. THE CAPITAL INSURANCE &
had been insuring yearly against fire its building, SURETY CO., INC.
machines and general merchandise with Domestic ISSUE 11 SCRA 63
Insurance Company (INSURER) since 1946. On May WON the premium payment for 1964-1965 was paid ABAD SANTOS; September 30, 1982.
14, 1962, ACME continued to insure its properties
with INSURER in the amount of P200,000 for the HELD NATURE
period May 15, 1962 up to May 15, 1963. NO Appeal from CFI decision on question of law.
- On May 14, 1963, INSURER issued Renewal Receipt - Not having paid the 1964-1965 premium within the
to cover the period May 15, 1963 to May 15, 1964. extension granted, and pursuant to R.A. No. 3540, FACTS
- On January 8, 1964, ACME paid P3,331.26 as the policy was automatically cancelled and there was - Arce (INSURED) owned a residential house which
premium. The INSURER applied the payment as no insurance coverage to speak of as of the date of was insured with the appellant COMPANY since 1961.
renewal premium for the period of May 15, 1963 to the fire on October 13, 1964. In November 1965, the COMPANY sent to the
May 15, 1964. - The pertinent provision of Republic Act No. 3540 INSURED a Renewal Certificate to cover the period
- On May 15, 1964, INSURER issued a Renewal reads: from December 5, 1965 to December 5,1966, and
Receipt for the period of May 15, 1964 to May 15, "Sec. 72. An insurer is entitled to payment of the requested payment of the corresponding premium.
1965 (for renewal premium of P3,331.26 yet to be premium as soon as the thing insured is exposed Anticipating that the premium could not be paid on
paid) with a stamped note that says that the to the peril insured against, unless there is clear time, the INSURED asked for an extension which was
insurance will be deemed valid and binding only agreement to grant the insured credit extension of granted by the COMPANY. After the lapse of the
when the premium and documentary stamps have the premium due. No policy issued by an requested extension, INSURED still failed to pay the
actually been paid in full and duly acknowledged in insurance company is valid and binding unless and premium. Thereafter, the house of the INSURED was
an official receipt. ACME was given 90 days to pay until the premium thereof has been paid." totally destroyed by fire. Upon INSURED's
otherwise the policy would automatically become - RA 3540 was approved on June 20, 2963 and was presentation of claim for indemnity, he was told that
void and ineffective. (ACME should pay short period put into effect on Oct 1, 1963. It could not be no indemnity was due because the premium was not
premium for 90 days before the period expires. If applied retroactively to the renewal of the policy for paid. The INSURED sued the COMPANY for
they are able to pay the whole amount before the the 1963-1964 period because said policy was indemnity.
90-day period, the automatic termination won’t renewed on May 14, 1963. (Laws have no retroactive - The trial court held the COMPANY liable to
apply anymore). effect unless the contrary is provided.) Therefore, indemnify the INSURED on the ground that since the
- On May 26, 1964, ACME, through its President, the Jan 8, 1964 payment was properly applied to the COMPANY could have demanded payment of the
signed a promissory note saying that they promise to 1963-1964 premium. The Trial Court's opinion that premium, mutuality of obligation required that it
pay the premium and documentary stamps and there was a clear agreement to grant ACME credit should be liable on the policy.
agreed to the automatic cancellation penalty for not extension for 1964-1965 is negated by ACME's
complying. Promissory Note binding itself to pay within ninety ISSUE
- On October 13, 1964, ACME’s properties were days from the effective date of this policy, 15th May, WON the COMPANY can be held liable on its policy
completely destroyed by fire. ACME filed insurance 1964. The credit extension was granted for 90 days
claim but the INSURER disclaimed liability on the only. (So wala na by August 16, 1964.) HELD
ground that as of the date of loss, the properties - If ACME was granted credit extensions in the past, NO.
burned were not covered by insurance. the promissory note it signed did away with such - The Court commiserates with the INSURED. They
- ACME claims that the January 8, 1964 payment credit arrangement. Also, before RA 3540, the are well aware that many insurance companies have
was for the period 1964-1965 and that INSURER had Renewal Receipts issued by INSURER did not contain fallen into the condemnable practice of collecting
no right to apply it to the period 1963-1964 because the auto-cancellation after 90 days note. By 1964, premiums promptly but resort to all kinds of excuses
under RA 3540, the policy was void and INSURER however, the situation had changed by the passage to deny or delay payment of just claims. Unhappily
could have validly disclaimed liability for loss had one of the RA: no policy could be valid and binding the instant case is one where the insurer has the law
occurred then. unless and until the premium thereof had been on its side.
paid.
INSURANCE Page 38

- Sec. 72 of the Insurance Act, as amended by R.A. of the premiums, at anytime between the 15th day decided to hold the same for thirty-five (35) days
No. 3540 reads: of December 1960 and one o'clock in the afternoon before presenting it for payment. Having held the
"SEC. 72. An insurer is entitled to payment of of the 15th day of December 1961, the insurance check for such an unreasonable period of time,
premium as soon as the thing insured is exposed company shall make good all such loss or damage in Capital Insurance was estopped from claiming a
to the perils insured against, unless there is clear an amount not exceeding P100,000.00. When the forfeiture of its policy for non-payment even if the
agreement to grant credit extension for the policy was delivered, Plastic Era failed to pay the check had been dishonored later. Where the check is
premium due. No policy issued by an insurance corresponding insurance premium. On January 8, held for an unreasonable time before presenting it
company is valid and binding unless and until the 1961, in partial payment of the insurance premium, for payment, the insurer may be held estopped from
premium thereof has been paid." Plastic Era delivered to Capital Insurance, a check for claiming a forfeiture if the check is dishonored.
- It is obvious from both the Insurance Act, as the amount of P1,000.00 postdated January 16, Disposition The decision of the CA is AFFIRMED in
amended, and the stipulation of the parties that time 1961. However, Capital Insurance tried to deposit toto.
is of the essence in respect of the payment of the the check only on February 20, 1961 and the same
insurance premium so that if it is not paid the was dishonored by the bank for lack of funds. MALAYAN INSURANCE CO., INC. v. ARNALDO
contract does not take effect unless there is still - Two days after the insurance premium became and PINCA
another stipulation to the contrary. In the instant due, at about 4:00 to 5:00 o'clock in the morning, 154 SCRA 672
case, the INSURED was given a grace period to pay the property insured by Plastic Era was destroyed by CRUZ; October 12, 1987
the premium but the period having expired with no fire. In less than a month Plastic Era demanded from
payment made, he cannot insist that the COMPANY Capital Insurance the payment of the sum of FACTS
is nonetheless obligated to him. P100,000.00 as indemnity for the loss of the insured - On June 7, 1981, Malayan Insurance Co. (MICO),
- Prior to the amendment (italicized portion above), property under Policy No. 22760 but the latter issued fire insurance for the amount of P14,000 on
an insurance contract was effective even if the refused for the reason that, among others, Plastic the property of private respondent, Pinca, effective
premium had not been paid so that an insurer was Era failed to pay the insurance premium. July 1981-1982. MICO later allegedly cancelled the
obligated to pay indemnity in case of loss and policy for non-payment of the premium and sent a
correlatively he had also the right to sue for payment ISSUES notice to Pinca. On Dec. 24 Adora, an agent of MICO,
of the premium. But the amendment to Sec. 72 has 1. WON a contract of insurance has been duly received Pinca’s payment, which was remitted to
radically changed the legal regime in that unless the perfected between petitioner and respondent MICO. On Jan. 18, 1982, Pinca’s property was
premium is paid there is no insurance. 2. WON the dishonored check constituted payment completely burned. On Feb. 5, MICO returned Pinca’s
Disposition The decision of the court a quo is payment to Adora on the ground that her policy had
reversed; the appellee's complaint is dismissed. No HELD been cancelled; the latter refused to accept it. Her
special pronouncement as to costs. 1. YES demand for payment having been rejected by MICO,
- Irrelevant facts: The premium costs P38.10. After - Tender of draft or check in order to effect payment Pinca went to the Insurance Commission. Public
the fire, the COMPANY issued a check for P300 to that would extinguish the debtor's liability should be respondent Arnaldo, the Insurance Commissioner,
Arce as donation. Arce accepted the check, but still actually cashed. If the delivery of the check of Plastic sustained Pinca, hence this petition from MICO.
sued the company. Era to Capital Insurance were to be viewed in the Records show MICO received Arnaldo’s decision on
CAPITAL INC. v. PLASTIC ERA CO. light of the foregoing, no payment of the premium April 10; MICO filed a MFR on April 25 which was
65 SCRA 134 had been effected. Significantly, Capital Insurance denied on June 4; MICO received notice of this denial
MARTIN; July 18, 1975 accepted the promise of Plastic Era to pay the on June 14; instant petition was filed on July 2.
insurance premium within 30 days from the effective
NATURE date of policy. By so doing, it has implicitly agreed to ISSUES
Petition for review of a decision of the CA affirming modify the tenor of the insurance policy and in Procedural
the decision of the CFI of Manila effect, waived the provision therein that it would only 1. WON the petition should be dismissed for late
pay for the loss or damage in case the same occurs filing
FACTS after the payment of the premium. Considering that Substantive
- On December 17, 1960, petitioner Capital the insurance policy is silent as to the mode of 2. WON there was a valid insurance contract at the
Insurance & Surety Co., Inc. delivered to the payment, Capital Insurance is deemed to have time of the loss
respondent Plastic Era Manufacturing Co., Inc., its accepted the promissory note in payment of the 3. WON Adora was authorized to receive such
open Fire Policy No. 22760 wherein the former premium. This rendered the policy immediately payment
undertook to insure the latter's building, equipments, operative on the date it was delivered. 4. WON an adjuster is indispensable in the
raw materials, products and accessories located at 2. YES valuation of the loss
Sheridan Street, Mandaluyong, Rizal. The policy - Although the check was due for payment on
expressly provides that if the property insured would January 16, 1961 and Plastic Era had sufficient funds HELD
be destroyed or damaged by fire after the payment to cover it as of January 19, 1961, Capital Insurance Procedural
INSURANCE Page 39

1. YES - MICO also suggests that Pinca knew the policy had - Plaintiff issued a number of life insurance policies in
- Petitioner invokes Sec 416 of the Insurance Code been cancelled and was paying the premium in order the Philippines containing stipulations referred to as
which grants it 30 days from notice of the Insurance to renew the policy. A close study of the transcripts NONFORFEITURE CLAUSES5
Commission within which to appeal by certiorari with show, however, that Pinca only meant to renew the - From January 1, 1942 to December 31, 1946,
the Court. MICO filed its MFR on April 25, 15 days policy had it been cancelled but not if it was still in Plaintiff head office at Toronto applied the provisions
after the notice; the reglementary period began to effect—it was conditional. Payment was thus legally of the automatic premium loan clauses upon the
run again after June 13. Since the petition was filed made on the original transaction and validly received nonpayment of the corresponding premiums by the
only on July 2, it was tardy by 4 days. Alternatively it by Adora, who was not informed of the alleged people who subscribed to the insurance. The net
invokes Rule 45 of the Rules of Court for certiorari cancellation and thus saw no reason to reject the amount of premiums advanced (by the company) or
but the petition still exceeds the 15 day limit from payment. loaned (to the insured) as payment for the premium
the June 13 notice. 3. YES due totaled P1,069,254.98.
-Respondents, on the other hand, invoke Sec. 39 of - Sec. 306 of the Insurance Code provides that any - Meer, the Collector of the National Internal
B.P. 129 which pegs the period for appeal from insurance company that delivers a policy to its agent Revenue assessed the net amount of premium at
decisions of any court in all cases at 15 days from is deemed to have authorized such agent to receive P17,917.12 pursuant to SEC.255, National Internal
the notice of the decision appealed from. Since the payment of premium on its behalf. It is a well-known Revenue Code6
MFR was filed only 15 days after receiving notice of principle under the law of agency that payment to an - Company protested the assessment, but paid the
the decision, it was already 18 days late by July 2. authorized agent is equivalent to payment to the taxes anyway. Then they filed a complaint to recover
So whichever is applied, the petition is still late. principal himself. MICO’s acknowledgement of Adora money paid under protest for taxes
Substantive as its agent thus defeats its contention that he was - CFI: Dissmiss complaint
2. YES not authorized to receive payments on its behalf. - PLAINTIFF’s MAIN CONTENTION: when it made
- A valid cancellation requires the following 4. NO premium loans or premium advances by virtue of the
conditions based on Sections 64-65 of the Code: - In absence of fraud, the amount of the loss may be non-forfeiture clauses, it did not collect premiums
prior notice which must be based on the occurrence determined on the basis of such proof offered by the within the meaning of the above sections of the law,
of one or more of the grounds mentioned in Sec 64 insured. Here. The certification of the Integrated and therefore it is not amenable to the tax therein
(in this case, non-payment of premium), after the National Police as the extent of the loss should provided.
effective date of the policy; the notice must be suffice.
written and mailed to the address on the policy; it Disposition petition is DENIED ISSUES
must state the ground(s) for cancellation and the 1. WON premium advances made by plaintiff-
insurer must furnish details upon the request of the MANUFACTURERS LIFE INSURANCE CO. v. MEER appellant under the automatic premium loan clause
insured. 89 PHIL 351 of its policies are premiums collected' by the
- It is undisputed that payment of premium was BENGZON, June 29, 1951 Company subject to tax
made. Petitioner relies heavily on Sec 77 of the 2. WON, in the application of the automatic premium
Insurance Code to contest this, the said provision NATURE loan clause of plaintiff-appellant's policies, there is
requiring payment of premium as soon as the thing APPEAL from a judgment of the Court of First 'payment in money, notes, credits, or any
is exposed to the peril insured against and that the Instance of Manila substitutes for money
policy is invalid without it. However, this is not 3. WON the collection of the alleged deficiency
applicable in the instant case as payment was FACTS premium taxes constitutes double taxation
eventually made. It is to be noted that the premium (this is a tax case. What’s really important here is
5"'8. Automatic Premium Loan.-This Policy shall not lapse for non-payment of any premium after it has been three full years in force, it, at the due date of such premium,
invoice was stamped “Payment Received”, indicating the definition of CASH SURRENDER VALUE).
an understanding between the parties that payment - Manufacturers Life Insurance Company is a duly the Cash Value of this Policy and of any bonus additions and dividends left on accumulation (after deducting any indebtedness to the company and the interest accrued
thereon) shall exceed the amount of said premium. In which event the company will, without further request, treat the premium then due as paid, and the amount of such
could be made later. This is furthered by the fact organized corporation which has its head office at premium, with interest from its actual due date at six per cent per annum, compounded yearly, and one per cent, compounded yearly, for expenses, shall be a first lien on

that Adora had earlier told her to call him anytime Toronto. It is duly registered and licensed to engage this Policy in the Company's favour in priority to the claim of any assignee or any other person. The accumulated lien may at any time, while the Policy is in force, be paid in

she was ready with her payment. The Court also in life insurance business in the Philippines, and,
whole or in part.
'When the premium falls due and is not paid in cash within the month's grace, if the Cash Value of this policy and of any bonus additions and dividends left on accumulation

finds it strange that MICO only sought to return maintains a branch office in Manila. It was engaged (after deducting any accumulated indebtedness) be less than the premium then due, the Company will, without further requests, continue this insurance in force for a period

Pinca’s Jan. 15 payment only on Feb. 5, long after in such business in the Philippines for more than five * * *.
'10. Cash and Paid-Up Insurance Values.-At the end of the third policy year or thereafter, upon the legal surrender of this Policy to the Company while there is no default in
her house had burned down—this makes petitioner’s years before and including the year 1941. But due to premium payments or within two months after the due date of the premium in default, the Company will (1) grant a cash value as specified in Column (A) increased by the

motives highly suspect. the exigencies of the war It closed the branch office cash value of any bonus additions and dividends left on accumulation, which have been alloted to this Policy, less all indebtedness to the Company on this Polley an the
date of ouch surrender, or (2) endorse this Policy as a Non-Participating Paid-up Polley for the amount as specified In Column (B) of the Table of Guaranteed Values * * *.
- MICO claims to have sent a notice to Pinca, who at Manila during 1942 up to September 1945. '11. Extended Insurance-After the premiums for three or more full years have been paid hereunder in cash, if any subsequent premium is not paid when due, and there is no

flatly denied receiving one. Pinca did not have to indebtedness to the Company on the written request of the insured * * *."

prove this since the strict language of Sec 64 6"SEC. 255. Taxes on insurance premiums.-There shall be collected from every person, company, or corporation (except purely cooperative companies or associations)
requires that MICO ensure the cancellation was doing insurance business of any sort in the Philippines a tax of one per centum of the total premiums collected * * * whether such permiums are paid in money, notes,

actually sent to and received by the insured. credits, or any substitute for money but premiums refunded within six months after payment on account of rejection of risk or returned for other reason to person insured
shall not be included in the taxable receipts * * *."
INSURANCE Page 40

4. WON the making of premium advances, granting - ON ARGUMENT THAT IF THE CREDIT IS PAID OUT OF THE withdrawal in good faith from this field of economic
for the sake of argument that it amounted to CASH SURRENDER VALUE, THERE WERE NO NEW FUNDS ADDED activity.
collection of premiums, were done in Toronto, TO THE COMPANY'S ASSETS”: Cash surrender value "as Disposition finding no prejudicial error in the
Canada applied to a life insurance policy, is the amount of appealed decision, we hereby affirm it with costs.
5. WON the fact that plaintiff-appellant was not money the company agrees to pay to the holder of
doing business in the Philippines during the period the policy if he surrenders it and releases his claims
from January 1, 1942 to September 30, 1945, upon it. The more premiums the insured has paid the
inclusive, exempts it from payment of premium greater will be the surrender value; but the
taxes corresponding to said period surrender value is always a lesser sum than the total
amount of premiums paid." (Cyclopedia Law
Dictionary 3d. ed. 1077.) The cash value or cash
HELD surrender value is therefore an amount which the
NOTE (example given by the plaintiff): insurance company holds In trust for the insured to ANDRES v. CROWN LIFE INSURANCE
"Suppose that 'A', 30 years of age, secures a 20- be delivered to him upon demand. It is therefore a 102 Phil. 919
year endowment policy for P5,000 from plaintiff- liability of the company to the insured. Now then, REYES, J.B.L., Jan.28, 1958
appellant Company and pays an annual premium of when the company's credit for advances is paid out
P250. 'A' pays the first ten yearly premiums of the cash value or cash surrender value, that value NATURE
amounting to P2,500 and on this amount plaintiff- and the company's liability is thereby diminished pro Appeal from judgment of CFI
appellant pays the corresponding taxes under section tanto.
255 of the National Internal Revenue Code. Suppose 2. YES FACTS
also that the cash value of said policy after the - the insurer agreed to consider the premium paid on - Feb. 13, 1950: For the sum of P5,000, defendant-
payment of the 10th annual premium amounts to the strength of the automatic loan. The premium appellee Crown Life issued an insurance policy in the
P1,000." When on the eleventh year the annual was therefore paid by means of a "note" or "credit" name of plaintiff-appellant Rufino and his wife, with
premium fell due and the insured remitted no money or "other substitute for money" and the tax is due the stipulation that the premiums are to be paid
within the mouth grace, the insurer treated the because section 255 above quoted levies taxes semi-annually.
premium then over due as paid from the cash value, according to the total premiums collected by the - The premiums for the 1 st and 2nd semester of the 1st
the amount being a loan to the policyholder1 insurer "whether such premiums are paid in money, year, in the amount of P165.15 were paid by Rufino
who could discharge it at any time with interest at 6 notes, credits or any substitute for money. but the premium for the third semester, in the same
per cent. The insurance contract, therefore, 3. NO amount, was not paid.
continued in force for the eleventh year. - No constitutional prohibition against double - Jan. 6, 1951, Crown Life, through its branch
1. YES taxation. secretary, wrote to Mr. and Mrs. Andres advising
- Based on the example given by the plaintiff, the 4. NO them that their insurance policy lapsed on Dec. 26,
insurer collected the amount of P250 as the annual - The loans are made to policyholders in the 1950 and the amount of P165.15 was overdue,
premium for the eleventh year on the said policy Philippines, who in turn pay therewith the premium giving them 60 days from the date of lapse to file an
when it loaned to “A” the sum of P250. The insurer to the insurer thru the Manila branch. Approval of application for reinstatement. Crown Life later sent
“became a creditor” of the loan, but not of the appellant's position will enable foreign insurers to another letter telling the spouses Andres that their
premium that had already been paid (advanced by evade the tax by contriving to require that premium insurance policy was no longer in force.
the insurer). The insurer is entitled to collect interest payments shall be made at their head offices. What - Feb. 1951: Plaintiff and his wife executed a
on the loan, not on the premium. "A" paid the is important, the law does not contemplate Statement of Health and application for
premium for the eleventh year; but in turn he premiums collected in the Philippines. It is enough reinstatement of the aforesaid policy.
became a debtor of the company for the sum of that the insurer is doing insurance business in the - Feb. 20, 1951: Plaintiff wrote a letter to the
P250. This debt he could repay either by later Philippines, irrespective of the place of its defendant, enclosed with a money order for P100.
remitting the money to the insurer or by letting the organization or establishment. Upon acceptance, defendant advised Rufino that its
cash value compensate for it. The debt may also be 5. NO main office had approved the application and that
deducted from the amount of the policy should "A" - Although during those years the appellant was not the reinstatement of the lapsed policy was subject to
die thereafter during the continuance of the policy. open for new business because its branch office was the payment of the remaining premium balance of
- ON ARGUMENT THAT THE ASSETS OF THE INSURER closed, still it was practically and legally, operating in P65.15.
REMAINED THE SAME AFTER THE APPLICATION OF THE this country by collecting premiums on its - May 3, 1951: Severa Andres died of dystocia,
AUTOMATIC PREMIUM LOAN CLAUSE:
there was an increase outstanding policies, incurring the risks and/or contracted pelvis.
in assets in the form of CREDIT for the advances enjoying the benefits consequent thereto, without - May 5, 1951: Plaintiff sent a letter enclosed with a
made (in the example, the P250 for the 11th year). having previously taken any steps indicating money order in the amount of P65, for the remaining
balance due.
INSURANCE Page 41

- May 15, 1951: Defendant sent a letter with official shall be made within 3 years from the date of lapse; NATURE
receipt of the P165.15 paid by Rufino as well as a (B) there should be a production of evidence of the Petition for review of the decision of theca.
Certificate of Reinstatement. good health of the insured; (C) if the rate of
- June 7, 1951: Rufino presented a death claim as premium depends upon the age of the Beneficiary, FACTS
survivor-beneficiary of his deceased wife. Payment there should likewise be a production of evidence of - Petitioner Arturo P. Valenzuela is a General Agent
was denied by the defendant. his or her good health; (D) there should be of private respondent Philippine American General
- April 1952: Rufino filed a complaint in CFI against presented such other evidence of insurability at the Insurance Company, Inc. (Philamgen for short) since
Crown Life for the recovery of the amount of P5,000 date of application for reinstatement; (E) there 1965. As such, he was authorized to solicit and sell
as the face value of a joint 20-year endowment should be no change which has taken place in such in behalf of Philamgen all kinds of non-life insurance,
insurance policy issued by defendant in favor of good health and insurability subsequent to the date and in consideration of services rendered was
plaintiff and his wife, on Feb. 13, 1950. In its of such application and before the policy is entitled to receive the full agent's commission of
answer, Crown Life disclaimed liability and set forth reinstated; and (F) all overdue premiums and other 32.5% from Philamgen under the scheduled
the special defense that the aforementioned policy indebtedness in respect of the policy, together with commission rates.
had already lapsed. interest at 6%, compounded annually, should first be - From 1973 to 1975, Valenzuela solicited marine
- Aug. 5, 1954: CFI rendered a decision absolving paid. insurance from one of his clients, the Delta Motors,
the defendant company from any liability on the - The plaintiff did not comply with the last condition; Inc. (Division of Electronics Airconditioning and
ground that the policy had lapsed and it was not for he only paid P100 before his wife’s death; and Refrigeration) in the amount of P4.4 Million from
reinstated at the time of the plaintiff’s wife’s death. despite the Company’s reminders, he only remitted which he was entitled to a commission of 32%.
Plaintiff later appealed to the CA but the same was the balance of P65.15 two days after his wife died. However, Valenzuela did not receive his full
certified by the CA to the SC for having no question On the face of such facts, the Company had the right commission which amounted to P1.6 Million from the
of fact. to treat the contract as lapsed and refuse payment P4.4 Million insurance coverage of the Delta Motors.
of the policy. During the period 1976 to 1978, premium payments
ISSUE - Rufino contends that the condition regarding amounting to P1,946,886.00 were paid directly to
WON the insurance policy, which has been in a state payment of the premium was waived by the Philamgen and Valenzuela's commission to which he
of lapse before May 3, 1951, has been validly and insurance Company through its letters, wherein it is entitled amounted to P632,737.00.
completely reinstated after said date (Was there a made statements such as: “If you are unable to pay - In 1977, Philamgen started to become interested in
perfected contract of reinstatement after the policy the full amount immediately, send as large amount and expressed its intent to share in the commission
lapsed due to non-payment of premiums?) as possible and advise us how soon you expect to be due Valenzuela on a fifty-fifty basis. Valenzuela
able to pay the balance; we will work out an refused.
HELD adjustment most beneficial to you.” The Court found - Because of the refusal of Valenzuela, Philamgen
NO the statements to be too vague and indefinite to and its officers took drastic action against
Ratio The stipulation in a life insurance policy giving indicate an intention on the insurer’s part to waive Valenzuela. They: (a) reversed the commission due
the insured the privilege to reinstate it upon written the full payment as prerequisite to the reinstatement him by not crediting in his account the commission
application does not give the insured absolute right of the lapsed policy. The Court reiterated the rule earned from the Delta Motors, Inc. insurance ; (b)
to such reinstatement by the mere filing of an that a waiver must be clear and positive, the intent placed agency transactions on a cash-and-carry
application. The Company has the right to deny the to waive shown clearly and convincingly. On the basis; (c) threatened the cancellation of policies
reinstatement if it is not satisfied as to the other hand, It found subsequent letters sent by issued by his agency; and (d) started to leak out
insurability of the insured and if the latter does not defendant indicating that they insisted on full news that Valenzuela has a substantial account with
pay all overdue premiums and all other indebtedness payment of the premium before the policy was Philamgen. All of these acts resulted in the decline of
to the Company. After the death of the insured the reinstated and that defendant did not consider his business as insurance agent.
insurance Company cannot be compelled to entertain partial payment as sufficient consideration for the - Then on December 27, 1978, Philamgen
an application for reinstatement of the policy reinstatement. Plaintiff-Appellant’s failure to remit terminated the General Agency Agreement of
because the conditions precedent to reinstatement the balance before the death of his wife operated to Valenzuela.
can no longer be determined and satisfied. deprive him of any right to waive the policy and - Lower court: the termination of Valenzuela as
Reasoning recover the face value thereof. General Agent was improper because the record will
- The stipulations of facts render it undisputable that Disposition Judgment appealed from is affirmed. show the principal cause of the termination of the
the original policy lapsed for non-payment of plaintiff as General Agent of defendant Philamgen
premiums on Dec. 26, 1950, upon expiration of the VALENZUELA v. CA (PHILIPPINE AMERICAN was his refusal to share his Delta commission.
31-day grace period. GENERAL INSURANCE COMPANY, INC.) - CA: In any event the principal's power to revoke an
- As found by the lower court, the conditions set 191 SCRA 1 agency at will is so pervasive, that the Supreme
forth in the policy for reinstatement as provided in GUTIERREZ; October 19, 1990 Court has consistently held that termination may be
the contract itself are the following: (A) application effected even if the principal acts in bad faith,
INSURANCE Page 42

subject only to the principal's liability for damages. policies of clients sourced from his agency. Worse, CHAPTER V – THE POLICY, PARTIES THERETO,
(CA ordered Valenzuela to pay Philamgen the despite the termination of the agency, Philamgen & RIGHTS THEREON
amount of One Million Nine Hundred Thirty-Two continued to hold Valenzuela jointly and severally
Thousand Five Hundred Thirty-Two and 17/100 liable with the insured for unpaid premiums. Under DE LIM v. SUN LIFE ASSURANCE COMPANY OF
Pesos (P1,932,532.17) with legal interest) these circumstances, it is clear that Valenzuela had CANADA
an interest in the continuation of the agency when it 41 PHIL 263
ISSUES was unceremoniously terminated not only because of MALCOLM; November 29, 1920
1. WON whether or not Philamgen and/or its officers the commissions he should continue to receive from
can be held liable for damages due to the the insurance business he has solicited and procured NATURE
termination of the General Agency Agreement it but also for the fact that by the very acts of the Appeal from an order of the CFI of Zamboanga
entered into with the petitioners respondents, he was made liable to Philamgen in the sustaining a demurrer to plaintiff's complaint upon
2. WON petitioners are liable to Philamgen for the event the insured fail to pay the premiums due. They the ground that it fails to state a cause of action.
unpaid and uncollected premiums are estopped by their own positive averments and
claims for damages. FACTS
HELD - "The principal may not defeat the agent's right to - On July 6, 1917, Luis Lim of Zamboanga made
1. YES indemnification by a termination of the contract of application to the Sun Life Assurance Company of
- If a principal acts in bad faith and with abuse of agency (Erskine v. Chevrolet Motors Co. 185 NC 479, Canada for a policy of insurance on his life in the
right in terminating the agency, then he is liable in 117 SE 706, 32 ALR 196). sum of P5,000. In his application Lim designated his
damages. - For the pivotal factor rendering Philamgen and the wife, Pilar de Lim, the plaintiff herein, as the
- There is an exception to the principle that an other private respondents liable in damages is that beneficiary. The first premium of P433 was paid by
agency is revocable at will and that is when the the termination by them of the General Agency Lim, and upon such payment the company issued
agency has been given not only for the interest of Agreement was tainted with bad faith. This is in what was called a ''provisional policy." Luis Lim died
the principal but for the interest of third persons or accordance with the precepts in Human Relations on August 23, 1917, after the issuance of the
for the mutual interest of the principal and the enshrined in our Civil Code. provisional policy but before approval of the
agent. In these cases, it is evident that the agency 2. NO. The respondent court erred in holding application by the home office of the insurance
ceases to be freely revocable by the sole will of the Valenzuela liable. There was no factual and legal company. Pilar de Lim brought an action to recover
principal basis for the award. Under Section 77 of the from the Sun Life sum of P5,000, the amount named
(PROCEDURAL: Where the findings of the Court of Insurance Code, the remedy for the non-payment in the provisional policy.
Appeals and the trial court are contrary to each of premiums is to put an end to and render the - The "provisional policy" reads: "Received (subject
other, this Court may scrutinize the evidence on insurance policy not binding - "Sec. 77 . . . to the following stipulations and agreements) the
record [N]otwithstanding any agreement to the contrary, no sum of P433, being the amount of the first year's
- After a painstaking review of the entire records of policy or contract of insurance is valid and binding premium for a Life Assurance Policy on the life of Mr.
the case and the findings of facts of both the court a unless and until the premiums thereof have been Luis D. Lim of Zamboanga for P5,000, for which an
quo and respondent appellate court, the Court paid except in the case of a life or industrial life application dated the 6th day of July, 1917, has been
affirmed the trial court’s findings.) policy whenever the grace period provision applies made to the Sun Life Assurance Company of Canada.
- The principal cause of the termination of (P.D. 612, as amended otherwise known as the - The above-mentioned life is to be assured in
Valenzuela as General Agent of Philamgen arose Insurance Code of 1974) accordance with the terms and conditions contained
from his refusal to share his Delta commission. The - This is buttressed by Section 776 of the or inserted by the Company in the policy which may
records sustain the conclusions of the trial court on Insurance Code (Presidential Decree No. 612, be granted by it in this particular case for four
the apparent bad faith of the private respondents in promulgated on December 18, 1974), which now months only from the date of the application,
terminating the General Agency Agreement of provides that no contract of Insurance by an provided that the Company shall confirm this
petitioners. insurance company is valid and binding unless and agreement by issuing a policy on said application
- It is also evident from the records that the agency until the premium thereof has been paid, when the same shall be submitted to the Head Office
involving petitioner and private respondent is one notwithstanding any agreement to the contrary." in Montreal. Should the Company not issue such a
"coupled with an interest," and, therefore, should not Disposition Petition is GRANTED. CA decision SET policy, then this agreement shall be null and void ab
be freely revocable at the unilateral will of the latter. ASIDE. The decision of the TC REINSTATED with the initio, and the Company shall be held not to have
- The private respondents by the simple expedient of MODIFICATIONS. And that the contractual been on the risk at all, but in such case the amount
terminating the General Agency Agreement relationship between Arturo P. Valenzuela and herein acknowledged shall be returned.
appropriated the entire insurance business of Philippine American General Insurance Company
Valenzuela. With the termination of the General shall be deemed terminated upon the satisfaction of ISSUE
Agency Agreement, Valenzuela would no longer be the judgment as modified.
entitled to commission on the renewal of insurance
INSURANCE Page 43

WON the contract of insurance between Luis Lim and 316 SCRA 677 1. WON CA erred in holding petitioner liable to DBP
Sun Life Assurance Company of Canada was QUISUMBING; October 13, 1999 as beneficiary in a group life insurance contract from
perfected a complaint filed by the widow of the
NATURE decedent/mortgagor
HELD Petition for Review of CA decision 2. WON CA erred in not finding that Dr. Leuterio
NO. concealed that he had hypertension, which would
- The document it is to be a provisional policy "for FACTS vitiate the insurance contract
four months only from the date of this application." - A contract of group life insurance was executed 3. WON CA erred in holding Grepalife liable for
Immediately following the words fixing the four between petitioner Great Pacific Life Assurance P86,200.00 without proof of the actual outstanding
months period comes the word "provided" which has Corporation (hereinafter Grepalife) and Development mortgage payable by the mortgagor to DBP
the meaning of "if." Otherwise stated, the policy for Bank of the Philippines (hereinafter DBP). Grepalife
four months is expressly made subject to the agreed to insure the lives of eligible housing loan HELD
affirmative condition that the company shall confirm mortgagors of DBP. 1. NO
this agreement by issuing a policy on said application - In Nov. 1983, Dr. Wilfredo Leuterio, a physician Ratio Insured, being the person with whom the
when the same shall be submitted to the head office and a housing debtor of DBP applied for membership contract was made, is primarily the proper person to
in Montreal. To re-enforce the same there follows the in the group life insurance plan. In an application bring suit. Subject to some exceptions, insured may
negative condition - "Should the company not issue form, Dr. Leuterio answered Qs concerning his health thus sue, although the policy is taken wholly or in
such a policy, then this agreement shall be null and condition as follows: part for the benefit of another person named or
void ab initio, and the company shall be held not to Q: Have you ever had, or consulted, a physician for unnamed, and although it is expressly made payable
have been on the risks." Certainly language could a heart condition, high blood pressure, cancer, to another as his interest may appear or otherwise.
hardly be used which would more clearly stipulate diabetes, lung, kidney or stomach disorder or any Although a policy issued to a mortgagor is taken out
that the agreement should not go into effect until the other physical impairment? No. for the benefit of the mortgagee and is made
home office of the company should confirm it by Q: Are you now, to the best of your knowledge, in payable to him, yet the mortgagor may sue thereon
issuing a policy. As we read and understand the so- good health? Yes. in his own name, especially where the mortgagee's
called provisional policy, it amounts to nothing but - Grepalife issued an insurance coverage of Dr. interest is less than the full amount recoverable
an acknowledgment on behalf of the company, that Leuterio, to the extent of his DBP mortgage under the policy. (See Sec. 8, Insurance Code)
it has received from the person named therein the indebtedness of P86,200.00. In Aug. 1984, Dr. Reasoning
sum of money agreed upon as the first year's Leuterio died due to "massive cerebral hemorrhage." [a] The insured private respondent did not cede to
premium upon a policy to be issued upon the DBP submitted a death claim to Grepalife. Grepalife the mortgagee all his rights or interests in the
application, if the application is accepted by the denied the claim because Dr. Leuterio was not insurance, the policy stating that: “In the event of
company. physically healthy when he applied for an insurance. the debtor's death before his indebtedness with the
- It is of course a primary rule that a contract of Grepalife insisted that Dr. Leuterio did not disclose Creditor (DBP) shall have been fully paid, an amount
insurance, like other contracts, must be assented to he had been suffering from hypertension, which to pay the outstanding indebtedness shall first be
by both parties either in person or by their agents. caused his death. Allegedly, such non-disclosure paid to the creditor and the balance of sum assured,
So long as an application for insurance has not been constituted concealment that justified the denial of if there is any, shall then be paid to the
either accepted or rejected, it is merely an offer or the claim. beneficiary/ies designated by the debtor.” When DBP
proposal to make a contract. The contract, to be - Herein respondent Medarda Leuterio, widow, filed a submitted the insurance claim against Grepalife, the
binding from the date of the application must have complaint with RTC against Grepalife for "Specific latter denied payment thereof, interposing the
been a completed contract, one that leaves nothing Performance with Damages." Dr. Mejia, who issued defense of concealment committed by the insured.
to be done, nothing to be completed, nothing to be the death certificate, testified that Dr. Leuterio Thereafter, DBP collected the debt from the
passed upon, or determined, before it shall take complained of headaches presumably due to high mortgagor and took the necessary action of
effect. There can be no contract of insurance unless blood pressure. The inference was not conclusive foreclosure on the residential lot of private
the minds of the parties have met in agreement. Our because Dr. Leuterio was not autopsied, hence, respondent.
view is, that a contract of insurance was not here other causes were not ruled out. [b] Since a policy of insurance upon life or health
consummated by the parties. - RTC ruled in favor of respondent widow and against may pass by transfer, will or succession to any
- The trial court committed no error in sustaining the Grepalife. CA sustained the RTC decision. Hence, the person, whether he has an insurable interest or not,
demurrer and dismissing the case. It is to be noted, present petition. and such person may recover it whatever the insured
however that counsel for appellee admits the liability might have recovered, the widow of the decedent Dr.
of the company for the return of the first premium to Leuterio may file the suit against the insurer,
the estate of the deceased. Grepalife.
ISSUES 2. NO
GREAT PACIFIC LIFE v. CA (LEUTERIO)
INSURANCE Page 44

Ratio The fraudulent intent on the part of the the outstanding indebtedness shall first be paid to entire shipment of logs covered by the two marines
insured must be established to entitle the insurer to the Creditor and the balance of the Sum Assured, if policies were received in good order at their point of
rescind the contract. Misrepresentation as a defense there is any shall then be paid to the beneficiary/ies destination. It was further stated that the said loss
of the insurer to avoid liability is an affirmative designated by the debtor." From this, it is clear that may not be considered as covered under cover note
defense and the duty to establish such defense by Grepalife is liable and that Dr. Leuterio’s heirs must because the said note had become 'null and void by
satisfactory and convincing evidence rests upon the get the proceeds. virtue of the issuance of two marine policies.
insurer. In the case at bar, the petitioner failed to Disposition Petition DENIED. CA Decision AFFIRMED - The CFI of Manila ruled in favour of the petitioner.
clearly and satisfactorily establish its defense, and is with modification. - The Court of Appeals reversed the decision of the
therefore liable to pay the proceeds of the insurance. CFI.
Reasoning PACIFIC TIMBER EXPORT CORPORATION v. CA
[a] The insured, Dr. Leuterio, had answered in his (WORKMEN’S INSURANCE CO) ISSUES
insurance application that he was in good health and 112 SCRA 199 1. WON the cover note is null and void for lack of
that he had not consulted a doctor or any of the DE CASTRO; February 25, 1982 valuable consideration because no separate
enumerated ailments, including hypertension; when premiums are collected by private respondent on all
he died the attending physician had certified in the FACTS its cover notes
death certificate that the former died of cerebral - March 19, 1963 - the plaintiff secured temporary 2. WON the court of appeals erred in holding that
hemorrhage, probably secondary to hypertension. insurance from the defendant for its exportation of private respondent was released from liability under
From this report, petitioner Grepalife refused to pay 1,250,000 board feet of Philippine Lauan and Apitong the cover note due to unreasonable delay in giving
the insurance claim. It alleged that the insured had logs to be shipped from the Diapitan Bay, Quezon to notice of loss because the court disregarded the
concealed the fact that he had hypertension. Okinawa and Tokyo, Japan. The defendant issued on proven fact that private respondent did not promptly
[b] Contrary to Grepalife’s allegations, there was no said date Cover Note No. 1010, insuring the said and specifically object to the claim on the ground of
sufficient proof that the insured had suffered from cargo of the plaintiff "Subject to the Terms and delay in giving notice of loss and, consequently,
hypertension. Aside from the statement of the Conditions of the WORKMEN'S INSURANCE objections on that ground are waived under section
insured's widow who was not even sure if the COMPANY, INC. printed Marine Policy form as filed 84 of the insurance act
medicines taken by Dr. Leuterio were for with and approved by the Office of the Insurance
hypertension, the appellant had not proven nor Commissioner. HELD
produced any witness who could attest to Dr. - April 2, 1963 - The two (2) regular marine cargo 1. NO
Leuterio's medical history. policies were issued by the defendant in favor of the Ratio Cover note is issued with a consideration
[c] Grepalife had failed to establish that there was plaintiff. The total cargo insured under the two when, by express stipulation, the cover note is made
concealment made by the insured, hence, it cannot marine policies accordingly consisted of 1,395 logs, subject to the terms and conditions of the marine
refuse payment of the claim. or the equivalent of 1,195,498 bd. ft. policies, and the payment of premiums is one of the
3. NO - After the issuance of cover note but before the terms of the policies.
- Considering the supervening event that DBP issuance of the two marine policies some of the logs Reasoning
foreclosed in 1995 their residential lot, in satisfaction intended to be exported were lost during loading a. the cover note in question is subject to the terms
of mortgagor's outstanding loan, the insurance operations in the Diapitan Bay due to bad weather. and conditions of the marine policies
proceeds shall inure to the benefit of the heirs of the - April 4, 1963 - The plaintiff informed the defendant b. Nature of the Cover Note: The fact that no
deceased person or his beneficiaries. Equity dictates about the loss of 'approximately 32 pieces of logs' separate premium was paid on the Cover Note
that DBP should not unjustly enrich itself at the during loading through a letter. before the loss insured against occurred, does not
expense of another. Hence, it cannot collect the - The plaintiff subsequently submitted a 'Claim militate against the validity of petitioner's
insurance proceeds, after it already foreclosed on the Statement' demanding payment of the loss under contention, for no such premium could have been
mortgage. The proceeds now rightly belong to Dr. the second marine cargo policy. paid, since by the nature of the Cover Note, it did
Leuterio's heirs represented by his widow, herein - July 17, 1963 - the defendant requested the First not contain, as all Cover Notes do not contain
private respondent. Philippine Adjustment Corporation to inspect the loss particulars of the shipment that would serve as basis
- The Court ruled this issue based on the clear and assess the damage. for the computation of the premiums. As a logical
provisions of the policy. The mortgagor paid the - August 23, 1963 - the adjuster reported that 'the consequence, no separate premiums are intended or
premium according to the coverage of his insurance, loss of 30 pieces of logs is not covered by the two required to be paid on a Cover Note.
which states that: "The policy states that upon policies inasmuch as said policies covered the actual c. The petitioner paid in full all the premiums as
receipt of due proof of the Debtor's death during the number of logs loaded on board. But it is covered by called for by the statement issued by private
terms of this insurance, a death benefit in the Cover Note. respondent after the issuance of the two regular
amount of P86,200.00 shall be paid… In the event of - On January 13, 1964 - the defendant wrote the marine insurance policies, thereby leaving no
the debtor's death before his indebtedness with the plaintiff denying the latter's claim, on the ground account unpaid by petitioner due on the insurance
creditor shall have been fully paid, an amount to pay that defendant's investigation revealed that the coverage, which must be deemed to include the
INSURANCE Page 45

Cover Note. If the Note is to be treated as a separate - On learning of this decision, the petitioner moved P2,500,000.00, and this must be considered, by
policy instead of integrating it to the regular policies to lift the order of default, invoking excusable agreement, the actual value of the property insured
subsequently issued, the purpose and function of the neglect, and to vacate the judgment by default. Its on the day the fire occurred. This valuation becomes
Cover Note would be set at naught or rendered motion was denied. even more believable if it is remembered that at the
meaningless, for it is in a real sense a contract, not a - On appeal, IAC affirmed the TC decision in toto. time the building was burned it was still under
mere application for insurance which is a mere offer. construction and not yet completed.
Had all the logs been lost during the loading ISSUE - The Court notes that the policy in this case is an
operations, but after the issuance of the Cover Note, 1. WON default of petitioner is based on excusable open policy and is subject to the express condition
liability on the note would have already arisen even neglect that:
before payment of premium. This is how the cover 2. What is the amount of indemnity due to the "Open Policy.
note as a "binder" should legally operate; otherwise, private respondent under its insurance contract? This is an open policy as defined in Sec57 of the
it would serve no practical purpose in the realm of WON CFI was correct in interpreting the contract Insurance Act. In the event of loss, whether total
commerce, and is supported by the doctrine that or partial, it is understood that the amount of the
where a policy is delivered without requiring HELD loss shall be subject to appraisal and the liability of
payment of the premium, the presumption is that a 1. NO the company, if established, shall be limited to the
credit was intended and policy is valid. - Summons was served through its vice-president. actual loss, subject to the applicable terms,
2. NO There were even several extensions to the original conditions, warranties and clauses of this Policy,
- The private respondent company never raised this period to answer. As a consequence, the TC, on and in no case shall exceed the amount of the
ground in the proceedings. It must be because it did motion of the private respondent filed declared the policy."
not find any delay, as this Court fails to find a real petitioner in default. This was done almost one - As defined in the aforestated provision, which is
and substantial sign thereof. But even on the month later. Even so, the petitioner made no move now Sec60 of the Insurance Code, "an open policy is
assumption that there was delay, this Court is at all for two months thereafter. It was only more one in which the value of the thing insured is not
satisfied and convinced that as expressly provided by than one month after the judgment of default was agreed upon but is left to be ascertained in case of
law, waiver can successfully be raised against private rendered by the TC that it filed a motion to lift the loss.".
respondent. Thus Section 84 of the Insurance Act order of default and vacate the judgment by default. - The actual loss has been ascertained in this case
provides: - There is a pattern of inexcusable neglect. and the Court will respect such factual determination
"Section 84. - Delay in the presentation to an 2. The policy is an open policy which means that the in the absence of proof that it was arrived at
insurer of notice or proof of loss is waived if actual loss, as determined, will represent the total arbitrarily. There is no such showing. Hence,
caused by any act of his or if he omits to take indemnity due the insured from the insurer except applying the open policy clause as expressly agreed
objection promptly and specifically upon that only that the total indemnity shall not exceed the upon by the parties in their contract, we hold that
ground." face value of the policy. the private respondent is entitled to the payment of
- From what has been said, We find duly - The petitioner argues that since at the time of the indemnity under the said contract in the total
substantiated petitioner's assignments of error. fire the building insured was worth P5,800,000.00, amount of P508,867.00.
Disposition The appealed decision is set aside and the private respondent should be considered its own - The refusal of its vice-president to receive the
the decision of the Court of First Instance is insurer for the difference between that amount and private respondent's complaint, as reported in the
reinstated in toto with the affirmance of this Court. the face value of the policy and should share pro rata sheriff's return, was the first indication of the
in the loss sustained. Accordingly, the private petitioner's intention to prolong this case and
DEVELOPMENT INSURANCE v. IAC (PHIL respondent is entitled to an indemnity of only postpone the discharge of its obligation to the
UNION REALTY DEVELOPMENT CORP) P67,629.31, the rest of the loss to be shouldered by private respondent under this agreement. That
143 SCRA 62 it alone. The petitioner cites Condition 17 of the intention was revealed further in its subsequent acts
CRUZ; July 16, 1986 policy, which provides: ---- or inaction ---- which indeed enabled it to avoid
"If the property hereby insured shall, at the payment for more than five years from the filing of
FACTS breaking out of any fire, be collectively of greater the claim against it in 1980.
- A fire occurred in the building of the private value than the sum insured thereon then the Disposition The appealed decision is affirmed in
respondent and it sued for recovery of damages from insured shall be considered as being his own full, with costs against the petitioner.
the petitioner on the basis of an insurance contract insurer for the difference, and shall bear a ratable
between them. The petitioner allegedly failed to proportion of the loss accordingly. Every item, if HARDING v. COMMERCIAL UNION ASSURANCE
answer on time and was declared in default by TC. A more than one, of the policy shall be separately 38 PHIL 464
judgment of default was rendered on the strength of subject to this condition." FISHER; August 10, 1918
the evidence submitted ex parte by the private - However, there is no evidence on record that the
respondent, which was allowed full recovery of its building was worth P5,800,000.00 at the time of the FACTS
claimed damages. loss. On the contrary, the building was insured at
INSURANCE Page 46

- Mrs. Harding was the owner of a Studebaker of "3,500" and under another heading "Present amount, upon the basis of which the premium was
automobile; in consideration of the payment to the value" is the amount of "3,000". paid, is bound by it and must pay the loss in
defendant of the premium of P150, by said plaintiff, - After the said proposal was made a representative accordance with the stipulated insured value.
Mrs. Henry E. Harding, with the consent of her of the Manila agent of defendant went to the Luneta
husband, the defendant by its duly authorized agent, Garage and examined said automobile and Mr. ISSUE
Smith, Bell & Company (limited), made its policy of Server, the General Manager of the Luneta Garage, 1. WON Mrs. Harding was not the owner of the
insurance in writing upon said automobile was set an experienced automobile mechanic, testified that automobile at the time of the issuance of the policy,
forth in said policy to be P3,000 that the value of at the time this automobile was insured it was worth and, therefore, had no insurable interest in it
said automobile was set forth in said policy to be about P3,000, and the defendant, by and through its 2. WON the statement regarding the cost of the
P3,000; that on March 24, 1916, said automobile said agent Smith, Bell & Company (limited), automobile was a warranty, that the statement was
was totally destroyed by fire; that the loss thereby to thereafter issued a policy of insurance upon proposal false, and that, therefore, the policy never attached
plaintiffs was the sum of P3,000. in which policy the said automobile was described as to the risk
- The defendant’s version is that by request of Mrs. of the "present value" of P3,000 and the said
Harding, it issued the policy of insurance on an defendant charged the said plaintiff Mrs. Henry E. HELD
automobile alleged by the said plaintiff to be her Harding as premium on said policy the sum of P150, 1. NO
property. It was made by means of a proposal in or 5 per cent of the then estimated value of P3,000. - Article 1334 of the Civil Code which provides that
writing signed and delivered by said plaintiff to the - The "Schedule" in said policy of insurance describes "All gifts between spouses during the marriage shall
defendant, guaranteeing the truth of the statements the automobile here in question, and provides in part be void. Moderate gifts which the spouses bestow on
contained therein which said proposal is referred to of follows: each other on festive days of the family are not
in the said policy of insurance made a part thereof; "That during the period above set forth and during included in this rule."
that certain of the statements and representations any period for which the company may agree to - Even assuming that defendant might have invoked
contained in said proposal and warranted by said renew this policy the company will subject to the article 1334 as a defense, the burden would be upon
plaintiff to be true, to wit: (a) the price paid by the exception and conditions contained herein or it to show that the gift in question does not fall
proposer for the said automobile; (b) the value of endorsed hereon indemnify the insured against loss within the exception therein established. We cannot
said automobile at the time of the execution and of or damage to any motor car described in the say, as a matter of law, that the gift of an
delivery of the said proposal and (c) the ownership schedule hereto (including accessories) by whatever automobile by a husband to his wife is not a
of said automobile, were false and known to be false cause such loss or damage may be occasioned and moderate one. Whether it is or is not would depend
by the said plaintiff at the time of signing and will further indemnify the insured up to the value of upon the circumstances of the parties, as to which
delivering the said proposal and were made for the the car or P3,000 whichever is the greater against nothing is disclosed by the record.
purpose of misleading and deceiving the defendant, any claim at common law made by any person (not - We are of the opinion that it would be unfair to
and inducing the defendant, relying upon the being a person in the said motor car nor in the hold the policy void simply because the outlay
warranties, statements, and representations insured's service) for loss of life or for accidental represented by the automobile was made by the
contained in the said proposal and believing the bodily injury or damage to property caused by the plaintiff's husband and not by his wife, to whom he
same to be true, issued the said policy of insurance. said motor car including law costs payable in had given the automobile. It cannot be assumed that
- The evidence shows that Hermanos, the Manila connection with such claim when incurred with the defendant should not have issued the policy unless it
agents for the Studebaker automobile, sold the consent of the company." were strictly true that the price representing the cost
automobile to Canson for P3,200 (testimony of Mr. - On March 24, 1916, the said automobile was totally of the machine had been paid by the insured and by
Diehl); who sold the said automobile to Henry destroyed by fire, and that the iron and steel no other person ? that it would no event insure an
Harding for the sum of P1,500. Harding sold the said portions of said automobile which did not burn were automobile acquired by gift, inheritance, exchange,
automobile to J. Brannigan for the sum of P2,000 taken into the possession of the defendant by and or any other title not requiring the owner to make a
who sold the said automobile Henry Harding for the through its agent Smith, Bell & Company (limited), specific cash outlay for its acquisition.
sum of P2,800; Henry Harding gave the said and sold by it for a small sum, which had never been 2. NO
automobile to his wife as a present; that said tendered to the plaintiff prior to the trial of this case, - It has not been shown by the evidence that the
automobile was repaired and repainted at the Luneta but in open court during the trial the sum of P10 as statement was false; on the contrary we believe that
Garage at a cost of some P900; that while the said the proceeds of such sale was tendered to plaintiff it shows that the automobile had in fact cost more
automobile was at the Luneta Garage; the latter and refused. than the amount mentioned. The court below found,
solicited of Mrs. Harding the insurance of said - Trial judge decided that there was no proof of fraud and the evidence shows, that the automobile was
automobile by the Company; that a proposal was on the part of plaintiff in her statement of the value bought by plaintiff's husband a few weeks before the
filled out by the said agent and signed by the plaintiff of the automobile, or with respect to its ownership; issuance of the policy in question for the sum of
Mrs. Henry E. Harding, and in said proposal under that she had an insurable interest therein; and that P2,800, and that between that time and the issuance
the heading "Price paid by proposer," is the amount defendant, having agreed to the estimated value, of the policy some P900 was spent upon it in repairs
P3,000, and having insured the automobile for that and repainting.
INSURANCE Page 47

- The witness Server, an expert automobile This petition for review assails the Decision of the - The test to determine if a contract is an insurance
mechanic, testified that the automobile was Court of Appeals, affirming the Decision of the contract or not, depends on the nature of the
practically as good as new at the time the insurance Insurance Commission. Both decisions held that promise, the act required to be performed, and the
was effected. The form of proposal upon which the there was no violation of the Insurance Code and the exact nature of the agreement in the light of the
policy was issued does not call for a statement respondents do not need license as insurer and occurrence, contingency, or circumstances under
regarding the value of the automobile at the time of insurance agent/broker. which the performance becomes requisite. It is not
its acquisition by the applicant for the insurance, but by what it is called. Basically, an insurance contract
merely a statement of its cost. The amount stated FACTS is a contract of indemnity. In it, one undertakes for
was less than the actual outlay which the automobile - White Gold procured a protection and indemnity a consideration to indemnify another against loss,
represented to Mr. Harding, including repairs, when coverage for its vessels from Steamship Mutual damage or liability arising from an unknown or
the insurance policy was issued. through Pioneer Insurance. Subsequently, White contingent event.
- The court below found and the evidence shows, Gold was issued a Certificate of Entry and - In particular, a marine insurance undertakes to
without dispute, that the proposal upon which the Acceptance. Pioneer also issued receipts evidencing indemnify the assured against marine losses, such as
policy in question was issued was made out by payments for the coverage. When White Gold failed the losses incident to a marine adventure. Section 99
defendant's agent by whom the insurance was to fully pay its accounts, Steamship Mutual refused of the Insurance Code enumerates the coverage of
solicited, and that appellee simply signed the same. to renew the coverage. marine insurance.
It also appears that an examiner employed by the - Steamship Mutual thereafter filed a case against - Relatedly, a mutual insurance company is a
defendant made an inspection of the automobile White Gold for collection of sum of money to recover cooperative enterprise where the members are both
before the acceptance of the risk, and that the sum the latter’s unpaid balance. White Gold on the other the insurer and insured. In it, the members all
after this examination. The trial court found that Mrs. hand, filed a complaint before the Insurance contribute, by a system of premiums or
Harding, in fixing the value of the automobile at Commission claiming that Steamship Mutual violated assessments, to the creation of a fund from which all
P3,000, acted upon information given her by her Sections 186 and 187 of the Insurance Code, while losses and liabilities are paid, and where the profits
husband and by Mr. Server, the manager of the Pioneer violated Sections 299, 300 and 301 in are divided among themselves, in proportion to their
Luneta Garage. She merely repeated the information relation to Sections 302 and 303, thereof. interest. Additionally, mutual insurance associations,
which had been given her by her husband, and at - The Insurance Commission dismissed the or clubs, provide three types of coverage, namely,
the same time disclosed to defendant's agent the complaint. It said that there was no need for protection and indemnity, war risks, and defense
source of her information. There is no evidence to Steamship Mutual to secure a license because it was costs.
sustain the contention that this communication was not engaged in the insurance business. It explained - A P & I Club is “a form of insurance against third
made in bad faith. We do not think that the facts that Steamship Mutual was a Protection and party liability, where the third party is anyone other
stated in the proposal can be held as a warranty of Indemnity Club (P & I Club). Likewise, Pioneer need than the P & I Club and the members.” By definition
the insured, even if it should have been shown that not obtain another license as insurance agent and/or then, Steamship Mutual as a P & I Club is a mutual
they were incorrect in the absence of proof of willful a broker for Steamship Mutual because Steamship insurance association engaged in the marine
misstatement. Under such circumstance, the Mutual was not engaged in the insurance business. insurance business.
proposal is to be regarded as the act of the insurer Moreover, Pioneer was already licensed, hence, a - The records reveal Steamship Mutual is doing
and not of the insured. separate license solely as agent/broker of Steamship business in the country albeit without the requisite
Disposition Plaintiff was the owner of the Mutual was already superfluous. certificate of authority mandated by Section 187 of
automobile in question and had an insurable interest - The Court of Appeals affirmed the decision of the the Insurance Code. It maintains a resident agent in
therein; that there was no fraud on her part in Insurance Commissioner. In its decision, the the Philippines to solicit insurance and to collect
procuring the insurance; that the valuation of the appellate court distinguished between P & I Clubs payments in its behalf. We note that Steamship
automobile, for the purposes of the insurance, is vis-à-vis conventional insurance. The appellate court Mutual even renewed its P & I Club cover until it was
binding upon the defendant corporation, and that the also held that Pioneer merely acted as a collection cancelled due to non-payment of the calls. Thus, to
judgment of the court below is, therefore, correct agent of Steamship Mutual. continue doing business here, Steamship Mutual or
and must be affirmed, with interest, the costs of this through its agent Pioneer, must secure a license
appeal to be paid by the appellant. ISSUES from the Insurance Commission.
1. WON Steamship Mutual, a P & I Club, is engaged - Since a contract of insurance involves public
WHITE GOLD MARINE SERVICES v. PIONEER in the insurance business in the Philippines interest, regulation by the State is necessary. Thus,
INSURANCE 2. WON Pioneer needs a license as an insurance no insurer or insurance company is allowed to
464 SCRA 448 agent/broker for Steamship Mutual engage in the insurance business without a license or
QUISUMBING; July 28, 2005 a certificate of authority from the Insurance
HELD Commission.
NATURE 1. YES 2. YES
- SEC. 299 . . .
INSURANCE Page 48

- No person shall act as an insurance agent or as an respondents. The Arbiter ordered all the
insurance broker in the solicitation or procurement of respondents, except Pandiman, to jointly and FILIPINAS COMPANIA DE SEGUROS V
applications for insurance, or receive for services in severally pay the widow the death benefits plus legal CHRISTERN, HUENEFELD AND CO INC
obtaining insurance, any commission or other fees. The NLRC, on appeal by Marine, limited the 89 PHIL 54
compensation from any insurance company doing liable parties to Pandiman and OMMIAL but PARAS; May 25, 1951
business in the Philippines or any agent thereof, maintained the money award. The CA sustained the
without first procuring a license so to act from the decision of the NLRC. Hence this appeal. FACTS
Commissioner, which must be renewed annually on - October 1, 1941 - Christern Huenefeld, & Co., Inc.,
the first day of January, or within six months after payment of corresponding premium, obtained
thereafter. ISSUE from the Filipinas Cia. de Seguros a fire policy in the
Disposition The petition is PARTIALLY GRANTED. 1. WON Pandiman may be held liable for the death sum of P1000,000, covering merchandise contained
The Decision dated July 30, 2002 of the Court of benefits in No. 711 Roman Street, Binondo Manila.
Appeals affirming the Decision dated May 3, 2000 of 2. WON Marine and its foreign principal, Fullwin, - February 27, 1942 or during the Japanese military
the Insurance Commission is hereby REVERSED AND should be absolved from the death claim liabilities occupation - building and insured merchandise were
SET ASIDE. The Steamship Mutual Underwriting burned. In due time the Huenefeld Co submitted to
Association (Bermuda) Ltd., and Pioneer Insurance HELD the Filipinas Cia its claim under the policy. The
and Surety Corporation are ORDERED to obtain 1. NO salvage goods were sold at public auction and, after
licenses and to secure proper authorizations to do - Pandiman is not an insurance agent as defined by deducting their value, the total loss suffered by the
business as insurer and insurance agent, Section 3007 of the Insurance Code. In this case, respondent was fixed at P92,650.
respectively. The petitioner’s prayer for the there was no showing that Pndiman in fact - Filipinas Cia refused to pay the claim on the ground
revocation of Pioneer’s Certificate of Authority and negotiated the insurance contract between Sun that the policy in favor of the respondent had ceased
removal of its directors and officers, is DENIED. Richie Five and the insurer OMMIAL. Even, if to be in force on the date the United States declared
Pandiman were an agent, payment for claims arising war against Germany, the respondent Corporation
PANDIMAN v. MARINE MANNING MNGT CORP. from peril insured against, to which the insurer is (though organized under and by virtue of the laws of
460 SCRA 418 liable, is definitely not one of the liabilities of an the Philippines) being controlled by the German
GARCIA; June 21, 2005 insurance agent. Thus, there is no legal basis subjects and the Filipinas Cia being a company under
whatsoever for holding petitioner solidarily liable with American jurisdiction when said policy was issued on
NATURE insurer OMMIAL for the widow’s claim for death October 1, 1941. Filipinas Cia, however, in
Petition for certiorari to review CA decision benefits. Also, Pandiman is not a party to the pursuance of the order of the Director of Bureau of
insurance contract and hence under Article 1311 of Financing, Philippine Executive Commission, dated
FACTS the Civil Code, it is not liable for the obligation April 9, 1943, paid to the Huenefeld Co the sum of
- Benito Singhid was hired as chief cook on board the arising out of the insurance contract. P92,650 on April 19, 1943.
vessel MV Sun Richie Five for a term of one year by 2. NO - August 6, 1946 – action filed in CFI Manila to
Fullwin Maritime Limited through its Philippine agent, - Fullwin, as Benito’s principal employer is liable recover from the Huenefeld Co the sum of P92,650
Marine Manning and Management Corporation. While under the employment contract. Marine is also above mentioned. The theory of the Filipinas Cia is
the said vessel was on its way to Shanghai from Ho bound by its undertaking pursuant to the Rules and that the insured merchandise were burned up after
Chih Minh City, Benito suffered a heart attack and Regulations Governing Overseas Employment that “it the policy issued in 1941 in favor of Huenefeld Co
subsequently died on June 24, 1997. shall assume joint and solidary liability with the has ceased to be effective because of the outbreak of
- Apparently, the vessel and the crew were insured employer for all the claims and liabilities which may the war between the United States and Germany on
with Ocean Marine Mutual Insurance Association arise in connection with the implementation of the December 10, 1941, and that the payment made by
Limited (OMMIAL), a Protective and Indemnity Club contract, including but not limited to the payment of the Filipinas Cia to Huenefeld Co during the Japanese
of which Sun Richie Five Bulkers S.A. is a member. wages, heath and disability compensation and military occupation was under pressure.
Pandiman Philippines, the petitioner, is the local repatriation”. In other words, both Fullwin and - CFI: dismissed the action without pronouncement
correspondent of OMMIAL. Marine should be held liable for whatever death as to costs.
- Benito’s widow, Rosita, filed a claim for death benefits the widow of Benito may be entitled to. - CA: CFI judgment affirmed, with costs. The case is
benefits with Marine which referred her to Pandiman. Disposition The petition is granted and the CA now before us on appeal by certiorari from the
After her submission of the required documentation, decision is reversed and set aside. decision of the Court of Appeals.
Pandiman recommended payment of the death
benefits amounting to $79,000. However, payment 7
ISSUE
has not been made. Section 300. Any person who for compensation solicits or obtains on behalf of any insurance WON the policy in question became null and void
company transmits for a person other than himself an application for a policy or contract of
- Rosita filed a complaint with the Labor Arbiter insurance to or from such company or offers or assumes to act in the negotiating of such insurance upon the declaration of war between United States
naming Marine, Pandiman, OMMIAL, and Fullwin as shall be an insurance agent within the intent of this section and shall thereby become liable to all and Germany
the duties, requirements, liabilities, and penalties to which an insurance agent is subject.
INSURANCE Page 49

war against Germany, relying on English and petitioner for the unexpired term of the policy in
HELD American cases which held that a corporation is a question, beginning December 11, 1941.
YES citizen of the country or state by and under the laws
Ratio The Philippine Insurance Law (Act No. 2427, of which it was created or organized. It rejected the
as amended,) in section 8, provides that "anyone theory that nationality of private corporation is
except a public enemy may be insured." It stands to determined by the character or citizenship of its
reason that an insurance policy ceases to be controlling stockholders.
allowable as soon as an insured becomes a public - There is no question that majority of the
enemy. stockholders of the respondent corporation were
> Effect of war, generally. - All intercourse German subjects. Therefore, Huenefeld Co became INSULAR LIFE ASSURANCE CO. v. EBRADO
between citizens of belligerent powers which is an enemy corporation upon the outbreak of the war 80 SCRA 181
inconsistent with a state of war is prohibited by between the United States and Germany. The MARTIN; October 28, 1977
the law of nations. Such prohibition includes all English and American cases relied upon by the Court
negotiations, commerce, or trading with the of Appeals have lost their force in view of the latest NATURE
enemy; all acts which will increase, or tend to decision of the Supreme Court of the United States Appeal from judgment of RTC.
increase, its income or resources; all acts of in Clark vs. Uebersee Finanz Korporation, decided on
voluntary submission to it; or receiving its December 8, 1947, in which the controls test has FACTS
protection; also all acts concerning the been adopted. In "Enemy Corporation" by Martin - Buenaventura Ebrado obtained a whole-life
transmission of money or goods; and all contracts Domke, a paper presented to the Second insurance policy from Insular, for P5,882.00 with a
relating thereto are thereby nullified. It further International Conference of the Legal Profession held rider for accidental death benefits for the same
prohibits insurance upon trade with or by the at the Hague (Netherlands) in August. 1948 also amount. He designated Carponia Ebrado as the
enemy, upon the life or lives of aliens engaged in discussed this dilemma revocable beneficiary, referring to her as the wife.
service with the enemy; this for the reason that > In Clark vs. Uebersee Finanz Korporation, A. G., - Afterwards, he died as a result of an accident when
the subjects of one country cannot be permitted to dealing with a Swiss corporation allegedly he was hit by a falling branch of a tree. Carponia
lend their assistance to protect by insurance the controlled by German interest, the Court: "The filed a claim for the proceeds as the designated
commerce or property of belligerent, alien property of all foreign interest was placed within beneficiary in the policy, although she admits that
subjects, or to do anything detrimental too their the reach of the vesting power (of the Alien she and Buenaventura were merely living as
country's interest. The purpose of war is to cripple Property Custodian) not to appropriate friendly or husband and wife without the benefit of marriage.
the power and exhaust the resources of the neutral assets but to reach enemy interest which The legal wife, Pascuala Vda De Ebrado, also filed
enemy, and it is inconsistent that one country masqueraded under those innocent fronts. . . . The her claim as the widow of the deceased.
should destroy its enemy's property and repay in power of seizure and vesting was extended to all - Insular then filed an interpleader in court (CFI
insurance the value of what has been so property of any foreign country or national so that Rizal) to determine to whom the proceeds should be
destroyed, or that it should in such manner no innocent appearing device could become a paid. CFI declared that Carponia was disqualified
increase the resources of the enemy, or render it Trojan horse." from becoming beneficiary of the insured and
aid, and the commencement of war determines, - The respondent having become an enemy directing the Insular to pay the proceeds to the
for like reasons, all trading intercourse with the corporation on December 10, 1941, the insurance estate of Buenaventura.
enemy, which prior thereto may have been lawful. policy issued in its favor on October 1, 1941, by the
All individuals therefore, who compose the petitioner (a Philippine corporation) had ceased to be ISSUE
belligerent powers, exist, as to each other, in a valid and enforcible, and since the insured goods 1. WON a common-law wife named as beneficiary in
state of utter exclusion, and are public enemies. (6 were burned after December 10, 1941, and during the insurance policy of a legally married man claim
Couch, Cyc. of Ins. Law, pp. 5352-5353.) the war, the respondent was not entitled to any the proceeds of the same
> In the case of an ordinary fire policy, which indemnity under said policy from the petitioner.
grants insurance only from year, or for some other However, elementary rules of justice (in the absence HELD
specified term it is plain that when the parties of specific provision in the Insurance Law) require 1. NO
become alien enemies, the contractual tie is that the premium paid by the respondent for the Ratio The prohibition that husband and wife cannot
broken and the contractual rights of the parties, period covered by its policy from December 11, donate to each other applies to common-law
so far as not vested. lost. (Vance, the Law on 1941, should be returned by the petitioner. relationships. As the appointment of a beneficiary in
Insurance, Sec. 44, p. 112.) Disposition the appealed decision is hereby insurance may be considered a donation, one cannot
Reasoning reversed and the respondent corporation is ordered name as beneficiary his common-law wife.
- The Court of Appeals overruled the contention of to pay to the petitioner the sum of P77,208.33, Reasoning
the petitioner that the respondent corporation Philippine currency, less the amount of the premium,
became an enemy when the United States declared in Philippine currency, that should be returned by the
INSURANCE Page 50

- It is quite unfortunate that the Insurance Code CONSUEGRA v. GSIS Berdin, his widow by the second marriage and their
does not contain any specific provision grossly 37 SCRA 315 seven children, on the other hand, who are entitled
resolutory of the prime question at hand. ZALDIVAR; January 30, 1971 to the remaining one-half, or 8/16, each of them to
- Rather, general rules of civil law should be applied receive an equal share of 1/16.
to resolve the issue. Art.2011, CC states: “The NATURE - Dissatisfied with the foregoing ruling and
contract of insurance is governed by special laws. Appeal from the decision of the Court of First apportionment made by the GSIS, Basilia Berdin and
Matters not expressly provided for in such special Instance of Surigao del Norte awarding the 8/16 part her children filed on October 10, 1966 a petition for
laws shall be regulated by this Code.” Thus, when of the proceeds of the deceased Consuegra’s mandamus with preliminary injunction in the Court
not otherwise specifically provided for by the retirement benefits to Rosario Diaz. of First Instance of Surigao.
Insurance Law, the contract of life insurance is - The CFI of Surigao ruled in favor of respondent
governed by the general rules of the civil law FACTS Rosario Diaz and upheld the ruling of GSIS in all
regulating contracts. - The late Jose Consuegra, at the time of his death, aspect. Thus, Basilia Berdin and her children
- Also, Art.2012 “any person who is forbidden from was employed as a shop foreman of the office of the appealed said decision to the Supreme Court.
receiving any donation under Article 739 cannot be District Engineer in the province of Surigao del
named beneficiary of a life insurance policy by the Norte. In his lifetime, Consuegra contracted two ISSUE
person who cannot make a donation to him.” marriages, the first with herein respondent Rosario WON GSIS was correct in awarding half of the
Common-law spouses are, definitely, barred from Diaz, solemnized in the parish church of San Nicolas retirement benefit of the deceased to Rosario Diaz,
receiving donations from each other. de Tolentino, Surigao, Surigao, on July 15, 1937, out the first wife, notwithstanding the fact that the
- Art.739, CC: The following donations shall be void: of which marriage were born two children, namely, petitioners were named as beneficiaries of the life
1. Those made between persons who were guilty of Jose Consuegra, Jr. and Pedro Consuegra, but both insurance
adultery or concubinage at the time of donation; predeceased their father; and the second, which was
- In essence, a life insurance policy is no different contracted in good faith while the first marriage was HELD
from a civil donation insofar as the beneficiary is subsisting, with herein petitioner Basilia Berdin, on YES
concerned. Both are founded upon the same May 1, 1957 in the same parish and municipality, out - The GSIS offers two separate and distinct systems
consideration: liberality. A beneficiary is like a of which marriage were born seven children, namely, of benefits to its members, one is the life insurance
donee, because from the premiums of the policy Juliana, Pacita, Maria Lourdes, Jose, Rodrigo, Lenida and the other is the retirement insurance. These two
which the insured pays out of liberality, the and Luz, all surnamed Consuegra. distinct systems of benefits are paid out from two
beneficiary will receive the proceeds or profits of said - Being a member of the Government Service distinct and separate funds that are maintained by
insurance. As a consequence, the proscription in Insurance System (GSIS, for short) when Consuegra the GSIS. Thus, it doesn’t necessarily mean that the
Art.739 CC should equally operate in life insurance died on September 26, 1965, the proceeds of his life beneficiaries in the life insurance are also the
contracts. The mandate of Art.2012 cannot be laid insurance under policy No. 601801 were paid by the beneficiaries in the retirement insurance.
aside: any person who cannot receive a donation GSIS to petitioner Basilia Berdin and her children - Consuegra started in the government service
cannot be named as beneficiary in the life insurance who were the beneficiaries named in the policy. sometime during the early part of 1943, or before
policy of the person who cannot make the donation. - However, Consuegra did not designate any 1943. In 1943 Com. Act 186 was not yet amended,
- Policy considerations and dictates of morality beneficiary who would receive the retirement and the only benefits then provided for in said Com.
rightly justify the institution of a barrier between insurance benefits due to him. Respondent Rosario Act 186 were those that proceed from a life
common-law spouses in regard to property relations Diaz, the widow by the first marriage, filed a claim insurance. Upon entering the government service
since such relationship ultimately encroaches upon with the GSIS asking that the retirement insurance Consuegra became a compulsory member of the
the nuptial and filial rights of the legitimate family. benefits be paid to her as the only legal heir of GSIS, being automatically insured on his life,
There is every reason to hold that the bar in Consuegra, considering that the deceased did not pursuant to the provisions of Com. Act 186 which
donations between legitimate spouses and those designate any beneficiary with respect to his was in force at the time. During 1943 the operation
between illegitimate ones should be enforced in life retirement insurance benefits. Petitioner Basilia of the Government Service Insurance System was
insurance policies since the same are based on Berdin and her children, likewise, filed a similar claim suspended because of the war, and the operation
similar consideration. with the GSIS, asserting that being the beneficiaries was resumed sometime in 1946. When Consuegra
- So long as marriage remains the threshold of named in the life insurance policy of Consuegra, they designated his beneficiaries in his life insurance he
family laws, reason and morality dictate that the are the only ones entitled to receive the retirement could not have intended those beneficiaries of his life
impediments imposed upon married couple should insurance benefits due the deceased Consuegra. insurance as also the beneficiaries of his retirement
likewise be imposed upon extra-marital relationship. Resolving the conflicting claims, the GSIS ruled that insurance because the provisions on retirement
If legitimate relationship is circumscribed by these the legal heirs of the late Jose Consuegra were insurance under the GSIS came about only when
legal disabilities, with more reason should an illicit Rosario Diaz, his widow by his first marriage who is Com. Act 186 was amended by Rep. Act 660 on June
relationship be restricted by these disabilities. entitled to one-half, or 8/16, of the retirement 16, 1951. Hence, it cannot be said that because
Disposition Decision AFFIRMED. insurance benefits, on the one hand; and Basilia herein appellants were designated beneficiaries in
INSURANCE Page 51

Consuegra's life insurance they automatically FACTS protection of the employee as well as of his family,
became the beneficiaries also of his retirement - Petronilo Davac, became a member of the Social but this purpose or intention of the law cannot be
insurance. Security System (SSS for short) on September 1, enforced to the extent of contradicting the very
- The provisions of subsection (b) of Section 11 of 1957. In the Member's Record he designated provisions of said law contained in Section 13,
Commonwealth Act 186, as amended by Rep. Act respondent, Candelaria Davac as his beneficiary and thereof”
660, clearly indicate that there is need for the indicated his relationship to her as that of "wife". - When the provisions of a law are clear and explicit,
employee to file an application for retirement - He died on April 5, 1959. It appears that the the courts can do nothing but apply its clear and
insurance benefits when he becomes a member of deceased contracted two marriages, the first, with explicit provisions (Velasco vs. Lopez)
the GSIS, and he should state in his application the Lourdes Tuplano on August 29, 1946, who bore him 2. NO
beneficiary of his retirement insurance. Hence, the a child, Romeo Davac, and the second, with - The disqualification mentioned in Article 739 is not
beneficiary named in the life insurance does not Candelaria Davac on January 18, 1949, with whom applicable to herein appellee Candelaria Davac
automatically become the beneficiary in the he had a minor daughter, Elizabeth Davac. Both filed because she was not guilty of concubinage, there
retirement insurance unless the same beneficiary in their claims for death benefit with the SSS. being no proof that she had knowledge of the
the life insurance is so designated in the application - Social Security Commission issued the resolution previous marriage of her husband Petronilo.
for retirement insurance. declaring respondent Candelaria Davac as the person ART. 2012. Any person who is forbidden from
- In the case of the proceeds of a life insurance, the entitled to receive the death benefits payable for the receiving any donation under Article 739 cannot be
same are paid to whoever is named the beneficiary death of Petronilo Davac. named beneficiary of a life insurance policy by the
in the life insurance policy. As in the case of a life person who cannot make any donation to him
insurance provided for in the Insurance Act, the ISSUES according to said article.
beneficiary in a life insurance under the GSIS may 1. WON the Social Security Commission Candelaria ART. 739. The following donations shall be void:
not necessarily be an heir of the insured. The Davac is entitled to receive the death benefits (1) Those made between persons who were guilty
insured in a life insurance may designate any 2. WON a beneficiary under the Social Security of adultery or concubinage at the time of the
person as beneficiary unless disqualified to be System partakes of the nature of a beneficiary in a donation; (the court did not decide whether this
so under the provisions of the Civil Code. And in life insurance policy and, therefore the designation partakes the nature of a life insurance policy)
the absence of any beneficiary named in the life made in the person DAVAC as bigamous wife is null 3. NO
insurance policy, the proceeds of the insurance will and void, because it contravenes the provisions of - The benefit receivable under the Act is in the
go to the estate of the insured. the Civil Code nature of a special privilege or an arrangement
- On the other hand, the beneficiary of the 3. WON the benefits accruing from membership with secured by the law pursuant to the policy of the
retirement insurance can only claim the proceeds of SSS forms part of the conjugal property thus the State to provide social security to the workingmen.
the retirement insurance if the employee dies before resolution deprives the lawful wife of her share in the The amounts that may thus be received cannot be
retirement. If the employee failed or overlooked to conjugal property as well as of her own and her considered as property earned by the member
state the beneficiary of his retirement insurance, the child's legitime in the inheritance during his lifetime. His contribution to the fund
retirement benefits will accrue to his estate and will constitutes only an insignificant portion thereof.
be given to his legal heirs in accordance with law, as HELD Then, the benefits are specifically declared not
in the case of a life insurance if no beneficiary is 1. YES transferable, and exempted from tax, legal
named in the insurance policy. - Section 13, RA1161 provides that the beneficiary processes, and lien. Furthermore, in the settlement
Disposition Petition Denied. It is Our view, "as recorded" by the employee's employer is the one of claims thereunder, the procedure to be observed
therefore, that the respondent GSIS had correctly entitled to the death benefits. is governed not by the general provisions of law, but
acted when it ruled that the proceeds of the - Section 13, Republic Act No. 1161, as amended by by rules and regulations promulgated by the
retirement insurance of the late Jose Consuegra Republic Act No. 1792, in force at the time of Commission. Thus, if the money is payable to the
should be divided equally between his first living wife Petronilo Davac's death provides: Upon the covered estate of a deceased member, it is the Commission,
Rosario Diaz, on the one hand, and his second wife employee's death or total and permanent disability not the probate or regular court that determines the
Basilia Berdin and his children by her. under such conditions as the Commission may person or persons to whom it is payable.
define, before becoming eligible for retirement and if - They are disbursed from a public special fund
either such death or disability is not compensable created by Congress.The sources of this special fund
SSS v. DAVAC
under the Workmen's Compensation Act, he or. in are the covered employee's contribution (equal to 2-
17 SCRA 863
case of his death, his beneficiaries, as recorded by 1/2 per cent of the employee's monthly
BARRERA: July 30, 1966
his employer shall be entitled to the following compensation) ; the employer's 'Contribution
benefit: (equivalent to 3-1/2 per cent of the monthly
NATURE
- In Tecson vs. Social Security System. Section 13 compensation of the covered employee) ;and the
APPEAL from a resolution Of the Social Security
was construed:"it may be true that the purpose of Government contribution which consists in yearly
Commission.
the coverage under the Social Security System is appropriation of public funds to assure the
INSURANCE Page 52

maintenance of an adequate working balance of the deceased; that the defendant account for 1. YES
funds of the System. Additionally, Section 21 of the P21,634.80, and that the sum be divided equally - The SC agreed with the finding of the trial court
Social Security Actprovides that the benefits among the plaintiffs and defendant along with the that the proceeds of the life-insurance policy belong
prescribed in this Act shall not be diminished and the other property of deceased. exclusively to the defendant as his individual and
Government of the Republic of the Philippines - The defendant denies the material allegations of separate property, we agree. That the proceeds of
accepts general responsibility for the solvency of the the complaint and sets up as special defense and an insurance policy belong exclusively to the
System. counterclaim that the redemption of the real estate beneficiary and not to the estate of the person
- The benefits under the Social Security Act are not sold by his father was made in the name of the whose life was insured, and that such proceeds are
intended by the lawmaking body to form part of the plaintiffs and himself instead of in his name alone the separate and individual property of the
estate of the covered –members. without his knowledge or consent. Andres contends beneficiary, and not of the heirs of the person whose
- Social Security Act is not a law of succession. that it was not his intention to use the proceeds of life was insured, is the doctrine in America. We
Disposition Resolution of the Social Security the insurance policy for the benefit of any person but believe that the same doctrine obtains in these
Commission appealed is affirmed himself, he alleging that he was and is the sole Islands by virtue of section 428 of the Code of
owner thereof and that it is his individual property. Commerce, which reads:
FRANCISCO DEL VAL v. ANDRES DEL VAL He, therefore, asks that he be declared the owner of "The amounts which the underwriter must deliver
29 PHIL 534 the real estate redeemed by the payment of the to the person insured, in fulfillment of the
MORELAND; February 16, 1915 P18,365.20, the owner of the remaining P21,634.80, contract, shall be the property of the latter, even
the balance of the insurance policy, and that the against the claims of the legitimate heirs or
NATURE plaintiffs account for the use and occupation of the creditors of any kind whatsoever of the person
Appeal from a judgment of the Court of First premises so redeemed since the date of the who effected the insurance in favor of the former."
Instance of the city of Manila dismissing the redemption. 2. NO
complaint with costs. - The trial court refused to give relief to either party - The contract of life insurance is a special contract
and dismissed the action. In this appeal, it is claimed and the destination of the proceeds thereof is
FACTS by the attorney for the plaintiffs that insurance determined by special laws which deal exclusively
- Plaintiffs and defendant are brothers and sisters; provisions in the Code of Commerce are with that subject. The Civil Code has no provisions
that they are the only heirs at law and next of kin of subordinated to the provisions of the Civil Code as which relate directly and specifically to life-insurance
Gregorio Nacianceno del Val, who died in Manila on found in article 1035. This article reads: contracts or to the destination of life insurance
August 4, 1910, intestate "An heir by force of law surviving with others of the proceeds. That subject is regulated exclusively by
- During the lifetime of the deceased he took out same character to a succession must bring into the the Code of Commerce which provides for the terms
insurance on his life for the sum of P40,000 and hereditary estate the property or securities he may of the contract, the relations of the parties and the
made it payable to the defendant ANDRES DEL VAL have received from the deceased during the life of destination of the proceeds of the policy.
as sole beneficiary. After his death the defendant the same, by way of dowry, gift, or for any good - Assuming that the proceeds of the life-insurance
collected the face of the policy. From said policy he consideration, in order to compute it in fixing the policy being the exclusive property of the defendant
paid the sum of P18,365.20 to redeem certain real legal portions and in the account of the division." and he having used a portion thereof in the
estate which the decedent had sold to third persons - Counsel also claims that the proceeds of the repurchase of the real estate sold by the decedent
with a right to repurchase. insurance policy were a donation or gift made by the prior to his death with right to repurchase, and such
- The redemption of said premises was made by the father during his lifetime to the defendant and that, repurchase having been made and the conveyance
attorney of defendant ANDRES in the name of the as such, its ultimate destination is determined by taken in the names of all of the heirs instead of the
plaintiffs and the defendant as heirs of the deceased those provisions of the Civil Code which relate to defendant alone, plaintiffs claim that the property
vendor. It further appears from the pleadings that donations, especially article 819. This article provides belongs to the heirs in common and not to the
the defendant, on the death of the deceased, took that "gifts made to children which are not defendant alone.
possession of most of his personal property, which betterments shall be considered as part of their legal - The Court rejected this contention unless the fact
he still has in his possession, and that he has also portion." appear or be shown that the defendant acted as he
the balance on said insurance policy amounting to did with the intention that the other heirs should
P21,634.80. ISSUES enjoy with him the ownership of the estate ---- in
- Plaintiffs contend that the amount of the insurance 1. WON the insurance belongs to the defendant and other words, that he proposed, in effect, to make a
policy belonged to the estate of the deceased and not to the decedent’s estate gift of the real estate to the other heirs. If it is
not to the defendant personally; that, therefore, they 2. WON the Civil code provisions on succession established by the evidence that was his intention
are entitled to a partition not only of the real and prevail over any other law with respect to the and that the real estate was delivered to the
personal property, but also of the P40,000 life insurance plaintiffs with that understanding, then it is probable
insurance. The complaint prays a partition of all the that their contention is correct and that they are
property, both real and personal, left by the HELD entitled to share equally with the defendant therein.
INSURANCE Page 53

If, however, it appears from the evidence in the case - On March 4, 1922, Hilario Gercio formally notified and specifically to life-insurance contracts or to the
that the conveyances were taken in the name of the the Sun Life that he had revoked his donation in destination of life-insurance proceeds. . . ."
plaintiffs without his knowledge or consent, or that it favor of Andrea Zialcita, and that he had designated - Insurance Act- there is likewise no provision
was not his intention to make a gift to them of the in her stead his present wife, Adela Garcia de Gercio, either permitting or prohibiting the insured to change
real estate, then it belongs to him. If the facts are as as the beneficiary of the policy. Gercio requested the the beneficiary.
stated, he has two remedies. The one is to compel insurance company to eliminate Andrea Zialcita as 2. NO
the plaintiffs to reconvey to him and the other is to beneficiary. This, the insurance company has refused Ratio The wife has an insurable interest in the life of
let the title stand with them and to recover from and still refuses to do. her husband. The beneficiary has an absolute vested
them the sum he paid on their behalf. interest in the policy from the date of its issuance
- For the complete and proper determination of the ISSUES and delivery. So when a policy of life insurance is
questions at issue in this case, the Court was of the 1. (Preliminary) WON the provisions of the Code of taken out by the husband in which the wife is named
opinion that the cause should be returned to the trial Commerce and the Civil Code shall be in force in as beneficiary, she has a subsisting interest in the
court with instructions to permit the parties to frame 1910, or the provisions of the Insurance Act now in policy. And this applies to a policy to which there are
such issues as will permit the settlement of all the force, or the general principles of law, guide the attached the incidents of a loan value, cash
questions involved and to introduce such evidence as court in its decision surrender value, an automatic extension by
may be necessary for the full determination of the 2. WON the insured, the husband, has the power to premiums paid, and to an endowment policy, as well
issues framed. Upon such issues and evidence taken change the beneficiary, the former wife, and to name as to an ordinary life insurance policy. If the husband
thereunder the court will decide the questions instead his actual wife, where the insured and the wishes to retain to himself the control and ownership
involved according to the evidence, subordinating his beneficiary have been divorced and where the policy of the policy he may so provide in the policy. But if
conclusions of law to the rules laid down in this of insurance does not expressly reserve to the the policy contains no provision authorizing a change
opinion. REMANDED. insured the right to change the beneficiary of beneficiary without the beneficiary's consent, the
insured cannot make such change. Accordingly, it is
GERCIO v. SUN LIFE ASSURANCE OF CANADA HELD held that a life insurance policy of a husband made
48 PHIL 53 1. Whether the case be considered in the light of the payable to the wife as beneficiary, is the separate
MALCOLM; September 28, 1925 Code of Commerce, the Civil Code, or the Insurance property of the beneficiary and beyond the control of
Act, the deficiencies in the law will have to be the husband.
NATURE supplemented by the general principles prevailing on - Unlike the statutes of a few jurisdictions, there is
Mandamus to compel Sun Life Assurance Co. of the subject. To that end, we have gathered the rules no provision in the Philippine Law permitting the
Canada to change the beneficiary in the policy issued which follow from the best considered American beneficiary in a policy for the benefit of the wife of
by the defendant company on the life of the plaintiff authorities. In adopting these rules, we do so with the husband to be changed after a divorce. It must
Hilario Gercio the purpose of having the Philippine Law of follow, therefore, in the absence of a statute to the
Insurance conform as nearly as possible to the contrary, that if a policy is taken out upon a
FACTS modern Law of Insurance as found in the United husband's life the wife is named as beneficiary
- On January 29, 1910, the Sun Life Assurance Co. of States proper. therein, a subsequent divorce does not destroy her
Canada issued an insurance policy on the life of - Court’s first duty is to determine what law should rights under the policy.
Hilario Gercio. The policy was what is known as a 20- be applied to the facts. The insurance policy was Reasoning
year endowment policy. By its terms, the insurance taken out in 1910, that the Insurance Act. No. 2427, - Yore vs. Booth
company agreed to insure the life of Hilario Gercio became effective in 1914, and that the effort to “. . . It seems to be the settled doctrine, with but
for the sum of P2,000, to be paid him on February 1, change the beneficiary was made in 1922. slight dissent in the courts of this country, that a
1930, or if the insured should die before said date, - Code of Commerce- there can be found in it no person who procures a policy upon his own life,
then to his wife, Mrs. Andrea Zialcita, should she provision either permitting or prohibiting the insured payable to a designated beneficiary, although he
survive him; otherwise to the executors, to change the beneficiary. pays the premiums himself, and keeps the policy
administrators, or assigns of the insured. The policy - Civil Code- it would be most difficult, if indeed it is in his exclusive possession, has no power to
did not include any provision reserving to the insured practicable, to test a life insurance policy by its change the beneficiary, unless the policy itself, or
the right to change the beneficiary. provisions. In the case of Del Val vs. Del Val, it the charter of the insurance company, so provides.
- On the date the policy was issued, Andrea Zialcita declined to consider the proceeds of the insurance In policy, although he has parted with nothing,
was the lawful wife of Hilario Gercio. Towards the policy as a donation or gift, saying "the contract of and is simply the object of another's bounty, has
end of the year 1919, she was convicted of the crime life insurance is a special contract and the acquired a vested and irrevocable interest in the
of adultery. On September 4, 1920, a decree of destination of the proceeds thereof is determined by policy, which he may keep alive for his own benefit
divorce was issued in civil case no. 17955, which had special laws which deal exclusively with that subject. by paying the premiums or assessments if the
the effect of completely dissolving their bonds of The Civil Code has no provisions which relate directly person who effected the insurance fails or refuses
matrimony to do so.”
INSURANCE Page 54

- Connecticut Mutual Life Insurance Company her husband, and he was thereafter again married - In 1968, Private Respondent Rodolfo Dimayuga
vs Schaefer to one who sustained the relation of wife to him at procured an ordinary life insurance policy from the
“We do not hesitate to say, however, that a policy the time of his death. petitioner company and designated his wife and
taken out in good faith and valid at its inception, is The rights of a beneficiary in an ordinary life children as irrevocable beneficiaries. On Feb. 22,
not avoided by the cessation of the insurable insurance policy become vested upon the issuance 1980, Dimayuga filed with the CFI a petition to
interest, unless such be the necessary effect of the of the policy, and can thereafter, during the life of amend the designation of the beneficiaries in his life
provisions of the policy itself.. . . .In our judgment the beneficiary, be defeated only as provided by policy from irrevocable to revocable. Petitioner filed
of life policy, originally valid, does not cease to be the terms of the policy.” an Urgent Motion to reset hearing as well as its
so by the cessation of the assured party's interest - On the admitted facts and the authorities comment and/or Opposition to the respondent’s
in the life insured.” supporting the nearly universally accepted principles petition.
- Central National Bank of Washington City vs. of insurance, we are irresistibly led to the conclusion - Respondent Judge denied petitioner’s Urgent
Hume that the question at issue must be answered in the Motion, thus allowing private respondent to adduce
“It is indeed the general rule that a policy, and the negative evidence, the consequence of which was the
money to become due under it, belong, the Disposition The judgment appealed from will be issuance of the questioned Order granting the
moment it is issued, to the person or persons reversed and the complaint ordered dismissed as to petition. Petitioner then filed a MFR which was also
named in it as the beneficiary or beneficiaries, and the appellant. denied hence this petition.
that there is no power in the person procuring the
insurance, by any act of his, by deed or by will, to SEPARATE OPINION ISSUE
transfer to any other person the interest of the 1. WON the designation of the irrevocable
person named.” JOHNSON [concur] beneficiaries could be changed or amended without
- In re Dreuil & Co. - I agree with the majority of the court, that the the consent of all the irrevocable beneficiaries
“In so far as the law of Louisiana is concerned, it judgment of the lower court should be revoked, but 2. WON the irrevocable beneficiaries herein, one of
may also be considered settled that where a policy for a different reason. The purpose of the petition is whom is already deceased while the others are all
is of the semitontine variety, as in this case, the to have declared the rights of certain persons in an minors could validly give consent to the change or
beneficiary has a vested right in the policy, of insurance policy which is not yet due and payable. It amendment in the designation of the irrevocable
which she cannot be deprived without her may never become due and payable. The premiums beneficiaries
consent” may not be paid, thereby rendering the contract of
- Wallace vs Mutual Benefit Life Insurance Co. insurance of non effect, and many other things may HELD
“As soon as the policy was issued Mrs. Wallace occur, before the policy becomes due, which would 1. NO
acquired a vested interest therein, of which she render it non effective. The plaintiff and the other - Based on the provision of their contract and the law
could not be deprived without her consent, except parties who are claiming an interest in said policy applicable, it is only with the consent of all the
under the terms of the contract with the insurance should wait until there is something due them under beneficiaries that any change or amendment in the
company. No right to change the beneficiary was the same. For the courts to declare now who are the policy concerning the irrevocable beneficiaries may
reserved. Her interest in the policy was her persons entitled to receive the amounts due, if they be legally and validly effected. Both the law and the
individual property, subject to be divested only by ever become due and payable, is impossible, for the Policy do not provide for any other exception.
her death, the lapse of time, or by the failure of reason that nothing may ever become payable under Reasoning
the insured to pay the premiums. She could keep the contract of insurance, and for many reasons such - Since the policy was procured in 1968, the
the policy alive by paying the premiums, if the persons may never have a right to receive anything applicable law in this case is the Insurance Act and
insured did not do so. It was contingent upon when the policy does become due and payable. In under that law, the beneficiary designated in a life
these events, but it was free from the control of my judgment, the action is premature and should insurance contract cannot be changed without the
her husband. He had no interest in her property in have been dismissed. consent of the beneficiary because he has a vested
this policy, contingent or otherwise. Her interest interest in the policy.
was free from any claim on the part of the insured PHIL. AMERICAN LIFE INSURANCE v. PINEDA - The Beneficiary Designation Indorsement in the
or his creditors. He could deprive her of her 175 SCRA 416 policy in the name of Dimayuga states that the
interest absolutely in but one way, by living more PARAS; July 19, 1989 designation of the beneficiaries is irrevocable: “no
than twenty years.” right or privilege under the Policy may be exercised,
- Filley vs. Illinois Life Insurance Company NATURE or agreement made with the Company to any
“The benefit accruing from a policy of life Petition for review on certiorari the orders of CFI change in or amendment to the Policy, without the
insurance upon the life of a married man, payable Judge Pineda consent of the said beneficiary/beneficiaries.”
upon his death to his wife, naming her, is payable - Contracts which are the private laws of the
to the surviving beneficiary named, although she FACTS contracting parties should be fulfilled according to
may have years thereafter secured a divorce from the literal sense of their stipulations, if their terms
INSURANCE Page 55

are clear and leave no room for doubt as to the "cash surrender value," indicated in an annexed the debtor, not exempt by law from execution, with
intention of the contracting parties, for contracts are table; but inasmuch as no more than two premiums all deeds, books and papers relating thereto; and
obligatory, no matter in what form they may be, had been paid upon the policy now in question up to while this language is broad, it nevertheless lacks
whenever the essential requisites for their validity the time of the death of the assured, this provision the comprehensiveness of section 70 (a) of the
are present. had not become effective; and it does not appear American Bankruptcy Law of 1898 in at least two
- Finally, the fact that the contract of insurance does that the company would in accordance with its own particulars; for under subsection 3 of section 70 (a)
not contain a contingency when the change in the usage or otherwise have made any concession to the of the last mentioned law, the trustee in bankruptcy
designation of beneficiaries could be validly effected assured in the event he had desired, before his acquires the right to exercise any powers which the
means that it was never within the contemplation of death, to surrender the policy. It must therefore be insolvent might have exercised for his own benefit,
the parties. accepted that this policy had no cash surrender and under subsection 5 the trustee acquires any
2. NO value, at the time of the assured's death, either by property of the insolvent which the latter could by
- The parent-insured cannot exercise rights and/or contract or by convention practice of the company in any means have assigned to another. The Insolvency
privileges pertaining to the insurance contract, for such cases. Law here in force, in common with the predecessor
otherwise, the vested rights of the irrevocable - Both Ingersoll, as assignee, and Tan Sit, as laws above-mentioned, contains nothing similar to
beneficiaries would be rendered inconsequential. The administratix of Dy Poco's estate, asserted claims to these provisions.
alleged acquiescence of the 6 children beneficiaries the proceeds of the policy. The lower court found On the applicability of the Insolvency Law
cannot be considered an effective ratification to the that Ingersoll had a better right and ordered Sun Life - Sec 32 of the Insolvency Law among other things,
change of the beneficiaries from irrevocable to to pay the insurance proceeds to him. declares that the assignment to be made by the clerk
revocable. They were minors at the time, and could of the court "shall operate to vest in the assignee all
not validly give consent. Neither could they act ISSUE of the estate of the insolvent debtor not exempt by
through their father-insured since their interests are WON Ingersoll, as assignee, has a right to the law from execution." Moreover, by section 24, the
quite divergent from one another. proceeds of the insurance court is required, upon making an order adjudicating
Disposition questioned Orders of respondent judge any person insolvent, to stay any civil proceedings
are nullified and set aside. HELD pending against him; and it is declared in section 60
NO that no creditor whose debt is provable under the Act
SUN LIFE ASSURANCE v. INGERSOLL On the Philippine Insolvency Law (Act No. 1956) shall be allowed, after the commencement of
41 PHIL 331 - The property and interests of the insolvent which proceedings in insolvency, to prosecute to final
STREET; November 8, 1921 become vested in the assignee of the insolvent are judgment any action therefor against the debtor. In
specified in section 32 of the Insolvency Law which connection with the foregoing may be mentioned
NATURE reads as follows: subsections 1 and 2 of section 36, as well as the
Action of interpleader "SEC. 32. As soon as an assignee is elected opening words of section 33, to the effect that the
or appointed and qualified, the clerk of the court assignee shall have the right and power to recover
FACTS shall, by an instrument under his hand and seal of and to take into his possession, all of the estate,
- April 16, 1918, Sun Life Assurance Company of the court, assign and convey to the assignee all the assets, and claims belonging to the insolvent, except
Canada (Sun Life), in consideration of the payment real and personal property, estate, and effects of the such as are exempt by law from execution.
of a stipulated annual premium during the period of debtor with all his deeds, books, and papers relating - These provisions clearly evince an intention to vest
the policy, or until the premiums had been thereto, and such assignment shall relate back to the in the assignee, for the benefit of all the creditors of
completely paid for twenty years, issued a policy of commencement of the proceedings in insolvency, the insolvent, such elements of property and
insurance on the life of Dy Poco for US$12,500, and shall relate back to the acts upon which the property right as could be reached and subjected by
payable to the said assured or his assigns on the adjudication was founded, and by operation of law process of law by any single creditor suing alone.
21st day of February, 1938, and if he should die shall vest the title to all such property, estate, and And this is exactly as it should be: for it cannot be
before that date then to his legal representatives. effects in the assignee, although the same is then supposed that the Legislature would suppress the
- June 23, 1919, the assured, Dy Poco, was attached on mesne process, as the property of the right of action of every individual creditor upon the
adjudged an involuntary insolvent by the CFI Manila, debtor. Such assignment shall operate to vest in the adjudication of insolvency, and at the same time
and Frank B. Ingersoll was appointed assignee of his assignee all of the estate of the insolvent debtor not allow the insolvent debtor to retain anything subject
estate. exempt by law from execution." to the payment of his debts in a normal state of
- July 10, 1919, Dy Poco died, and on August 21, - the Insolvency Law is in great part a copy of the solvency.
1919, Tan Sit, was duly appointed as the Insolvency Act of California, enacted in 1895, though - "leviable assets" and "assets in insolvency" are
administratrix of his intestate estate. it contains a few provisions from the American practically coextensive terms. Hence, in determining
- By the terms of the policy it was provided that after Bankruptcy Law of 1898 what elements of value constitute assets in
the payment of three full premiums, the assured - Under each of said laws the assignee acquires all insolvency, SC is at liberty to consider what
could surrender the policy to the company for a the real and personal property, estate, and effects of
INSURANCE Page 56

elements of value are subject to be taken upon paid is legally the sole property of the company, still practically prohibitive to many. Insolvency is a
execution, and vice versa. in practical effect, though not in law, it is moneys of disaster likely to overtake men in mature life; and
On whether a policy of insurance having no cash the assured deposited with the company in advance one who has gone through the process of bankruptcy
surrender value, but payable to insured or his legal to make up the deficiency in later premiums to cover usually finds himself in his declining years with the
representative, is property that may be taken upon the annual cost of insurance, instead of being accumulated savings of years swept away and
execution against him. retained by the assured and paid by him to the earning power diminished. The courts are therefore
- Philippine laws declare no exemption with respect company in the shape of greatly-increased practically unanimous in refusing to permit the
to insurance policies; and this species of property is premiums, when the risk is greatest. It is the 'net assignee in insolvency to wrest from the insolvent a
not enumerated, in section 48 of the Insolvency Law, reserve' required by law to be kept by the company policy of insurance which contains in it no present
among items from the ownership of which the for the benefit of the assured, and to be maintained realizable assets.
assignee is excluded. Moreover, all life insurance to the credit of the policy. So long as the policy On the applicability of the Insolvency Law
policies are declared by law to be assignable, remains in force the company has not practically any - Sec 32 of the Insolvency Law among other things,
regardless of whether the assignee has an insurable beneficial interest in it, except as its custodian, with declares that the assignment to be made by the clerk
interest in the life of the insured or not (Insurance the obligation to maintain it unimpaired and suitably of the court "shall operate to vest in the assignee all
Act No. 2427, sec. 166). invested for the benefit of the insured. This is the of the estate of the insolvent debtor not exempt by
- SC has held that insurance policies having a practical, though not the legal, relation of the law from execution." Moreover, by section 24, the
present cash surrender value are subject to be taken company to this fund. "Upon the surrender of the court is required, upon making an order adjudicating
upon execution. (Misut Garcia vs. West Coast San policy before the death of the assured, the company, any person insolvent, to stay any civil proceedings
Francisco Life Ins. Co.) to be relieved from all responsibility for the increased pending against him; and it is declared in section 60
- a policy devoid of a cash surrender value cannot be risk, which is represented by this accumulating that no creditor whose debt is provable under the Act
either "leviable assets" or "assets in insolvency." reserve, could well afford to surrender a considerable shall be allowed, after the commencement of
- the assignee in insolvency acquired no beneficial part of it to the assured, or his representative. A proceedings in insolvency, to prosecute to final
interest in the policy of insurance in question; that return of a part in some form or other is now Usually judgment any action therefor against the debtor. In
its proceeds are not liable for any of the debts made." (In re McKinney) connection with the foregoing may be mentioned
provable against the insolvent in the pending - the stipulation providing for a cash surrender value subsections 1 and 2 of section 36, as well as the
proceedings, and that said proceeds should therefore is a comparatively recent innovation in life insurance. opening words of section 33, to the effect that the
be delivered to his administratrix. Formerly the contracts provided — as they still assignee shall have the right and power to recover
On applicable US case commonly do in the policies issued by fraternal and to take into his possession, all of the estate,
- In re McKinney: no beneficial interest in the policy organizations and benefit societies — for the assets, and claims belonging to the insolvent, except
had ever passed to the assignee over and beyond payment of a premium sufficient to keep the such as are exempt by law from execution.
what constituted the surrender value, and that the estimated risk covered; and in case of a lapse the - These provisions clearly evince an intention to vest
legal title to the policy was vested in the assignee policy-holder received nothing. Furthermore, the in the assignee, for the benefit of all the creditors of
merely in order to make the surrender value- practice is common among insurance companies the insolvent, such elements of property and
available to him. The assignee should surrender the even now to concede nothing in the character of property right as could be reached and subjected by
policy upon the payment to him of said value, as he cash surrender value, until three full premiums have process of law by any single creditor suing alone.
was in fact directed to do. The assignee in been paid, as in this case. And this is exactly as it should be: for it cannot be
bankruptcy had no right to keep the estate unsettled - CONLUSION (from this case and other English and supposed that the Legislature would suppress the
for an indefinite period, for the mere purpose of American cases cited following the same opinion): right of action of every individual creditor upon the
speculating upon the chances of the bankrupt's the assignee acquires no beneficial interest in adjudication of insolvency, and at the same time
death. As regards everything beyond the surrender insurance effected on the life of the insolvent, except allow the insolvent debtor to retain anything subject
value, the assignee in bankruptcy would, after the to the extent that such insurance contains assets to the payment of his debts in a normal state of
discharge of the bankrupt, have no insurable interest which can be realized upon as of the date when the solvency.
in the life of the bankrupt. petition of insolvency is filed. The explanation is to - "leviable assets" and "assets in insolvency" are
- surrender value of a policy "arises from the fact be found in the consideration that the destruction of practically coextensive terms. Hence, in determining
that the fixed annual premiums is much in excess of a contract of life insurance is not only highly what elements of value constitute assets in
the annual risk during the earlier years of the policy, prejudicial to the insured and those dependent upon insolvency, SC is at liberty to consider what
an excess made necessary in order to balance the him, but is inimical to the interests of society. elements of value are subject to be taken upon
deficiency of the same premium to meet the annual Insurance is a species of property that should be execution, and vice versa.
risk during the latter years of the policy. This excess conserved and not dissipated. As is well known, life On whether a policy of insurance having no cash
in the premium paid over the annual cost of insurance is increasingly difficult to obtain with surrender value, but payable to the insured or his
insurance, with accumulations of interest, constitutes advancing years, and even when procurable after the legal representative, is property that may be taken
the surrender value. Though this excess of premiums age of fifty, the cost is then so great as to be upon execution against him.
INSURANCE Page 57

- Philippine laws declare no exemption with respect Insular Life denied his application for reinstatement `affirmative’ defense. The duty to establish such a
to insurance policies; and this species of property is of his lapsed life insurance policy defense by satisfactory and convincing evidence
not enumerated, in section 48 of the Insolvency Law, 2) he gave the appellant's medical examiner false rests upon the defendant. The evidence before the
among items from the ownership of which the and misleading information as to his ailment and Court does not clearly and satisfactorily establish
assignee is excluded. Moreover, all life insurance previous operation when he said he was that defense."
policies are declared by law to be assignable, “operated on for a Tumor [mayoma] of the -Kwong Nam had informed the appellant's medical
regardless of whether the assignee has an insurable stomach… associated with ulcer of stomach. Tumor examiner that the tumor for which he was operated
interest in the life of the insured or not (Insurance taken out was hard and of a hen's egg size. on was ''associated with ulcer of the stomach." In
Act No. 2427, sec. 166). Operation was two years ago in Chinese General the absence of evidence that the insured had
- SC has held that insurance policies having a Hospital by Dr. Yap. Claims he is completely sufficient medical knowledge as to enable him to
present cash surrender value are subject to be taken recovered.” Medical report show that insured was distinguish between "peptic ulcer" and "a tumor", his
upon execution. (Misut Garcia vs. West Coast San operated on for "peptic ulcer", involving the excision statement that said tumor was "associated with ulcer
Francisco Life Ins. Co., 41 Phil., 258.) of a portion of the stomach, not tumor. of the stomach" should be construed as an
- a policy devoid of a cash surrender value cannot be expression made in good faith of his belief as to the
either "leviable assets" or "assets in insolvency." ISSUE nature of his ailment and operation. Indeed, such
- the assignee in insolvency acquired no beneficial WON there was concealment (Was appellant, statement must be presumed to have been made by
interest in the policy of insurance in question; that because of insured's aforesaid representation, misled him without knowledge of its incorrectness and
its proceeds are not liable for any of the debts or deceived into entering the contract or in accepting without any deliberate intent on his part to mislead
provable against the insolvent in the pending the risk at the rate of premium agreed upon?) the appellant.
proceedings, and that said proceeds should therefore 3) Waiver:
be delivered to his administratrix. HELD While it may be conceded that, from the viewpoint of
Disposition Judgment reversed. Sun Life is directed NO a medical expert, the information communicated was
to pay the proceeds of the policy to Tan Sit. -"concealment exists where the assured had imperfect, the same was nevertheless sufficient to
knowledge of a fact material to the risk, and have induced appellant to make further inquiries
CHAPTER VI – RESCISSION OF INSURANCE honesty, good faith, and fair dealing requires that he about the ailment and operation of the insured.
CONTRACTS: CONCEALMENT, should communicate it to the assurer, but he Section 32 of Insurance Law [Act No. 2427]
MISREPRESENTATION, & BREACH OF designedly and intentionally withholds the same." provides:
WARRANTIES - It has also been held "that the concealment must, “The right to information of material facts may be
in the absence of inquiries, be not only material, but waived either by the terms of insurance or by
NG v. ASIAN CRUSADER LIFE ASSURANCE CORP fraudulent, or the fact must have been intentionally neglect to make inquiries as to such facts where they
122 SCRA 461 withheld." are distinctly implied in other facts of which
ESCOLIN; May 30, 1983 Reasoning information is communicated.”
1) The evidence shows that the Insular Life It has been held that where, "upon the face of the
FACTS Assurance Co., Ltd. approved Kwong Nam's request application, a question appears to be not answered
- On May 12, 1962, Kwong Nam applied for a 20- for reinstatement and amendment of his lapsed at all or to be imperfectly answered, and the insurers
year endowment insurance on his life for the sum of insurance policy on April 24, 1962…. It results, issue a policy without any further inquiry, they waive
P20,000, with his wife, Ng Gan Zee, as beneficiary. therefore, that when on May 12, 1962 Kwong Nam the imperfection of the answer and render the
- He died on Dec 1963 of cancer of the liver with answered `No' to the question whether any life omission to answer more fully immaterial.
metastasis. All premiums had been paid at the time insurance company ever refused his application for Disposition the judgment appealed from is hereby
of his death. reinstatement of a lapsed policy he did not affirmed, with costs against appellant
- Ng presented a claim for payment of the face value misrepresent any fact.
of the policy. Appellant (Asian Crusader) denied the 2) Assuming that the aforesaid answer given by the CANILANG v. CA (GREAT PACIFIC LIFE
claim on the ground that the answers given by the insured is false, Sec. 278 of the Insurance Law ASSURANCE CORP.)
insured to the questions appearing in his application nevertheless requires that fraudulent intent on the 223 SCRA 443
for life insurance were untrue. part of the insured be established to entitle the FELICIANO; June 17, 1993
-Appellant: the insured was guilty of insurer to rescind the contract. And as correctly
misrepresentation when observed by the lower court, "misrepresentation as a NATURE
1) he answered "No" to the question (in the defense of the insurer to avoid liability is an Petition for review on certiorari of the decision of the
application) of "Has any life insurance company ever Court of Appeals
refused your application for insurance or for 8
"Sec. 27. Such party to a contract of insurance must communicate to the other, in good faith, all
reinstatement of a lapsed policy or offered you a facts within his knowledge which are material to the contract, and which the other has not the FACTS
policy different from that applied for?" when in fact, means of ascertaining, and as to which he makes no warranty."
INSURANCE Page 58

- June 18, 1982 – Jaime Canilang was diagnosed by > It also found that the failure of Jaime to disclose reasonable influence of the farts" concealed must, of
Dr. Claudio to have sinus tachycardia. He was previous medical consultation and treatment course, be determined objectively, by the judge
directed by the doctor to take a tranquilizer constituted material information which should have ultimately.
(Trazepam) and a beta-blocker drug (Aptin). been communicated to Great Pacific to enable the Reasoning
- August 3, 1982 – Jaime consulted Dr. Claudio again latter to make proper inquiries. - The information which Jaime failed to disclose was
and was diagnosed to have acute bronchitis. material to the ability of Great Pacific to estimate the
- August 4, 1982 – Jaime applied for a nonmedical ISSUES probable risk he presented as a subject of life
insurance policy with Great Pacific Life Assurance 1. WON Jaime intentionally withheld information insurance.
Company. He named his wife Thelma as his from Great Pacific - Had Canilang disclosed his visits to his doctor, the
beneficiary. He was issue the policy with a face 2. WON the information withheld would have been diagnosis made and the medicines prescribed by
value of P19,700 effective August 9, 1982. material to Great Pacific’s decision to grant Jaime the such doctor, in the insurance application, it may be
- August 5, 1983 – Jaime died of congestive heart insurance policy reasonably assumed that Great Pacific would have
failure, anemia and chronic anemia. Thelma filed her made further inquiries and would have probably
claim but the insurance company refused to grant it HELD refused to issue a non-medical insurance policy or,
on the ground that Jaime had concealed information. 1. YES at the very least, required a higher premium for the
- Thelma filed a complaint against Great Pacific to Ratio Section 27 of the Insurance Code of 1978 is same coverage.
recover the insurance proceeds. She testified that properly read as referring to "any concealment - As held in the case of Saturnino vs. Philippine-
she was not aware of her husband’s ailments and without regard to whether such concealment is American Life Insurance, “the waiver of medical
that she thought he had died from a kidney disorder. intentional or unintentional. The restoration in 1985 examination in a non-medical insurance contract
- Great Pacific presented as witness Dr. Quismorio by B.P. Blg. 874 of the phrase "whether intentional renders even more material the information inquired
who testified that Jaime’s insurance application was or unintentional" merely underscored the fact that all of the applicant concerning previous condition of
the basis of his medical declaration and she throughout (from 1914 to 1985), the statute did not health and diseases suffered, for such information
explained that an applicant was required to undergo require proof that concealment must be "intentional" necessarily constitutes an important factor which the
medical examination only if the applicant had in order to authorize rescission by the injured party. insurer takes into consideration in deciding whether
disclosed that he had previously been consulted with Reasoning to issue the policy or not.”
a doctor and had been hospitalized. - Art. 27 of the 1978 Insurance Code reads that “a Disposition the Petition for Review is DENIED for
- The Insurance Commissioner ordered Great Pacific concealment entitles the injured party to rescind a lack of merit and the Decision of the Court of
to pay Thelma the insurance proceeds, including contract of insurance,” which does not include the Appeals dated 16 October 1989 in C.A.-G.R. SP No.
attorney’s fees, holding that Jaime’s illness was not words “whether intentional or unintentional” from 08696 is hereby AFFIRMED.
that serious as to Great Pacific’s decision to insure the previous statutes. The Insurance Commissioner
him and that there was no concealment on the part relied on this deletion in arguing that the statute YU PANG CHENG v. CA
of Jaime with regard to his illness. intended to limit the kinds of concealment which 105 PHIL 930
Petitioners’ Claim: generate a right to rescind on the part of the injured BAUTISTA ANGELO; May 29, 1959
> Thelma argues that the non-disclosure of Jaime party to "intentional concealments."
did not amount to fraud. - In the case at bar, the nature of the facts not FACTS
> She also argues that the CA erred in not holding conveyed to the insurer was such that the failure to - September 5, 1950: Yu Pang Eng submitted parts
that the issue in the case agreed upon between the communicate must have been intentional rather than II and III of his application for insurance consisting
parties before the Insurance Commission is whether merely inadvertent. of the medical declaration made by him to the
or not Jaime 'intentionally' made material > Jaime could not have been unaware that his medical examiner of defendant and the medical
concealment in stating his state of health; heart beat would at times rise to high and examiner's report
Respondents’ Comments: alarming levels and that he had consulted a doctor - September 7: he submitted part I of his application
> The CA reversed the Insurance Commissioner’s twice two months before applying for non-medical which is the declaration made by him to an agent of
decision, holding that the use of the word insurance. defendant
'intentionally" by the Insurance Commissioner in > The last medical consultation took place just the - September 8: defendant issued to the insured
defining and resolving the issue agreed upon by the day before the insurance application was filed. Policy No. 812858
parties at pre-trial before the Insurance 2. YES - December 27, 1950: the insured entered St.
Commissioner was not supported by the evidence Ratio Materiality relates rather to the "probable and Luke's Hospital for medical treatment but he died on
and that the issue agreed upon by the parties had reasonable influence of the facts" upon the party to February 27, 1951.
been whether Jaime made a material concealment as whom the communication should have been made, in - According to the death certificate, he died of
to the state of his health at the time of the filing of assessing the risk involved in making or omitting to "infiltrating medullary carcinoma, Grade 4, advanced
insurance application, justifying the denial of the make further inquiries and in accepting the cardiac and of lesser curvature, stomach metastases
claim. application for insurance; that "probable and spleen."
INSURANCE Page 59

- Plaintiff, brother and beneficiary of the insured, - The negative answers given by the insured
demanded from defendant the payment of the regarding his previous ailment, or his concealment of
proceeds of the insurance policy and when the the fact that he was hospitalized and treated for
demand was refused, he brought the present action. sometime of peptic ulcer and had suffered from
- The insured, in his application for insurance, "dizziness, anemia, abdominal pains and tarry
particularly in his declarations to the examining stools", deprived defendant of the opportunity to GREAT PACIFIC LIFE v. CA (supra p.34)
physician, stated the following in answering the make the necessary inquiry as to the nature of his
questions propounded to him: past illness so that it may form its estimate relative PACIFIC BANKING CORP v. CA (ORIENTAL
14. Have you ever had any of the following diseases to the approval of his application. ASSURANCE CORPORATION)
or symtoms? Each question must be read and - Had defendant been given such opportunity, 168 SCRA 1
answered "Yes" or "No.". considering the previous illness of the insured as PARAS; November 28, 1988
"Gastritis, Ulcer of the Stomach or any disease of disclosed by the records of the Chinese General
that organ? No. Hospital, defendant would probably had never NATURE
"Vertigo, Dizziness, Fainting-spells or consented to the issuance of the policy in question. Petition for review on certiorari of the CA decision,
Unconsciouness? No. In fact, according to the death certificate, the which set aside the decision of CFI Manila, which had
"Cancer, Tumors or Ulcers of any kind? No. insured died of "infiltrating medullary carcinoma, in turn granted the complaint for a sum of money in
- 15. Have you ever consulted any physician riot Grade, 4, advanced cardiac and of lesser curvature, civil case filed by Pacific Banking against Oriental
included in any of the above answers? Give names stomach metastases spleen", which may have a Assurance.
and address or physicians list ailments or accidents direct connection with his previous illness.
and date. No." - Our Insurance Law provides that "A neglect to FACTS
- It appears that the insured entered the Chinese communicate that which a party knows and ought to - October 21,1963: an open Fire Policy was issued to
General Hospital for medical treatment on January communicate, is called concealment" (Section 25, the Paramount Shirt Manufacturing Co. (insured), by
29, 1950 having stayed there up to February 11, Act No. 2427). Whether intentional or unintentional, which Oriental Assurance Corporation bound itself to
1950. the concealment entitles the insurer to rescind the indemnify the insured for any loss or damage, not
- An X-ray picture of his stomach was taken and the contract of insurance (Section 26). exceeding P61,000.00, caused by fire to its property
diagnosis made of him by his doctors showed that - Our law even requires the insured to communicate consisting of stocks, materials and supplies usual to
his illness was "peptic ulcer, bleeding." to the insurer all facts within his knowledge which a shirt factory, including furniture, fixtures,
are material to the contract and which the other machinery and equipment while contained in the
ISSUE party has not the means of ascertaining (Section ground, second and third floors of the building
WON the insured is guilty of concealment of some 27), and the materiality is to be determined not by situated at number 256 Jaboneros St., San Nicolas,
facts material to the risk insured against which has the event but solely by the probable and reasonable Manila, for a period of one year commencing from
the effect of avoiding the policy as found by influence of the facts upon the party to whom the that date to October 21, 1964.
respondent court. communication is due (Section 30). - Insured was at the time of the issuance of the
- Argente vs. West Coast Life Insurance Co.: "One policy and is up to this time, a debtor of Pacific
HELD ground for the rescission of a contract of insurance Banking in the amount of not less P800,000.00 and
- It should be noted that the insured's confinement under the Insurance Act is 'a concealment', which in the goods described in the policy were held in trust
in the Chinese General Hospital took place from section 25 is defined 'A neglect to communicate that by the insured for the Pacific Banking under thrust
January 29, 1950 to February 11, 1950, whereas his which a party knows and ought to communicate.' receipts.
application for insurance wherein he stated his Appellant argues that the concealment was - Said policy was duly endorsed to Pacific Banking as
answers to the questions propounded to him by the immaterial and insufficient to avoid the policy. We mortgagee/trustor of the properties insured, with the
examining physician of defendant was submitted to cannot agree. In an action on a life insurance policy knowledge and consent of Oriental Assurance to the
defendant on September 5, 1950. where the evidence conclusively shows that the effect that "loss if any under this policy is payable to
- It is apparent that when the insured gave his answers to questions concerning diseases were the Pacific Banking Corporation".
answers regarding his previous ailment, particularly untrue, the truth or falsity of the answers become - While the aforesaid policy was in full force and
with regard to "Gastritis, Ulcer of the Stomach or any the determining factor. If the policy was procured by effect, a fire broke out on the subject premises
disease of that organ" and "Vertigo, Dizziness, fraudulent representations, the contract of insurance destroying the goods contained in its ground and
Fainting-spells or Unconsciousness", he concealed apparently set forth therein was never legally second floors. Counsel for the Pacific Banking sent a
the ailment of which he was treated in the Chinese existent. It can fairly be assumed that had the true letter of demand to Oriental Assurance for indemnity
General Hospital which precisely has direct facts been disclosed by the assured, the insurance due to the loss of property by fire. Oriental
connection with the subject of the questions would never have been granted." Assurance informed counsel that it was not yet ready
propounded. Disposition Decision affirmed. to accede to the latter's demand as the former is
INSURANCE Page 60

awaiting the final report of the insurance adjuster, effect from the very beginning, as if it had never
H.H. Bayne Adjustment Company. ISSUES been entered into, and which cannot be validated
- Said insurance adjuster notified counsel for the 1. WON insured is guilty of fraud either by time or by ratification.
Pacific Banking that the insured under the policy had 2. WON mortgagee/assignee can still claim from the - As the insurance policy against fire expressly
not filed any claim with it, nor submitted proof of insurance required that notice should be given by the insured
loss which is a clear violation of Policy Condition of other insurance upon the same property, the total
No.11, and for which reason, determination of the HELD absence of such notice nullifies the policy.
liability of Oriental Assurance could not be had. 1. YES - Argument that notice of co-insurances may be
Pacific Banking's counsel replied asking the insurance - The crux of the controversy centers on two points: made orally is preposterous and negates policy
adjuster to verify from the records of the Bureau of (a) unrevealed co-insurances which violated policy condition No. 20 which requires every notice and
Customs the entries of merchandise taken into the conditions No. 3; and (b) failure of the insured to file other communications to the insurer to be written or
customs bonded warehouse razed by fire as a the required proof of loss prior to court action. printed.
reliable proof of loss. - Policy Condition No. 3 explicitly provides: “The 2. NO
- For failure of the insurance company to pay the Insured shall give notice to the Company of any - Subject mortgage clause pecifically provides: “Loss,
loss as demanded, Pacific Banking field before CFI an insurance already effected, or which may if any, under this policy, shall be payable to the
action for a sum of money against the Oriental subsequently be effected, covering any of the PACIFIC BANKING CORPORATION Manila
Assurance, in the principal sum of P61,000.00 issued property hereby insured, and unless such notice be mortgagee/trustor as its interest may appear, it
in favor of Paramount Shirt Manufacturing Co. given and the particulars of such insurance or being hereby understood and agreed that this
Oriental Assurance defenses insurances be stated in or endorsed on this Policy by insurance as to the interest of the mortgagee/trustor
(a) lack of formal claim by insured over the loss and or on behalf of the Company before the occurrence only herein, shall not be invalidated by any act or
(b) premature filing of the suit as neither plaintiff nor of any loss or damage, all benefit under this policy neglect except fraud or misrepresentation, or arson
insured had submitted any proof of loss on the basis shall be forfeited.” of the mortgagor or owner/trustee of the property
of which defendant would determine its liability and - It is not disputed that the insured failed to reveal insured; provided, that in case the mortgagor or
the amount thereof, either to the Oriental Assurance before the loss three other insurances. By reason of owner/ trustee neglects or refuses to pay any
or its adjuster H.H. Bayne Adjustment Co. said unrevealed insurances, the insured had been premium, the mortgagee/ trustor shall, on demand
Pacific Banking guilty of a false declaration; a clear pay the same.”
> presented evidence that insured has undeclared misrepresentation and a vital one because where the - The paragraph clearly states the exceptions to the
co-insurances with the following: P30,000.00 with insured had been asked to reveal but did not, that general rule that insurance as to the interest of the
Wellington Insurance; P25,000. 00 with Empire was deception. Otherwise stated, had the insurer mortgagee, cannot be invalidated; namely: fraud, or
Surety and P250,000.00 with Asian Surety; known that there were many co-insurances, it could misrepresentation or arson.
undertaken by insured Paramount on the same have hesitated or plainly desisted from entering into - Concealment of the aforecited co-insurances can
property covered by its policy with Oriental such contract. Hence, the insured was guilty of clear easily be fraud, or in the very least,
Assurance whereas the only co-insurances declared fraud. misrepresentation. It is but fair and just that where
in the subject policy are those of P30,000.00 with - Pacific Banking's contention that the allegation of the insured who is primarily entitled to receive the
Malayan, P50,000.00 with South Sea, and fraud is but a mere inference or suspicion is proceeds of the policy has by its fraud and/or
P25.000.00 with Victory untenable. Concrete evidence of fraud or false misrepresentation, forfeited said right, with more
- NOTE: the defense of fraud and/or violation of non- declaration by the insured was furnished by the reason Pacific Banking which is merely claiming as
declaration of co-insurances was not pleaded in the Pacific Banking itself when the facts alleged in the indorsee of said insured, cannot be entitled to such
answer, also not pleaded in the Motion to Dismiss. policy under clauses "Co-Insurances Declared" and proceeds.
- CFI denied Oriental Assurance's motion on the "Other Insurance Clause" are materially different - The fact of fraud was tried by express or at least
ground that since the defense was raised for the first from the actual number of co-insurances taken over implied consent of the parties. Pacific Banking did
time, it must be deemed to have waived the the subject property. Consequently, the whole not only object to the introduction of evidence but on
requirement of proof of loss. Case was submitted for foundation of the contract fails, the risk does not the contrary, presented the very evidence that
decision. But upon MR, Oriental Asurance was attach and the policy never becomes a contract proved its existence.
allowed to present additional evidence, "in order to between the parties. Representations of facts are the - Be that as it may, SC has ample authority to give
prove that 'insured has committed a violation of foundation of the contract and if the foundation does beyond the pleadings where in the interest of justice
condition No. 3 of the policy in relation to the other not exist, the superstructure does not arise. and the promotion of public policy, there is a need to
Insurance Clause.' " CFI eventually adjudged Falsehood in such representations is not shown to make its own finding to support its conclusion.
Oriental Assurance liable to the Pacific Banking under vary or add to the contract, or to terminate a Otherwise stated, the Court can consider a fact
the said contract of insurance. contract which has once been made, but to show which surfaced only after trial proper.
- Court of Appeals reversed. Pacific Banking's MR that no contract has ever existed (Tolentino). A void - Generally, the cause of action on the policy accrues
denied. or inexistent contract is one which has no force and when the loss occurs, but when the policy provides
INSURANCE Page 61

that no action shall be brought unless the claim is - Pacific Banking prematurely filed the civil case and prompted it to reject the claim on the ground that
first presented extrajudicially in the manner provided dismissal thereof was warranted under the the insured did not disclose facts material to the
in the policy, the cause of action will accrue from the circumstances. While it is a cardinal principle of issuance of the policy. The insured gave false
time the insurer finally rejects the claim for insurance law that a policy or contract of insurance is statements in the application when he answered in
payment. to be construed liberally in favor of the insured and the negative to the question “have you ever had or
- In the case at bar, policy condition No. 11 strictly as against the insurer company yet, contracts sought advice for urine, kidney, bladder
specifically provides that the insured shall on the of insurance, like other contracts, are to be disorder?”
happening of any loss or damage give notice to the construed according to the sense and meaning of the - Sunlife discovered that two weeks prior to the
company and shall within fifteen (15) days after such terms which the parties themselves have used. If issuance, insured was diagnosed with renal failure,
loss or damage deliver to the Oriental Assurance (a) such terms are clear and unambiguous, they must was confined, and underwent tests.
a claim in writing giving particular account as to the be taken and understood in their plain, ordinary and - November 17, 1988: Bacani and her husband filed
articles or goods destroyed and the amount of the popular sense. for specific performance against Sunlife. RTC granted
loss or damage and (b) particulars of all other - Contracts of insurance are contracts of indemnity the plea on the ground that that the facts concealed
insurances, if any. Likewise, insured was required "at upon the terms and conditions specified in the policy. by the insured were made in good faith and under
his own expense to produce, procure and give to the The parties have a right to impose such reasonable the belief that they need not be disclosed, and that
company all such further particulars, plans, conditions at the time of the making of the contract the disclosure was not material since the policy was
specifications, books, vouchers, invoices, duplicates as they may deem wise and necessary. The non-medical.
or copies thereof, documents, proofs and information agreement has the force of law between the parties. - Sunlife appealed to the CA, but the latter denied
with respect to the claim". The terms of the policy constitute the measure of the the appeal on the ground that the cause of death
- Evidence adduced shows that 24 days after the insurer's liability, and in order to recover, the insured was unrelated to the facts concealed by the insured.
fire, Pacific Banking merely wrote letters to Oriental must show himself within those terms. The
Assurance to serve as a notice of loss, thereafter, compliance of the insured with the terms of the Petitioner’s Claim
the former did not furnish the latter whatever policy is a condition precedent to the light of > The insured did not disclose facts relevant to the
pertinent documents were necessary to prove and recovery. issuance of the policy, thus rescission of the contract
estimate its loss. Instead, Pacific Banking shifted - It appearing that insured has violated or failed to may be invoked by the insurance company.
upon Oriental Assurance the burden of fishing out perform the conditions under No. 3 and 11 of the Respondents’ Comments
the necessary information to ascertain the particular contract, and such violation or want of performance > The actual cause of death was not relevant to the
account of the articles destroyed by fire as well as has not been waived by the insurer, the insured concealed information, and the policy was entered
the amount of loss. cannot recover, much less the herein Pacific Banking. into by the insured in good faith.
- Oriental Assurance and its adjuster notified Pacific Courts are not permitted to make contracts for the
Banking that insured had not yet filed a written claim parties; the function and duty of the courts is simply ISSUE
nor submitted the supporting documents in to enforce and carry out the contracts actually made. WON the concealment renders the insurance policy
compliance with the requirements set forth in the Disposition Petition dismissed. CA affirmed. rescissible
policy. Despite the notice, the latter remained
unheedful. Since the required claim by insured, SUNLIFE ASSURANCE COMPANY v. CA (SPS. HELD
together with the preliminary submittal of relevant BACANI) YES
documents had not been complied with, it follows 245 SCRA 268 Ratio The terms of the contract are clear. The
that Oriental Assurance could not be deemed to have QUIASON; June 22, 1995 insured is specifically required to disclose to the
finally rejected Pacific Banking's claim and therefore insurer matters relating to his health.
the latter's cause of action had not yet arisen. NATURE Reasoning
Compliance with condition No. 11 is a requirement A petition for review on certiorari. SEC. 26 (IC)
sine qua non to the right to maintain an action as A neglect to communicate that which a party
prior thereto no violation of Pacific Banking's right FACTS knows and ought to communicate, is called a
can be attributable to Oriental Assurance. As before - April 15, 1986: Robert John B. Bacani procured a concealment.
such final rejection, there was no real necessity for life insurance contract for himself from SUNLIFE SEC. 31 (IC)
bringing suit. Pacific Banking should have (petitioner) valued at P100K. The designated Materiality is to be determined not by the event,
endeavored to file the formal claim and procure all beneficiary was his mother, Bernarda Bacani but solely by the probable and reasonable influence
the documents, papers, inventory needed by Oriental (respondent). of the facts upon the party to whom communication
Assurance or its adjuster to ascertain the amount of - June 26, 1987: the insured died in a plane crash. is due, in forming his estimate of the disadvantages
loss and after compliance await the final rejection of Bernarda Bacani filed a claim with Sunlife, seeking of the proposed contract or in making his inquiries
its claim. Indeed, the law does not encourage the benefits of the insurance policy taken by her son. - The information which the insured failed to disclose
unnecessary litigation. Petitioner conducted an investigation and its findings was material and relevant to the approval and the
INSURANCE Page 62

issuance of the insurance policy. The matters Albay had been obtained through fraud and deceit - Qua Chee Gan insured 4 of his bodegas with Law
concealed would have definitely affected petitioner's known and consented to by the interested parties Union & Rock Insurance Co in 1937. These bodegas
action on his application, either by approving it with and is therefore completely illegal, void, and were used for the storage of stocks of copra and of
the corresponding adjustment for a higher premium ineffective. hemp, baled and loose.
or rejecting the same. - A criminal case for frustrated estafa was filed by - Fire of undetermined origin that broke out in the
- Good faith is no defense in concealment. It appears defendant against Ponciano Remigio, Castor Garcia early morning of July 21, 1940, and lasted almost
that such concealment was deliberate on the part of and Francisca Eguaras. They were acquitted, and one week, gutted and completely destroyed Bodegas
the insured. claim that the judgment produces the effect of res Nos. 1, 2 and 4, with the merchandise stored
- The waiver of a medical examination [in a non- judicata in the present suit. therein.
medical insurance contract] renders even more - Qua Chee Gan informed the insurance company of
material the information required of the applicant ISSUE the fire. Fire adjusters of the company conducted an
concerning previous condition of health and diseases WON the life insurance obtained by Dominador Albay extensive investigation. Qua Chee Gan submitted the
suffered, for such information necessarily constitutes was issued through fraud and deceit corresponding fire claims, totaling P398,562.81 (but
an important factor which the insurer takes into reduced to the full amount of the insurance,
consideration in deciding whether to issue the policy HELD P370,000), the Insurance Company resisted
or not. YES payment, claiming violation of warranties and
- Anent the finding that the facts concealed had no Ratio In a contract where one of the contracting conditions, filing of fraudulent claims, and that the
bearing to the cause of death of the insured, it is parties may have given his consent through error, fire had been deliberately caused by the insured or
well settled that the insured need not die of the violence, intimidation, or deceit, and in any of such by other persons in connivance with him.
disease he had failed to disclose to the insurer. It is cases the contract is void, even though, despite this - Qua Chee Gan, his brother and his employees were
sufficient that his non-disclosure misled the insurer nullity, no crime was committed. There may not tried for arson, where counsel of the insurance
in forming his estimates of the risks of the proposed have been estafa in the case at bar, but it was company acted as a private prosecutor. They were
insurance policy or in making inquiries conclusively demonstrated by the trial that deceit acquitted.
Disposition Petition is granted and the decision of entered into the insurance contract, fulfillment - This civil suit was then instituted to claim against
CA is reversed and set aside. whereof is claimed, and therefore the conclusions the insurance company. The CFI ruled in favor of
reached by the court in the judgment it rendered in Qua Chee Gan and ordered Law Union Rock Co. to
EGUARAS v. GREAT EASTERN the criminal proceedings for estafa do not affect this pay.
33 PHIL. 263 suit, nor can they produce in the present suit the
TORRES.; January 24, 1916 force of res adjudicata. ISSUES
Reasoning 1. WON there was a breach of the fire hydrant
NATURE - It is proven that the signatures on the insurance warranty
Appeal filed through bill of exceptions from the applications reading "Dominado Albay" are false and 2. WON the insured violated the Hemp warranty
judgment of the CFI forged; that the person who presented himself to Dr. 3. WON Qua Chee Gan is guilty of overvaluation
Vidal to be examined was not the real Dominador 4. WON Qua Chee Gan caused the fire
FACTS Albay, but Castor Garcia who was positively 5. WON there was an error in the amount of copra
- Francisca Eguaras filed a written complaint in court, identified by Dr. Vidal; that at the time of the and hemp lost
alleging as a cause of action that her son-in-law application for insurance and the issuance of the 6. WON the claims contained false and fraudulent
Dominador Albay had applied in writing to the policy which is the subject matter of this suit the real statements
defendant insurance company to insure his life for Dominador Albay was informed of all those
the sum of P5,000, naming as the beneficiary in case machinations, wherefore it is plain that the insurance HELD
of his death the plaintiff Francisca Eguaras; that contract between the defendant and Dominador 1. NO
after compliance with the requisites and the Albay is null and void because it is false, fraudulent - It is argued that he should have 11 fire hydrants in
investigation carried on by the defendant company, and illegal. the compound, but he only had 2. We are in
it accepted the application for insurance and issued Disposition The judgment appealed from is agreement with the trial Court that the appellant is
the policy; that, said policy being in force, the reversed and the defendant absolved from the barred by waiver (or rather estoppel) to claim
insured died, and despite the fact that the complaint without special finding as to the costs. violation of the so-called fire hydrants warranty, for
beneficiary submitted satisfactory proofs of his death the reason that knowing fully all that the number of
and that the defendant company investigated the QUA CHEE GAN v. LAW UNION AND ROCK hydrants demanded therein never existed from the
event, still it refused and continues to refuse to pay 98 PHIL 85 very beginning, the appellant nevertheless issued
to the plaintiff the value of the policy. REYES; December 17, 1955 the policies in question subject to such warranty, and
- Defendant set forth in special defense that the received the corresponding premiums.
insurance policy issued in the name of Dominador FACTS 2. NO
INSURANCE Page 63

- The insurance company avers that the insured established fact that, notwithstanding the insurer's the omission in Exhibits EE and FF was due to
violated the hemp warranty when it admitted that it refusal to pay the value of the policies the extensive inadvertance, for the insured could hardly expect
had 36 cans of gasoline in the building. It is well to resources of the insured enabled him to pay off the under such circumstances, that the 1939 would pass
note that gasoline is not specifically mentioned National Bank in a short time; and if he was able to unnoticed by the insurance agents. Similarly, the 20
among the prohibited articles listed in the so-called do so, no motive appears for attempt to defraud the per cent overclaim on 70 per cent of the hemo stock,
"hemp warranty." The cause relied upon by the insurer. While the acquittal of the insured in the was explained by the insured as caused by his belief
insurer speaks of "oils (animal and/or vegetable arson case is not res judicata on the present civil that he was entitled to include in the claim his
and/or mineral and/or their liquid products having a action, the insurer's evidence, to judge from the expected profit on the 70 per cent of the hemp,
flash point below 300o Fahrenheit", and is decidedly decision in the criminal case, is practically identical in because the same was already contracted for and
ambiguous and uncertain; for in ordinary parlance, both cases and must lead to the same result, since sold to other parties before the fire occurred.
"Oils" mean "lubricants" and not gasoline or the proof to establish the defense of connivance at Compared with other cases of over-valuation
kerosene. And how many insured, it may well be the fire in order to defraud the insurer "cannot be recorded in our judicial annals, the 20 per cent
wondered, are in a position to understand or materially less convincing than that required in order excess in the case of the insured is not by itself
determine "flash point below 003o Fahrenheit. Here, to convict the insured of the crime of arson. sufficient to establish fraudulent intent. Certainly,
again, by reason of the exclusive control of the 5. NO the insured's overclaim of 20 per cent in the case at
insurance company over the terms and phraseology - As to the defense that the burned bodegas could bar, duly explained by him to the Court a quo,
of the contract, the ambiguity must be held strictly not possibly have contained the quantities of copra appears puny by comparison (compared to other
against the insurer and liberally in favor of the and hemp stated in the fire claims, the insurer's case cases cited by the court), and can not be regarded
insured, especially to avoid a forfeiture rests almost exclusively on the estimates, inferences as "more than misstatement, more than
- Another point that is in favor of the insured is that and conclusions of its adjuster investigator, inadvertence of mistake, more than a mere error in
the gasoline kept in Bodega No. 2 was only incidental Alexander D. Stewart, who examined the premises opinion, more than a slight exaggeration" that would
to his business, being no more than a customary 2 during and after the fire. His testimony, however, entitle the insurer to avoid the policy. It is well to
day's supply for the five or six motor vehicles used was based on inferences from the photographs and note that the overcharge of 20 per cent was claimed
for transporting of the stored merchandise). "It is traces found after the fire, and must yield to the only on a part (70 per cent) of the hemp stock; had
well settled that the keeping of inflammable oils on contradictory testimony of engineer Andres Bolinas, the insured acted with fraudulent intent, nothing
the premises though prohibited by the policy does and specially of the then Chief of the Loan prevented him from increasing the value of all of his
not void it if such keeping is incidental to the Department of the National Bank's Legaspi branch, copra, hemp and buildings in the same proportion.
business." (Bachrach vs. British American Ass. Co., Porfirio Barrios, and of Bank Appraiser Loreto This also applies to the alleged fraudulent claim for
17 Phil. 555, 560) Samson, who actually saw the contents of the burned empty sacks, that was likewise explained to
3. NO bodegas shortly before the fire, while inspecting our satisfaction and that of the trial Court. The rule
- The charge that the insured failed or refused to them for the mortgagee Bank is that to avoid a policy, the false swearing must be
submit to the examiners of the insurer the books, 6. NO willful and with intent to defraud which was not the
vouchers, etc. demanded by them was found - Appellant insurance company also contends that cause. Of course, the lack of fraudulent intent would
unsubstantiated by the trial Court, and no reason the claims filed by the insured contained false and not authorize the collection of the expected profit
has been shown to alter this finding. fraudulent statements that avoided the insurance under the terms of the polices, and the trial Court
- In view of the discrepancy in the valuations policy. But the trial Court found that the correctly deducted the same from its award.
between the insured and the adjuster Stewart for the discrepancies were a result of the insured's Disposition Decision affirmed
insurer, the Court referred the controversy to a erroneous interpretation of the provisions of the
government auditor, Apolonio Ramos; but the latter insurance policies and claim forms, caused by his ARGENTE v. WEST COAST LIFE
reached a different result from the other two. Not imperfect knowledge of English, and that the 51 PHIL 725
only that, but Ramos reported two different misstatements were innocently made and without MALCOLM; March 19, 1928
valuations that could be reached according to the intent to defraud. The trial court’s ruling must be
methods employed. Clearly then, the charge of upheld. FACTS
fraudulent overvaluation cannot be seriously - For example, the occurrence of previous fires in the - This is an action upon a joint life insurance policy
entertained. premises insured in 1939, altho omitted in the for P15,000 issued by the West Coast Life Insurance
4. NO claims, Exhibits EE and FF, were nevertheless Co., on May 15, 1925, in favor of Bernardo Argente,
- This defense is predicted on the assumption that revealed by the insured in his claims Exhibits Q (filed and his wife, Vicenta de Ocampo, the latter having
the insured was in financial difficulties and set the simultaneously with them), KK and WW. Considering died on November 18, 1925. Fraud in obtaining the
fire to defraud the insurance company, presumably that all these claims were submitted to the smae policy was pleaded by way of special defense. On the
in order to pay off the Philippine National Bank, to agent, and that this same agent had paid the loss issue thus suggested, the court adopted the theory
which most of the insured hemp and copra was caused by the 1939 fire, we find no error in the trial of the defendant, and held the insurance policy null
pledged. This defense is fatally undermined by the Court's acceptance of the insured's explanation that
INSURANCE Page 64

and void, with the result that the complaint was evidence on these points consists of the testimony of risk upon a false basis that it does not exist. The
dismissed, with costs. the plaintiff and his subordinate clerk, Apolonio principal question, therefore, must be, Was the
-Bernardo Argente signed an application for joint Espiritu, on the one hand, and of the testimony of assurer misled or deceived into entering a contract
insurance with his wife in the sum of P2,000. The Doctor Sta. Ana and Jose Geronimo del Rosario on obligation or in fixing the premium of insurance by
wife, Vicenta de Ocampo, signed a like application the other. This was rejected by the Trial Court. Trial a withholding of material information or facts
for the same policy. judge found with the insurance company with regard within the assured's knowledge or presumed
- Bernardo Argente and his wife was examined by to the question of fact. SC agrees. There appears no knowledge?
Dr. Cesareo Sta. Ana, a medical examiner for the motive whatever on the part of Doctor Sta. Ana to "It therefore follows that the assurer in assuming a
West Coast Life Insurance Co. which did not show falsify the Medical Examiner's Reports and thereby risk is entitled to know every material fact of which
previous and existing health problems. not only jeopardize his career as a physician, but the assured has exclusive or peculiar knowledge,
- A temporary policy for P15,000 was issued to also gravely implicate himself criminally. as well as all material facts which directly tend to
Bernardo Argente and his wife as of May 15, 1925. increase the hazard or risk which are known by the
In view of the fact that more than thirty days had ISSUE assured, or which ought to be or are presumed to
elapsed since the applicants were examined by the WON the contract of insurance may be rescinded be known by him. And a concealment of such facts
company's physician, each of them was required to vitiates the policy. 'It does not seem to be
file a certificate of health before the policy was necessary . . . that the . . . suppression of the
delivered to them. truth should have been willful.' If it were but an
- On November 18, 1925, Vicenta de Ocampo died of HELD inadvertent omission, yet if it were material to the
cerebral apoplexy. Thereafter Bernardo Argente YES risk and such as the plaintiff should have known to
presented a claim. Following investigation conducted - Bernardo Argente and his wife applications’ were be so, it would render the policy void. But it is held
by the Manager of the Manila office of the insurance false with respect to their state of health during the that if untrue or false answers are given in
company, it was apparently disclosed that the period of five years preceding the date of such response to inquiries and they relate to material
answers given by the insured in their medical applications and that they knew the representations facts the policy is avoided without regard to the
examinations with regard to their health and made by them in their applications were false. The knowledge or fraud of assured, although under the
previous illnesses and medical attendance were question arises as to the state of the law in relation statute statements are representations which must
untrue. West Coast Life Insurance Co. refused to pay thereto. be fraudulent to avoid the policy. So under certain
the claim of Bernardo Argente, and wrote him to the - One ground for the rescission of a contract of codes the important inquiries are whether the
effect that the claim was rejected because the insurance under the Insurance Act is "a concealment was willful and related to a matter
insurance was obtained through fraud and concealment," which in section 25 is defined as "A material to the risk.
misrepresentation. neglect to communicate that which a party knows xxx xxx xxx
- It is admitted that it appears in the Medical and ought to communicate." In an action on a life "If the assured has exclusive knowledge of
Examiner's Report that Bernardo Argente gave false insurance policy where the evidence conclusively material facts, he should fully and fairly disclose
responses. As well as with the Medical Examiner's shows that the answers to questions concerning the same, whether he believes them material or
Report that Vicenta de Ocampo. It is, however, not diseases were untrue, the truth or falsity of the not. But notwithstanding this general rule it will
disputed that Vicenta de Ocampo was taken by a answers become the determining factor. If the policy not infrequently happen, especially in life risks,
patrolman, at the request of her husband, Bernardo was procured by fraudulent representations, the that the assured may have a knowledge actual or
Argente, on May 19, 1924, to the Meisic police contract of insurance apparently set forth therein presumed of material facts, and yet entertain an
station, and from there was transferred to the San was never legally existent. It can fairly be assumed honest belief that they are not material. . . . The
Lazaro Hospital. In San Lazaro Hospital, her case that had the true facts been disclosed by the determination of the point whether there has or
was diagnosed by the admitting physician as assured, the insurance would never have been has not been a material concealment must rest
"alcoholism," but later Doctor Domingo made a granted. largely in all cases upon the form of the questions
diagnosis of probable "manic-depressive psychosis," - In Joyce, The Law of Insurance, second edition, propounded and the exact terms of the contract.
and still, later in Mary Chiles Hospital, made a final volume 3, Chapter LV, is found the following: Thus, where in addition to specifically named
diagnosis of "phycho-neurosis." "The basis of the rule vitiating the contract in diseases the insured was asked whether he had
- Bernardo Argente, while readily conceding most of cases of concealment is that it misleads or had any sickness within ten years, to which he
the facts herein narrated, yet alleges that both he deceives the insurer into accepting the risk, or answered 'No,' and it was proven that within that
and his wife revealed to the company's physician, accepting it at the rate of premium agreed upon; period he had had a slight attack of pharyngitis, it
Doctor Sta. Ana, all the facts concerning their The insurer, relying upon the belief that the was held a question properly for the jury whether
previous illnesses and medical attendance, but that assured will disclose every material fact within his such an inflammation of the throat was a 'sickness'
Doctor Sta. Ana, presumably acting in collusion with actual or presumed knowledge, is misled into a within the intent of the inquiry, and the court
the insurance agent, Jose Geronimo del Rosario, belief that the circumstance withheld does not remarked on the appeal decision that if it could be
failed to record them in the medical reports. The exist, and he is thereby induced to estimate the held as a matter of law that the policy was thereby
INSURANCE Page 65

avoided, then it was a mere device on the part of [1927], p. 80, ante.) The second answer is that the customers had been asking for such coverage since
insurance companies to obtain money without insurance company more than one month previous 1954.
rendering themselves liable under the policy. . . . to the commencement of the present action wrote -On May 28, 1957, Helen died of influenza. Ngo Hing
" . . . The question should be left to the jury the plaintiff and informed him that the insurance sought the payment of the proceeds of the
whether the assured truly represented the state of contract was void because it had been procured insurance, but having failed to do so, filed an action
his health so as not to mislead or deceive the through fraudulent representations, and offered to for recovery with the CFI of Cebu. The Court ordered
insurer; and if he did not deal in good faith with refund to the plaintiff the premium which the latter Pacific Life to pay P50k with 6% interest, hence this
the insurer in that matter, then the inquiry should had paid upon the return of the policy for petition.
be made, Did he know the state of his health so as cancellation. As held in California as to a fire
to be able to furnish a proper answer to such insurance policy, where any of the material ISSUE
questions as are propounded? A Massachusetts representations are false, the insurer's tender of the WON the binding deposit receipt constituted a
case, if construed as it is frequently cited, would premium and notice that the policy is canceled, temporary contract of the life insurance in question
be opposed to the above conclusion; but, on the before the commencement of suit thereon, operate
contrary, it sustains it, for the reason that to rescind the contract of insurance. (Rankin vs. HELD
symptoms of consumption had so far developed Amazon Insurance Co. [1891], 89 Cal., 203.) NO
themselves within a few months prior to effecting Disposition Judgment affirmed, with the costs of - The binding deposit receipt is merely a provisional
the insurance as to induce a reasonable belief that this instance against the appellant. contract and only upon compliance with the ff
the applicant had that fatal disease, and we should conditions: (1) that the company be satisfied that
further construe this case as establishing the rule GREAT PACIFIC LIFE v. CA (NGO HING) the applicant was insurable on standard rates (2)
that such a matter cannot rest alone upon the 89 SCRA 543 that if the company does not accept the application
assured's belief irrespective of what is a DE CASTRO, J; April 30, 1979 and offers a different policy, the insurance contract
reasonable belief, but that it ought to be judged by shall not be binding until the applicant accepts the
the criterion whether the belief is one fairly NATURE new policy (3) that if the applicant is not found to be
warranted by the circumstances. A case in Petition for certiorari insurable on standard rates and the application is
Indiana, however, holds that if the assured has disapproved, the insurance shall not be in force at
some affection or ailment of one or more of the FACTS any time and the premium be returned to the
organs inquired about so well defined and marked - On March 14, 1957, private respondent Ngo Hing applicant.
as to materially derange for a time the functions of filed an application with the Great Pacific Life -This implies the receipt is merely an
such organ, as in the case of Bright's disease, the Assurance Co. (Pacific Life) for a 20 year endowment acknowledgement, on behalf of the company, that
policy will be avoided by a nondisclosure, policy of P50k on the life of his 1 year old daughter, the Cebu branch of Pacific Life had received the
irrespective of the fact whether the assured knew Helen. Ngo Hing supplied the essetntial data which premium and had accepted the application subject to
of such ailment or not. . . ." petitioner Mondragon, branch manager of the Pacific processing by the insurance company, which will
- Lastly, appellant contends that even if the Life in Cebu, wrote on the corresponding form in his approve or reject it depending on whether the
insurance company had a right to rescind the own handwriting, later typing the data on an applicant is insurable on standard rates. As such, the
contract, such right cannot now be enforced in view application form signed by Ngo Hing. The latter paid receipt was never in force—it does not insure
of the provisions of section 47 of the Insurance Act the P1077.75 annual premium but retained P1,317 outright. No liability attaches until the principal
providing "Whenever a right to rescind a contract of as commission as he was also a duly authorized approves the risk and a receipt is given by the
insurance is given to the insurer by any provision of agent of Pacific Life. The binding deposit receipt was agent; because private respondent failed to accept
this chapter, such right must be exercised previous then issued to Ngo Hing; Mondragon handwrote his Pacific Life’s offer for the Juvenile Triple Action plan,
to the commencement of an action on the contract." strong recommendation for the approval of the there was no meeting of the minds and thus no
This section was derived from section 2583 of the application on the back of the form. contract. Also, being an authorized agent of Pacific
California Civil Code, but in contrast thereto, makes - On April 30, Mondragon received a letter from Life, Ngo Hing must have known the company did
use of the imperative "must" instead of the Pacific Life which stated that the 20 year endowment not offer the insurance applied for and merely took a
permissive "may." Nevertheless, there are two plan was not available for minors below 7, but that chance on Mondragon’s recommendation.
answers to the problem as propounded. The first is Pacific Life could consider the same under the Disposition the decision appealed from is set aside,
that the California law as construed by the code Juvenile Triple Action Plan, advising that if the offer absolving Pacific Life from their civil liabilities
examiners, at whose recommendation it was was acceptable, the Juvenile Non-Medical Declaration
adopted, conceded that "A failure to exercise the be sent to the company. EDILLON v. MANILA BANKERS LIFE
right (of rescission), cannot, of course, prejudice any -Mondragon allegedly failed to inform Ngo Hing of 117 SCRA 187
defense to the action which the concealment may the non-acceptance of the insurance plan, instead VASQUEZ; September 30, 1982
furnish." (Codes of California Annotated; Tan Chay writing Pacific Life again, recommending the
Heng vs. West Coast Life Insurance Company approval of the endowment plan to children since NATURE
INSURANCE Page 66

Appeal from a decision of the CFI - The age of the insured Carmen 0. Lapuz was not Said application was approved and was issued
concealed to the insurance company. Her application effective November 6, 1973
FACTS for insurance coverage which was on a printed form - On April 26,1975, Tan Lee Siong died of hepatoma
- Sometime in April 1969, Carmen O, Lapuz applied furnished by private respondent and which contained (Exhibit B). Petitioners then filed with respondent
with respondent insurance corporation for insurance very few items of information clearly indicated her company their claim for the proceeds of the life
coverage against accident and injuries. In the age of the time of filing the same to be almost 65 insurance policy
application form which was dated April 15, 1969, she years of age. Despite such information which could -respondent company denied petitioners' claim and
gave the date of her birth as July 11, 1904. On the hardly be overlooked in the application form, rescinded the policy by reason of the alleged
same date, she paid the sum of P20.00 representing considering its prominence thereon and its misrepresentation and concealment of material facts
the premium for which she was issued the materiality to the coverage applied for, the made by the deceased Tan Lee Siong in his
corresponding receipt signed by an authorized agent respondent insurance corporation received her application for insurance. The premiums paid on the
of the respondent insurance corporation. Upon the payment of premium and issued the corresponding policy were thereupon refunded
filing of said application and the payment of the certificate of insurance without question. The - Petitioners filed on November 27, 1975, a
premium on the policy applied for, the respondent accident which resulted in the death of the insured, a complaint against the former with the Office of the
insurance corporation issued to Carmen O. Lapuz its risk covered by the policy, occurred on May 31, 1969 Insurance Commissioner. Commissioner denied
Certificate of Insurance. The policy was to be or FORTY-FIVE (45) DAYS after the insurance petition. CA affirmed Commissioners decision
effective for a period of 90 days. coverage was applied for. There was sufficient time
- On May 31, 1969 or during the effectivity of the for the private respondent to process the application ISSUE
Insurance, Carmen O. Lapuz died in a vehicular and to notice that the applicant was over 60 years of WON according to Sec. 48 of the Insurance Code,
accident. age and thereby cancel the policy on that ground if it insurance company is barred from rescinding
- On June 7, 1969, petitioner Regina L. Edillon, a was minded to do so. If the private respondent failed contract
sister of the insured and who was the named to act, it is either because it was willing to waive
beneficiary in the policy, filed her claim for the such disqualification; or, through the negligence or HELD
proceeds of the insurance, submitting all the incompetence of its employees for which it has only - Section 48. Whenever a right to rescind a contract
necessary papers and other requisites with the itself to blame, it simply overlooked such fact. Under of insurance is given to the insurer by any provision
private respondent. Her claim having been denied, the circumstances, the insurance corporation is of this chapter, such right must be exercised
Regina L. Edillon instituted this action in the Court of already deemed in estoppel. Its inaction to revoke previous to the commencement of an action on the
First Instance of Rizal. the policy despite a departure from the exclusionary contract.
- In resisting the claim of the petitioner, the condition contained in the said policy constituted a After a policy of life insurance made payable on the
respondent insurance corporation relies on a waiver of such condition. death of the insured shall have been in force during
provision contained in the Certificate of Insurance, Disposition Judgment appealed from is REVERSED the lifetime of the insured for a period of two years
excluding its liability to pay claims under the policy and SET ASIDE and respondent insurance from the date of its issue or of its last reinstatement,
in behalf of "persons who are under the age of corporation is ordered to pay to the petitioner the the insurer cannot prove that the policy is void ab
sixteen (16) years of age or over the age of sixty proceeds of Insurance initio or is rescindable by reason of the fraudulent
(60) years ..." It is pointed out that the insured concealment or misrepresentation of the insured or
being over sixty (60) years of age when she applied HARDING v. COMMERCIAL UNION (supra p.36) his agent.
for the insurance coverage, the policy was null and - According to the petitioners, the Insurance Law
void, and no risk on the part of the respondent TAN v. CA (PHILIPPINE AMERICAN LIFE was amended and the second paragraph of Section
insurance corporation had arisen therefrom. INSURANCE COMPANY) 48 added to prevent the insurance company from
- RTC dismissed the complaint. 174 SCRA 403 exercising a right to rescind after the death of the
GUTIERREZ; June 29, 1989 insured
ISSUE - The so-called "incontestability clause" precludes the
WON the acceptance by the private respondent NATURE insurer from raising the defenses of false
insurance corporation of the premium and the Review on certiorari of the decision of the Court of representations or concealment of material facts
issuance of the corresponding certificate of insurance Appeals affirming the decision of the Insurance insofar as health and previous diseases are
should be deemed a waiver of the exclusionary Commissioner concerned if the insurance has been in force for at
condition of overage stated in the said certificate of least two years during the insured's lifetime. The
insurance FACTS phrase "during the lifetime" found in Section 48
- On September 23,1973, Tan Lee Siong, father of simply means that the policy is no longer considered
HELD herein petitioners, applied for life insurance in the in force after the insured has died. The key phrase in
YES amount of P 80,000.00 with respondent company. the second paragraph of Section 48 is "for a period
of two years."
INSURANCE Page 67

- The policy was issued on November 6,1973 and the and deceit perpetrated against this defendant in the agents of the defendant insurance company in
insured died on April 26,1975. The policy was thus in following manner, to wit: the Province of Occidental Negros; that the
force for a period of only one year and five months. 1. Go, Sanchez and Locsin, caused Tan Caeng to defendant, believing that the representations
Considering that the insured died before the two- sign an application for insurance with the made in said document were true, and relying
year period had lapsed, respondent company is not, defendant in the sum of P10,000, in which it was thereon, provisionally accepted the said
therefore, barred from proving that the policy is void said that Tan Ceang was single and was a application for insurance on the life of Tan
ab initio by reason of the insured's fraudulent merchant, and that the plaintiff Tan Chai Heng, Ceang in the sum of P10,000 and issued a
concealment or misrepresentation. the beneficiary, was his nephew, whereas in temporary policy pending the final approval or
- The petitioners contend that there could have been truth and in fact and as the plaintiff and his said disapproval of said application by defendant's
no concealment or misrepresentation by their late coconspirators well knew, the said Tan Ceang home-office in San Francisco, California, where
father because Tan Lee Siong did not have to buy was not single but was married and had several in case of approval a permanent policy was to be
insurance. He was only pressured by insistent children; and was not a merchant but a mere issued; that such permanent policy was never
salesmen to do so employee of Tan Quina from whom he received delivered to the plaintiff because defendant
-The legislative answer to the arguments posed by only a meager salary, and that plaintiff was not discovered the fraud before its delivery.
the petitioners is the "incontestability clause" added a nephew of the said Tan Ceang. 6. That the first agreed annual premium on the
by the second paragraph of Section 48. The insurer 2. Tan Ceang was seriously ill, suffering from insurance in question of P936.50 not having
has two years from the date of issuance of the pulmonary tuberculosis of about three years' been paid within 60 days after medical
insurance contract or of its last reinstatement within duration, which illness was incurable and was examination of the applicant as required by the
which to contest the policy, whether or not, the well known to the plaintiff and his said regulations of the defendant insurance company,
insured still lives within such period. After two years, coconspirators. plaintiff and coconspirators caused Tan Ceang to
the defenses of concealment or misrepresentation, 3. Locsin, in his capacity as medical examiner for sign a health certificate for reinstatement; that
no matter how patent or well founded, no longer lie the defendant, prepared and falsified the the said temporary policy was delivered by
necessary medical certificate, in which it was defendant to the insured on April 10, 1925, in
TAN CHAY HENG v. WEST COAST LIFE made to appear, among other things, that Tan the belief that said statements and
INSURANCE Ceang had never used morphine, cocaine or any representations were true and in reliance
51 PHIL 80 other drug; that he was then in good health and thereon.
JOHNS; November 21, 1927 had never consulted any physician; that he had 7. 2 ½ months after the supposed medical
never spit blood; and that there was no sign of examination above referred to, and exactly 1
FACTS either present or past disease of his lungs; month after the date of the health certificate for
- Plaintiff alleges that defendant accepted and whereas in truth and in fact, plaintiff and reinstatement above set forth, Tan Ceang died
approved a life insurance policy of for the sum of coconspirators well knew, Tan Ceang was in Valladolid, Occidental Negros, of pulmonary
P10,000 in which the plaintiff was the sole addicted to morphine, cocaine, and opium and tuberculosis, the same illness from which
beneficiary; that the policy was issued upon the had been convicted and imprisoned therefor, suffering at the time it is supposed he was
payment by the said Tan Ceang of the first year's and for about three year prior thereto had been examined by Dr. V. S. Locsin, but that the
premium amounting to P936; that in and by its suffering from pulmonary tuberculosis. plaintiff coconspirators, pursuant to their
terms, the defendant agreed to pay the plaintiff as 4. Plaintiff caused a confidential report to the conspiracy, caused the said Dr. V. S. Locsin to
beneficiary the amount of the policy upon the receipt defendant insurance company to be signed by state falsely in the certificate of death that the
of the proofs of the death of the insured while the one V. Sy Yock Kian, who was an employee of said Tan Ceang had died of cerebral
policy was in force; that without any premium being Go Chulian, in which it was falsely represented hemorrhage.
due or unpaid, Tan Ceang died on May 10, 1925; that Tan Ceang was worth about P40,000, had - Defendant also alleges that plaintiff was, like V. Sy
that in June, 1925, plaintiff submitted the proofs of an annual income of from eight to ten thousand Yock Kian, an employee of Go Chulian; that the
the death of Tan Ceang with a claim for the payment pesos net, had the appearance of good health, latter was the ringleader of a gang of malefactors,
of the policy which the defendant refused to pay, for and never had tuberculosis. who, during, and for some years previous to the
which he prays for a corresponding judgment, with 5. After said application for insurance, medical dates above mentioned, were engaged in the illicit
legal interest from the date of the policy, and costs. certificate and confidential report had been enterprise of procuring fraudulent life insurances
- Defendant alleges that the insurance policy on the prepared and falsified, plaintiff and from the present defendant, similar to the one in
life of Tan Ceang, upon which plaintiff's action is coconspirators caused the same to be forwarded question, and which enterprise was capitalized by
based, was obtained by the plaintiff in confabulation to the defendant at its office in Manila, the him by furnishing the funds with which to pay the
with one Go Chulian, of Bacolod, Negros Occidental; medical certificate thru the said Dr. V. S. Locsin premium on said fraudulent insurance; that the said
Francisco Sanchez of the same place; and Dr. V. S. as medical examiner, and said application for Go Chulian was the one who furnished the money
Locsin, of La Carlota, Negros Occidental, thru fraud insurance and confidential report thru the said with which to pay the first and only annual premium
Francisco Sanchez in his capacity as one of the on the insurance here in question, amounting to
INSURANCE Page 68

P936.50; that the said Go Chulian, on August 28, the existence of the contract which is sought to be insurance on the property hereby covered and
1926, was convicted by the Court of First Instance of rescinded. If all of the material matters set forth and no other insurance is allowed except by the
the City of Manila, in criminal case No. 31425 of that alleged in the defendant's special plea are true, there consent of the Company endorsed hereon. Any
court, of the crime of falsification of private was no valid contract of insurance, for the simple false declaration or breach or this condition will
documents in connection with an fraudulent reason that the minds of the parties never met and render this policy null and void.
insurance, similar to the present, committed against never agreed upon the terms and conditions of the - At the time of insurance of Policy 4219(April 19,
this defendant in the month of September, 1924; contract. We are clearly of the opinion that, if such 1962), an insurance policy for P20,000 issued by the
that in the same case the said Francisco Sanchez matters are known to exist by a preponderance of Great American Insurance Company covering the
was one of the coaccused of the said Go Chulian but the evidence, they would constitute a valid defense same properties was noted on said policy as co-
was discharged from the complaint, because he to plaintiff's cause of action. Upon the question as to insurance.
offered himself and was utilized as a state's witness; whether or not they or are not true, we do not at - August 29, 1962 : parties executed an
that there is another civil action now pending against this time have or express any opinion, but we are endorsement on Policy 4219 stating:
Go Chulian and Sanchez in the Court of First clear that section 47 does not apply to the It is hereby declared and agreed that the co-
Instance of Manila (civil case No. 28680), in which allegations made in the answer, and that the trial insurance existing at present under this policy is
the present defendant is the plaintiff, for the court erred in sustaining the demurrer. as follows: P20,000.00 � Northwest Ins., and not
recovery of the amounts of two insurance policies Disposition The judgment of the lower court is as originally stated. (emphasis supplied)
aggregating P19,000, fraudulently obtained by the reversed and the case is remanded for such other Except as varied by this endorsement, all other
said Go Chulian and Sanchez. and further proceedings as are not inconsistent with terms and conditions remain unchanged.
- To this, plaintiff filed a demurrer which was this opinion, with costs against the plaintiff. - September 26, 1962: Yap took out another fire
granted. insurance policy for P20,000 covering the same
PIONEER INSURANCE AND SURETY properties, from Federal Insurance Company. This
ISSUE CORPORATION v. YAP policy was procured without notice to and the
WON defense is barred by Art. 47 61 SCRA 426 written consent of Pioneer, and was therefore not
FERNANDEZ; December 19, 1974 noted as a co-insurance in Policy 4219.
HELD - December 19, 1962: Fire burned Yap’s store
NO NATURE
Ratio The word "rescind" has a well defined legal Appeal by certiorari from CA decision affirming a CFI ISSUE
meaning, and as applied to contracts, it presupposes decision which declared plaintiff Yap entitled to WON petitioner should be absolved from liability on
the existence of a contract to rescind. recover from defendant Pioneer Insurance and Fire insurance Policy No. 4219 on account of any
Reasoning Surety Corp, the full amount of the damage inquired violation by respondent Yap of the co-insurance
- Plaintiff vigorously contends that section 47 of the in Policy No. 4219 clause therein
Insurance Act should be applied, and that when so
applied, defendant is barred and estopped to plead FACTS HELD
and set forth the matters alleged in its special - Yap owned a store in a 2 storey building, where YES
defense. That section is as follows: she sold shopping bags and footwear. Her son-in-law - The petitioner should be absolved.
Whenever a right to rescind a contract of was in charge of the store Reasoning
insurance is given to the insurer by any provision - April 19, 1962- Yap took out Fire Insurance Policy - There was a violation by Yap of the co-insurance
of this chapter, such right must be exercised No. 4216 from Pioneer with a face value of P25,000 clause contained in Policy No. 4219 which resulted in
previous to the commencement of an action on the covering her stocks, office furniture, fixtures, etc. the avoidance of the petitioner’s liability.
contract. - among the conditions set forth: - By the plain terms of the policy, other insurance
- It will be noted that defendant does not seek to The Insured shall give notice to the without the consent of petitioner would ipso facto
have the alleged insurance contract rescinded. It Company of any insurance or insurances already avoid the contract. It required no affirmative act of
denies that it ever made any contract of insurance effected, or which may subsequently be election on the part of the company to make
on the life of Tan Ceang or that any such a contract effected, covering any of the property hereby operative the clause avoiding the contract, wherever
ever existed, and that is the question which it seeks insured, and unless such notice be given and the specified conditions should occur. Its obligations
to have litigated by its special defense. In the very the particulars of such insurance or insurances ceased, unless, being informed of the fact, it
nature of things, if the defendant never made or be stated in, or endorsed on this Policy by or on consented to the additional insurance.
entered into the contract in question, there is no behalf of the Company before the occurrence of - The obvious purpose of the aforesaid requirement
contract to rescind, and, hence, section 47 upon any loss or damage, all benefits under this in the policy is to prevent over-insurance and thus
which the lower based its decision in sustaining the Policy shall be forfeited. (emphasis supplied) avert the perpetration of fraud. The public, as well as
demurrer does not apply. As stated, an action to It is understood that, except as may be the insurer, is interested in preventing the situation
rescind a contract is founded upon and presupposes stated on the face of this policy there is no other in which a fire would be profitable to the insured.
INSURANCE Page 69

According to Justice Story: "The insured has no right HELD avert the perpetration of fraud. The public, as well as
to complain, for he assents to comply with all the YES the insurer, is interested in preventing the situation
stipulation on his side, in order to entitle himself to - Petitioners admit that the respective insurance in which a fire would be profitable to the insured.
the benefit of the contract, which, upon reason or policies issued by private respondents did not state The insured has no right to complain, for he assents
principle, he has no right to ask the court to or endorse thereon the other insurance coverage to comply with all the stipulations on his side, in
dispense with the performance of his own part of the obtained or subsequently effected on the same order to entitle himself to the benefit of the contract,
agreement, and yet to bind the other party to stocks in trade for the loss of which compensation is which, upon reason or principle, he has no right to
obligations, which, but for those stipulation would claimed by petitioners. It is further admitted by ask the court to dispense with the performance of his
not have been entered into." petitioners that Equitable's policy stated "nil" in the own part of the agreement, and yet to bind the other
Disposition the appealed judgment of the Court of space thereon requiring indication of any co- party to obligations, which, but for those
Appeals is reversed and set aside, and the petitioner insurance although there were 3 policies subsisting stipulations, would not nave been entered into.
absolved from all liability under the policy. on the same stocks in trade at the time of the loss, - It is not disputed that the insured failed to reveal
namely, that of Western in the amount of before the loss three other insurances. By reason of
NEW LIFE ENTERPRISES v. CA P350,000.00 and two 2 policies of Reliance in the said unrevealed insurances, the insured had been
207 SCRA 609 total amount of P1,000,000.00. guilty of a false declaration; a clear
REGALADO; March 31, 1992 - The coverage by other insurance or co-insurance misrepresentation and a vital one because where the
effected or subsequently arranged by petitioners insured had been asked to reveal but did not, that
FACTS were neither stated nor endorsed in the policies of was deception. Otherwise stated, had the insurer
- Julian Sy and Jose Sy Bang are partners engaged the 3 private respondents, warranting forfeiture of all known that there were many co-insurances, it could
in the business of selling construction materials benefits thereunder if we are to follow the express have hesitated or plainly desisted from entering into
under the business name “New Life Enterprises.” stipulation in Policy Condition No. 3. such contract. Hence, the insured was guilty of clear
Julian Sy insured against fire the stocks in trade of - The terms of the contract are clear and fraud.
New Life Enterprises with Western Guaranty unambiguous. The insured is specifically required to - As the insurance policy against fire expressly
Corporation, Reliance Surety and Insurance Co. Inc., disclose to the insurer any other insurance and its required that notice should be given by the insured
and Equitable Insurance Corporation in the particulars which he may have effected on the same of other insurance upon the same property, the total
aggregate amount of P1,550,000.00. When the subject matter. The knowledge of such insurance by absence of such notice nullifies the policy.
building where New Life Enterprises was located, the insurer's agents, even assuming the acquisition - Additionally, insofar as the liability of respondent
along with the stocks in trade therein, were gutted thereof by the former, is not the "notice" that would Reliance is concerned, it is not denied that the
by fire, petitioners filed an insurance claim against stop the insurers from denying the claim. Besides, complaint for recovery was filed in court by
the three companies. The insurance companies all the so-called theory of imputed knowledge, that is, petitioners only on January 31, 1984, or after more
denied Julian Sy’s claim on the ground of “breach of knowledge of the agent is knowledge of the principal, than one (1) year had elapsed from petitioners'
policy condition,” (i.e., the “other insurance” clause aside from being of dubious applicability here has receipt of the insurers' letter of denial on November
which required New Life Enterprises to inform each likewise been roundly refuted by respondent court 29, 1982.
of the insurance companies in case the former whose factual findings we find acceptable. The mere - The condition contained in an insurance policy that
insures with another company the same property fact that Yap Kam Chuan was an agent for both claims must be presented within one year after
already insured by each of the insurance Reliance and Equitable does not justify the allegation rejection is not merely a procedural requirement but
companies). that the two are sister companies. Availment of the an important matter essential to a prompt
- Because of the denial of their claims for payment services of the same agents and adjusters by settlement of claims against insurance companies as
by the 3 insurance companies, petitioners filed different companies is a common practice in the it demands that insurance suits be brought by the
separate civil actions against the former before the insurance business and such facts do not warrant the insured while the evidence as to the origin and cause
Regional Trial Court of Lucena City, which cases were speculative conclusion of the trial court. of destruction have not yet disappeared.
consolidated for trial. The trial court ruled in favor of - Considering the terms of the policy which required
petitioner. However, the Court of Appeals reversed the insured to declare other insurances, the QUA CHEE GAN v. LAW UNION (supra p.48)
the trial court’s decision, found petitioner to have statement in question must be deemed to be a
violated Clauses 3 and 27 of the separate insurance statement (warranty) binding on both insurer and
policies issued by the 3 companies, and exonerated insured, that there were no other insurance on the
the insurance companies from liability. property. The annotation then, must be deemed to
be a warranty that the property was not insured by
ISSUE any other policy. Violation thereof entitled the
WON petitioners violated the “Other Insurance insurer to rescind.
Clause” of the insurance policies - The obvious purpose of the aforesaid requirement YOUNG v. MIDLAND TEXTILE INSURANCE CO.
in the policy is to prevent over-insurance and thus 30 PHIL 617
INSURANCE Page 70

JOHNSON; March 31, 1915 contends that under all the facts and circumstances terms of the contract constitute the measure of the
of the case, they were not “stored” in said building, insurer's liability. If the contract has been
FACTS and that the placing of them in the building was not terminated, by a violation of its terms on the part of
- K.S. Young had a candy and fruit store on the a violation of the terms of the contract. the insured, there can be no recovery. Compliance
Escolta, Manila, and occupied a building at 321 Calle - Whether a particular article is "stored" or not must, with the terms of the contract is a condition
Claveria, as a residence and bodega. The Midland in some degree, depend upon the intention of the precedent to the right of recovery.
Textile Insurance Co. in consideration of the parties. Nearly all of the cases cited by the lower - Young argues that since the "storing" of the
payment of a premium of P60, entered into a court are cases where the article was being put to fireworks on the premises did not contribute in any
contract of insurance with Young by the terms of some reasonable and actual use, which might easily way to the damage occasioned by the fire, he should
which the company, upon certain conditions, have been permitted by the terms of the policy, and be permitted to recover. That argument, however, is
promised to pay Young the sum of P3,000 in case within the intention of the parties, and excepted beside the question, if the "storing" was a violation
said residence and bodega and contents should be from the operation of the warranty, like the present. of the terms of the contract. The violation of the
destroyed by fire. - (1) Where merchants have had or kept the terms of the contract, by virtue of the provisions of
- One of the conditions of the contract is: "Warranty "hazardous" articles in small quantities, and for the policy itself, terminated, at the election of either
B – It is hereby declared and agreed that during the actual daily use, for sale, such as gasoline, party, the contractual relations.
pendency of this policy no hazardous goods be gunpowder, etc.; (2) Where such articles have been - Young paid a premium based upon the risk at the
stored or kept for sale, and no hazardous trade or brought on the premises for actual use thereon, and time the policy was issued. Certainly, the placing of
process be carried on, in the building to which this in small quantities, such as oil, paints, etc; and (3) the firecrackers in the building insured increased the
insurance applies, or in any building connected Where such articles or goods were used for lighting risk. Young had not paid a premium based upon the
therewith." purposes, and in small quantities. increased risk, neither had the defendant issued a
- Young placed in the residence and bodega three - In the present case no claim is made that the policy upon the theory of a different risk. He was
boxes filled with fireworks. Said residence and "hazardous goods" were placed in the bodega for enjoying, if his contention may be allowed, the
bodega and the contents thereof were partially present or daily use. It is admitted that they were benefits of an insurance policy upon one risk,
destroyed by fire. placed in the bodega "for future use," or for future whereas, as a matter of fact, it was issued upon an
- The fireworks had been given to Young by the consumption, or for safe keeping. It seems clear to entirely different risk. The defendant had neither
former owner of the Luneta Candy Store. He us that the "hazardous goods" in question were been paid nor had issued a policy to cover the
intended to use them in the celebration of the "stored" in the bodega, as that word is generally increased risk. An increase of risk which is
Chinese New Year. However, the authorities of the defined. That being true, suppose the defendant had substantial and which is continued for a considerable
city of Manila had prohibited the use of fireworks on made an examination of the premises, even in the period of time, is a direct and certain injury to the
said occasion, so Young then placed them in the absence of a fire, and had found the "hazardous insurer, and changes the basis upon which the
bodega where they remained from the 4th or 5th of goods" there, would it not have been justified in contract of insurance rests.
February, 1913 until after the fire of March 18, 1913. declaring the policy null and of no effect by reason of Disposition Decision of the lower court is
- Both parties agree that the fireworks come within a violation of its terms? If it might, then may it not REVERSED.
the phrase "hazardous goods," mentioned in repudiate its liability, even after the fire? If the
"Warranty B" of the policy; that the fireworks were "warranty" is a term of the contract, will not its TAN v. CA (supra p.51)
found in a part of the building not destroyed by the violation cause a breach and justify noncompliance
fire and that they in no way contributed to the fire, or repudiation? AREOLA v. CA (supra p.26)
or to the loss that resulted. - Contracts of insurance are contracts of indemnity,
- The lower court rendered a judgment in favor of upon the terms and conditions specified therein. TAN CHAY v. WEST COAST (supra p.51)
Young for the sum of P2,708.78, and costs. Parties have a right to impose such reasonable
conditions at the time of the making of the contract FILIPINAS LIFE ASSURANCE v. NAVA
ISSUE as they deem wise and necessary. The rate of 17 SCRA 210
1. WON the placing of the fireworks in the building premium is measured by the character of the risk BAUTISTA ANGELO; May 20, 1966
insured, they being "hazardous goods," was a assumed. The insurer, for a comparatively small
violation of the terms of the contract of insurance consideration, undertakes to guarantee the insured NATURE
and especially of "Warranty B." against loss or damage, upon the terms and Petition for review of a decision of the Court of
conditions agreed upon, and upon no other. When Appeals
HELD the insurer is called upon to pay, in case of loss, he
1. YES. may justly insist upon a fulfillment of the terms of FACTS
Reasoning It is admitted by both parties that the the contract. If the insured, cannot bring himself - Before the war, Nava entered into a contract of
fireworks are hazardous goods. The defendant within the terms and conditions of the contract, he is insurance with Insular Life Assurance Co., Ltd. (face
alleged that they were "stored." The plaintiff not entitled to recover for any loss suffered. The value of P5k), and 17 separate contracts of life
INSURANCE Page 71

insurance with Filipinas Life Assurance Co. (total face Japanese occupation when they relate to life apparently did not give any importance to such
value of P90k). Each and everyone of the 18 policies insurance policies. decision for in their opinion it does not have any
issued by defendants to plaintiff contains a loan - Feb 4, 1949: Nava was again refused even if the application to transactions which have any relation to
clause of the following tenor: total amount of the cash surrender values of the 18 payment of premiums on life insurance policies.
Policy loans. After three full years' policies reached the sum of P9,468.29. - It cannot be denied that a life insurance policy
premiums have been paid upon this Policy, if no - Feb 10, 1949: Nava brought case to the CFI Manila involves a contractual obligation wherein the insured
premium payment is in default, the Company, praying for the rescission of the abovementioned 18 becomes duty bound to pay the premiums agreed
subject to its then existing rules, will advance on policies and for the refund to him of all the premiums upon, lest he runs the risk of having his insurance
proper assignment and delivery of this Policy and so far paid by him to defendants in the amount of policy lapse if he fails to pay such premiums.
on the sole security thereof a sum equal to, or at P31,633.80, plus 6% interest thereon as damages - The fact that if the insured had paid in full the
the option of the owner less than, the cash value - Nov 28, 1951: companies passed a resolution premiums corresponding to the first 3 years of the
specified in the Schedule of Policy Values, less any which was approved by the Insurance Commissioner, life of his policy he cannot be considered delinquent
existing indebtedness on or secured by this Policy giving full credit to all premium payments made by that would cause the lapse of his policy if the same
and any unpaid balance of the premium for the their policyholders in fiat currency during the contains an automatic premium payment clause
current policy-year; provided interest at six per Japanese occupation on account of pre-war policies cannot divest such policy of its contractual nature,
centum per annum on the whole amount of the for which reason they filed an amended answer for the result of such failure would only be for him to
loan is paid in advance to the end of the current offering to pay plaintiff the amount of P9,468.29 pay later the premium plus the corresponding
policy-year. At the end of the current policy-year which represents the aggregate cash surrender interest depending upon the condition of the policy.
interest at the same rate for one year in advance values of all the policies in question as of February But certainly it does not cease to be a contractual
will be due and payable, and annually thereafter, 10, 1949, but apparently this offer was refused. liability insofar as the payment of that premium is
and if not so paid will be added to the principal - CFI: (1) rescinded the insurance contracts; (2) concerned for whether he likes it or not that
and bear the same rate of interest. Failure to ordered defendant Filipinas Life Assurance Co. to pay premium has to be paid lest he allows the lapse of
repay any such loan or interest shall not avoid this plaintiff the amount of P32,072.60; and (3) ordered his policy. Consequently, the payment of premiums
Policy unless the total indebtedness shall equal or defendant Insular Life Assurance Co., Ltd. to pay on the life insurance policies made by Nava before
exceed the full amount of the loan value available plaintiff the amount of P2,574.00 and during the war up to the time he applied for the
hereunder. - CA affirmed. loan in question with petitioners should be
Any indebtedness on this Policy shall first considered likewise as valid payments upon the
be deducted from any money payable or in any ISSUES theory that such insurance policies are in the nature
settlement under this Policy. 1. WON CA erred in ruling that as a consequence of of a contractual obligation within the meaning of the
- Nava had so far paid to Insular a total of P2,574; the decision in the Haw Pia case petitioners violated civil law. In effect, therefore, those payments were
and to Filipinas Life, a total of P32,072.60. the loan clause contained in the insurance policies made by a debtor to a creditor within the meaning of
- April 28, 1948: Nava applied to the companies for thereby entitling respondent to their rescission the requirement of the regulations of the Insurance
a P5k loan in line with the loan clause, but they 2. WON CA erred in ruling that by virtue of Article Commissioner and as such they can offer no excuse
refused to grant it because certain regulations issued 1295 of the old Civil Code petitioners should refund to petitioners for refusing to grant the loan as
by the Insurance Commissioner required the to defendant all the premiums paid on his insurance contemplated in the loan clause embodied in the
insurance companies to withhold the payments on policies as a consequence of their rescission policies in question.
premiums made during the Japanese occupation 3. WON CA erred in not ruling that, even if - It is clear from the foregoing that the petitioners
because the same shall be subject to future respondent is entitled to the rescission of said violated the loan clause embodied in each of the 18
adjustments " as soon as debtor-creditor relationship insurance policies, he can only recover their cash life insurance policies issued to respondent to rescind
is established" and because of such process of surrender value at the time the complaint was filed all said policies under Section 69 of the Insurance
"withholding" plaintiff was not entitled to borrow any Act, which provides: "The violation of a material
amount until such adjustment has been made. HELD warranty, or other material provision of a policy, on
- Sept 30, 1948: Nava called the attention of the 1. NO. the part of either party thereto, entitles the other to
insurance companies to the SC decision (Haw Pia v. - Even assuming the validity of the Insurance rescind."
China Banking Corporation) establishing and Commissioner’s regulations, the fact however is that - "The general rule is that a breach of the agreement
recognizing the relationship of debtor and creditor such requirement has already lost its legal effect and to make the loan does not entitle the insured to
with respect to payments in fiat currency made value when our Supreme Court rendered its decision rescind the contract," is not controlling in this
during the Japanese occupation on pre-war in the Haw Pia case wherein it was declared, among jurisdiction. Firstly, it was not shown that the
obligations. others, that all payments made in fiat currency insurance laws in the states where said ruling
- Companies still refused saying that the SC decision during the Japanese occupation in relation with any prevails contain a provision identical to Section 69 of
was not applicable to transactions undertaken during contractual obligation executed before the war were our Insurance Law we quoted above, and secondly,
valid to all intents and purposes, and yet petitioners the rule cited by Vance is not a rule uniformly
INSURANCE Page 72

followed by all states in the US, for on this matter Disposition Decision appealed from is AFFIRMED. - The coming of the men with the torch was to be
there is a marked divergence of opinion. Costs against petitioners expected, and was a natural sequence of the
2. NO overturning of the bus, the trapping of some of its
- Considering that our Insurance Law does not CHAPTER VII. RISKS AND COVERAGES passengers and the call for outside help.
contain an express provision as to what the court - The burning of bus can also in part be attributed to
should do in cases of rescission of an insurance VDA. DE BATACLAN v. MEDINA negligence of carrier, through its driver and
policy under Section 69, the provision that should 102 PHIL 181 conductor. They, or at least the driver, should have
apply is that embodied in Article 1225 of the old Civil MONTEMAYOR; October 22, 1957 known that in the position in which the overturned
Code, as postulated in Article 16 of the same Code, bus was, gasoline could and must have leaked from
which provides that on matters which are not FACTS the gasoline tank and soaked the area in and around
governed by special laws the provisions of said Code - Juan Bataclan rode Bus 30 of Medina Transport, the bus. Gasoline can be smelt and detected even
shall supplement its deficiency. And said Article 1295 driven by Saylon, shortly after midnight. The bus from a distance, and yet neither the driver nor the
provides: was running very fast. One of the front tires burst. conductor would appear to have cautioned or taken
ART. 1295. Rescission makes necessary the return Bus fell into canal and turned turtle. 4 passengers steps to warn rescuers not to bring the lighted torch
of the things which were the subject-matter of the couldn’t get out, including Bataclan. Gasoline began too near the bus.
contract, with their fruits, and of the price paid, to leak from the overturned bus. 10 men came to
with interest thereon. ...xxx help. 1 carried a torch and when he approached the FINMAN GENERAL ASSURANCE CORPORATION
- Said the petitioners: "Recovery of the full amount bus, fire started, killing the trapped passengers. v. CA (SURPOSA)
of the premium after the insurer has sustained for - TC opined that proximate cause of Bataclan’s 213 SCRA 493
sometime the risk of the insurance and the insured death was not the overturning of bus but the fire. At NOCON; September 2, 1992
has enjoyed the benefit of protection is obviously the time fire started, Bataclan, though injured, was
unjust and is so recognized by the better still alive and damages were awarded, not for his NATURE
authorities." The ruling above quoted merely death, but for physical injuries suffered. Certiorari
represents the minority rule in the US, the majority
rule being that the insured can recover all premiums ISSUE/S FACTS
paid, in some cases with interest in case of wrongful WON the proximate cause is the overturning of the - Oct. 22, 1986: Carlie Surposa was insured with
cancellation, repudiation, termination or rescission of bus or the fire Finman General Assurance Corporation under Finman
the contract of life insurance. General Teachers Protection Plan Master Policy No.
- Contention that because respondent cannot restore HELD 2005 and Individual Policy No. 08924 with his
to petitioners the "value of the benefit of protection" - The proximate cause is the overturning of the bus. parents, spouses Julia and Carlos Surposa, and
which he might have received under the 18 life - Ordinarily, when a bus overturns and pins down brothers Christopher, Charles, Chester and Clifton,
insurance policies in question he is not entitled to passenger, merely causing him injuries. If through all surnamed, Surposa, as beneficiaries.
rescind them under the provision of Article 1295 of some event, unexpected and extraordinary, the bus - While said insurance policy was in full force and
the old Civil Code, is untenable because said article is set on fire, and passenger is burned to death, one effect, the insured, Carlie Surposa, died on October
only contemplates a transaction whether material might contend that the proximate cause was the fire 18, 1988 as a result of a stab wound inflicted by one
things are involved, and do not refer to intangible and not the overturning of the vehicle. of the 3 unidentified men without provocation and
ones which cannot be the subject of restoration, for - But here, the proximate cause of Bataclan’s death warning on the part of the former as he and his
to interpret it otherwise would be to defeat the law is the overturning of the bus, this for the reason that cousin, Winston Surposa, were waiting for a ride on
itself with the result that rescission can never be had when the vehicle turned not only on its side but their way home after attending the celebration of the
under Section 69 of our Insurance Law. completely on its back, leaking of gasoline from the "Maskarra Annual Festival."
- It cannot be denied that petitioners had in turn tank was not unnatural or unexpected. - Thereafter, Julia Surposa and the other
already derived material benefits from the use of - The coming of the men with the torch was in beneficiaries of said insurance policy filed a written
premiums paid to them by respondent before, during response to the call for help, made only not by the notice of claim with the FINMAN Corp which denied
and after the last war from which they must have passengers but even the driver and conductor, and said claim contending that murder and assault are
realized huge profits, and in this light alone because it was very dark, about 2:30 am, rescuers not within the scope of the coverage of the insurance
petitioners cannot claim prejudice or unfairness if had to carry a light with them. Coming as they did policy.
they are ordered to refund the premiums paid by from rural area where lanterns and flashlights were - Feb. 24, 1989: Surposa filed a complaint with the
respondents. not available, they had to use a torch. What was Insurance Commission which subsequently ordered
3. NO. more natural than that said rescuers should FINMAN to pay Surposa the proceeds of the policy
- Issue is corollary to preceding issue. No need to innocently approach the overturned vehicle to extend with interest.
refute. aid. - CA affirmed said decision.
INSURANCE Page 73

ISSUE the policies insuring against death or injury from in the contract and have the effect of exempting the
WON CA committed GAD in applying the principle of accident. company from liability.
"expresso unius exclusio alterius" in a personal - The personal accident insurance policy involved - It is contended in behalf of the company that
accident insurance policy (since death resulting from herein specifically enumerated only 10 circumstances Basilio was killed which "making an arrest as an
murder and/or assault are impliedly excluded in said wherein no liability attaches to FINMAN for any officer of the law" or as a result of an "assault or
insurance policy considering that the cause of death injury, disability or loss suffered by the insured as a murder" committed in the place and therefore his
of the insured was not accidental but rather a result of any of the stimulated causes. death was caused by one of the risks excluded by
deliberate and intentional act of the assailant in -The principle of " expresso unius exclusio alterius" the supplementary contract which exempts the
killing the former as indicated by the location of the the mention of one thing implies the exclusion of company from liability. This contention was upheld
lone stab wound on the insured) [TF they cannot be another thing is therefore applicable in the instant by the Court of Appeals. Hence, this petition.
made to indemnify the Surposa heirs] case since murder and assault, not having been
expressly included in the enumeration of the ISSUE
HELD circumstances that would negate liability in said WON the death of the victim comes within the
NO insurance policy: the failure of the FINMAN to purview of the exception clause of the
- The record is barren of any circumstance showing include death resulting from murder or assault supplementary policy and, hence, exempts the
how the stab wound was inflicted. While the act may among the prohibited risks leads inevitably to the company from liability
not exempt the unknown perpetrator from criminal conclusion that it did not intend to limit or exempt
liability, the fact remains that the happening was a itself from liability for such death. HELD
pure accident on the part of the victim. The insured - A1377 NCC: The interpretation of obscure words NO
died from an event that took place without his or stipulations in a contract shall not favor the party - Basilio was a watchman of the Manila Auto Supply
foresight or expectation, an event that proceeded who caused the obscurity. which was a block away from the house of Atty.
from an unusual effect of a known cause and, - NPC vs. CA [1986]~ It is well settled that Ojeda where something suspicious was happening
therefore, not expected. contracts of insurance are to be construed liberally in which caused the latter to ask for help. While at first
Reasoning favor of the insured and strictly against the insurer. he declined the invitation of Atty. Ojeda to go with
- De la Cruz vs. Capital Insurance & Surety Co., Inc Thus ambiguity in the words of an insurance contract him to his residence to inquire into what was going
(1966)~ The terms "accident" and "accidental" as should be interpreted in favor of its beneficiary. on because he was not a regular policeman, he later
used in insurance contracts have not acquired any Disposition DENIED for lack of merit. agreed to come along when prompted by the traffic
technical meaning, and are construed by the courts policeman, and upon approaching the gate of the
in their ordinary and common acceptation. Thus, the CALANOC v. CA residence he was shot and died. The circumstance
terms have been taken to mean that which happen 98 PHIL 79 that he was a mere watchman and had no duty to
by chance or fortuitously, without intention and BAUTISTA; December 16, 1955 heed the call of Atty. Ojeda should not be taken as a
design, and which is unexpected, unusual, and capricious desire on his part to expose his life to
unforeseen. An accident is an event that takes place FACTS danger considering the fact that the place he was in
without one's foresight or expectation an event that - Basilio was a watchman of the Manila Auto Supply duty-bound to guard was only a block away. In
proceeds from an unknown cause, or is an unusual located at the corner of Avenida Rizal and Zurbaran. volunteering to extend help under the situation, he
effect of a known cause and, therefore, not He secured a life insurance policy from the Philippine might have thought, rightly or wrongly, that to know
expected. American Life Insurance Company in the amount of the truth was in the interest of his employer it being
Ratio The generally accepted rule is that, death or P2,000 to which was attached a supplementary a matter that affects the security of the
injury does not result from accident or accidental contract covering death by accident. On January 25, neighborhood. No doubt there was some risk coming
means within the terms of an accident-policy if it is 1951, he died of a gunshot wound on the occasion of to him in pursuing that errand, but that risk always
the natural result of the insured's voluntary act, a robbery committed in the house of Atty. Ojeda at existed it being inherent in the position he was
unaccompanied by anything unforeseen except the the corner of Oroquieta and Zurbaran streets. holding. He cannot therefore be blamed solely for
death or injury. There is no accident when a Calanoc, the widow, was paid the sum of P2,000, doing what he believed was in keeping with his duty
deliberate act is performed unless some additional, face value of the policy, but when she demanded the as a watchman and as a citizen. And he cannot be
unexpected, independent, and unforeseen happening payment of the additional sum of P2,000 considered as making an arrest as an officer of the
occurs which produces or brings about the result of representing the value of the supplemental policy, law, as contended, simply because he went with the
injury or death. In other words, where the death or the company refused alleging, as main defense, that traffic policeman, for certainly he did not go there for
injury is not the natural or probable result of the the deceased died because he was murdered by a that purpose nor was he asked to do so by the
insured's voluntary act, or if something unforeseen person who took part in the commission of the policeman.
occurs in the doing of the act which produces the robbery and while making an arrest as an officer of - Much less can it be pretended that Basilio died in
injury, the resulting death is within the protection of the law which contingencies were expressly excluded the course of an assault or murder considering the
very nature of these crimes. In the first place, there
INSURANCE Page 74

is no proof that the death of Basilio is the result of TEEHANKEE [dissent]


either crime for the record is barren of any HELD - Calanoc v. CA is controlling in this case because
circumstance showing how the fatal shot was fired. YES the insurance company wasn’t able to prove that the
Perhaps this may be clarified in the criminal case - Whether the robbers had the intent to kill or merely killing was intentional. (Burden of proof is with the
now pending in court as regards the incident but to scare the victim or to ward off any defense he insurance company)
before that is done anything that might be said on might offer, it cannot be denied that the act itself of - Insurance, being contracts of adhesion, must be
the point would be a mere conjecture. Nor can it be inflicting the injuries was intentional. construed strictly against insurance company in
said that the killing was intentional for there is the - The exception in the accidental benefit clause cases of ambiguity.
possibility that the malefactor had fired the shot invoked by the appellant does not speak of the - The supplementary contract enumerated
merely to scare away the people around for his own purpose — whether homicidal or not — of a third exceptions. The only exception which is not
protection and not necessarily to kill or hit the party in causing the injuries, but only of the fact that susceptible of classification is that provided in
victim. In any event, while the act may not exempt such injuries have been "intentionally" inflicted — paragraph 5(e), the very exception herein involved,
the triggerman from liability for the damage done, this obviously to distinguish them from injuries which would also except injuries "inflicted
the fact remains that the happening was a pure which, although received at the hands of a third intentionally by a third party, either with or without
accident on the part of the victim. The victim could party, are purely accidental. provocation on the part of the insured, and whether
have been either the policeman or Atty. Ojeda for it - Examples of unintentional: or not the attack or the defense by the third party
cannot be pretended that the malefactor aimed at >> A gun which discharges while being cleaned and was caused by a violation of the law by the insured."
the deceased precisely because he wanted to take kills a bystander; - This ambiguous clause conflicts with all the other
his life. >> a hunter who shoots at his prey and hits a four exceptions in the same paragraph 5 particularly
Disposition Decision set aside person instead; that immediately preceding it in item (d) which
>> an athlete in a competitive game involving excepts injuries received where the insured has
BIAGTAN v. THE INSULAR LIFE ASSURANCE physical effort who collides with an opponent and violated the law or provoked the injury, while this
COMPANY, LTD. fatally injures him as a result. clause, construed as the insurance company now
44 SCRA 58 - In Calanoc vs. CA: Where a shot was fired and it claims, would seemingly except also all other
MAKALINTAL; March 29, 1972 turned out afterwards that the watchman was hit in injuries, intentionally inflicted by a third party,
the abdomen, the wound causing his death, the regardless of any violation of law or provocation by
NATURE Court held that it could not be said that the killing the insured, and defeat the very purpose of the
Appeal from decision of CFI Pangasinan. was intentional for there was the possibility that the policy of giving the insured double indemnity in case
malefactor had fired the shot to scare the people of accidental death by "external and violent means"
FACTS around for his own protection and not necessarily to — in the very language of the policy.'
- Juan Biagtan was insured with Insular for P5k and kill or hit the victim. A similar possibility is clearly - It is obvious from the very classification of the
a supplementary contract “Accidental Death Benefit” ruled out by the facts in this case. For while a single exceptions and applying the rule of noscitus a sociis,
clause for another P5k if "the death of the Insured shot fired from a distance, and by a person who was that the double-indemnity policy covers the insured
resulted directly from bodily injury effected solely not even seen aiming at the victim, could indeed against accidental death, whether caused by fault,
through external and violent means sustained in an have been fired without intent to kill or injure, nine negligence or intent of a third party which is
accident . . . and independently of all other causes." wounds inflicted with bladed weapons at close range unforeseen and unexpected by the insured. All the
The clause, however, expressly provided that it cannot conceivably be considered as innocent insofar associated words and concepts in the policy plainly
would not apply where death resulted from an injury as such intent is concerned. exclude the accidental death from the coverage of
"intentionally inflicted by a third party." - In Hucthcraft's Ex'r vs. Travelers' Ins. Co. (US the policy only where the injuries are self-inflicted or
- One night, a band of robbers entered their house. case): where the insured was waylaid and attended by some proscribed act of the insured or
Juan went out of his room and he was met with 9 assassinated for the purpose of robbery, the court are incurred in some expressly excluded calamity
knife stabs. He died. The robbers were convicted of rendered judgment for the insurance company and such as riot, war or atomic explosion.
robbery with homicide. held that while the assassination of the insured was - The untenability of insurer's claim that the
- The family was claiming the additional P5k from as to him an unforeseen event and therefore insured's death fell within the exception is further
Insular, under the Accidental Death Benefit clause. accidental, "the clause of the proviso that excludes heightened by the stipulated fact that two other
Insular refused on the ground that the death the (insurer's) liability, in case death or injury is insurance companies which likewise covered the
resulted from injuries intentionally inflicted by 3 rd intentionally inflicted by any other person, applies to insured for much larger sums under similar
parties and was therefore not covered. Biagtans filed this case." accidental death benefit clauses promptly paid the
against Insular. CFI ruled in favor of Biagtans. Disposition CFI decision reversed. benefits thereof to plaintiffs beneficiaries.

ISSUE SEPARATE OPINION SUN INSURANCE v. CA (LIM)


WON the injuries were intentionally inflicted 211 SCRA 554
INSURANCE Page 75

CRUZ; July 17, 1992 - The petitioner, however, cites one of the four company for payment of the indemnity, but it was
exceptions provided for in the insurance contract and denied.
FACTS contends that the private petitioner's claim is barred - He instituted the action in the CFI of Pangasinan
- The petitioner issued Personal Accident Policy to by such provision. It is there stated: for specific performance.
Felix Lim, Jr. with a face value of P200,000.00. Two Exceptions —The company shall not be liable in - Defendant insurer set up the defense that the
months later, he was dead with a bullet wound in his respect of. death of the insured, caused by his participation in a
head. As beneficiary, his wife Nerissa Lim sought 1. Bodily injury. boxing contest, was not accidental and, therefore,
payment on the policy but her claim was rejected. xxx xxx xxx not covered by insurance
The petitioner agreed that there was no suicide. It b. consequent upon. - The court rendered the decision in favor of the
argued, however, that there was no accident either. i) The insured persons attempting to commit plaintiff, hence, the present appeal.
Pilar Nalagon, Lim's secretary, was the only suicide or wilfully exposing himself to needless peril
eyewitness to his death. According to Nalagon, Lim except in an attempt to save human life. ISSUE
was in a happy mood (but not drunk) and was - To repeat, the parties agree that Lim did not WON the death of the insured was not accidental
playing with his handgun, from which he had commit suicide. Nevertheless, the petitioner and, therefore, not covered by insurance
previously removed the magazine. As she watched contends that the insured willfully exposed himself to HELD
the television, he stood in front of her and pointed needless peril and thus removed himself from the NO
the gun at her. She pushed it aside and said it might coverage of the insurance policy. That posture is - The terms "accident" and "accidental", as used in
be loaded. He assured her it was not and then arguable. But what is not is that, as the secretary insurance contracts, have not acquired any technical
pointed it to his temple. The next moment there was testified, Lim had removed the magazine from the meaning, and are construed by the courts in their
an explosion and Lim slumped to the floor. He was gun and believed it was no longer dangerous. He ordinary and common acceptation. Thus, the terms
dead before he fell. expressed assured her that the gun was not loaded. have been taken to mean that which happen by
- The term "accident" has been defined as follows: It is submitted that Lim did not willfully expose chance or fortuitously, without intention and design,
The words "accident" and "accidental" have never himself to needless peril when he pointed the gun to and which is unexpected, unusual, and unforeseen.
acquired any technical signification in law, and his temple because the fact is that he thought it was An accident is an event that takes place without
when used in an insurance contract are to be not unsafe to do so. The act was precisely intended one's foresight or expectation, an event that
construed and considered according to the to assure Nalagon that the gun was indeed harmless. proceeds from an unknown cause, or is an unusual
ordinary understanding and common usage and Disposition CA Affirmed effect of a known cause and, therefore, not
speech of people generally. In substance, the expected.
courts are practically agreed that the words DE LA CRUZ v. CAPITAL INSURANCE - The generally accepted rule is that, death or injury
"accident" and "accidental" mean that which 17 SCRA 554 does not result from accident or accidental means
happens by change or fortuitously, without BARRERA; June 30, 1966 within the terms of an accident-policy if it is the
intention or design, and which is unexpected, natural result of the insured's voluntary act,
unusual, and unforeseen. The definition that has NATURE unaccompanied by anything unforeseen except the
usually been adopted by the courts is that an Appeal from the decision of the CFI of Pangasinan death or injury. There is no accident when a
accident is an event that takes place without one's deliberate act is performed unless some additional,
foresight or expectation — an event that proceeds FACTS unexpected, independent, and unforeseen happening
from an unknown cause, or is an unusual effect of - Eduardo de la Cruz, employed in the Itogon-Suyoc occurs which produces or brings about the result of
a known case, and therefore not expected. Mines, Inc., was the holder of an accident insurance injury or death. In other words, where the death or
- An accident is an event which happens without any policy underwritten by the Capital Insurance & injury is not the natural or probable result of the
human agency or, if happening through human Surety Co., Inc., for the period beginning November insured's voluntary act, or if something unforeseen
agency, an event which, under the circumstances, is 13, 1956 to November 12, 1957. occurs in the doing of the act which produces the
unusual to and not expected by the person to whom - On January 1, 1957, the Itogon-Suyoc Mines, Inc. injury, the resulting death is within the protection of
it happens. It has also been defined as an injury sponsored a boxing contest wherein the insured policies insuring against death or injury from
which happens by reason of some violence or Eduardo de la Cruz participated. accident.
casualty to the insured without his design, consent, - In the course of his bout, Eduardo slipped and was - In the present case, while the participation of the
or voluntary co-operation. hit by his opponent on the left part of the back of the insured in the boxing contest is voluntary, the injury
head, causing Eduardo to fall, with his head hitting was sustained when he slid, giving occasion to the
ISSUE the rope of the ring. infliction by his opponent of the blow that threw him
WON what happened was an accident - He was brought to the Baguio General Hospital, but to the ropes of the ring.
he died as a result of hemorrhage, intracranial, left. - The fact that boxing is attended with some risks of
HELD - Simon de la Cruz, the father and named beneficiary external injuries does not make any injuries received
YES of the insured, filed a claim with the insurance in the course of the game not accidental. In boxing
INSURANCE Page 76

as in other equally physically rigorous sports, such were saved were loaded to another vessel for liable for the loss, damage, or deterioration of the
as basketball or baseball, death is not ordinarily delivery to their original ports of destination. ESLI goods transported by them but who, among the
anticipated to result. If, therefore, it ever does, the charged the consignees several amounts carrier, consignee or insurer of the goods, is liable
injury or death can only be accidental or produced by corresponding to additional freight and salvage for the additional charges or expenses incurred by
some unforeseen happening or event as what charges. the owner of the ship in the salvage operations and
occurred in this case. - The charges were all paid by Philippine Home in the transshipment of the goods via a different
- Furthermore, the policy involved herein specifically Assurance Corporation (PHAC) under protest for and carrier. In absolving respondent carrier of any
excluded from its coverage: in behalf of the consignees. PHAC, as subrogee of liability, CA sustained the trial court's finding that the
(e) Death or disablement consequent upon the the consignees, then filed a complaint before the fire that gutted the ship was a natural disaster or
Insured engaging in football, hunting, pigsticking, RTC of Manila, against ESLI to recover the sum paid calamity.
steeplechasing, polo-playing, racing of any kind, under protest on the ground that the same were
mountaineering, or motorcycling. actually damages directly brought about by the fault, ISSUE
- Death or disablement resulting from engagement in negligence, illegal act and/or breach of contract of WON the burning of the SS Eastern Explorer
boxing contests was not declared outside of the ESLI. rendering it a constructive total loss was a natural
protection of the insurance contract. Failure of the - ESLI contended that it exercised the diligence disaster or calamity
defendant insurance company to include death required by law in the handling, custody and carriage
resulting from a boxing match or other sports among of the shipment; that the fire was caused by an HELD
the prohibitive risks leads inevitably to the unforeseen event; that the additional freight charges NO
conclusion that it did not intend to limit or exempt are due and demandable pursuant to the Bill of Ratio In our jurisprudence, fire may not be
itself from liability for such death. Lading; and that salvage charges are properly considered a natural disaster or calamity since it
Disposition The decision appealed from is affirmed collectible under Act No. 2616, known as the Salvage almost always arises from some act of man or by
Law. human means. It cannot be an act of God unless
FORTUNE INSURANCE v. CA (supra p.7) - RTC: dismissed PHAC's complaint and ruled in caused by lightning or a natural disaster or casualty
favor of ESLI. not attributable to human agency.
PHIL HOME ASSURANCE CORP v. CA (EASTERN - The burning of the vessel was not the fault or Reasoning
SHIPPING) negligence of defendant but a natural disaster or - There was no showing, and none was alleged by
257 SCRA 468 calamity. Salvage operations conducted by Fukuda the parties, that the fire was caused by a natural
KAPUNAN; June 20, 1996 Salvage Company was perfectly a legal operation disaster. On the contrary, there is strong evidence
and charges made on the goods recovered were indicating that the acetylene cylinder caught fire
NATURE legitimate charges. Section 19 of Act No. 2616, the because of the fault and negligence of respondent
- Eastern Shipping Lines, Inc. loaded on board SS Salvage Law is applicable. With respect to the ESLI, its captain and its crew:
Eastern Explorer in Kobe, Japan, the following additional freight charged by defendant from the (1) The acetylene cylinder which was fully loaded
shipment for carriage to Manila and Cebu, freight consignees of the goods, the same are also validly should not have been stored near the engine room
pre-paid and in good order and condition: (a) 2 demandable. where the heat generated therefrom could cause the
boxes internal combustion engine parts, consigned to - The burning of "EASTERN EXPLORER" while off acetylene cylinder to explode by reason of
William Lines, Inc.; (b) 10 metric tons (334 bags) Okinawa rendered it physically impossible for spontaneous combustion. ESLI should have easily
ammonium chloride, consigned to Orca's Company; defendant to comply with its obligation of delivering foreseen that the acetylene cylinder, containing
(c) 200 bags Glue 300, consigned to Pan Oriental the goods to their port of destination pursuant to the highly inflammable material, was in a real danger of
Match Company; and (d) garments, consigned to contract of carriage. Under Article 1266 of the Civil exploding.
Ding Velayo. All consignations were made by virtue Code, the physical impossibility of the prestation (2) ESLI should have known that by storing the
of a Bill of Lading. extinguished defendant's obligation. acetylene cylinder in the accommodation area
- While the vessel was off Okinawa, a small flame - Note: The goods subject of the present supposed to be reserved for passengers, it
was detected on the acetylene cylinder located in the controversy were neither lost nor damaged in transit unnecessarily exposed its passengers to grave
accommodation area near the engine room. As the by the fire that razed the carrier. In fact, these were danger and injury.
crew was trying to extinguish the fire, the cylinder all delivered to the consignees, even if the (3) The fact that the acetylene cylinder was checked,
suddenly exploded, thus causing death and severe transshipment took longer than necessary. What is tested and examined and subsequently certified as
injuries to the crew and instantly setting fire to the at issue therefore is not whether or not the carrier is having complied with the safety measures and
whole vessel. standards by qualified experts before it was loaded
9
- SS Eastern Explorer was then found to be a Section 1. When in case of shipwreck, the vessel or its cargo shall be beyond the control of the in the vessel only shows to a great extent that
constructive total loss and its voyage was declared crew, or shall have been abandoned by them, and picked up and conveyed to a safe place by other negligence was present in the handling of the
persons, the latter shall be entitled to a reward for the salvage.
abandoned. Those who, not being included in the above paragraph, assist in saving a vessel or its cargo from acetylene cylinder after it was loaded and while it
- After the fire was extinguished, the cargoes which shipwreck, shall be entitled to like reward. was on board the ship.
INSURANCE Page 77

- From the foregoing premises, it indubitably follows sought reconsideration but was denied, saying premium that the obligation to pay the margin fee
that the cargo consignees cannot be made liable to reinsurance treaty NOT EXEMPTED. arises.
respondent carrier for additional freight and salvage 2. NO
charges. ISSUES Ratio. Existing laws form part of the contract "as the
Disposition Judgment appealed from is REVERSED 1. WON the premia remitted were in pursuance of measure of the obligation to perform them by the
and SET ASIDE. Respondent Eastern Shipping Lines, the reinsurance treaty between Philamlife and Airco one party and the right acquired by the other. If the
Inc. is ORDERED to return to petitioner Philippine of January 1959, a contract antedating the Margin obligation does not inhere and subsist in the
Home Assurance Corporation the amount it paid Law, and therefore, Philamlife exempted from paying contract itself, propio vigore, but in the law
under protest in behalf of the consignees herein. margin fee applicable to the contract.
2. WON Margin Law impairs the obligation of contract
3. WON reinsurance contracts abroad would be made
impractical by the imposition of the 25% margin fee
Reasoning
HELD - . When petitioner entered into the reinsurance
1. NO treaty of January 1, 1950 with Airco, it did so with
- For an exemption to come into play, there must be the understanding that the municipal laws of the
a reinsurance policy or, as in the reinsurance treaty Philippines at the time said treaty was executed,
PHILIPPINE AMERICAN LIFE INSURANCE provided, a "reinsurance cession" which may be became an unwritten condition thereof. Such
COMPANY v. THE AUDITOR GENERAL automatic or facultative. municipal laws constitute part of the obligation of
22 SCRA 135 Ratio A reinsurance policy is thus a contract of contract.
SANCHEZ, JANUARY 18, 1968 indemnity one insurer makes with another to protect -Rationale of Margin Law: to reduce the excessive
the first insurer from a risk it has already assumed. . demand on and prevent further decline of our
NATURE . . In contradistinction, a reinsurance treaty is international reserves; to provide the Central Bank
PETITION FOR REVIEW of a ruling of the Auditor merely an agreement between two insurance with an additional instrument for effectively coping
General. companies whereby one agrees to cede and the with the problem and achieving domestic and
other to accept reinsurance business pursuant to international stability of our currency; to reduce the
FACTS provisions specified in the treaty. The practice of excessive demand-for foreign exchange.
- Philamlife, a domestic life insurance corp., and issuing policies by insurance companies includes, - implementation of Margin Law in accordance with
American International Reinsurance Company among other things, the issuance of reinsurance police power
(Airco), a corporation organized under the laws of policies on standard risks and also on substandard 3. NO
the Republic of Panama, entered into a risks under special arrangements. The lumping of the Reasoning
REINSURANCE TREATY wherein Philamlife agrees to different agreements under a contract has resulted - First, there is no concrete evidence that such
reinsure with Airco on January 1950. Philamlife in the term known to the insurance world as imposition of the 25% margin fee is unreasonable,
agreed to pay premiums for all reinsurances on an 'treaties.' Such a treaty is, in fact, an agreement Second, if really continuance of the existing
annual premium basis. between insurance companies to cover the different reinsurance treaty becomes unbearable, that
- In July 16, 1959, the Margin Law was approved situations described. Reinsurance treaties and contract itself provides that petitioner may
and became effective, which exempts certain reinsurance policies are not synonymous. Treaties potestatively write finis thereto on ninety days'
“obligations from payment of margin fees, are contracts for insurance; reinsurance written notice. Petitioner is not forced to continue
particularly contractual obligations calling for policies or cessions are contracts of insurance. its reinsurance treaty indefinitely with Airco.
payment of foreign exchange issued, approved and Reasoning Disposition For the reasons given, the petition for
outstanding as of the date this Act takes place”. - Even if reinsurance treaty preceded the Margin Law review is hereby denied, and the ruling of the
- Central Bank of the Philippines collected by over nine years, nothing in the treaty obligates Auditor General of October 24, 1961 denying refund
P268,747.48 as foreign exchange margin on Philamlife to remit to Airco a fixed, certain, and is hereby affirmed.
Philamlife remittances to Airco purportedly totalling obligatory sum by way of reinsurance premiums. The Costs against petitioner. So ordered.
$610,998.63 and made subsequent to July 16, 1959. reinsurance treaty per se cannot give rise to a
Philamlife filed a claim for refund on the ground that contractual obligation for the payment of foreign FIELDMEN'S INSURANCE CO INC v. ASIAN
the reinsurance premiums remitted were paid in exchange. Philamlife’s obligation to remit reinsurance SURETY & INSURANCE CO INC
pursuant to the January 1950 reinsurance treaty, premiums becomes fixed and definite upon the 34 SCRA 36
and therefore exempted. execution of the reinsurance cession. It is only MAKALINTAL; July 31, 1970
- Monetary Board exempted Philamlife from payment after a reinsurance cession is made that payment of
of margin fee. However, Auditor of CB refused to reinsurance premium may be exacted, as it is only FACTS
pass in audit Philamlife’s claim for refund. Philamlife after Philamlife seeks to remit that reinsurance
INSURANCE Page 78

- On various dates between April 11, 1960 and Jan. into between them had terminated as of December "that in the event of termination of this
9, 1961 the Asian Surety & Insurance Company, 31, 1961 and to obtain an order directing ASIAN to Agreement . . ., the liability of the Fieldmen's under
Inc. and the Fieldmen's Insurance Company, Inc. render final accounting of the transactions between current cessions shall continue in full force and effect
entered into 7 reinsurance agreements under which them with respect to said reinsurance treaties as of until their natural expiry . . .;" and the 4th
the former, as the ceding company undertook to the cut-off date. paragraph of Article VI of the Personal Accident
cede to the latter, as the reinsuring company, a - In its answer below ASIAN denied having received Reinsurance Treaty states:
specified portion of the amount of insurance FIELDMEN'S letter dated Sep 19, 1961, and argued "4. On the termination of this Agreement from
underwritten by ASIAN upon payment to that even assuming it did, FIELDMEN'S could not any cause whatever, the liability of the
FIELDMEN'S of a proportionate share of the gross have terminated the reinsurance treaties as of Dec REINSURER (Fieldmen's) under any current
rate of the premium applicable with respect to each 31, 1961 because the letter was merely an cession including any amounts due to be ceded
cession after deducting a commission. Said expression of FIELDMEN'S desire to cancel the under the terms of this Agreement and which are
agreements were to take effect from certain specific treaties and not a formal notice of cancellation as not cancelled in the ordinary course of business
dates and were to be in force until cancelled by contemplated in their reinsurance agreements. By shall continue in full force until their expiry unless
either party upon previous notice of at least 3 way of special defense Asian contended that even if the COMPANY (Asian) shall, prior to the thirty-first
months by registered mail to the other party, the the Sep. 19 letter were considered sufficient notice December next following such notice, elect to
cancellation to take effect as of Dec. 31 of the year of cancellation — thereby rendering the reinsurance withdraw the existing cessions . . ."
in which the notice was given. agreements terminated as of December 31, 1961 — - Thus, insofar as the two reinsurance agreements
- On Sep. 19, 1961 FIELDMEN'S, by means of the liability of FIELDMEN'S with respect to policies or are concerned, there is clearly no merit in
registered mail, served notice to ASIAN of the cessions issued under two of the said agreements FIELDMEN'S claim that their cancellation carried with
former's desire to be relieved from all participation in prior to their cancellation continued to have full force it ipso facto the termination of all reinsurance
its various agreements with the latter effective Dec. and effect until the stated expiry dates of such cessions thereunder. Such cessions continued to be
31, 1961. This communication, although admittedly policies or cessions. in force until their respective dates of expiration.
received by ASIAN on Sep. 25, 1961, did not elicit - On Dec. 4, 1962, the trial court declared 6 of the 7 Since it was under one of said agreements that the
any reply from ASIAN. reinsurance agreements in question cancelled as of reinsurance cession corresponding to the GSIS policy
- On Dec. 7, 1961 FIELDMEN'S sent another letter to Dec 31, 1961. At the same time, it upheld ASIAN'S had been made, FIELDMEN'S cannot avoid liability
ASIAN expressing regrets at alleged violations position that all cessions of reinsurance made by it to which arose by reason of the burning of the insured
committed by the latter with respect to the various FIELDMEN'S prior to the cancellation of the property.
agreements between them and reiterated its position reinsurance treaties continued in full force and effect - With respect to the other 4 agreements, it would
that it would consider itself "no longer at risk for any until expiry dates and ordered FIELDMEN'S to make seem that the petition for declaratory relief is moot,
reinsurance and/or cession" given by ASIAN which an accounting of its business transactions with and that no useful purpose would be served by
might be in force on Dec. 31, 1961. Not having ASIAN within 30 days. defining the respective rights and obligations of the
received any formal reply from ASIAN, FIELDMEN'S - On appeal to the CA, the decision of the trial court parties thereunder. The said agreements have been
sent a new a letter on Feb. 17, 1962 reminding was substantially affirmed, with the slight cancelled, and it does not appear that any claim by
ASIAN of the cancellation of all the reinsurance modification that the order for accounting was or liability in favor of the insured has actually arisen
treaties and cessions as of Dec. 31, 1961 and eliminated, without prejudice to the filing of a proper under any of the reinsurance cessions made prior to
requested ASIAN to submit its final accounting of all action between the parties for that purpose. such cancellation. Future conflicts of the same nature
cessions made to the former for the preceding as those which have motivated the present action
months when the reinsurance agreements were in ISSUE can of course be obviated by using more precise and
force. WON the cancellation as of Dec. 31, 1961 of the definite terminology in the reinsurance agreements
- Meanwhile one of the risks reinsured with reinsurance treaties had the effect of terminating which the parties may enter into henceforth.
FIELDMEN'S issued in favor of the GSIS became a also the liability of FIELDMEN'S as reinsurer with
liability when the insured property was burned on respect to policies or cessions issued prior to the EQUITABLE INSURANCE v. RURAL INSURANCE
February 16, 1962. Since the policy was issued on termination of the principal reinsurance contracts or 4 SCRA 343
July 1, 1961, it was supposed to expire on July 1, treaties BARRERA; January 31, 1962
1962. 2 The next day, Feb. 17, ASIAN immediately
notified FIELDMEN'S of said fire loss. HELD FACTS
- FIELDMEN'S, relying on the sufficiency of its notice NO to the 2 reinsurance contracts - Plaintiff Equitable Insurance file a complaint with
of termination dated September 19, 1961 and - Of the 6 reinsurance contracts, 2 contain the CFI of Manila against defendant Rural Insurance
obviously bent on avoiding its liability under the provisions, which clearly and expressly recognize the alleging, as first cause of action, that they entered
reinsurance agreements with ASIAN, filed a petition continuing effectivity of policies ceded under them into a reciprocal facultative reinsurance agreement,
for declaratory relief with the CFI of Manila to seek a for reinsurance notwithstanding the cancellation of wherein they agreed to cede to each other. Pursuant
declaration that all the reinsurance contracts entered the contracts themselves. The said treaties provide to said agreement, plaintiff reinsured for P2k with
INSURANCE Page 79

defendant the stock covered by fire insurance Policy dispute between the parties; in the stipulation of interruption loss, leaving a balance of P3,624,683.43
No. 5880 issued by plaintiff which was later burned; facts defendant admitted that plaintiff had paid its and P1,748,460.00, respectively.
the share of the loss of defendant as per insurance liability and defendant likewise admitted that it - The counsel for Artex filed a Manifestation saying
agreement was computed at P2,024 for which ignored plaintiff’s demands for reimbursement for that in view of the Deeds of Discharge and Collateral
plaintiff sent to defendant a statement of account for defendant’s failure to pay its share as reinsurer. As Agreement, the only remaining liability subject of
payment by the latter. Despite repeated demands held in Maligad v United Assurance Co., if in the litigation shall be the proportion of the loss reinsured
by plaintiff, defendant refused to pay. course of the settlement of a loss, the action of the with or through Alexander and Alexander, Inc. of
- On the second cause of action, plaintiff reinsured company or its agents amounts to refusal to pay, the New York, USA, namely, P397,813.00.
for P2k with defendant stock covered by fire company will be deemed to have waived the - The document recited further that Artex
insurance Policy No. 6062 which also burned. Again, condition precedent with reference to arbitration and acknowledges receipt of the sum of P3.6M paid by
defendant refused to pay its share of the loss of a suit upon the policy will lie. the insurer in full and final settlement of all or any
P1,334 hence said complaint. 2. NO claims of Artex against its insurer. It discharges its
- Defendant filed a motion to dismiss on the ground - There is no connection between Art 1206 NCC and insurer from all actions, proceedings, claims,
that it states no cause of action, as pursuant to Art the agreement of this action. The term “facultative” demands, costs and expenses in respect thereof.
VIII of the Reinsurance Agreement between the is used in reinsurance contracts, and it is so used in - With regard the balance unpaid, Wellington
parties, before a court action can be brought, the this particular case, merely to define the right of the contends that Artex should have been directed
parties agreed to submit all disputes to a board of reinsurer to accept or not to accept participation in against the reinsurers to cover the liability and not
arbitrators. The Court denied the motion and the risk insured. But once the share is accepted, as it against Wellington.
required defendant to answer. was in the case at bar, the obligation is absolute and
- Defendant filed its answer, alleging that the nature the liability assumed thereunder can be discharged ISSUE
of the agreement is “self-liquidating between the by only one way—the payment of the share of the WON the insured (Artex) has a cause of action
parties”, the reinsurer becoming the reinsured and losses. against the reinsurer
vice versa; and that said agreement has not yet Disposition judgment appealed from the TC is
been abrogated so the liability of either to the other affirmed HELD
is not yet known. Defendant prayed that the NO
complaint be dismissed and plaintiff filed a motion ARTEX DEVELOPMENT CO INC v. WELLINGTON - Unless there is a specific grant in, or assignment
for judgment on the pleadings which the court INSURANCE CO INC of, the reinsurance contract in favor of the insured or
denied. 51 SCRA 352 a manifest intention of the contracting parties to the
- Instead of going into a formal hearing, the parties TEEHANKEE; June 27, 1973 reinsurance contract to grant such benefit or favor to
submitted their case for decision stipulating the ff the insured, the insured, not being privy to the
facts: defendant admits the allegations of the FACTS reinsurance contract, has no cause of action against
complaint and that plaintiff admits that the issues of - Wellington Insurance Co. Inc. insured for the reinsurer. It is expressly provided in Section 91
the complaint were not submitted to a Board of P24,346,509.00 the buildings, stocks and machinery the Insurance Act 1 that "(T)he original insured has
Arbitrators as provided in par VIII of the complaint, of plaintiff Artex Development Co. Inc. against loss no interest in a contract of insurance."
but instead referred it to the Insurance or damage by fire or lighting upon payment of the
Commissioner. The CFI rendered judgment in favor plaintiff of the corresponding premiums; that said PERLA COMPANIA DE SEGUROS v. CA(LIM)
of plaintiff. Hence this appeal. properties were insured for an additional sum of 208 SCRA 487
P883,034.00; that defendant insured plaintiff against NOCON; May 7, 1992
ISSUES business interruption (use and occupancy) for
1. WON Equitable had no cause of action as the P5,200,000.00; Wellington entered into a contract NATURE
matter was not referred to the decision of arbitrators of reinsurance with Alexander and Alexander, Inc. of Petition for certiorari by Perla Compania de Seguros
2. WON in a facultative obligation the right to choose New York. USA. and FOC Credit Corporation seeking to annul and set
an alternative remedy lies only with the debtor (here - The buildings, stocks and machineries of plaintiffs aside CA decision revering the RTC decision for
the defendant) under Art 1206 spinning department were burned. replevin and damages.
- Notice of the loss and damage was given the
HELD defendant; that as per report of the adjusters, the FACTS
1. NO total property loss of the plaintiff was the sum of - Private respondents spouses Herminio and Evelyn
- The requirement of submission for decision to 2 P10,106,554.40 and the total business interruption Lim executed a promissory note in favor of
arbitrators or an umpire the matter of losses by fire loss was P3,000,000.00; Supercars, Inc. in the sum of P77,940.00, payable in
or the liability of the parties thereto under Art VIII of - That defendant has paid to the plaintiff the sum of monthly installments according to the schedule of
the agreement arises only if the same is disputed by P6,481,870.07 of the property loss suffered by payment indicated in said note, and secured by a
one of the parties. In the instant case, there is no plaintiff and P1,864,134.08 on its business chattel mortgage over a brand new red Ford Laser,
INSURANCE Page 80

which is registered under the name of private - On July25, 1983, petitioner FCP filed a complaint - If the insured vehicle had figured in an accident at
respondent Herminio Lim and insured with the against private respondents, who in turn filed an the time she drove it with an expired license, then,
petitioner Perla Compania de Seguros, Inc. (Perla for amended third party complaint against petitioner appellee Perla Compania could properly resist
brevity) for comprehensive coverage. Perla on Dec8, 1983. After trial on the merits, TC appellants' claim for indemnification for the loss or
- On the same date, Supercars, Inc., with notice to ordered sps Lim to pay jointly and severally, plaintiff destruction of the vehicle resulting from the
private respondents spouses, assigned to petitioner the sum of P55,055.93 plus interest thereon at the accident. But in the present case, the loss of the
FCP Credit Corporation (FCP for brevity) its rights, rate of 24% per annum from July 2, 1983 until fully insured vehicle did not result from an accident where
title and interest on said promissory note and chattel paid; as well as the cost of suit. It also ordered the intent was involved; the loss in the present case was
mortgage as shown by the Deed of Assignment. dismissal of the Third party complaint against Third- caused by theft, the commission of which was
- At around 2:30pm Nov9, 1982, said vehicle was Party Defendant. attended by intent."
carnapped while parked at the back of Broadway - Upon appeal, CA reversed said decision - There is no causal connection between the
Centrum. Evelyn Lim, who was driving said car - After petitioners' separate MFRs were denied by possession of a valid driver's license and the loss of a
before it was carnapped, immediately called up the CA, petitioners filed these separate petitions for vehicle. To rule otherwise would render car insurance
Anti-Carnapping Unit of the Philippine Constabulary review on certiorari. practically a sham since an insurance company can
to report said incident and thereafter, went to the ISSUE easily escape liability by citing restrictions which are
nearest police substation to make a police report 1. WON there was grave abuse of discretion on the not applicable or germane to the claim, thereby
regarding said incident. part of the appellate court in holding that private reducing indemnity to a shadow.
- On Nov10, 1982, Evelyn Lim reported said incident respondents did not violate the insurance contract 2. The court agrees with FCP that Lim spouses are
to the LTO in compliance with the insurance because the authorized driver clause is not not relieved of their obligation to pay the former the
requirement. She also filed a complaint with the applicable to the "Theft" clause of said Contract installments due on the promissory note on account
Headquarters. Constabulary Highway Patrol Group. 2. WON the loss of the collateral exempted the of the loss of the automobile. The chattel mortgage
- On Nov11, 1982, private respondent filed a claim debtor from his admitted obligations under the constituted over the automobile is merely an
for loss with the petitioner Perla but said claim was promissory note particularly the payment of interest, accessory contract to the promissory note. Being the
denied on Nov18, 1982 on the ground that Evelyn litigation expenses and attorney's fees principal contract, the promissory note is unaffected
Lim, who was using the vehicle before it was by whatever befalls the subject matter of the
carnapped, was in possession of an expired driver's HELD accessory contract.
license at the time of the loss of said vehicle which is 1. NO - The unpaid balance on the promissory note should
in violation of the authorized driver clause of the - The comprehensive insurance policy issued by be paid, and not just the installments due and
insurance policy, which states, to wit: petitioner Perla undertook to indemnify the private payable before the automobile was carnapped, as
"AUTHORIZED DRIVER: respondents against loss or damages to the car (a) erronously held by the CA
Any of the following: (a) The Insured (b) Any by accidental collision or overturning, or collision or - However, this does not mean that private
person driving on the Insured's order, or with his overturning consequent upon mechanical breakdown respondents are bound to pay the interest, litigation
permission. Provided that the person driving is or consequent upon wear and tear; (b) by fire, expenses and attorney's fees stipulated in the
permitted, in accordance with the licensing or external explosion, self-ignition or lightning or promissory note. Because of the peculiar relationship
other laws or regulations, to drive the Scheduled burglary, housebreaking or theft; and (c) by between the three contracts in this case, i. e., the
Vehicle, or has been permitted and is not malicious act. promissory note, the chattel mortgage contract and
disqualified by order of a Court of Law or by - Where a car is unlawfully and wrongfully taken the insurance policy, the Court is compelled to
reason of any enactment or regulation in that without the owner's consent or knowledge, such construe all three contracts as intimately interrelated
behalf." taking constitutes theft, and, therefore, it is the to each other, despite the fact that at first glance
- On Nov17, 1982, private respondents requested "THEFT" clause, and not the "AUTHORIZED DRIVER" there is no relationship whatsoever between the
from petitioner FCP for a suspension of payment on clause, that should apply.The risk against accident is parties thereto.
the monthly amortization agreed upon due to the distinct from the risk against theft. The 'authorized - Under the promissory note, Lim spouses are
loss of the vehicle and, since the carnapped vehicle driver clause' in a typical insurance policy as in obliged to pay Supercars, Inc. the amount stated
was insured with petitioner Perla, said insurance contemplation or anticipation of accident in the legal therein in accordance with the schedule provided for.
company should be made to pay the remaining sense in which it should be understood, and not in To secure said promissory note, private respondents
balance of the promissory note and the chattel contemplation or anticipation of an event such as constituted a chattel mortgage in favor of Supercars,
mortgage contract. theft. The distinction often seized upon by insurance Inc. over the automobile the former purchased from
- Perla, however, denied private respondents' claim. companies in resisting claims from their assureds the latter. The chattel mortgage, in turn, required
Consequently, petitioner FCP demanded that private between death occurring as a result of accident and private respondents to insure the automobile and to
respondents pay the whole balance of the death occurring as a result of intent may apply to the make the proceeds thereof payable to Supercars,
promissory note or to return the vehicle but the case at bar. Inc. The promissory note and chattel mortgage were
latter refused. assigned by Supercars, Inc. to petitioner FCP, with
INSURANCE Page 81

the knowledge of private respondents. Private pay petitioner FCP the amount of P55,055.93, with ISSUE
respondents were able to secure an insurance policy legal interest from July 2, 1983 until fully paid. The WON the court a quo erred in dismissing petitioner's
from petitioner Perla, and the same was made decision appealed from is hereby affirmed as to all third party complaint on the ground that petitioner
specifically payable to petitioner FCP. other respects. No pronouncement as to costs. had no cause of action yet against the insurance
- From the abovementioned provision that upon the company
loss of the insured vehicle, the insurance company SHAFER v. JUDGE
Perla undertakes to pay directly to the mortgagor or 167 SCRA 386 HELD
to their assignee, FCP, the outstanding balance of PADILLA; November 14, 1988 YES
the mortgage at the time of said loss under the - There is no need on the part of the insured to wait
mortgage contract. If the claim on the insurance NATURE for the decision of the trial court finding him guilty of
policy had been approved by petitioner Perla, it Petition for review on certiorari reckless imprudence. The occurrence of the injury to
would have paid the proceeds thereof directly to the third party immediately gave rise to the liability
petitioner FCP, and this would have had the effect of FACTS of the insurer under its policy. Respondent
extinguishing private respondents' obligation to - Sherman Shafer obtained a private car policy over insurance company's contention that the third party
petitioner FCP. Therefore, private respondents were his Ford Laser from Makati Insurance Company, Inc., complaint involves extraneous matter which will only
justified in asking petitioner FCP to demand the for third party liability. During the effectivity of the clutter, complicate and delay the criminal case is
unpaid installments from petitioner Perla. policy, an information for reckless imprudence without merit. The civil aspect of the offense
- Because petitioner Perla had unreasonably denied resulting in damage to property and serious physical charged, i.e., serious physical injuries allegedly
their valid claim, private respondents should not be injuries was filed against shafer. The information suffered by Jovencio Poblete, Sr., was impliedly
made to pay the interest, liquidated damages and said that on or about the 17th day of May 1985, in instituted with the criminal case. Petitioner may thus
attorney's fees as stipulated in the promissory note. the City of Olongapo. Shafer hit and bumped a raise all defenses available to him insofar as the
As mentioned above, the contract of indemnity was Volkswagen car owned and driven by Felino llano y criminal and civil aspects of the case are concerned.
procured to insure the return of the money loaned Legaspi, thereby causing damage in the total amount The claim of petitioner for payment of indemnity to
from petitioner FCP, and the unjustified refusal of of P12,345.00 and as a result thereof one Jovencio the injured third party, under the insurance policy,
petitioner Perla to recognize the valid claim of the Poblete, Sr. who was on board of the said for the alleged bodily injuries caused to said third
private respondents should not in any way prejudice Volkswagen car sustained physical injuries which party, arose from the offense charged in the criminal
the latter. injuries causing deformity on the face. The owner of case, from which the injured (Jovencio Poblete, Sr.)
- Private respondents can not be said to have unduly the damaged Volkswagen car filed a separate civil has sought to recover civil damages. Hence, such
enriched themselves at the expense of FCP since action against petitioner for damages, while Jovencio claim of petitioner against the insurance company
they will be required to pay the latter the unpaid Poblete, Sr., who was a passenger in the Volkswagen cannot be regarded as not related to the criminal
balance of its obligation under the promissory note. car, did not reserve his right to file a separate civil action.
- In view of the foregoing discussion, We hold that action for damages. Instead, in the course of the - A third party complaint is a device allowed by the
the Court of Appeals did not err in requiring trial in the criminal case, Poblete, Sr. testified on his rules of procedure by which the defendant can bring
petitioner Perla to indemnify private respondents for claim for damages for the serious physical injuries into the original suit a party against whom he will
the loss of their insured vehicle. However, the latter which he claimed to have sustained as a result of the have a claim for indemnity or remuneration as a
should be ordered to pay petitioner FCP the amount accident. result of a liability established against him in the
of P55,055.93, representing the unpaid installments - The court issued an order dismissing the third party original suit. 13 Third party complaints are allowed to
from December 30, 1982 up to July 1, 1983, as complaint on the ground that it was premature, minimize the number of lawsuits and avoid the
shown in the statement of account prepared by based on the premise that unless the accused necessity of bringing two (2) or more actions
petitioner FCP, 18 plus legal interest from July 2, (herein petitioner) is found guilty and sentenced to involving the same subject matter. They are
1983 until fully paid. pay the offended party (Poblete Sr.) indemnity or predicated on the need for expediency and the
- As to the award of moral damages, exemplary damages, the third party complaint is without cause avoidance of unnecessary lawsuits. If it appears
damages and attorney's fees, private respondents of action. The court further stated that the better probable that a second action will result if the
are legally entitled to the same since Perla had acted procedure is for the accused (petitioner) to wait for plaintiff prevails, and that this result can be avoided
in bad faith by unreasonably refusing to honor the the outcome of the criminal aspect of the case to by allowing the third party complaint to remain, then
insurance claim of the private respondents. Besides, determine whether or not the accused, also the third the motion to dismiss the third party complaint
awards for moral and exemplary damages, as well as party plaintiff, has a cause of action against the third should be denied.
attorney's fees are left to the sound discretion of the party defendant for the enforcement of its third party - Compulsory Motor Vehicle Liability Insurance (third
Court. Such discretion, if well exercised, will not be liability (TPL) under the insurance contract. 6 party liability, or TPL) is primarily intended to
disturbed on appeal. Petitioner moved for reconsideration of said order, provide compensation for the death or bodily injuries
Disposition the assailed decision of the CA is but the motion was denied; hence, this petition. suffered by innocent third parties or passengers as a
hereby MODIFIED to require private respondents to result of a negligent operation and use of motor
INSURANCE Page 82

vehicles. The victims and/or their dependents are Davao City. One day, when he was on his way to his - AFISCO argued that since the Insurance Code
assured of immediate financial assistance, regardless work, he met an accident that resulted in his death. does not expressly provide for a solidary obligation,
of the financial capacity of motor vehicle owners. He died on the spot. the presumption is that the obligation is joint.
- The liability of the insurance company under the - The PUJ jeep that bumped the deceased was driven - The lower court denied the motion for
Compulsory Motor Vehicle Liability Insurance is for by Pepito Into, operated and owned by defendant reconsideration ruling that since the insurance
loss or damage. Where an insurance policy insures Destrajo. From the investigation conducted by the contract "is in the nature of suretyship, then the
directly against liability, the insurer's liability accrues traffic investigator, the PUJ jeep was overtaking liability of the insurer is secondary only up to the
immediately upon the occurrence of the injury or another passenger jeep that was going towards the extent of the insurance coverage."
event upon which the liability depends, and does not city poblacion. While overtaking, the PUJ jeep of - Petitioners filed a second motion for
depend on the recovery of judgment by the injured defendant Destrajo running abreast with the reconsideration reiterating that the liability of the
party against the insured. overtaken jeep, bumped the motorcycle driven by insurer is direct, primary and solidary with the
- The injured for whom the contract of insurance is the deceased. The point of impact was on the lane of jeepney operator because the petitioners became
intended can sue directly the insurer. The general the motorcycle and the deceased was thrown from direct beneficiaries under the provision of the policy
purpose of statutes enabling an injured person to the road and met his untimely death. which, in effect, is a stipulation pour autrui. This
proceed directly against the insurer is to protect - Heirs of Lope Maglana, Sr. filed an action for motion was likewise denied for lack of merit.
injured persons against the insolvency of the insured damages and attorney's fees against operator
who causes such injury, and to give such injured Patricio Destrajo and the Afisco Insurance ISSUE
person a certain beneficial interest in the proceeds of Corporation (AFISCO). An information for homicide WON AFISCO can be held directly liable
the policy, and statutes are to be liberally construed thru reckless imprudence was also filed against
so that their intended purpose may be accomplished. Pepito Into. HELD
It has even been held that such a provision creates a - During the pendency of the civil case, Into was YES
contractual relation which inures to the benefit of sentenced to suffer an indeterminate penalty, with - As this Court ruled in Shafer vs. Judge, RTC of
any and every person who may be negligently all the accessory penalties provided by law, and to Olongapo City, Br. 75, "[w]here an insurance policy
injured by the named insured as if such injured indemnify the heirs of Lope Maglana, Sr. in the insures directly against liability, the insurer's liability
person were specifically named in the policy. amount of twelve thousand pesos with subsidiary accrues immediately upon the occurrence of the
- In the event that the injured fails or refuses to imprisonment in case of insolvency, plus five injury or even upon which the liability depends, and
include the insurer as party defendant in his claim thousand pesos in the concept of moral and does not depend on the recovery of judgment by the
for indemnity against the insured, the latter is not exemplary damages with costs. No appeal was injured party against the insured."
prevented by law to avail of the procedural rules interposed by accused who later applied for - The underlying reason behind the third party
intended to avoid multiplicity of suits. Not even a "no probation. liability (TPL) of the Compulsory Motor Vehicle
action" clause under the policy-which requires that a - The lower court rendered a decision finding that Liability Insurance is "to protect injured persons
final judgment be first obtained against the insured Destrajo had not exercised sufficient diligence as the against the insolvency of the insured who causes
and that only thereafter can the person insured operator of the jeepney ordering him to pay plaintiffs such injury, and to give such injured person a certain
recover on the policy can prevail over the Rules of the sum for loss of income; funeral and burial beneficial interest in the proceeds of the policy . . ."
Court provisions aimed at avoiding multiplicity of expenses of the deceased; moral damages, and Since petitioners had received from AFISCO the sum
suits. attorney's fees and costs of suit. The defendant of P5,000.00 under the no-fault clause, AFISCO's
Disposition instant petition is GRANTED. The insurance company is ordered to reimburse liability is now limited to P15,000.00.
questioned order dated 24 April 1987 is SET ASIDE defendant Destrajo whatever amounts the latter - However, we cannot agree that AFISCO is likewise
and a new one entered admitting petitioner's third shall have paid only up to the extent of its insurance solidarily liable with Destrajo. In Malayan Insurance
party complaint against the private respondent coverage. Co., Inc. v. Court of Appeals, this Court had the
Makati Insurance Company, Inc. - Petitioners filed a motion for the reconsideration of opportunity to resolve the issue as to the nature of
the second paragraph of the decision contending that the liability of the insurer and the insured vis-a-vis
VDA DE MAGLANA v. CONSOLACION AFISCO should not merely be held secondarily liable the third party injured in an accident. We
212 SCRA 268 because the Insurance Code provides that the categorically ruled thus: While it is true that where
ROMERO; August 6, 1992 insurer's liability is "direct and primary and/or jointly the insurance contract provides for indemnity against
and severally with the operator of the vehicle, liability to third persons, such third persons can
NATURE although only up to the extent of the insurance directly sue the insurer, however, the direct liability
Petition for certiorari coverage." Hence, they argued that the P20,000.00 of the insurer under indemnity contracts against
coverage of the insurance policy issued by AFISCO, third party liability does not mean that the insurer
FACTS should have been awarded in their favor. can be held solidarily liable with the insured and/or
- Lope Maglana was an employee of the Bureau of the other parties found at fault. The liability of the
Customs whose work station was at Lasa, here in insurer is based on contract; that of the insured is
INSURANCE Page 83

based on tort. In the case at bar, petitioner as 1957. The taxi they were riding in collided with a accepted the responsibility for damages or injuries to
insurer of Sio Choy, is liable to respondent Vallejos gravel and sand truck resulting to injuries to both passengers even if it had no fault at all.
(the injured third party), but it cannot, as incorrectly Misa and Pinto. - In the case of the insurance company, the SC ruled
held by the trial court, be made "solidarily" liable - The two passengers instituted a suit for damages that it neither authorized nor consented to the
with the two principal tortfeasors, namely against La Mallorca who, while denying representations made by the taxi company to its
respondents Sio Choy and San Leon Rice Mill, Inc. responsibility, instituted a third party complaint passengers. As such, the liability of the said
For if petitioner-insurer were solidarily liable with against Far Eastern Surety to recoup from the latter insurance company based on its insurance contract
said, two (2) respondents by reason of the any award for damages that might be recovered by is limited to the recovery by the insured of all sums,
indemnity contract against third party liability under the passengers. cost and expenses which the insured shall become
which an insurer can be directly sued by a third party - It would appear from the case that a sticker was legally liable. The insurance company therefore
this will result in a violation of the principles placed in all the taxis of La Mallorca stating that cannot be held liable for the award.
underlying solidary obligation and insurance passengers of the taxis were insured against - The taxi company is adjudged to be the sole party
contracts. accidents. This was done to entice the public into responsible for the award.
- While in solidary obligations, the creditor may patronizing La Mallorca. Disposition The decision of the CA is modified by
enforce the entire obligation against one of the - The trial court awarded to Misa and Pinto actual, eliminating the award against Far Eastern.
solidary debtors, in an insurance contract, the moral and exemplary damages, and attorney’s fees
insurer undertakes for a consideration to indemnify payable by La Mallorca and sentenced Far Eastern to PEZA v. ALIKPALA
the insured against loss, damage or liability arising pay La Mallorca P10,000. on its third party liability 160 SCRA 31
from an unknown or contingent event. insurance. NARVASA; April 15, 1988
- Similarly, petitioners herein cannot validly claim - On appeal, the CA, while holding that the collision
that AFISCO, whose liability under the insurance was due to the fault of the driver of the gravel and NATURE
policy is also P20,000.00, can be held solidarily liable sand truck, found the taxi company liable for Motion praying that Judge Alikpala be declared guilty
with Destrajo for the total amount of P53,901.70 in damages to the passengers on the strength of its of contempt of court for having decided the case on
accordance with the decision of the lower court. representation contained in the sticker above noted the merits despite the pendency in this Court of the
Since under both the law and the insurance policy, that the passengers were insured against accidents. certiorari action instituted by the plaintiffs
AFISCO's liability is only up to P20,000.00, the In so ruling, the CA overruled the defense of the
second paragraph of the dispositive portion of the insurance company to the effect that it was FACTS
decision in question may have unwittingly sown responsible only if the insured, La Mallorca, was - vehicular accident with 2 children running across
confusion among the petitioners and their counsel. involved in accidents caused by, or arising out of, the the path of a Chevrolet "Carry-All", belonging to a
What should have been clearly stressed as to leave use of the motor vehicle. A motion for partnership known as Diman & Company driven by
no room for doubt was the liability of AFISCO under reconsideration was filed in and dismissed by the CA. its driver, Perfecto Amar, as it was passing a
the explicit terms of the insurance contract. national highway at barrio Makiling Calamba,
Disposition present petition is hereby GRANTED. ISSUE Laguna. They were killed. It was insured with the
The award of P28,800.00 representing loss of income WON Far Eastern Surety is liable to the insured on its Empire Insurance Co., Inc. under a so-called
is INCREASED to P192,000.00 and the death insurance policy 'comprehensive coverage" policy, loss by theft
indemnity of P12,000.00 to P50,000.00. excluded. The policy was in force at the time of the
HELD accident.
NO - Placida Peza, the managing partner of Diman & Co.
- The award for damages made to the passengers filed a claim with Empire, for payment of
was exclusively predicated on the representation compensation to the family of the 2 children who
made by La Mallorca that its passengers were died as a result of the accident. Empire refused to
insured against accidents and not because it was at pay on the ground that the driver had no authority to
FAR EASTERN SURETY v. MISA fault in causing the accident. operate the vehicle, a fact which it expressly
25 SCRA 663 Reasoning excepted from liability under the policy. What Peza
REYES; October 26, 1968 - In this case, the findings of the CA and the trial did was to negotiate directly with the deceased
court that the causative factor of the mishap was the children father for an out-of-court settlement. The
NATURE negligence of the gravel and truck driver would have father agreed to accept P6,200.00 in fun settlement
Appeal by petition for review from a CA judgment been sufficient to relieve the taxi company of any of the liability of the vehicles owner and driver, and
liability arising from the accident. However, in view Peza paid him this sum.
FACTS of the sticker in all of its taxicabs, La Mallorca has - Peza thereafter sued Empire to recover this sum of
- Socorro Dancel Vda.de Misa and Araceli Pinto hired insured its passengers against accidents, whether it P6,200.00 as actual damages, as well as P20,000.00
a taxi cab operated by La Mallorca on September 3, was at fault or not. In other words, La Mallorca as moral damages, P10,000.00 as exemplary
INSURANCE Page 84

damages, and P10,000.00 as attorney's fees. She assigned as errors and reviewed in the appeal this chapter shall be paid without the necessity of
amended her complaint shortly thereafter to include properly taken from the decision rendered by the proving fault or negligence of any kind. Provided,
Diman & Co. as alternative party plaintiff. trial court on the merits of the case. That for purposes of this section
- Empire's basic defense to the suit was anchored - In the meantime, Judge Alikpala rendered (i) The indemnity in respect of any one person shall
on the explicit requirement in the policy limiting the judgment on the merits, since the case was then not exceed P5,000;
operation of the insured vehicle to the "authorized already ripe for adjudication. The judgment ordered (ii) The following proofs of loss, when submitted
driver" therein defined, namely, (a) the insured, or dismissal of the case for failure on the part of the under oath, shall be sufficient evidence to
(b) any person driving on the insured order or with plaintiff to prove their cause of action against substantiate the claim:
his permission, provided that- Empire. Notice of the judgment was served on the (a) Police report of accident, and
... that the person driving is permited in parties in due course. (b) Death certificate and evidence sufficient to
accordance with the licensing or other laws or 2. YES establish, the proper payee, or
regulations to drive the Motor vehicle or has been - It would seem fairly obvious that whether the LTC (c) Medical report and evidence of medical or
so permitted and is not disqualified by order of the agent was correct or not in his opinion that driver hospital disbursement in respect of which refund is
Court of Law of by reason of any enactment or Amar had violated some traffic regulation warranting claimed;
regulation in that behalf from driving such Motor confiscation of his license and issuance of a TVR in (iii) Claim may be made against one motor vehicle
Vehicle.- lieu thereof, this would not alter the undisputed fact only. In the case of an occupant of a vehicle, claim
- driver Perfecto Amar, only having a temporary that Amar's licence had indeed been confiscated and shall lie against the insurer of the vehicle in which
operator's permit (TVR) [already expired] his driver’s a TVR issued to him, and the TVR had already the occupant is riding, mounting or dismounting
license having earlier been confiscated by an agent expired at the time that the vehicle being operated from. In any other case, claim shall lie against the
of the Land Transportation Commission for an by him killed two children by accident. Neither would insurer of the directly offending vehicle. In all cases,
alleged violation of Land Transportation and Traffic proof of the renewal of Amar's license change the the right of the party paying the claim to recover
Rules, was not permitted by law and was in truth fact that it had really been earlier confiscated by the against the owner of the vehicle responsible for the
disqualified to operate any motor vehicle; Peza LTC agent. accident shall be maintained.
attempted to neutralize that fact by(1) the issuance Disposition petition is DISMISSED for lack of merit Perla denied its liability under the above provision
of the TVR by the LTC officer to Amar; in proof of the and said that the insurer of the vehicle that the
proposition that there was no reason for confiscation respondents were riding (Malayan Insurance in this
of Amar's license (2) Amar's license had not expired, case) should be liable. Its 2 MFRs denied, Perla filed
but had been renewed. this action
- Judge Alikpala did not admit such evidence
ISSUE
ISSUES WON Perla is the insurer liable to indemnify under
1. WON Judge Alikapala committed grave abuse of Sec. 378
discretion in not admitting evidence PERLA COMPANIA DE SEGUROS v. ANCHETA
2. WON confiscation of license and expiration of TVR 164 SCRA 144 HELD
of the driver would serve as bar for Peza in CORTES; August 8, 1988 NO
recovering from Empire Ratio The law is very clear – the claim shall lie
NATURE against the insurer of the vehicle in which the
HELD Petition for certiorari and prohibition with prelim “occupant xxx is riding,” and no other. The claimant
1. NO injunction to review orders of CFI Camarines Norte is not free to choose from which insurer he will claim
- Even positing error in the Judge's analysis of the the “no fault indemnity,” as the law, by using the
evidence attempted to be introduced and his FACTS word “shall”, makes it mandatory that the claim be
rejection thereof, it is clear that it was at most an - Perla was the insurer of a Superlines bus which made against the insurer of the vehicle in which the
error of judgment, not such an error as may be figured in a collision with a III Scout (it’s a kind of occupant is riding, mounting or dismounting from.
branded a grave abuse of discretion, i.e., such vehicle). Injured passengers of the latter (and Reasoning
capricious and whimsical exercise of judgment as is respondents in this case) filed a complaint for - the rules on claims under the “no fault indemnity”
equivalent to lack of jurisdiction, against which the damages against Superlines, the bus driver, and provision, where proof of fault or negligence is not
writ of certiorari will lie. In any event, the Perla (as insurer of the bus). CFI Judge Ancheta necessary for payment of any claim for death or
established principle is "that ruling of the trial court ordered that Perla should pay the respondents injury to a passenger or to a 3rd party, are
on procedural questions and on admissibility of immediately the P5000 under the “no fault clause” as established:
evidence during the course of the trial are provided in Sec. 378. 1. A claim may be made against one motor vehicle
interlocutory in nature and may not be the subject of Sec. 378: Any claim for death or injury to any only.
separate appeal or review on certiorari, but are to be passenger or 3rd party pursuant to the provisions of
INSURANCE Page 85

2. If the victim is an occupant of a vehicle, the claim ground, hitting her forehead. She was treated at the
shall lie against the insurer of the vehicle in which he Protacio Emergency Hospital and later on HELD
is riding, mounting or dismounting from. hospitalized at the San Juan De Dios Hospital. Her YES
3. In any other case (i.e. if the victim is not an face was permanently disfigured, causing her serious - The Schedule of Indemnities does not purport to
occupant of a vehicle), the claim shall lie against the anxiety and moral distress. Respondent bus restrict the kinds of damages that may be awarded
insurer of the directly offending vehicle. company was insured with petitioner Western against Western once liability has arisen. It was
4. In all cases, the right of the party paying the Guaranty Corporation ("Western") under its Master merely meant to set limits to the amounts the
claim to recover against the owner of the vehicle Policy which provided, among other things, for movant would be liable for in cases of claims for
responsible for the accident shall be maintained. protection against third party liability, the relevant death, bodily injuries of, professional services and
-That the vehicle ridden might not be the one that section reading as follows: hospital charges, for services rendered to traffic
caused the accident is of no moment since the law Section 1. Liability to the Public ? Company will, accident victims,' and not necessarily exclude claims
itself provides that the party paying the claim under subject to the Limits of Liability, pay all sums against the insurance policy for other kinds of
Sec. 378 may recover against the owner of the necessary to discharge liability of the insured in damages, such as those in question.
vehicle responsible for the accident. This is precisely respect of ? - It will be seen that the above quoted Schedule of
the essence of “no fault indemnity” insurance which (a) death of or bodily injury to or damage to Indemnities establishes monetary limits which
was introduced to and made part of our laws in order property of any passenger as defined herein. Western may invoke in case of occurrence of the
to provide victims of vehicular accidents or their (b) death of or bodily injury or damage to property particular kinds of physical injury there listed.
heirs immediate compensation, although in a limited of any THIRD PARTY as defined herein in any - It must be stressed, however, that the Schedule of
amount, pending final determination of who is accident caused by or arising out of the use of the Indemnities does not purport to limit, or to
responsible for the accident and liable for the Schedule Vehicle, provided that the liability shall enumerate exhaustively, the species of bodily injury
victims' injuries or death. In turn, the “no fault have first been determined. In no case, however, occurrence of which generate liability for petitioner
indemnity” provision is part and parcel of the Code shall the Company's total payment under both Western. A car accident may, for instance, result in
provisions on compulsory motor vehicle liability Section I and Section 11 combined exceed the injury to internal organs of a passenger or third
insurance and should be read together with the Limits of Liability set forth herein. With respect to party, without any accompanying amputation or loss
requirement for compulsory passenger and/or 3 rd death of or bodily injury to any third party or of an external member (e.g., a foot or an arm or an
party liability insurance (Sec. 377) which was passenger, the company's payment per victim in eye). But such internal injuries are surely covered by
mandated in order to ensure ready compensation for any one accident shall not exceed the limits Section I of the Master Policy, since they certainly
victims of vehicular accidents. indicated in the Schedule of indemnities provided constitute bodily injuries.
-Irrespective of whether or not fault or negligence for in this policy excluding the cost of additional - The Schedule of Indemnities does not purport to
lies with the driver of the Superlines bus, as medicines, and such other burial and funeral restrict the kinds of damages that may be awarded
respondents were not occupants of the bus, they expenses that might have been incurred. against Western once liability has arisen. Section 1,
cannot claim the “no fault indemnity” provided in - Respondent Priscilla Rodriguez filed a complaint for quoted above, does refer to certain "Limits of
Sec. 378 from Perla. The claim should be made damages before the Regional Trial Court of Makati Liability" which in the case of the third party liability
against the insurer of the vehicle they were riding. against De Dios Transportation Co. and Walter A. section of the Master Policy, is apparently
Disposition Petition GRANTED. Orders of CFI Saga Respondent De Dios Transportation Co., in P50,000.00 per person per accident. Within this
ordering Perla to pay respondents immediately turn, filed a third-party complaint against its over-all quantitative limit, all kinds of damages
P5000 ANNULLED and SET ASIDE insurance carrier, petitioner Western. On 6 August allowable by law" actual or compensatory
1985, the trial court rendered a decision in favor of damages"; "moral damages'; "nominal damages";
WESTERN GUARANTY CORPORATION v. CA respondent Priscilla E. Rodriguez, awarding moral "temperate or moderate damages"; "liquidated
(RODRIGUEZ & DE DIOS TRANS) damages, lossof earning and attorney's fees among damages"; and "exemplary damages" ? may be
185 SCRA 652 others. awarded by a competent court against the insurer
FELICIANO; July 20, 1990 - On appeal, the Court of Appeals affirmed in toto once liability is shown to have arisen, and the
the decision of the trial court. essential requisites or conditions for grant of each
NATURE - Petitioner contends that it cannot be held liable for species of damages are present. It appears to us
Petition for review the decision of CA affirming in loss of earnings, moral damages and attorney's fees self-evident that the Schedule of Indemnities was
toto the damages awarded to private respondent by because these items are not among those included in not intended to be an enumeration, much less a
the trial court. the Schedule of Indemnities set forth in the closed enumeration, of the specific kinds of damages
insurance policy. which may be awarded under the Master Policy
FACTS Western has issued.
- Respondent Priscilla E. Rodriguez was struck by a ISSUE - The reading urged by Western of the Schedule of
De Dios passenger bus owned by respondent De Dios WON petitioner can be held liable for loss of Indemnities comes too close to working fraud upon
Transportation Co., Inc. Priscilla was thrown to the earnings, moral damages and attorney's fees both the insured and the third party beneficiary of
INSURANCE Page 86

Section 1, quoted above. For Western's reading demanded from Summit through counsel on be considered in fixing the meaning of any of its
would drastically and without warning limit the February 28, 1978 the payment of the damages parts and in order to produce a harmonious whole.
otherwise unlimited and comprehensive scope of sustained by the car of Olaso but to no avail. A statute must be so construed as to harmonize
liability assumed by the insurer Western under - Hence on May 22, 1978 FGU filed a case in the and give effect to all its provisions whenever
Section 1: "all sums necessary to discharge liability Insurance Commissioner's Office against Summit for possible.'
of the insured in respect of [bodily injury to a third recovery of said amount. - Petitioner company is trying to use Section 384 of
party]". This result- which is not essentially different - Summit filed a motion to dismiss on the ground of the Insurance Code as a cloak to hide itself from its
from taking away with the left hand what had been prescription under Section 384 of PD No. 612. liabilities. The facts of these cases evidently reflect
given with the right hand we must avoid as obviously Averring that the accident happened on November the deliberate efforts of petitioner company to
repugnant to public policy. If what Western now 26, 1976 while the complaint was filed on May 22, prevent the filing of a formal action against it.
urges is what Western intended to achieve by its 1978 beyond the one-year period from the time of Bearing in mind that if it succeeds in doing so until
Schedule of Indemnities, it was incumbent upon the accident provided for by the said provision. one year lapses from the date of the accident it could
Western to use language far more specific and set up the defense of prescription, petitioner
precise than that used in fact by Western, so that ISSUE company made private respondents believe that
the insured, and potential purchasers of its Master WON the action must be dismissed on the ground of their claims would be settled in order that the latter
Policy, and the Office of the Insurance prescription under Section 384 of PD No. 612 will not find it necessary to immediately bring suit. In
Commissioner, may be properly informed and act violation of its duties to adopt and implement
accordingly. HELD reasonable standards for the prompt investigation of
- Moreover, an insurance contract is a contract of NO claims and to effectuate prompt, fair and equitable
adhesion. The rule is well entrenched in our - The case do not fall within the meaning of proper settlement of claims, and with manifest bad faith,
jurisprudence that the terms of such contract are to cases' as contemplated in Section 384 of the petitioner company devised means and ways of
be construed strictly against the party which Insurance Code. stalling the settlement proceedings. In G.R. No. L-
prepared the contract, which in this case happens to 50997, no steps were taken to process the claim and
be petitioner Western. no rejection of said claim was ever made even if
Disposition Petition denied. Reasoning private respondent had already complied with all the
- Section 384 of PD 612 (Insurance Code) requirements.
SUMMIT GUARANTY & INSURANCE COMPANY v. Any person having any claim upon the policy - In G.R. No. L-48758-petitioner company even
ARNALDO issued pursuant to this chapter shall, without any provided legal assistance to one of the private
158 SCRA 332 unnecessary delay, present to the insurance respondents in the criminal case filed against him
GANCAYCO; February 29, 1988 company concerned a written notice of claim leading Private respondents to believe that it was
setting forth the amount of his loss, and/or the ready to pay. In the same case, petitioner company
NATURE nature, extent and duration of the injuries admits that it took no final action or adjudication of
PETITION to review the order of the Insurance sustained as certified by a duly licensed physician. the claim. Worse still, in G.R. No. L-48679,
Commissioner. Notice of claim must be filed within six months assurances of payment were constantly given and
from date of the accident, otherwise, the claim petitioner company even said that a check was ready
FACTS shall be deemed waived. Action or suit for for release. This Court has made the observation
- On Nov. 26, 1976, a Ford Pick-up truck owned by recovery of damage due to loss or injury must be that some insurance companies have been inventing
Marcos Olasco was bumped by a cargo truck owned brought, in proper cases, with the Commissioner excuses to avoid their just obligations and it is only
by Floralde. or the Courts within one year from date of the State that can give the protection which the
FGU Insurance Corporation (FG U) by reason of accident, otherwise, the claimant's right of action insuring public needs from possible abuses of the
Motor Vehicle Insurance Policy No. IC-VF-07185 paid shall prescribe. insurers. In view of the foregoing,
Olaso the sum of P2,817.50 as its share in the repair - It is very clear that the one-year period is only - It is not denied that an extrajudicial demand for
cost of the said Ford Pick-up. Having been required In proper cases. Had the lawmakers payment was made by respondent FGU on petitioner
subrogated to the rights and causes of action of intended it to be the way Petitioner Company but petitioner failed to respond to the same.
Olaso in the said amount FGU formally demanded assumes it to be, then the phrase 'in proper cases' Nevertheless the complaint was filed even before a
payment of said amount from Floralde and would not have been inserted. denial of the claim was made by petitioner. For all
attempted to verify Floralde's insurance carrier but - in Aisporna. vs. Court of Appeals: legal purposes, the one-year prescriptive period
failed to do so. In 1978 FGU was able to ascertain 'Legislative intent must be ascertained from a provided for in Section 384 of the Insurance Code
the identity of Floralde's insurance carrier to be the consideration of the statute as a whole. The has not begun to run.The cause of action arises only
Summit Guaranty and Insurance Company, Inc. particular words, clauses and phrases should not and starts to run upon the denial of the claim by the
(Summit) and thus requested the insurance be studied as detached and isolated expressions, insurance company.The court takes note of the
commissioner for a conference with Summit and but the whole and every part of the statute must dilatory tactics employed by petitioner in this as in
INSURANCE Page 87

the several cases aforecited to avoid payment of its himself or any person on his (insured's) permission. reposed in the shop by the insured car owner does
liabilities. Apparently, the Insurance commission sees the not mean that the "authorized driver" clause has
unauthorized taking of the vehicle for a joyride as a been violated such as to bar recovery, provided that
VILLACORTA v. THE INSURANCE COMMISSION violation of the 'Authorized Driver' clause of the such employee is duly qualified to drive under a valid
100 SCRA 467 policy." driver's license.
TEEHANKEE; October 30, 1980 - Respondent commission likewise upheld private - Secondly, and independently of the foregoing
respondent's assertion that the car was not stolen (since when a car is unlawfully taken, it is the theft
FACTS and therefore not covered by the Theft clause, ruling clause, not the "authorized driver" clause, that
- JEWEL VILLACORTA was the owner of a Colt that "(T)he element of 'taking' in Article 308 of the applies), where a car is admittedly as in this case
Lancer, Model 1976, insured with respondent Revised Penal Code means that the act of depriving unlawfully and wrongfully taken by some people, be
company for P35,000.00 - Own Damage; P30,000.00 another of the possession and dominion of a they employees of the car shop or not to whom it
- Theft; and P30,000.00 - Third Party Liability, movable thing is coupled . . . with the intention, at had been entrusted, and taken on a long trip to
effective May 16, 1977 to May 16, 1978. the time of the 'taking', of withholding it with the Montalban without the owner's consent or
- On May 9, 1978, the vehicle was brought to the character of permanency knowledge, such taking constitutes or partakes of
Sunday Machine Works, Inc., for general check-up the nature of theft as defined in Article 308 of the
and repairs. On May 11, 1978, while it was in the ISSUE Revised Penal Code.
custody of the Sunday Machine Works, the car was WON the Insurance commission’s findings are in - The Court rejects respondent commission's
allegedly taken by six (6) persons and driven out to accord with law premise that there must be an intent on the part of
Montalban, Rizal. While travelling along Mabini St., the taker of the car "permanently to deprive the
Sitio Palyasan, Barrio Burgos, going North at HELD insured of his car" and that since the taking here was
Montalban, Rizal, the car figured in an accident, NO for a "joy ride" and "merely temporary in nature," a
hitting and bumping a gravel and sand truck parked - First, respondent commission's ruling that the "temporary taking is held not a taking insured
at the right side of the road going south. As a person who drove the vehicle in the person of Benito against."
consequence, the gravel and sand truck veered to Mabasa, who, according to its own finding, was one - The insurer must therefore indemnify the petitioner
the right side of the pavement going south and the of the residents of the Sunday Machine Works, Inc. owner for the total loss of the insured car in the sum
car veered to the right side of the pavement going to whom the car had been entrusted for general of P35,000.00 under the theft clause of the policy,
north. The driver, Benito Mabasa, and one of the check-up and repairs was not an "authorized driver" subject to the filing of such claim for reimbursement
passengers died and the other four sustained of petitioner-complainant is too restrictive and or payment as it may have as subrogee against the
physical injuries. The car, as well, suffered extensive contrary to the established principle that insurance Sunday Machine Works, Inc.
damage. Complainant, thereafter, filed a claim for contracts, being contracts of adhesion where the
total loss with the respondent company but claim only participation of the other party is the signing of
was denied. Hence, complainant was compelled to his signature or his "adhesion" thereto, "obviously
institute the present action." call for greater strictness and vigilance on the part of
- The comprehensive motor car insurance policy for courts of justice with a view of protecting the weaker
P35,000.00 issued by respondent Empire Insurance party from abuse and imposition, and prevent their
Company admittedly undertook to indemnify the becoming traps for the unwary."
petitioner-insured against loss or damage to the car - The main purpose of the "authorized driver" clause, CHAPTER VIII. MARINE INSURANCE
(a) by accidental collision or overturning, or collision as may be seen from its text, supra, is that a person
or overturning consequent upon mechanical other than the insured owner, who drives the car on MAGSAYSAY INC v. AGAN
breakdown or consequent upon wear and tear; (b) the insured's order, such as his regular driver, or 96 PHIL 504
by fire, external explosion, self-ignition or lightning with his permission, such as a friend or member of REYES; January 31, 1955
or burglary, housebreaking or theft; and (c) by the family or the employees of a car service or repair
malicious act. shop must be duly licensed drivers and have no FACTS
- Respondent insurance commission, however, disqualification to drive a motor vehicle. A car owner - The S S "San Antonio", vessel owned and operated
dismissed petitioner's complaint for recovery of the who entrusts his car to an established car service by plaintiff, left Manila on October 6, 1949, bound for
total loss of the vehicle against private respondent, and repair shop necessarily entrusts his car key to Basco, Batanes, vis Aparri, Cagayan, with general
sustaining respondent insurer's contention that the the shop owner and employees who are presumed to cargo belonging to different shippers, among them
accident did not fall within the provisions of the have the insured's permission to drive the car for the defendant. The vessel reached Aparri, but while
policy either for the Own Damage or Theft coverage, legitimate purposes of checking or road-testing the still in the port, it ran aground at the mouth of the
invoking the policy provision on "Authorized Driver" car. The mere happenstance that the employee(s) of Cagayan river, and, attempts to refloat it under its
clause, which clause limits the use of the insured the shop owner diverts the use of the car to his own own power having failed, plaintiff had it refloated by
vehicle to two (2) persons only, namely: the insured illicit or unauthorized purpose in violation of the trust the Luzon Stevedoring Co. at an agreed
INSURANCE Page 88

compensation. Once afloat the vessel returned to - Tolentino, in his commentaries on the Code of
Manila to refuel and then proceeded to Basco, the Commerce, gives the following requisites for FACTS
port of destination. There the cargoes were delivered general average: - Plaintiff’s motorboat, “Pandan” was insured on a
to their respective owners or consignees, who, with First, there must be a common danger. This means, marine insurance policy with National Union Fire
the exception of defendant, made a deposit or signed that both the ship and the cargo, after has been Insurance Company (NUFIC) for P45K. According to
a bond to answer for their contribution to the loaded, are subject to the same danger, whether the provisions of a “rider” attached to the policy, the
average. during the voyage, or in the port of loading or insurance was against the “absolute total loss of the
- On the theory that the expenses incurred in unloading; that the danger arises from the accidents vessel only.” On Oct. 31, 1928, the ship ran into
floating the vessel constitute general average to of the sea, dispositions of the authority, or faults of very heavy sea and it became necessary to jettison a
which both ship and cargo should contribute, plaintiff men, provided that the circumstances producing the portion of the cargo. As a result of the jettison, the
brought the present action in the CFI of Manila to peril should be ascertained and imminent or may NUFIC was assessed P2,610.86 as its contribution to
make defendant pay his contribution, which, as rationally be said to be certain and imminent. This the general average.
determined by the average adjuster, amounts to last requirement exclude measures undertaken - The insurance company, insisting that its obligation
P841.40. against a distant peril. did not extend beyond the insurance of the “absolute
- Defendant denies liability to his amount, alleging, Second, that for the common safety part of the total loss of the vessel only, and to pay proportionate
among other things, that the stranding of the vessel vessel or of the cargo or both is sacrificed salvage of the declared value,” refused to contribute
was due to the fault, negligence and lack of skill of deliberately. to the settlement of the gen. ave. The present action
its master, that the expenses incurred in putting it Third, that from the expenses or damages caused was thereupon instituted, and after trial the court
afloat did not constitute general average, and that follows the successful saving of the vessel and cargo. below rendered judgment in favor of the plaintiff and
the liquidation of the average was not made in Fourth, that the expenses or damages should have ordered the defendant to pay the plaintiff P2,610.86
accordance with law. been incurred or inflicted after taking proper legal as its part of the indemnity for the gen. ave. brought
- The lower court found for plaintiff steps and authority. about by the jettison of cargo. The insurance
- With respect to the first requisite, the evidence company then appealed to the SC.
ISSUE does not disclose that the expenses sought to be - The insurance contract is printed in the English
WON the expenses incurred in floating a vessel so recovered from defendant were incurred to save common form of marine policies. One of the clauses
stranded should be considered general average and vessel and cargo from a common danger...it is the of the document originally read as follows:
shared by the cargo owners safety of the property, and not of the voyage, which “Touching the Adventures and Perils which the
constitutes the true foundation of the general said NUFIC is content to bear, and to take upon
HELD average. them in this Voyage; they are of the Seas, Men-of-
NO - As to the second requisite, we need only repeat War, Fire, Pirates, Thieves, Jettison, Letters of
Ratio The law on averages is contained in the Code that the expenses in question were not incurred for Mart and Countermart, Surprisals, and Takings at
of Commerce. Under that law, averages are the common safety of vessel and cargo, since they, Sea. Arrests, Restraints and Detainments, of all
classified into simple or particular and general or or at least the cargo, were not in imminent peril. Kings, Princes and People of what Nation,
gross. Generally speaking, simple or particular - With respect to the third requisite, the salvage Condition or Quality soever; Barratry of the Master
averages include all expenses and damages caused operation, it is true, was a success. But as the and Marines, and of all other Perils, Losses and
to the vessel or cargo which have not inured to the sacrifice was for the benefit of the vessel to enable it Misfortunes, that have or shall come to the Hurt,
common benefit (Art. 809), and are, therefore, to be to proceed to destination and not for the purpose of Detriment, or Damage of the said Vessel or any
borne only by the owner of the property gave rise to saving the cargo, the cargo owners are not in law part thereof; and in case of any Loss or
same (Art. 810); while general or gross averages bound to contribute to the expenses. Misfortunes, it shall be lawful for the Assured, his
include "all the damages and expenses which are - The final requisite has not been proved, for it does or their Factors, Servants, or assigns, to sue,
deliberately caused in order to save the vessel, its not appear that the expenses here in question were labour and travel for, in and about the Defence.
cargo, or both at the same time, from a real and incurred after following the procedure laid down in Safeguard, and recovery of the said Vessel or any
known risk" (Art. 811). Being for the common article 813. part thereof, without Prejudice to this Insurance;
benefit, gross averages are to be borne by the Disposition Wherefore, the decision appealed from to the Charges whereof the said Company, will
owners of the articles saved (Art. 812). is reversed. contribute, according to the rate and quantity of
Reasoning the sum herein assured...”
- the stranding of plaintiff's vessel was due to the JARQUE v. SMITH, BELL & CO. - Attached to the policy over and above the said
sudden shifting of the sandbars at the mouth of the 56 PHIL 758 clause is a “rider” containing typewritten provisions,
river which the port pilot did not anticipate. The OSTRAND; November11, 1932 among which appears in capitalized type the
standing may, therefore, be regarded as accidental. following clause:
NATURE
Appeal from judgment of the lower court
INSURANCE Page 89

“AGAINST THE ABSOLUTE TOTAL LOSS OF THE simply places the insurer on the same footing as before the voyage was begun, and an attempt
VESSEL ONLY, AND TO PAY PROPORTIONATE other persons who have an interest in the vessel, or had been made to repair it by filling with
SALVAGE CHARGES OF THE DECLARED VALUE.” the cargo therein, at the time of the occurrence of cement and bolting over it a strip of iron. The
the general average and who are compelled to effect of loading the boat was to submerge
ISSUES contribute (Art. 812, Code of Commerce). the vent, or orifice, of the pipe until it was
1. WON the lower court erred in disregarding the - In the present case it is not disputed that the ship about 18 inches or 2 feet below the level of
typewritten clause endorsed upon the policy, was in grave peril and that the jettison of part of the the sea. As a consequence the sea water rose
expressly limiting insurer's liability thereunder of the cargo was necessary. If the cargo was in peril to the in the pipe. Navigation under these conditions
total loss of the wooden vessel Pandan and to extent of call for general average, the ship must also resulted in the washing out of the cement-
proportionate salvage charges have been in great danger, possibly sufficient to filling from the action of the sea water, thus
2. WON lower court erred in concluding that cause its absolute loss. The jettison was therefore as permitting the continued flow of the salt water
defendant and appellant, NUFIC is liable to much to the benefit of the underwriter as to the into the compartment of rice.
contribute to the general average resulting from the owner of the cargo. The latter was compelled to - The court found in effect that the opening above
jettison of a part of said vessel's cargo contribute to the indemnity; why should not the described had resulted in course of time from
insurer be required to do likewise? If no jettison had ordinary wear and tear and not from the straining of
HELD taken place and if the ship by reason thereof had the ship in rough weather on that voyage. The court
1. NO foundered, the underwriter's loss would have been also found that the repairs that had been made on
Ratio In case repugnance exists between written many times as large as the contribution now the pipe were slovenly and defective and that, by
and printed portions of a policy, the written portion demanded. reason of the condition of this pipe, the ship was not
prevails. Disposition Appealed judgment is affirmed properly equipped to receive the rice at the time the
Reasoning voyage was begun. For this reason the court held
- Section 291 of the Code of Civil Procedure provides GO TIACO v. UNION INSURANCE that the ship was unseaworthy.
that “when an instrument consists partly of written 40 PHIL 40 - The policy purports to insure the cargo from the
words and partly of a printed form and the two are STREET; September 1, 1919 following among other risks: "Perils . . . of the seas,
inconsistent, the former controls the latter.” men, of war, fire, enemies, pirates, rovers,
FACTS thieves, .jettisons, . . . barratry of the master and
- Union Insurance Society of Canton, Ltd., issued a mariners, and of all other perils, losses, and
2. NO marine insurance policy upon a cargo of rice misfortunes that have or shall come to the hurt,
Ratio The liability for contribution in general average belonging to the Go Tiaoco Brothers, which was detriment, or damage of the said goods and
is not based on the express terms of the policy, but transported in the early days of May, 1915, on the merchandise or any part thereof."
rests upon the theory that from the relation of the steamship Hondagua from the port of Saigon to
parties and for their benefit, a quasi contract is Cebu.
implied by law. - On discharging the rice from one of the ISSUE
Reasoning compartments in the after hold, upon arrival at WON Union Insurance is liable for the loss of the Go
- In the absence of positive legislation to the Cebu, it was discovered that 1473 sacks had been Tiaco Brothers
contrary, the liability of the defendant insurance damaged by sea water. The loss was P3,875.25.
company on its policy would, perhaps, be limited to - The trial court found that the inflow of the sea HELD
“absolute loss of the vessel only, and to pay water during the voyage was due to a defect in one NO
proportionate salvage of the declared value.” But the of the drain pipes of the ship and concluded that the - the words "all other perils, losses, and misfortunes"
policy was executed in this jurisdiction and loss was not covered by the policy of insurance. The are to be interpreted as covering risks which are of
“warranted to trade within the waters of the trial court made the ff findings: like kind (ejusdem generis) with the particular risks
Philippine Archipelago only.” Here, Art. 859 of the The drain pipe which served as a discharge which are enumerated in the preceding part of the
Code of Commerce is still in force: from the water closet passed down through same clause of the contract. ''According to the
“ART. 859. The underwriters of the vessel, of the the compartment where the rice in question ordinary rules of construction, these words must be
freight, and of the cargo shall be obliged to pay for was stowed and thence out to sea through the interpreted with reference to the words which
the indemnity of the gross average in so far as is wall of the compartment, which was a part of immediately precede them. They were no doubt
required of each one of these objects the wall of the ship. The joint or elbow where inserted in order to prevent disputes founded on nice
respectively.” the pipe changed its direction was of cast distinctions. X x x For example, if the expression
- The article is mandatory in its terms, and the iron; and in course of time it had become 'perils of the seas' is given its widest sense the
insurers (whether for the vessel or for the freight or corroded and abraded until a longitudinal general words have little or no effect as applied to
for the cargo) are bound to contribute to the opening had appeared in the pipe about one that case. If on the other hand that expression is to
indemnity of the general average. The provision inch in length. This hole had been in existence receive a limited construction, as apparently it did in
INSURANCE Page 90

Cullen vs. Butler (5 M. & S., 461), and loss by perils unseaworthy for the purpose of insurance upon the WON rusting is a “peril of the sea”
of the seas is to be confined to loss ex marine cargo (Act No. 2427, sec. 106).
tempestatis discrimine, the general words become Disposition Decision of trial court is affirmed HELD
most important. X x x" (Thames and Mersey Marine YES
Insurance Co. vs. Hamilton, Fraser & Co.) CATHAY INSURANCE CO. v. CA (REMINGTON - There is no question that the rusting of steel pipes
- a loss which, in the ordinary course of events, INDUSTRIAL SALES CORP.) in the course of a voyage is a "peril of the sea" in
results from the natural and inevitable action of the 151 SCRA 710 view of the toll on the cargo of wind, water, and salt
sea, from the ordinary wear and tear of the ship, or PARAS; June 30 1987 conditions. At any rate if the insurer cannot be held
from the negligent failure of the ship's owner to accountable therefor, We would fail to observe a
provide the vessel with proper equipment to convey FACTS cardinal rule in the interpretation of contracts,
the cargo under ordinary conditions, is not a peril of - Remington Industrial Sales Corp insured its namely, that any ambiguity therein should be
the sea. Such a loss is rather due to what has been shipment of seamless steel pipes. It incurred losses construed against the maker/issuer/drafter thereof,
aptly called the "peril of the ship." The insurer and damages (I gather the steel pipes rusted during namely, the insurer. Besides the precise purpose of
undertakes to insure against perils of the sea and the voyage from Japan to the Phils. on board vessel insuring cargo during a voyage would be rendered
similar perils, not against perils of the ship. There SS "Eastern Mariner”) and filed complaint against fruitless.
must, in order to make the insurer liable, be "some Cathay Insurance Co seeking collection of the sum of Disposition WHEREFORE, this petition is hereby
casualty, something which could not be foreseen as P868,339.15 DENIED, and the assailed decision of the Court of
one of the necessary incidents of the adventure. The - TC decided for Remington. Cathay filed MR, which Appeals is hereby AFFIRMED.
purpose of the policy is to secure an indemnity was denied. CA affirmed.
against accidents which may happen, not against - CA said (among other things): 1. Coverage of ROQUE v. IAC (PIONEER INSURANCE AND
events which must happen." (Wilson, Sons & Co. vs. private respondent's loss under the insurance policy SURETY CORP.)
Owners of Cargo per the Xantho) issued by petitioner is unmistakable; 139 SCRA 596
- In the present case the entrance of the sea water 2. Alleged contractual limitations contained in GUTIERREZ; November 11, 1985
into the ship's hold through the defective pipe insurance policies are regarded with extreme caution
already described was not due to any accident which by courts and are to be strictly construed against the NATURE
happened during the voyage, but to the failure of the insurer; obscure phrases and exceptions should not Petition for certiorari to review the decision of the
ship's owner properly to repair a defect of the be allowed to defeat the very purpose for which the IAC
existence of which he was apprised. The loss was policy was procured;
therefore more analogous to that which directly 3. Rust is not an inherent vice of the seamless steel FACTS
results from simple unseaworthiness than to that pipes without interference of external factors - February 19, 1972 – Common carrier Manila Bay
which results from perils of the sea. - Cathay contend (among other things): 1. private Lighterage Corp. entered into a contract with Roque
- there is no room to doubt the liability of the respondent has admitted that the questioned Timber Enterprises and Chiong. The contract stated
shipowner for such a loss as occurred in this case. By shipment is not covered by a "square provision of that Manila Bay would carry 422.18 cu. meters of
parity of reasoning the insurer is not liable; for, the contract," but private respondent claims implied logs on its vessel Mable 10 from Malampaya Sound,
generally speaking, the shipowner excepts the perils coverage from the phrase "perils of the sea" Palawan to Manila North Harbor. Roque insured the
of the sea from his engagement under the bill of mentioned in the opening sentence of the policy; 2. logs with Pioneer Insurance for P100,000.
lading, while this is the very peril against which the The insistence of private respondent that rusting is a - February 29, 1972 – 811 logs were loaded in
insurer intends to give protection. As applied to the peril of the sea is erroneous; 3. Rusting is not a Malampaya but en route to Manila, Mable 10 sank.
present case it results that the owners of the risk insured against, since a risk to be insured - March 8,1972 – Roque and Chiong wrote a letter to
damaged rice must look to the shipowner for redress against should be a casualty or some casualty, Manila Bay, demanding payment of P150,000.00 for
and not to the insurer. something which could not be foreseen as one of the the loss of the shipment plus P100,000.00 as
The same conclusion must be reached if the question necessary incidents of adventure; 4. A fact capable unrealized profits but the latter ignored the demand.
be discussed with reference to the seaworthiness of of unquestionable demonstration or of public - A letter was also sent to Pioneer, claiming the full
the ship. It is universally accepted that in every knowledge needs no evidence. This fact of amount of P100,000.00 under the insurance policy
contract of insurance upon anything which is the unquestionable demonstration or of public knowledge but Pioneer refused to pay on the ground that its
subject of marine insurance, a warranty is implied is that heavy rusting of steel or iron pipes cannot liability depended upon the "Total Loss by Total Loss
that the ship shall be seaworthy at the time of the occur within a period of a seven (7) day voyage. of Vessel only".
inception of the voyage. This rule is accepted in our Besides, petitioner had introduced the clear cargo - After hearing, the trial court favored Roque.
own Insurance Law (Act No. 2427, sec. 106). It is receipts or tally sheets indicating that there was no Pioneer and Manila Bay were ordered to pay Roque
also well settled that a ship which is seaworthy for damage on the steel pipes during the voyage. P100,000. Pioneer appealed the decision.
the purpose of insurance upon the ship may yet be - January 30, 1984 – Pioneer was absolved from
ISSUE liability after finding that there was a breach of
INSURANCE Page 91

implied warranty of seaworthiness on the part of the defense in order to recover on the marine insurance - The court found in effect that the opening above
petitioners and that the loss of the insured cargo was policy. described had resulted in course of time from
caused by the "perils of the ship" and not by the - Since the law provides for an implied warranty of ordinary wear and tear and not from the straining of
"perils of the sea". It ruled that the loss is not seaworthiness in every contract of ordinary marine the ship in rough weather on that voyage. The court
covered by the marine insurance policy. insurance, it becomes the obligation of a cargo also found that the repairs that had been made on
- It was alleged that Mable 10 was not seaworthy owner to look for a reliable common carrier which the pipe were slovenly and defective and that, by
and that it developed a leak keeps its vessels in seaworthy condition. The shipper reason of the condition of this pipe, the ship was not
- The IAC found that one of the hatches was left of cargo my have no control over the vessel but he properly equipped to receive the rice at the time the
open, causing water to enter the barge and because has full control in the choice of the common carrier voyage was begun. For this reason the court held
the barge was not provided with the necessary cover that will transport his goods. that the ship was unseaworthy.
or tarpaulin, the splash of sea waves brought more - In marine cases, the risks insured against are
water inside the barge. 'perils of the sea.’ The term extends only to losses ISSUE
- Petitioners contend that the implied warranty of caused by sea damage, or by the violence of the WON the insurer is liable
seaworthiness provided for in the Insurance Code elements, and does not embrace all losses
refers only to the responsibility of the shipowner who happening at sea. HELD
must see to it that his ship is reasonably fit to make - It is quite unmistakable that the loss of the cargo - The question whether the insurer is liable on this
in safety the contemplated voyage. was due to the perils of the ship rather than the policy for the loss caused in the manner above
- The petitioners state that a mere shipper of cargo, perils of the sea. stated presents two phases which are in a manner
having no control over the ship, has nothing to do - Loss which, in the ordinary course of events, involved with each other. One has reference to the
with its seaworthiness. They argue that a cargo results from the natural and inevitable action of the meaning of the expression "perils of the seas and all
owner has no control over the structure of the ship, sea, from the ordinary wear and tear of the ship, or other perils, losses, and misfortunes," as used in the
its cables, anchors, fuel and provisions, the manner from the negligent failure of the ship's owner to policy; the other has reference to the implied
of loading his cargo and the cargo of other shippers, provide the vessel with proper equipment to convey warranty, on the part of the insured, as to the
and the hiring of a sufficient number of competent the cargo under ordinary conditions, is not a ‘peril of seaworthiness of the ship.
officers and seamen. the sea’ but is called ‘peril of the ship.’ - The meaning of the expression "perils * * * of the
ISSUE Disposition Decision appealed from is affirmed. seas * * * and all other perils, losses, and
WON the loss should have been covered by the misfortunes," used in describing the risks covered by
marine insurance policy LA RAZON v. UNION INSURANCE SOCIETY OF policies of marine insurance, has been the subject of
CANTON, LTD. frequent discussion; and certain propositions relative
HELD 40 PHIL 40 thereto are now so generally accepted as to be
NO STREET; September 1, 1919 considered definitely settled.
Ratio It is universally accepted that in every - The words "all other perils, losses, and
contract of insurance upon anything which is the FACTS misfortunes" are to be interpreted as covering risks
subject of marine insurance, a warranty is implied - This is an action on a policy of marine insurance which are of like kind (ejusdem generis) with the
that the ship shall be seaworthy at the time of the issued by the Union Insurance Society of Canton, particular risks which are enumerated in the
inception of the voyage. In marine insurance, the Ltd., upon a cargo of rice belonging to the plaintiffs, preceding part of the same clause of the contract.
risks insured against are classified as 'perils of the Go Tiaoco Brothers, which was transported on the - A loss which, in the ordinary course of events,
sea,’ which includes such losses that are of steamship Hondagua from the port of Saigon to results from the natural and inevitable action of the
extraordinary nature, or arise from some Cebu. sea, from the ordinary wear and tear of the ship, or
overwhelming power, which cannot be guarded - On discharging the rice from one of the from the negligent failure of the ship's owner to
against by the ordinary exertion of human skill and compartments in the after hold, upon arrival at provide the vessel with proper equipment to convey
prudence. Cebu, it was discovered that 1,473 sacks had been the cargo under ordinary conditions, is not a peril of
Reasoning damaged by sea water. the sea. Such a loss is rather due to what has been
- Based on Sec. 113 and Sec. 99 of the Insurance - The loss so resulting to the owners of rice, after aptly called the "peril of the ship." The insurer
Code, the term "cargo" can be the subject of marine proper deduction had been made for the portion undertakes to insure against perils of the sea and
insurance and that once it is so made, the implied saved, was P3,875.25. similar perils, not against perils of the ship.
warranty of seaworthiness immediately attaches to - The trial court found that the inflow of the sea - As was said by Lord Herschell in Wilson, Sons & Co.
whoever is insuring the cargo whether he be the water during the voyage was due to a defect in one vs. Owners of Cargo per the Xantho, there must, in
shipowner or not. of the drain pipes of the ship and concluded that the order to make the insurer liable, be "some casualty,
- The fact that the un-seaworthiness of the ship was loss was not covered by the policy of insurance. something which could not be foreseen as one of the
unknown to the insured is immaterial in ordinary Judgment was accordingly entered in favor of the necessary incidents of the adventure. The purpose of
marine insurance and may not be used by him as a defendant and the plaintiffs appealed. the policy is to secure an indemnity against accidents
INSURANCE Page 92

which may happen, not against events which must - As a consequence, the molasses at the cargo tanks affreightment (time or voyage charter party) or
happen." were contaminated and rendered unfit for the use it not a charter party at all.
- In the present case the entrance of the sea water was intended. This prompted consignee Pag-asa On the other hand a contract of affreightment is
into the ship's hold through the defective pipe Sales to reject the shipment of molasses as a total one in which the owner of the vessel leases part
already described was not due to any accident which loss. Thereafter, Pag-asa Sales filed a formal claim or all of its space to haul goods for others. It is a
happened during the voyage, but to the failure of the with the insurer of its lost cargo (PhilGen) and contract for special service to be rendered by
ship's owner properly to repair a defect of the against the carrier (Coastwise). Coastwise denied the the owner of the vessel and under such contract
existence of which, he was apprised. The loss was claim and it was PhilGen which paid Pag-asa Sales the general owner retains the possession,
therefore more analogous to that which directly the amount of P700k representing the value of the command and navigation of the ship, the
results from simple unseaworthiness than to that damaged cargo of molasses. charterer or freighter merely having use of the
which results from perils of the sea. - PhilGen then filed an action against Coastwise space in the vessel in return for his payment of
- It is universally accepted that in every contract of before the RTC Manila, seeking to recover the P700k the charter hire...
insurance upon anything which is the subject of which it paid to Pag-asa Sales for the latter's lost An owner who retains possession of the ship
marine insurance, a warranty is implied that the ship cargo. PhilGen now claims to be subrogated to all the though the hold is the property of the charterer,
shall be seaworthy at the time of the inception of the contractual rights and claims which the consignee remains liable as carrier and must answer for
voyage. This rule is accepted in our own Insurance may have against the carrier, which is presumed to any breach of duty as to the care, loading and
Law (Act No. 2427, see. 106). have violated the contract of carriage. unloading of the cargo.”
- It is also well settled that a ship which is - RTC awarded the amount prayed for by PhilGen. CA - Although a charter party may transform a common
seaworthy for the purpose of insurance upon the affirmed. Hence, this petition. carrier into a private one, the same however is not
ship may yet be unseaworthy for the purpose of true in a contract of affreightment on account of the
insurance upon the cargo (Act No. 2427, see. 106). ISSUES aforementioned distinctions between the two.
Disposition Jjudgment affirmed. 1. WON Coastwise Lighterage was transformed into a - SC agrees with Coastwise's admission that the
private carrier, by virtue of the contract of contract it entered into with the consignee was one
MALAYAN INSURANCE v. CA (supra p.10) affreightment which it entered into with the of affreightment. Pag-asa Sales, Inc. only leased
consignee, Pag-asa Sales, Inc. (Corollarily, if it were three of petitioner's vessels, in order to carry cargo
FILIPINO MERCHANTS INS. CO. v. CA (supra in fact transformed into a private carrier, did it from one point to another, but the possession,
p.19) exercise the ordinary diligence to which a private command and navigation of the vessels remained
carrier is in turn bound?) with Coastwise. As such, Coastwise, by the contract
COASTWISE LIGHTERAGE CORP v. CA 2. WON the insurer was subrogated into the rights of of affreightment, was not converted into a private
(PHILIPPINE GENERAL INSURANCE COMPANY) the consignee against the carrier, upon payment by carrier, but remained a common carrier and was still
245 SCRA 796 the insurer of the value of the consignee's goods lost liable as such.
FRANCISCO; July 12, 1995 while on board one of the carrier's vessels - Therefore, the mere proof of delivery of goods in
good order to a carrier and the subsequent arrival of
NATURE HELD the same goods at the place of destination in bad
Petition for review of CA Decision affirming decision 1. NO order makes for a prima facie case against the
of RTC Manila holding that Coastwise is liable to pay - The distinction between the two kinds of charter carrier. The presumption of negligence that attaches
PhilGen Insurance the amount of P700thou plus legal parties (i.e. bareboat or demise and contract of to common carriers, once the goods it transports are
interest thereon, another sum of P100thou as affreightment) is more clearly set out in the case of lost, destroyed or deteriorated, applies to Coastwise.
attorney's fees and the cost of the suit. Puromines, Inc. vs. Court of Appeals, wherein SC This presumption, which is overcome only by proof
ruled: of the exercise of extraordinary diligence, remained
FACTS “Under the demise or bareboat charter of the unrebutted in this case.
- Pag-asa Sales, Inc. entered into a contract to vessel, the charterer will generally be regarded - The damage to the barge which carried the cargo
transport molasses from the province of Negros to as the owner for the voyage or service of molasses was caused by its hitting an unknown
Manila with Coastwise, using the latter's dumb stipulated. The charterer mans the vessel with sunken object as it was heading for Pier 18. The
barges. The barges were towed in tandem by the his own people and becomes the owner pro hac object turned out to be a submerged derelict vessel.
tugboat MT Marica, also owned by Coastwise. Upon vice, subject to liability to others for damages The evidence on record appeared that far from
reaching Manila Bay, while approaching Pier 18, one caused by negligence. To create a demise, the having rendered service with the greatest skill and
of the barges struck an unknown sunken object. The owner of a vessel must completely and utmost foresight, and being free from fault, the
forward buoyancy compartment was damaged, and exclusively relinquish possession, command and carrier was culpably remiss in the observance of its
water gushed in through a hole "two inches wide and navigation thereof to the charterer, anything duties.
twenty-two inches long." short of such a complete transfer is a contract of - Jesus R. Constantino, the patron of the vessel
"Coastwise 9" admitted that he was not licensed. The
INSURANCE Page 93

Code of Commerce, which subsidiarily governs has been obligated to pay. Payment by the insurer unseaworthiness as she was put to sea in an
common carriers (which are primarily governed by to the assured operated as an equitable assignment unstable condition. It further alleged that the
the provisions of the Civil Code) provides: “Art. 609. to the former of all remedies which the latter may vessel was improperly manned and that its
Captains, masters, or patrons of vessels must be have against the third party whose negligence or officers were grossly negligent in failing to take
Filipinos, have legal capacity to contract in wrongful act caused the loss. The right of appropriate measures to proceed to a nearby port or
accordance with this code, and prove the skill subrogation is not dependent upon, nor does it grow beach after the vessel started to list.
capacity and qualifications necessary to command out of, any privity of contract or upon written - FELMAN filed a motion to dismiss based on the
and direct the vessel, as established by marine and assignment of claim. It accrues simply upon payment affirmative defense that no right of subrogation in
navigation laws, ordinances or regulations, and must of the insurance claim by the insurer. favor of PHILAMGEN was transmitted by the shipper,
not be disqualified according to the same for the - Undoubtedly, upon payment by respondent insurer and that, in any event, FELMAN had abandoned all
discharge of the duties of the position.” PhilGen of the amount of P700,000.00 to Pag-asa its rights, interests and ownership over “MV Asilda”
- Clearly, Coastwise Lighterage's embarking on a Sales, Inc., the consignee of the cargo of molasses together with her freight and appurtenances for the
voyage with an unlicensed patron violates this rule. totally damaged while being transported by purpose of limiting and extinguishing its liability
It cannot safely claim to have exercised petitioner Coastwise Lighterage, the former was under Art. 587 of the Code of Commerce.
extraordinary diligence, by placing a person whose subrogated into all the rights which Pag-asa Sales, - Trial court dismissed the complaint of PHILAMGEN.
navigational skills are questionable, at the helm of Inc. may have had against the carrier, herein On appeal the Court of Appeals set aside the
the vessel which eventually met the fateful accident. petitioner Coastwise Lighterage. dismissal and remanded the case to the lower court
It may also logically, follow that a person without Disposition Petition denied. CA affrimed. for trial on the merits. FELMAN filed a petition for
license to navigate, lacks not just the skill to do so, certiorari with this Court but it was subsequently
but also the utmost familiarity with the usual and THE PHILIPPINE AMERICAN GENERAL denied on 13 February 1989.
safe routes taken by seasoned and legally authorized INSURANCE COMPANY INC v. CA (FELMAN - Trial court rendered judgment in favor of FELMAN.
ones. Had the patron been licensed, he could be SHIPPING LINES) It ruled that “MV Asilda” was seaworthy when it left
presumed to have both the skill and the knowledge 273 SCRA 226 the port of Zamboanga as confirmed by certificates
that would have prevented the vessel's hitting the BELLOSILLO; June 11, 1997 issued by the Philippine Coast Guard and the
sunken derelict ship that lay on their way to Pier 18. shipowner’s surveyor attesting to its seaworthiness.
- As a common carrier, Coastwise is liable for breach FACTS Thus the loss of the vessel and its entire shipment
of the contract of carriage, having failed to overcome - Coca-Cola Bottlers Philippines, Inc., loaded on could only be attributed to either a fortuitous event,
the presumption of negligence with the loss and board “MV Asilda,” a vessel owned and operated by in which case, no liability should attach unless there
destruction of goods it transported, by proof of its Felman 7,500 cases of 1-liter Coca-Cola softdrink was a stipulation to the contrary, or to the
exercise of extraordinary diligence. bottles to be transported from Zamboanga City to negligence of the captain and his crew, in which
2. YES Cebu for consignee Coca-Cola Bottlers Philippines, case, Art. 587 of the Code of Commerce should
- Coastwise is liable for breach of the contract of Inc., Cebu. The shipment was insured with petitioner apply.
carriage it entered into with Pag-asa Sales, Inc. Philippine American General under Marine Open - CA ruled that “MV Asilda” was unseaworthy for
However, for the damage sustained by the loss of Policy. being top- heavy as 2,500 cases of Coca-Cola
the cargo which petitioner-carrier was transporting, - The vessel sank in the waters of Zamboanga del softdrink bottles were improperly stowed on deck.
it was not the carrier which paid the value thereof to Norte bringing down her entire cargo with her Nonetheless, the appellate court denied the claim of
Pag-asa Sales, Inc. but the latter's insurer, herein including the subject 7,500 cases of 1-liter Coca-Cola PHILAMGEN on the ground that the assured’s implied
private respondent PhilGen. softdrink bottles. warranty of seaworthiness was not complied with.
- Article 2207 of the Civil Code: If the plaintiffs - The consignee filed a claim with respondent Perfunctorily, PHILAMGEN was not properly
property has been insured, and he has received FELMAN for recovery of damages it sustained as a subrogated to the rights and interests of the shipper.
indemnity from the insurance company for the injury result of the loss of its softdrink bottles that sank Furthermore, respondent court held that the filing of
or loss arising out of the wrong or breach of contract with “MV Asilda.” Respondent denied the claim notice of abandonment had absolved the
complained of, the insurance company shall be thus prompting the consignee to file an insurance shipowner/agent from liability under the limited
subrogated to the rights of the insured against the claim with PHILAMGEN which paid its claim of liability rule.
wrongdoer or the person who violated the contract.” P755,250.00.
- This legal provision is founded on the well-settled - Claiming its right of subrogation PHILAMGEN ISSUES
principle of subrogation. If the insured property is sought recourse against respondent FELMAN which 1. WON “MV Asilda” was seaworthy when it left the
destroyed or damaged through the fault or disclaimed any liability for the loss. Consequently, port of Zamboanga
negligence of a party other than the assured, then PHILAMGEN sued the shipowner for sum of money 2. WON the limited liability under Art. 587 of the
the insurer, upon payment to the assured will be and damages. Code of Commerce should apply
subrogated to the rights of the assured to recover - PHILAMGEN alleged that the sinking and total loss 3. WON PHILAMGEN was properly subrogated to the
from the wrongdoer to the extent that the insurer of “MV Asilda” and its cargo were due to the vessel’s rights and legal actions which the shipper had
INSURANCE Page 94

against FELMAN, the shipowner Bottlers Philippines, Inc., gave the former the right and 2213 of the Civil Code.
to bring an action as subrogee against FELMAN.
HELD Having failed to rebut the presumption of fault, the PHILIPPINE MFTG. CO. v. UNION INSURANCE
1. YES liability of FELMAN for the loss of the 7,500 cases of SOCIETY OF CANTON
- “MV Asilda” was unseaworthy when it left the port 1-liter Coca-Cola softdrink bottles is inevitable. 42 PHIL 378
of Zamboanga. We subscribe to the findings of the - Sec. 113 of the Insurance Code provides that “(i)n JOHNS; November 22, 1921
Elite Adjusters, Inc., and the Court of Appeals that every marine insurance upon a ship or freight, or
the proximate cause of the sinking of “MV Asilda” freightage, or upon anything which is the subject of FACTS
was its being top-heavy. Contrary to the ship marine insurance, a warranty is implied that the ship - The plaintiff’s steel tank lighter was insured by
captain’s allegations, evidence shows that is seaworthy.” Under Sec. 114, a ship is “seaworthy defendant company for absolute total loss. As a
approximately 2,500 cases of softdrink bottles were when reasonably fit to perform the service, and to result of a typhoon, the lighter sunk in Manila Bay.
stowed on deck. Several days after “MV Asilda” encounter the ordinary perils of the voyage, The plaintiff demanded payment from the defendant
sank, an estimated 2,500 empty Coca-Cola plastic contemplated by the parties to the policy.” Thus it insurance company but the latter refused. The
cases were recovered near the vicinity of the sinking. becomes the obligation of the cargo owner to look company asked the plaintiff to salvage the ship,
Considering that the ship’s hatches were properly for a reliable common carrier which keeps its which it was able to do so.
secured, the empty Coca-Cola cases recovered could vessels in seaworthy condition. He may have no - With the plaintiff able to raise the lighter,
have come only from the vessel’s deck cargo. It is control over the vessel but he has full control in reconstruct it and placed it in commission, the
settled that carrying a deck cargo raises the the selection of the common carrier that will defendant insurance company claims that it was only
presumption of unseaworthiness unless it can be transport his goods. He also has full discretion in the liable for a total absolute loss and that there was no
shown that the deck cargo will not interfere with the choice of assurer that will underwrite a particular total destruction of the lighter.
proper management of the ship. However, in this venture. - The trial court decided in favor of the defendant,
case it was established that “MV Asilda” was not - In policies where the law will generally imply a saying that the policy only covered an actual total
designed to carry substantial amount of cargo on warranty of seaworthiness, it can only be excluded loss, not a constructive total loss.
deck. The inordinate loading of cargo deck resulted by terms in writing in the policy in the clearest
in the decrease of the vessel’s metacentric height language. And where the policy stipulates that the ISSUES
thus making it unstable. The strong winds and seaworthiness of the vessel as between the assured 1. WON there was an absolute total loss that can be
waves encountered by the vessel are but the and the assurer is admitted, the question of covered by the policy
ordinary vicissitudes of a sea voyage and as such seaworthiness cannot be raised by the assurer 2. WON the Marine Law of Great Britain applies
merely contributed to its already unstable and without showing concealment or misrepresentation
unseaworthy condition. by the assured. HELD
2. NO - PHILAMGEN’s action against FELMAN is squarely 1. YES
- The ship agent is liable for the negligent acts of the sanctioned by Art. 2207 of the Civil Code which - At the time that the lighter was at the bottom of
captain in the care of goods loaded on the vessel. provides: the bay, it was of no value to the owner, thus there
This liability however can be limited through Art. 2207. If the plaintiff’s property has been was an actual total loss.
abandonment of the vessel, its equipment and insured, and he has received indemnity from - The ship was sunk in July 1, 1918. After several
freightage as provided in Art. 587. Nonetheless, the insurance company for the injury or loss futile attempts, it was finally raised on Sept. 20,
there are exceptional circumstances wherein the arising out of the wrong or breach of contract 1918. It is faitr to assume that in its then condition
ship agent could still be held answerable despite the complained of, the insurance company shall much further time would be required to make the
abandonment, as where the loss or injury was due to be subrogated to the rights of the insured necessary repairs and install the new machinery
the fault of the shipowner and the captain. The against the wrongdoer or the person who has before it could again be placed in commission.
international rule is to the effect that the right of violated the contract. If the amount paid by During that time the owner would be deprived of the
abandonment of vessels, as a legal limitation of a the insurance company does not fully cover use of its vessel or the interest on its investment.
shipowner’s liability, does not apply to cases where the injury or loss, the aggrieved party shall When those questions are considered the testimony
the injury or average was occasioned by the be entitled to recover the deficiency from the is conclusive that the cost of salvage, repair and
shipowner’s own fault. person causing the loss or injury. reconstruction was more than the original cost of the
3. YES Disposition Petition is GRANTED. Respondent ship at the time the policy was issued. As found by
- The doctrine of subrogation has its roots in equity. FELMAN SHIPPING LINES is ordered to pay petitioner the trial court, “t is difficult to see how there could
It is designed to promote and to accomplish justice PHILIPPINE AMERICAN GENERAL INSURANCE CO., have been a more complete loss of the vessel than
and is the mode which equity adopts to compel the INC., Seven Hundred Fifty-five Thousand Two that which actually occurred”. Upon the facts shown
ultimate payment of a debt by one who in justice, Hundred and Fifty Pesos (P755,250.00) plus legal here, any other construction would nullify the statute
equity and good conscience ought to pay. Therefore, interest thereon counted from 29 November 1983, and as applied to the conditions existing in the
the payment made by PHILAMGEN to Coca-Cola the date of judicial demand, pursuant to Arts. 2212 Manila Bay, this kind of policy would be worthless,
INSURANCE Page 95

and there would not be any consideration for the 600 bags delivered to petitioner, 403 were in bad insured. Claims recoverable hereunder shall be
premium. order. The surveys showed that the bad order bags payable irrespective of percentage."
2. NO suffered spillage and loss later valued at P33,117.63. - The terms of the policy are so clear and require no
- The defendant argues that the policy contains the Petitioner filed a claim for said loss dated February interpretation. The insurance policy covers all loss or
provision that it “shall be of as force and effect as 16, 1977 against respondent insurance company in damage to the cargo except those caused by delay
the surest writing or policy of insurance made in the amount of P33,117.63 as the insured value of or inherent vice or nature of the cargo insured. It is
London”. However, for such law to apply to our the loss. the duty of the respondent insurance company to
courts the existence of such law must be proven. It - Respondent insurance company rejected the claim establish that said loss or damage falls within the
cannot apply when such proof is lacking. alleging that assuming that spillage took place while exceptions provided for by law, otherwise it is liable
Nevertheless, in the English practice, a ship is a total the goods were in transit, petitioner and his agent therefor.
loss when she has sustained such extensive failed to avert or minimize the loss by failing to - An "all risks" provision of a marine policy creates a
damages that it would not be reasonably practical to recover spillage from the sea van, thus violating the special type of insurance which extends coverage to
repair her. terms of the insurance policy sued upon; and that risks not usually contemplated and avoids putting
Disposition Decision reversed assuming that the spillage did not occur while the upon the insured the burden of establishing that the
cargo was in transit, the said 400 bags were loaded loss was due to peril falling within the policy's
in bad order, and that in any case, the van did not coverage. The insurer can avoid coverage upon
carry any evidence of spillage. demonstrating that a specific provision expressly
- Petitioner filed a complaint in the RTC against the excludes the loss from coverage.
insurance company seeking payment of the sum of - In this case, the damage caused to the cargo has
P33,117.63 as damages plus attorney's fees and not been attributed to any of the exceptions provided
expenses of litigation. Insurance company denied all for nor is there any pretension to this effect. Thus,
the material allegations of the complaint and raised the liability of respondent insurance company is
several special defenses as well as a compulsory clear.
counterclaim. Insurance company filed a third-party Disposition the decision appealed from is hereby
CHOA TIEK SENG v. CA (FILIPINO MERCHANTS complaint against respondents Ben Lines and broker. REVERSED AND SET ASIDE and another judgment is
INSURANCE) - RTC dismissed the complaint, the counterclaim and hereby rendered ordering the respondent Filipinas
183 SCRA 223 the third-party complaint with costs against the Merchants Insurance Company, Inc. to pay the sum
GANCAYO; March 15, 1990 petitioner. Appealed in CA but denied. MFR was of P33,117.63 as damages to petitioner with legal
denied as well. interest from the filing of the complaint, plus
NATURE attorney's fees and expenses of litigation in the
Appeal from a decision of the Court of Appeals ISSUE amount of P10,000.00 as well as the costs of the
WON insurance company should be held liable even suit.
FACTS if the technical meaning in marine insurance of an
- Petitioner imported some lactose crystals from “insurance against all risk" is applied FILIPINO MERCHANTS INS. CO. v. CA (supra
Holland. p.19)
- The importation involved fifteen (15) metric tons HELD
packed in 600 6-ply paper bags with polythelene YES ABOITIZ SHIPPING v. PHILAMGEN INSURANCE
inner bags, each bag at 25 kilos net. The goods were - In Gloren Inc. vs. Filipinas Cia. de Seguros, 12 it 179 SCRA 357
loaded at the port at Rotterdam in sea vans on board was held that an all risk insurance policy insures GANCAYCO; October 5, 1989
the vessel "MS Benalder' as the mother vessel, and against all causes of conceivable loss or damage,
thereafter aboard the feeder vessel "Wesser Broker except as otherwise excluded in the policy or due to NATURE
V-25" of respondent Ben Lines Container, Ltd. (Ben fraud or intentional misconduct on the part of the Petition for review on certiorari
Lines for short). The goods were insured by the insured. It covers all losses during the voyage
respondent Filipino Merchants' Insurance Co., Inc. whether arising from a marine peril or not, including FACTS
(insurance company for short) for the sum of pilferage losses during the war. - Marinduque Mining Industrial Corporation
P98,882.35, the equivalent of US$8,765.00 plus - In the present case, the "all risks" clause of the (Marinduque) shipped on board SS Arthur Maersk
50% mark-up or US $13,147.50, against all risks policy sued upon reads as follows: from Boston, U.S.A. a shipment of 1 skid carton
under the terms of the insurance cargo policy. Upon "5. This insurance is against all risks of loss or parts for valves. The shipment was ordered from
arrival at the port of Manila, the cargo was damage to the subject matter insured but shall in Jamesbury, Singapore PTE, LTD., which issued the
discharged into the custody of the arrastre operator no case be deemed to extend to cover loss, cargo's packing list and Invoice number showing the
respondent E. Razon, Inc. (broker for short), prior to damage, or expense proximately caused by delay contents of the carton. The Philippine Consulate in
the delivery to petitioner through his broker. Of the or inherent vice or nature of the subject matter Singapore issued invoice for the shipment showing
INSURANCE Page 96

the contents and its total price of $39,419.60 and [a] Records of the case show that Phil-Am executed - Panama demanded payment for the loss but
the freight and other charges of $2,791.73. When a continuous and open insurance coverage covering Oriental Assurance refuse on the ground that its
the cargo arrived in Manila, it was received and goods of Marinduque imported into and exported contracted liability was for "TOTAL LOSS ONLY."
deposited in the office of Aboitiz Shipping from the Philippines which took effect after Sept. 1, - Unable to convince Oriental Assurance to pay its
Corporation (Aboitiz) for transhipment to Nonoc 1975, as contained in Marine Open Policy No. claim, Panama filed a Complaint for Damages against
Island. 100184. A similar insurance coverage was also Oriental Assurance before the Regional Trial Court.
- In July 1980, Marinduque, as consignee of the executed by petitioner in favor of Marinduque for all - RTC ordered Oriental Assurance to pay Panama
cargo, made a report that said cargo was pilfered on its goods shipped or moved within the territorial with the view that the insurance contract should be
July 3, 1980 due to heavy rain at the Aboitiz limits of the Philippines also effective after Sept. 1, liberally construed in order to avoid a denial of
terminal and that of the total value of the cargo of 1975 and contained in Marine Open Policy No. substantial justice; and that the logs loaded in the
$42,209.33, only $7,412.00 worth remains of the 100185. two barges should be treated separately such that
cargo with the recommendation that the claim be [b] TC in dismissing the complaint apparently relied the loss sustained by the shipment in one of them
made against Aboitiz. on Marine Risk Note No. 017545 issued by private may be considered as "constructive total loss" and
- The services of the Manila Adjusters and Surveyors respondent Phil-Am only on July 28, 1980 after the correspondingly compensable. CA affirmed in toto.
Co. (Manila Adjusters) were engaged by the Phil- shipment in question was already pilfered. Obviously
American General Insurance Co., Inc. (Phil Am) TC mistook said Marine Risk Note as an insurance ISSUE
which came out with the report that the cargo in policy when it is NOT. It is only an acknowledgment WON Oriental Assurance can be held liable under its
question, when inspected, showed that it was or declaration of the private respondent confirming marine insurance policy based on the theory of a
pilfered. A confirmatory report was submitted by the the specific shipment covered by its Marine Open divisible contract of insurance and, consequently, a
Manila Adjusters. Policy, the evaluation of the cargo and the constructive total loss
- On August 11, 1980 Marinduque then filed a claim chargeable premium.
against Aboitiz in the amount of P246,430.80 [c] The contention of the Aboitiz that it could not be HELD
representing the value of the pilfered cargo. On the liable for the pilferage of the cargo as it was stolen NO
same day Marinduque filed a claim for the same even before it was loaded on its vessel is untenable. - The terms of the contract constitute the measure of
amount against the Phil-Am on the latter's policy. Aboitiz received cargo when it arrived in Manila at its the insurer liability and compliance therewith is a
Phil-Am paid Marinduque the sum of P246,430.80 as offices, and it was while in its possession and before condition precedent to the insured's right to recovery
insurer of the cargo. loading it in its vessel that the cargo was pilfered. Its from the insurer. Whether a contract is entire or
- Phil-Am then filed a complaint in RTC Manila liability is clear. severable is a question of intention to be determined
against Aboitiz for recovery of same amount alleging Disposition Petition DISMISSED. by the language employed by the parties. The policy
that it has been subrogated to the rights of in question shows that the subject matter insured
Marinduque. Complaint dismissed and MFR denied. ORIENTAL ASSURANCE v. CA (PANAMA SAW was the entire shipment of 2,000 cubic meters of
CA reversed. MFR thereof was denied. Hence, this MILL) apitong logs. The fact that the logs were loaded on
petition. 200 SCRA 459 two different barges did not make the contract
MELENCIO-HERRERA; August 9, 1991 several and divisible as to the items insured. The
ISSUE logs on the two barges were not separately valued or
WON petitioner Aboitiz was properly held liable to NATURE separately insured. Only one premium was paid for
the private respondent Phil-Am by the appellate Petition for review on certiorari the entire shipment, making for only one cause or
court consideration. The insurance contract must,
FACTS therefore, be considered indivisible.
HELD - Sometime in January 1986, private respondent - More importantly, the insurer's liability was for
YES Panama Sawmill Co., Inc. (Panama) bought, in "total loss only." A total loss may be either actual or
- The questioned shipment is covered by a Palawan, 1,208 pieces of apitong logs, with a total constructive (Sec. 129, Insurance Code). An actual
continuing open insurance coverage (which took volume of 2,000 cubic meters. It hired Transpacific total loss is caused by:
effect after Sept. 1, 1975, as contained in Marine Towage, Inc., to transport the logs by sea to Manila (a) A total destruction of the thing insured;
Open Policy No. 100184) from the time it was loaded and insured it against loss for P1-M with petitioner (b) The irretrievable loss of the thing by sinking, or
aboard the SS Arthur Maersk in Boston, U.S.A. to the Oriental Assurance Corporation (Oriental Assurance). by being broken up;
time it was delivered to the possession of petitioner - While the logs were being transported, rough seas (c) Any damage to the thing which renders it
at its offices at Pier 4 in Manila until it was pilfered and strong winds caused damage to one of the two valueless to the owner for the purpose for which
when the great majority of the cargo was lost on July barges resulting in the loss of 497 pieces of logs out he held it; or
3, 1980. Hence, petitioner Aboitiz was properly held of the 598 pieces loaded thereon. (d) Any other event which effectively deprives the
liable to Phil-Am. owner of the possession, at the port of destination,
Reasoning
INSURANCE Page 97

of the thing insured. (Section 130, Insurance petitions winch is the subject of the present action. however, claims that, for all intents and purposes, it
Code). The cargo was loaded on board LUZTEVECO Barge has practically lost its total or entire shipment in this
- A constructive total loss is one which gives to a No. LC-3000 and consisted of 34,122 bags of IR-36 case, inclusive of expenses, premium fees, and so
person insured a right to abandon, under Section certified rice seeds purchased by FAO from the forth, despite the alleged recovery by defendant
139 of the Insurance Code. This provision reads: Bureau of Plant Industry for P4,602,270.00 LUZTEVECO. As found by the court below and
SECTION 139. A person insured by a contract of - FAO secured insurance coverage in the amount of reproduced with approval by respondent court, FAO
marine insurance may abandon the thing insured, P5,250,000.00 from petitioner, Pan Malayan "has never been compensated for this total loss or
or any particular portion thereof separately valued Insurance Corporation damage, a fact which is not denied nor controverted
by the policy, or otherwise separately insured, and - On June 16, 1980, FAO gave instructions to - If there were some cargoes saved, by LUZTEVECO,
recover for a total loss thereof, when the cause of LUZTEVECO to leave for Vaung Tau, Vietnam to private respondent abandoned it and the same was
the loss is a peril injured against, deliver the cargo which, by its nature, could not sold or used for the benefit of LUZTEVECO or Pan
(a) If more than three-fourths thereof in value is withstand delay because of the inherent risks of Malayan Corporation. Under Sections 129 and 130 of
actually lost, or would have to be expended to termination and/or spoilage. On the same date, the the New Insurance Code, a total loss may either be
recover it from the peril; insurance premiums on the shipment was paid by actual or constructive. In case of total loss in Marine
(b) If it is injured to such an extent as to reduce FAO petitioner Insurance, the assured is entitled to recover from
its value more than three-fourths; - On June 26, 1980, FAO was advised of the sinking the underwriter the whole amount of his subscription
xxx xxx xxx of the barge in the China Sea, hence it informed - SEC. 130. An actual total loss is caused by: (c) Any
- The requirements for the application of Section 139 petitioner thereof and, later, formally filed its claim damage to the thing which renders it valueless to the
of the Insurance Code, quoted above, have not been under the marine insurance policy. On July 29, 1980, owner for the purpose for which he held it; or
met. The logs involved, although placed in two FAO was informed by LUSTEVECO of the recovery of (d) Any other event which effectively deprives the
barges, were not separately valued by the policy, nor the lost shipment, for which reason FAO formally owner of the possession, at the port of destination of
separately insured. Resultantly, the logs lost in the filed its claim with LUZTEVECO for compensation of the thing insured.
damaged barge in relation to the total number of damage to its cargo -as said and proven, the seeds were of fragile
logs loaded on the same barge cannot be made the - LUZTEVECO failed and refused to pay. Pan Malayan nature. And the wetting of said seeds affected the
basis for determining constructive total loss. The logs likewise failed to pay for the losses and damages state of seeds. Thus rendering them useless for FAO.
having been insured as one inseparable unit, the sustained by FAO by reason of its inability to recover Although there were bags which were recovered,
correct basis for determining the existence of the value of the shipment from LUZTEVECO these were “stained” and not in the same condition it
constructive total loss is the totality of the shipment - Pan Malayan claims that part of the cargo was was brought in. in addition to this, FAO did not
of logs. Of the entirety of 1,208, pieces of logs, only recovered and thus the claim by FAO was receive any compensation for said recovered bags as
497 pieces thereof were lost or 41.45% of the entire unwarranted. This is evidenced by two surveys upon the same were distributed by LUZVETECO without
shipment. Since the cost of those 497 pieces does the cargo wherein it was found that only around authorization of FAO
not exceed 75% of the value of all 1,208 pieces of 78% was lost. - the complete physical destruction of the subject
logs, the shipment cannot be said to have sustained - FAO filed a civil case against both LUZTEVECO and matter is not essential to constitute an actual total
a constructive total loss under Section 139(a) of the Pan Malayan. Trial court found in favor of FAO and loss. Such a loss may exist where the form and
Insurance Code. ordered both to pay jointly and severally the full specie of the thing is destroyed, although the
Disposition judgment under review is SET ASIDE amount of the claim. This was affirmed by CA materials of which it consisted still exist (Great
Western Ins. Co. vs. Fogarty, N.Y., 19 Wall 640, 22
PAN MALAYAN INSURANCE v. CA ( THE FOOD ISSUE L. Ed. 216), as where the cargo by the process of
AND AGRICULTURAL ORGANIZATION OF THE 1. WON respondent court committed a reversible decomposition or other chemical agency no longer
UNITED NATIONS) error in holding that the trial court is correct in remains the same kind of thing as before (Williams
201 SCRA 382 holding that there is a total loss of the shipment vs. Cole, 16 Me. 207).
REGALADO; September 5, 1991 - It is thus clear that FAO suffered actual total loss
HELD under Section 130 of the Insurance Code, specifically
FACTS 1. NO under paragraphs (c) and (d) thereof, recompense
- The Food and Agricultural Organization of the - The law classifies loss into either total or partial. for which it has been denied up to the present
United Nations (hereinafter referred to as FAO), Total loss may be actual or absolute, or it may -Section 135 of the Insurance Code explicitly
ntended and made arrangements to send to otherwise be constructive or technical. Petitioner provides that "(u)pon an actual total loss, a person
Kampuchea 1,500 metric petitions of IR-36 certified submits that respondent court erred in ruling that insured is entitled to payment without notice of
rice seeds to be distributed to the people for seedling there was total loss of the shipment despite the fact abandonment." This is a statutory adoption of a long
purposes that only 27,922 bags of rice seeds out of 34,122 standing doctrine in maritime insurance law that in
- LUZTEVECO was to ship the cargo amounting to bags were rendered valueless to FAO and the case of actual total loss, the right of the insured to
US$83,325.92 in respect of one lot of 1,500 metric shipment sustained only a loss of 78%. - FAO,
INSURANCE Page 98

claim the whole insurance is absolute, without need indicated that the insured was already dead at the likewise unable to make out any clear motive as to
of a notice of abandonment time the policy was applied for. It also why Ramon Piganto would purposely lie. Mere
counterclaimed for attorney’s fees. The first report, allegations of fraud could not substitute for the full
PHILIPPINE AMERICAN LIFE INSURANCE prepared by one Dr. Benedicto Briones, was dated and convincing evidence that is required to prove it.
COMPANY v. CA (ELIZA PULIDO) April 1, 1992, and had attached to it a questionnaire, A failure to do so would leave intact the presumption
344 SCRA 260 responded to by one Ramon Piganto, who of good faith and regularity in the performance of
GONZAGA-REYES; November 15, 2000 represented to be the brother-in-law of the insured public duties, which was the basis of both
and the barangay chairman of Cardiz, Bagulin, La respondent court and the trial court in finding the
NATURE Union. To the question “Where does [Florence date of Florence Pulido’s death to be as plaintiff-
This petition for review on certiorari seeks to reverse Pulido] reside now?”, Piganto had replied that private respondent maintained.
the Decision of the Special Second Division of the Florence Pulido used to live in Cardiz, but was dead - We cannot likewise give credence to petitioner’s
Court of Appeals since 1988. Piganto’s statement was signed by him, submission that the inconsistencies in the
and witnessed by his wife, Nenita Piganto. This testimonies of the witnesses for plaintiff-private
FACTS report was petitioner’s basis for treating the disputed respondent are in themselves evidence of fraud.
- On January 9, 1989, petitioner received from one policy as void since April 1992, even before receipt Such alleged inconsistencies are matters of
Florence Pulido an application for life insurance, of private respondent’s claim. credibility which had been ably passed upon by the
dated December 16, 1988, in the amount of lower court.
P100,000.00 which designated her sister, herein ISSUE - The absence of fraud, as a factual finding of the
private respondent, as its principal beneficiary. WON there was fraud (whether the insured, Florence lower court adopted by the Court of Appeals, entirely
Because the insurance applied for was non-medical, Pulido, was in fact dead before the application for consistent with the evidence on record, will not be
petitioner did not require a medical examination and insurance on her life was made) reversed and, hence, is final and conclusive upon
issued a policy on the sole basis of the application on this Court.
February 11, 1989. On April 1992, petitioner HELD Disposition The instant petition is DENIED
received private respondent’s claim, which declared NO
that the insured, Florence Pulido, died of acute - This the lower courts had effected ruled on, upon a CHAPTER IX. CLAIMS SETTLEMENT &
pneumonia on September 10, 1991. preponderance of the evidence duly received from SUBROGATION
- Petitioner withheld payment on the ground that the both parties. We see no reversible error in the
policy claimed under was void from the start for finding of both respondent court and the trial court in LONDRES v. NATIONAL LIFE INSURANCE
having been procured in fraud. It is petitioner’s favor of the correctness of the entries in Certificate 94 PHIL 627
contention that even before they received private of Death, duly registered with the Local Civil BAUTISTA ANGELO; March 29, 1954
respondent’s claim for death benefits, their Registrar of Bagulin, La Union, which declared that
investigation concerning the subject policy yielded Florence Pulido died of acute pneumonia on NATURE
the information that the insured, Florence Pulido, September 10, 1991. Dr. Irineo Gutierrez, the Appeal from a decision of the Court of First Instance
died in 1988, before the application for insurance on Municipal Health Officer of Bagulin, La Union whose of Manila ordering defendant to pay to plaintiff the
her life was made. While this was communicated to signature appeared in the death certificate, testified sum of P3,000, Philippine currency, plus legal
private respondent in a letter dated April 29, 1992, in addition that he ministered to the ailing Florence interest thereon from the time of the filing of the
private respondent had already filed her claim earlier Pulido for two days immediately prior to her death. complaint until its full payment.
that month. In another letter dated July 27, 1992, This fact is likewise noted in the death certificate.
however, petitioner confirmed to private respondent - Death certificates, and notes by a municipal health FACTS
receipt of the claim papers and assured her that her officer prepared in the regular performance of his - On April 14, 1943, the National Life Insurance
case was “being given preferential attention and duties, are prima facie evidence of facts therein Company of the Philippines issued a policy on the life
prompt action”. stated. A duly-registered death certificate is of Jose C. Londres whereby it undertook to pay its
- Following the filing by private respondent of her considered a public document and the entries found beneficiary upon his death the sum of P3,000. All the
claim, petitioner caused another investigation therein are presumed correct, unless the party who premiums due under the policy were actually paid on
respecting the subject policy. Pursuant to the contests its accuracy can produce positive evidence their dates of maturity and the policy was in force
findings of this second investigation, petitioner stood establishing otherwise. Petitioner’s contention that when the insured died on February 7, 1945.
by its initial decision to treat the policy as void and the death certificate is suspect because Dr. Gutierrez Salvacion V. Londres, as beneficiary, demanded from
not to honor the claim. On November 9, 1992, was not present when Florence Pulido died, and the company the payment of the proceeds of the
private respondent enlisted the services of counsel in knew of Florence’s death only through Ramon policy, and her demand having been refused, she
reiterating her claim for death benefits. Petitioner Piganto, does not merit a conclusion of fraud. No instituted the present action against the company in
still refused to make payment and thus, this action. motive was imputed to Dr. Gutierrez for seeking to the Court of First Instance of Manila.
- Petitioner: the results of its investigations having perpetuate a falsity in public records. Petitioner was - Defendant in its answer denied, for lack of
INSURANCE Page 99

sufficient proof, the allegation that the insured died itself to pay to the insured under the policy upon his "The parties herein gambled and speculated on the
on February 7, 1945, and set up the following death should be paid in accordance with the present date of the termination of the war and the
special defenses: (a) that plaintiff's claim is covered currency or should be adjusted under the Ballantyne liberation of the Philippines by the Americans. This
by the Moratorium Law; (b) that the policy having scale of values can be gleaned from the stipulation about
been issued during the Japanese occupation, it is redemption, particularly that portion to the effect
presumed that its face value should be paid in HELD that redemption could be effected not before the
Japanese currency, there being no provision in the YES, present currency. expiration of one year from June 24, 1944. This
policy from which can be inferred that the parties Reasoning kind of agreement is permitted by law. We find
contemplated payment in any other currency; (c) - In the case of Rutter vs. Esteban, 93 Phil., 68, the nothing immoral or unlawful in it." (Gomez vs.
that the money paid by the insured as premiums, Moratorium Law was declared invalid and Tabia)
together with the money received from other policy- unconstitutional. Disposition Wherefore, the decision appealed from
holders, was all deposited by the defendant in the - During those days of liberation, while the people is affirmed, with costs against appellant.
Philippine National Bank and said deposit was were rejoicing because of the happy event, the
declared without value by Executive Order No. 49 of banks, the insurance companies, and for that matter
the President of the Philippines; and (d) that the other commercial and business firms, were still
policy having been issued under abnormal feeling the adverse effects of the sudden fall of
circumstances, it should be considered in the light of values and were uncertain and apprehensive as to
equity which does not permit anyone to enrich the manner the readjustment would be made by the VDA. DE FERNANDEZ v. NATIONAL LIFE
himself at the expense of another. Defendant, new Government. It is for this reason that the INSURANCE CO OF THE PHILS
however, as a proof of good faith, offered to pay the beneficiary, after realizing the truth about the death 105 PHIL 59
value of the policy in accordance with the Ballantyne of her husband, and after gathering evidence to ENDENCIA; January 27, 1959
scale of values, or the sum of P2,400, Philippine substantiate his death, had difficulty in effecting the
currency. collection of her claim from the insurance company NATURE
- It appears that the deceased took up the policy because at that time it had not yet reopened for Appeal from CFI decision applying the Ballantyne
under consideration on April 15, 1943 for the sum of business purposes. Although the record does not scale of values upon the proceeds of life insurance
P3,000. All the premiums due under the policy were disclose the exact date on which the insurance taken and maturing during the Japanese occupation
actually paid on their dates of maturity and the company reopened for this purpose, this Court can but claimed after liberation
policy was in force when the insured died on take judicial notice that it only did so after liberation.
February 7, 1945. On said date, the battle of the At that time the legal tender was already the FACTS
liberation of the City of Manila was still raging. While present currency. - National Life Insurance Company (NLIC) insured J.
the northern part may have been liberated, not so - As final plea, appellant invokes equity in its favor in Fernandez’s life for P10,000 upon his payment of
the southern part, as shown from the very affidavits view of the nullification of the deposits made by it P444 from July 15, 1944 to July 14, 1945
submitted by appellee wherein it was stated that on with the Philippine National Bank of all fiat money - The insured died on November 2, 1944, while the
the aforesaid date, the insured, Jose Londres, and received from its policyholders, which money was policy was in force
his two sons were taken by the Japanese soldiers declared without value by Executive Order No. 49 of - After more than 7 years, in 1952, Atty de la Torre,
from their house at Singalong Street and were the President of the Philippines. Appellant claims representing the benficiaries of the policy, informed
massacred by their captors. It may therefore be said that, considering the unexpected circumstances that the company that Fernandez had died in 1944, and
that the policy became due when the City of Manila developed, the indemnity to be paid by it should be claimed the proceeds of the policy. The company
was still under the yoke of the enemy and became suffered by it under Article 307 of the Code of said that the status of the policies issued during the
payable only after liberation which took place on Commerce which provides: "When the deposits are Japanese occupation was still pending consideration
March 10, 1945 when President Osmena issued of cash, with a specification of the coins constituting before the courts. NLIC said that because the policy
Proclamation No. 6 following the restoration of the them, . . . the increase or reduction which their matured upon the insured’s death in November,
civil government by General Douglas Mac Arthur. value may suffer shall be for the account of the 1944, they should compute the value of their claim
And we say that the policy became payable only depositor." Appellant, by entering into an under the Ballantyne scale of values (which would
after liberation even if it matured sometime before, insurance contract, cannot claim, if it suffers amount only to P500)
because before that eventuality the insurance loss, that the beneficiary cannot enrich herself - beneficiaries commenced suit, and the lower court
company, appellant herein, was not yet in a position at its expense. This is a risk attendant to any sustained the stand of the company, dismissed the
to pay the value of the policy for the simple reason wagering contract. One who gambles and loses complaint.
that it had not yet reopened. cannot be heard to complain of his loss. To - beneficiaries maintain that the obligation of the
appellant, we can only repeat the following company to pay accrued not upon the death of
ISSUE admonition: Fernandez, but only upon the receipt and approval
WON the amount of P3,000 which appellant bound by the company, on proof of death of the insured,
INSURANCE Page
100

which was in 1954. The policy reads: proceeds are payable to his beneficiaries within sixty - The trial court, upon motion by petitioner, issued a
National Life Insurance Company of the Philippine days after their filing of proof of death. writ of execution against EASCO. The sheriff
hereby agrees to pay at its Home Office, Manila, Reasoning enforcing the writ reportedly fixed the legal rate of
Ten Thousand Pesos to Juan D. Fernandez - The sixty day period fixed by law within which to interest at 12%. Respondent EASCO moved to quash
(hereinafter called the insured) on the 15th day of pay the proceeds after presentation of proof of death the writ alleging that the legal interest to be
July, 1964, if the Insured is living and this Policy is Is merely procedural in nature, evidently to computed should be 6% per cent per annum in
in force, or upon receipt and approved at its Office determine the exact amount to be paid and the accordance with Article 2209 of the Civil Code. The
of due proofs of the title of the claimant and of the interest thereon to which the beneficiaries may be trial court denied EASCO's motion. On appeal, the
prior death of the Insured while this Policy is in entitled to collect in case of unwarranted refusal of Court of Appeals reversed the trial court’s denial of
force to Teresa Duat Vda. De Fernandez, Maria T. the company to pay, and also to enable the insurer EASCO’s motion and ruled that the applicable
and Manuela Fernandez, mother and sisters to verify or check on the fact of death which it may interest is 6% per annum. Hence, this petition.
respectively of the Insured (Hereinafter called the even validly waive. It is the happening of the
Beneficiary) subject to the right of the Insured to suspensive condition of death that renders a life ISSUE
change the beneficiary as stated on the second policy matured, and not ht efiling of proof of death WON the applicable rate of interest is 12% per
page of this Policy. which, as above stated, is merely procedural. The annum
- The above stipulation is apparently based on Sec. insured having died during the Japanese occupation,
91-A of the Insurance Law which provides as follows: the proceeds of his policy should be adjusted HELD
The proceeds of a life insurance policy shall be paid accordingly, for “The rule is already settled that NO
immediately upon maturity of the policy, unless such where a debtor could have paid his obligation at any - Sections 243 and 244 of the Insurance Code apply
proceeds are made payable in installments or a as an time during the Japanese occupation, payment after only when there is an unjustified refusal or
annuity, in which case the installments or annuities liberation must be adjusted in accordance with the withholding of payment on the insured’s claim. In
shall be paid as they become due: Provided, Ballantyne schedule (De Asis vs. Agdamag, among this case, EASCO's refusal to settle the claim to Tio
however, That in case of a policy maturing by the other cases). (Collaboration is defined as the acts Khe Chio was based on some ground which, while
death of the insured, the proceeds thereof shall be of working together in a joint project. not sufficient to free it from liability under its policy,
paid within sixty days after presentation of the claim Disposition Judgment affirmed nevertheless is sufficient to negate any assertion
and filing of the proof of the death of the insured. that in refusing to pay, it acted unjustifiably. Simply
Refused to pay the claim within the time prescribed TIO KHE CHIO v. CA (EASTERN ASSURANCE & put, the said provisions of the Insurance Code are
herein will entitle the beneficiary to collect interest SURETY) not pertinent to the instant case. They apply only
on the proceeds of the policy for the duration of the 202 SCRA 119 when the court finds an unreasonable delay or
delay at the rate of six per centum per annum, FERNAN; September 30, 1991 refusal in the payment of the claims.
unless such failure or refusal to pay is based on the - Circular No. 416 of the Central Bank, which raised
ground that the claim is fraudulent . . . . FACTS the legal rate of interest from 6% to 12% per annum
- Based on the foregoing provision of law and the - Petitioner Tio Khe Chio imported 1,000 bags of refers only to loans or forbearances of money, goods
aforequoted stipulation as well as on the allegation fishmeal valued at $36,000.30 from Agro Impex, or credits and court judgments thereon but not to
that the filing of proof of death by the beneficiaries is S.A. Dallas, Texas, U.S.A. The goods were insured court judgments for damages arising from injury to
a condition precedent of the demandability of the with respondent EASCO and shipped on board the persons and loss of property which does not involve
obligation of the insurer to pay the proceeds, M/V Peskov, a vessel owned by Far Eastern Shipping a loan. Clearly, the applicable law is Article 2209 of
appellants claim that they should be paid P10,000 in Company. When the goods reached Manila, they the Civil Code.
Philippine currency and not under the Ballantyne were found to have been damaged by sea water - And in the light of the fact that the contending
scale of values. which rendered the fishmeal useless. Petitioner filed parties did not allege the rate of interest stipulated in
a claim with EASCO and Far Eastern Shipping. Both the insurance contract, the legal interest was
ISSUE refused to pay. Whereupon, petitioner sued them properly pegged by the Appellate Court at 6% per
WON the policy matured upon the death of the before the then Court of First Instance of Cebu for cent.
insured damages. EASCO, as the insurer, filed a
counterclaim against the petitioner for the recovery CATHAY INSURANCE v. CA(LUGAY)
HELD of the unpaid insurance premiums. 174 SCRA 11
YES - The trial court rendered judgment in favor of GRINO-AQUINO; June 5, 1989
Ratio In life insurance, the policy matures either petitioner. The judgment became final as to EASCO
upon the expiration of the term set forth therein, or but the shipping company appealed to the Court of FACTS
upon his death occuring at any time prior to the Appeals and was absolved from liability by the said - Petitioners are 6 insurance companies that issued
expiration of such stipulated term, in which case, the court. fire insurance policies for the total sum of
INSURANCE Page
101

P4,000,000 to the Cebu Filipina Press owned by loss were submitted from January 15, 1982 through was proven is refuted by the trial court's explicit
Emilia Chan Lugay. The fire policies described the June 21, 1982 in compliance with the adjusters' finding that "there was a delay that was not
insured property as "stocks of Printing materials, numerous requests for various documents, payment reasonable in processing the claim and doing
papers and general merchandise usual to the should have been made within 90 days thereafter payments". Under Section 244, a prima facie
Assured's trade" stored in a one-storey building of (Sec 243), or on or before September 21, 1982. evidence of unreasonable delay in payment of the
strong materials housing the Cebu Filipina Press Hence, when the assured filed her complaint on claim is created by the failure of the insurer to pay
located at UNNO Pres. Quirino cor. Don V. Sotto December 15, 1982, her cause of action had already the claim within the time fixed in both Sec. 242 and
Sts., Mabolo, Cebu City. The co-insurers were accrued. 243 of the IC.
indicated in each of the policies. All, except one 2. YES - In view of the not insubstantial value of the private
policy (Paramount's), were renewals of earlier - There is no merit in the petitioners' contention that respondent's claims and the considerable time and
policies issued for the same property. the proofs of loss were insufficient because Lugay effort expended by them and their counsel in
- On December 18, 1981, the Cebu Filipina Press was failed to comply with the adjuster's request for the prosecuting these claims for the past 8 years,
razed by electrical fire together with all the stocks submission of her bank statements. Condition No. 13 attorney's fees were properly awarded to the private
and merchandise stored in the premises. On of the policy does not require the insured to produce respondents.
January 15, 1982, Lugay submitted sworn her bank statements. Therefore, the insured was not
Statements of Loss and Formal Claims to the obligated to produce them and the insurers had no 5. YES
insurers, through their adjusters. She claimed a total right to ask for them. Condition No. 13 was prepared - An award equivalent to 10% of the proceeds of the
loss of P4,595,000. by the insurers themselves, hence, it should be policies would be more reasonable than the 20%
- After nearly 10 months of waiting, she sued to taken most strongly against them. awarded by the trial court and the CA.
collect on December 15, 1982. The insurance 3. NO Disposition Decision of the CA AFFIRMED with
companies denied liability, alleging violation of - Both the trial court and the CA noted that the MODIFICATION.
certain conditions of the policy, misdeclaration, and proofs were ample and more than enough for
even arson which was not seriously pressed for, defendant insurers to do a just assessment NODA v. CRUZ
come the pre-trial, the petitioners offered to pay supporting the 1981 fire claim for an amount 151 SCRA 227
50% of her claim, but she insisted on full recovery. exceeding four million pesos. FERNAN; June 22, 1987
- Trial court rendered judgment in her favor ordering 4. NO
the insurers to pay her a total of P4,000,000 as - The award of double interest on the claim is lawful NATURE
indemnity, P48,000 representing expenses of the and justified under Sections 243 and 244 of the Petition to review decision of the Insurance
plaintiff, a separate amount of 20% of the Insurance Code which provide: Commissioner
P4,000,000 representing fees of counsel, interests at Sec. 243 Refusal or failure to pay the loss or
the rate of twice the ceiling being prescribed by the damage within the time prescribed herein will FACTS
Monetary Board starting from the time when the entitle the assured to collect interest on the - In 1977, Noda obtained from Zenith Insurance
case was filed, and finally, with costs. CA affirmed. proceeds of the policy for the duration of the delay Corporation 2 fire insurance policies: [1] No. F-
at the rate of twice the ceiling prescribed by the 03724 with a face value of P30k covering the goods
ISSUES Monetary Board. and stocks in trade in his business establishment at
1. WON the insured's cause of action had already Sec. 244 In case of any litigation for the the market site in Mangagoy, Bislig, Surigao del Sur
accrued before she filed her complaint enforcement of any policy or contract of insurance, and [2] No. F-03734 with a face value in the
2. WON sufficient proofs of loss had been presented it shall be the duty of the Commissioner or the aggregate amount of P100k and consisting of Item 1
by the insured Court, as the case may be, to make a finding as to for P40k on household furniture, fixtures, fittings and
3. WON the private respondents claim for loss was whether the payment of the claim of the insured other personal effects, and Item 2 for P60k on stocks
inflated has been unreasonably denied or withheld; and in in trade usual to petitioner's retail business situated
4. WON lower court erred in awarding damages to the affirmative case, the insurance company shall in a two-storey building at 039 Barreda St.,
the private respondent in the form of interest be adjudged to pay damages which shall consist of Mangagoy, Bislig, Surigao del Sur.
equivalent to double the interest ceiling set by the attorney's fees and other expenses incurred by the - While both policies were in force, fire destroyed
Monetary Board insured person by reason of such unreasonable petitioner's insured properties at the market site on
5. WON attorney's fees awarded were exorbitant denial or withholding of payment plus interest of September 5, 1977 and at Barreda St. on November
twice the ceiling prescribed by the Monetary Board 9, 1977.
HELD of the amount of claim due the insured. - When petitioner failed to obtain indemnity on his
1. YES - The petitioners' contention that the charging of claims from Zenith, he filed a complaint with the
- As the fire which destroyed the Cebu Filipina Press double interest was improper because no Insurance Commission praying that Zenith be
occurred on December 19, 1981 and the proofs of unreasonable delay in the processing of the fire claim ordered to pay him P130kj representing the value of
INSURANCE Page
102

the 2 policies insured by respondent with interest at documentary exhibits. The foregoing evidence for Disposition Zenith Insurance Corporation ordered
12% per annum, plus damages, attorney's fees and petitioner preponderantly showed the presence of to pay petitioner Norman R. Noda the sum of
other expenses of litigation. ... some P590k worth of goods in his retail store during P60,592.10 with legal interest from the filing of the
- Zenith interposed that petitioner had no cause of the fire of November 9, 1977. complaint until full payment, but deducting
action; that Policy No. F-03724 was not in full force - While the insurer, and the Insurance Commissioner therefrom the amount of P15,472.50 which it had
and effect at the time of the fire because the for that matter, have the right to reject proofs of loss earlier paid to petitioner.
premium on the policy was not paid; that Zenith's if they are unsatisfactory, they may not set up for
liability under Policy No. F-03734, if any, was limited themselves an arbitrary standard of satisfaction. DELSAN TRANSPORT, INC. v. CA (AMERICAN
to P15,472.50 in view of the co-insurance; and that Substantial compliance with the requirements will HOME ASSURANCE)
petitioner failed to substantiate his claim as to the always be deemed sufficient. 369 SCRA 24
value of the goods reputedly destroyed by fire. - Zenith introduced in evidence the final report on DE LEON, JR; November 15, 2001
- While the case was pending, Zenith settled Policy No. F-03734 submitted by its own adjuster,
petitioner’s fire loss claim under Item 1 of Policy No. Dela Merced Adjustment Corporation. Respondent NATURE
03734 in the amount of P15,472.50. Commissioner however ignored such report, A petition for review on certiorari of the decision of
- Insurance Commissioner allowed petitioner to reasoning that with regard to Item 2 of Policy No. F- CA.
recover under said policy and ordered Zenith to pay 03734 the claim for loss of the stocks in trade was
him the amount of P20k with legal interest from the not successfully proven in view of petitioner's failure FACTS
date the complaint was filed, including P1k as to present evidence; that the adjuster's report - Caltex entered into a contract of affreightment with
attorney's fees but excluding the actual, moral and deserved scant consideration since the allegations the petitioner, Delsan Transport Lines, Inc.
exemplary damages prayed for. As for petitioner's therein were not substantiated, and that said report (petitioner), for a period of one year whereby the
claim under Policy No. F-03734, she held that in view did not even make a recommendation for payment. said common carrier agreed to transport Caltex’s
of the payment of P15,472.50 to petitioner, Zenith - A scrutiny of the abovementioned adjuster's report industrial fuel oil from the Batangas-Bataan Refinery
had fully discharged its liability under said policy reveals that together with the formal demand for full to different parts of the country. Delsan took on
which covered furniture, fixtures, fittings and other indemnity, petitioner submitted his income tax board its vessel, MT Maysun, 2,277.314 kiloliters of
personal belongings of petitioner. return for 1978, purchase invoices, certification from industrial fuel oil of Caltex to be delivered to the
- In allowing recovery under Policy No. F-03734, his suppliers as to his purchases, and other Caltex Oil Terminal in Zamboanga City. The
Commissioner placed much weight on the final report supporting papers. The report even took into account shipment was insured by American Home Assurance
prepared by Dela Merced Adjustment Corporation, an the appraisals of the other adjusters and concluded Corporation (respondent).
independent fire, marine and casualty adjuster that the total loss sustained by petitioner in his - August 14, 1986: MT Maysun set sail from
contracted by Zenith to investigate the claims of its household effects and stocks in trade reached Batangas for Zamboanga City. The vessel sank in
various policyholders. Said report concluded that P379,302.12. But after apportioning said amount the early morning of August 16, 1986 near Panay
"the sound value of P26,666.67 represented the among petitioner's six different insurers [the co- Gulf in the Visayas taking with it the entire cargo of
whole loss and damage" incurred by petitioner, but insurance being known to Zenith], the liability of fuel oil.
with the application of the three-fourths loss clause, Zenith was placed at P60,592.10. It therefore - Respondent paid Caltex P5,096,635.57
Zenith's liability was reduced to P20k. recommended that Zenith pay the petitioner the representing the insured value of the lost cargo.
amount of P60, 592.10. Exercising its right of subrogation under Article 2207
ISSUES - Said document was offered as evidence by Zenith of the New Civil Code, the private respondent
1. WON Insurance Commissioner erred in denying itself and could very well be considered as an demanded of the petitioner the same amount it paid
petitioner's demand for P60k under Item 2 of Policy admission of its liability up to the amount to Caltex. Delsan refused to pay, forcing American
No. F-03734 recommended. Being in the nature of an admission home to file a case for collection in the RTC.
2. WON Insurance Commissioner erred in not against interest, it is the best evidence which affords - RTC found that the vessel, MT Maysun, was
awarding in favor of petitioner exemplary damages the greatest certainty of the facts in dispute. seaworthy to undertake the voyage, and that the
for Zenith's unjustified and wanton refusal to pay Respondent Commissioner should not have incident was caused by an unexpected inclement
petitioner's claim under the said two insurance perfunctorily dismissed that particular evidence as a weather condition or force majeure, thus exempting
contracts worthless piece of paper. the common carrier from liability for the loss of its
2. NO cargo.
HELD - There is no showing that Zenith, in contesting - CA reversed RTC decision on the basis of evidence
1. YES payment, had acted in a wanton, oppressive or from PAG-ASA that there were no 20 ft. waves in the
- To prove the existence of the stocks in trade malevolent manner to warrant the imposition of area. CA ruled that the petitioner is liable on its
covered by Policy No. F-03734, petitioner offered his corrective damages. obligation as common carrier to respondent
testimony and that of his wife as well as insurance company as subrogee of Caltex.
INSURANCE Page
103

Petitioner’s Claim debt by one who in justice and good conscience the relationship of respondent as insurer and Caltex,
> In every marine insurance upon a ship or freight, ought to pay. It is not dependent upon, nor does it as the assured shipper of the lost cargo of industrial
or freightage, or upon any thing which is the subject grow out of, any privity of contract or upon written fuel oil, but also the amount paid to settle the
of marine insurance there is an implied warranty by assignment of claim. It accrues simply upon insurance claim. The right of subrogation accrues
the shipper that the ship is seaworthy.10 When payment by the insurance company of the insurance simply upon payment by the insurance company of
private respondent paid Caltex the value of its lost claim. the insurance claim.
cargo, the act of the private respondent is equivalent 2. NO Disposition Petition is denied, and the decision of
to a tacit recognition that the ill-fated vessel was Ratio Seaworthiness relates to a vessel’s actual the CA is affirmed.
seaworthy. condition. Neither the granting of classification or
the issuance of certificates establishes
Respondent’s Comment seaworthiness.
> American Home Assurance is entitled to payment Reasoning
by its right of subrogation. - Common carriers are bound to observe FINMAN GENERAL ASSURANCE CORP v.
extraordinary diligence in the vigilance over the INOCENCIO
ISSUES goods and for the safety of passengers transported 179 SCRA 480
1. WON payment made by American Home to Caltex by them, according to all the circumstances of each FELICIANO; November 15, 1989
for the insured value of the lost cargo amounted to case. There is no liability if the loss, destruction or
an admission that the vessel was seaworthy, thus deterioration is by force majeure. FACTS
precluding any action for recovery against the - The tale of strong winds and big waves by the said - Pan Pacific is a recruitment and employment
petitioner officers of the petitioner however, was effectively agency. It posted surety bond issued by Finman
2. WON MT Maysun was seaworthy at the time of the rebutted and belied by the weather report from PAG- General Assurance and was granted license to
voyage (outline topic) ASA. MT Maysun sank with its entire cargo for the operate by POEA.
3. WON non-presentation of the marine insurance reason that it was not seaworthy. There was no - Inocencio, Palero, Cardones, Hernandez filed with
policy bars the complaint for recovery of sum of squall or bad weather or extremely poor sea POEA complaints against Pan Pacific for violation of
money for lack of cause of action condition in the vicinity when the said vessel sank. Labor Code and for refund of placement fees. POEA
- Petitioner may not escape liability by presenting in Administrator motu propio impleaded Finman as
HELD evidence certificates that tend to show that at the surety for Pan Pacific.
1. NO time of dry-docking and inspection by the Philippine - Pan Pacific moved out and no notice of transfer was
Ratio The fact of payment grants American Home Coast Guard MT Maysun, was fit for voyage. These furnished to POEA as required. POEA considered
the subrogatory right which enables it to exercise pieces of evidence do not necessarily take into that constructive service of complaints had been
legal remedies that would otherwise be available to account the actual condition of the vessel at the time effected.
Caltex as owner of the lost cargo against the of the commencement of the voyage. At the time of - Finman denied liability and said that
petitioner common carrier. dry-docking and inspection, the ship may have - POEA had no jurisdiction over surety bonds;
Reasoning appeared fit. The certificates issued, however, do jurisdiction is vested in Insurance Commission
Art. 2207. (Civil Code) not negate the presumption of unseaworthiness - Finman had not violated Labor Code
If the plaintiff’s property has been insured, and he triggered by an unexplained sinking. - Complainants have no cause of action against
has received indemnity from the insurance - Authorities are clear that diligence in securing Finman
company for the injury or loss arising out of the certificates of seaworthiness does not satisfy the - Amounts claimed were paid as deposits and not as
wrong or breach of contract complained of, the vessel owner’s obligation. Also securing the approval placement fees.
insurance company shall be subrogated to the of the shipper of the cargo, or his surveyor, of the - POEA Administrator issued Order that respondents
rights of the insured against the wrongdoer or the condition of the vessel or her stowage does not should pay. Finman appealed to Secretary of Labor.
person who has violated the contract. If the establish due diligence if the vessel was in fact Secretary upheld the POEA order.
amount paid by the insurance company does not unseaworthy, for the cargo owner has no obligation
fully cover the injury or loss, the aggrieved party in relation to seaworthiness. ISSUE
shall be entitled to recover the deficiency from the 3. NO WON Finman can be held liable for complainants’
person causing the loss or injury. Ratio The presentation in evidence of the marine claims against Pan Pacific
- The right of subrogation is designed to promote insurance policy is not indispensable in this case
and to accomplish justice and is the mode which before the insurer may recover from the common HELD
equity adopts to compel the ultimate payment of a carrier the insured value of the lost cargo in the YES
exercise of its subrogatory right. The subrogation
10
Sec. 113 Insurance Code receipt, by itself, is sufficient to establish not only
INSURANCE Page
104

- Under Insurance Code, liability of surety in a Chia Yu brought the present action against both, to this amendment to the Insurance Act cannot be
surety bond is joint and several with the principal including their respective agents in the Philippines. given effect in an action in our courts.
obligor. - An action was filed at the CFI after more than 2 SEC. 61-A. (Insurance Code) ~ Any condition,
- Conditions of a bond specified and required in the years after delivery of the damaged bales and the stipulation or agreement in any policy of
provisions of a statute providing for submission of date when the missing bales should have been insurance, limiting the time for commencing an
the bond, are incorporated into all bonds tendered delivered, the action was resisted by the Atkins and action thereunder to a period of less than one year
under that statute even though not set out in Eagle Star principally on the ground of prescription. from the time when the cause of action accrues, is
printer’s ink. -TC favored Chia Yu and CA affirmed. void.
- POEA held and Secretary of Labor affirmed that *** CARRIER’s defense of prescription is made to - The prescription clause could be harmonized with
Pan Pacific had violated Labor Code, and at least one rest on the following stipulation of the bill of lading: section 61-A of the Insurance Act by taking it to
of the conditions for the grant and continued use of In any event the carrier and the ship shall be mean that the time given the insured for bringing his
the recruitment license. POEA and Secretary of discharged from all liability in respect of loss or suit is twelve months after the cause of action
Labor can require Pan Pacific to refund the damage unless suit is brought within one year accrues.
placement fees and to impose the fine. after the delivery of the goods or the date when - If so, when did the cause of action accrue? Chia
- If Pan Pacific is liable, and if Finman is solidarily the goods should have been delivered. (This Yu’s action did not accrue until his claim was finally
liable with Pan Pacific, then Finman is liable both to stipulation is but a repetition of a provision in the rejected by the insurance company. This is because,
private respondents and to POEA. CA 65 which says that bills of lading covering before such final rejection, there was no real
- Cash and surety bonds are required from shipments from the US to the Phils should be necessity for bringing suit.
recruitment companies as means of ensuring prompt brought w/in one year after the delivery of the - As the policy provides that the insured should file
and effective recourse against such companies when goods or the date when the goods should have his claim, first, with the carrier and then with the
held liable. Public policy will be effectively negated if been delivered to hold the carrier liable.) insurer, he had a right to wait for his claim to be
POEA and the DoLE were held powerless to compel a *** INSURER’s claim of prescription is founded upon finally decided before going to court.
surety company to make good on its solidary the terms of the policy and not upon the bill of - Furthermore, there is nothing in the record to show
undertaking. lading. (But in our jurisdiction, as per A1144, that the claim was rejected in the year 1947, either
prescription is 10 years after action accrues.) by the insurance company in London or its settling
EAGLE STAR INSURANCE CO LTD v. CHIA YU No suit action on this Policy, for the recovery of agents in the Philippines.
96 PHIL 696 any claim, shall be sustainable in any Court of law - For the purpose of this action, Chia Yu's claim was
REYES; March 31, 1955 or equity unless the insured shall have fully considered to have been finally rejected by the
complied with all the terms and conditions of this insurer on April 22, 1948. Having been filed within
NATURE Policy nor unless commenced with twelve (12) twelve months form that date, the action cannot be
Certiorari months next after the happening of the loss . . . deemed to have prescribed even on the supposition
that the period given the insured for bringing suit
FACTS ISSUE under the prescriptive clause of the policy is twelve
- Atkin, Kroll & Co., loaded on the S. S. Roeph WON ATKIN’ s action has prescribed months after the accrual of the cause of action.
Silverlight owned and operated by Leigh Hoegh & - Contractual limitations contained in insurance
Co., A/S, of San Francisco California, 14 bales of HELD policies are regarded with extreme jealousy by
assorted underwear valued at P8,085.23 consigned NO courts and will be strictly construed against the
to Chia Yu in the City of Manila. - Being contrary to the law of the forum, the insurer and should not be permitted to prevent a
- The shipment was insured against all risks by Eagle stipulation in the policy cannot be given effect as it recovery when their just and honest application
Star Ins. Co. of San Francisco, California, under a would reduce the period allowed the insured for would not produce that result. (46 C. J. S. 273.)
policy issued to the shipper and by the latter bringing his action to less than one year (because Disposition Judgment appealed from is REVERSED
assigned to the consignee. the prescription period begins from the “happening with respect to the carrier and its agents but
- The vessel arrived in Manila but of the 14 bales of the loss” and that before any suit could be AFFIRMED with respect to the insurance company
(a.k.a. freights =p) consigned to Chia Yu only 10 sustained the insured shall have to comply with the and its agents.
were delivered to him as the remaining 3 could not terms and conditions of the policy first TF lessening
be found.3 of those delivered were also found the period to less than a year. )
damaged to the extent of 50 per cent. - Insular Government vs. Frank(13 Phil. 236)~
-Chia Yu claimed indemnity for the missing and "matters respecting a remedy, such as the bringing
damaged bales. But the claim was declined, first, by of suit, admissibility of evidence, and statute of
the carrier and afterward by the insurer, whereupon limitations, depend upon the law of the place where ACCFA v. ALPHA INSURANCE
the suit is brought" TF any policy clause repugnant 24 SCRA 151
INSURANCE Page
105

REYES; July 29, 1968 ISSUE brought within the statutory period of limitation for
WON the provision of a fidelity bond that no action written contracts (New Civil Code, Article 1144).
FACTS shall be had or maintained thereon unless
- In order to guarantee the Asingan Farmers' commenced within one year from the making of a ANG v. FULTON FIRE INSURANCE CO.
Cooperative Marketing Association, Inc. (FACOMA) claim for the loss upon which the action is based, is 2 SCRA 945
against loss on account of "personal dishonesty, valid, in view of Section 61-A of the Insurance Act LABRADOR; July 31, 1961
amounting to larceny or estafa of its Secretary- invalidating stipulations limiting the time for
Treasurer, Ladines, the appellee, Alpha Insurance & commencing an action thereon to less than one year NATURE
Surety Company had issued, on 14 February 1958, from the time the cause of action accrues Appeal from judgment of the CFI ordering the
its bond, No. P-FID-15-58, for the sum of P5,000 defendant Fulton Fire Insurance Co. to pay the
with said Ladines as principal and the appellee as HELD plaintiffs the sum of P10,000.00, with interest, and
solidary surety. On the same date, the Asingan NO an additional sum of P2,000.00 as attorney's fees,
FACOMA assigned its rights to the appellant, - A fidelity bond is, in effect, in the nature of a and costs.
Agricultural Credit Cooperative and Financing contract of insurance against loss from misconduct,
Administration (ACCFA for short), with approval of and is governed by the same principles of FACTS
the principal and the surety. interpretation. Consequently, the condition of the - The stocks of general merchandise in the store of
- During the effectivity of the bond, Ladines bond in question, limiting the period for bringing the Ang spouses are insured with Fulton. While the
converted and misappropriated, to his personal action thereon, is subject to the provisions of Section insurance was in force, fire destroyed the goods. The
benefit, some P11,513.22 of the FACOMA funds, of 61-A of the Insurance Act (No. 2427), as amended Angs filed their first claim immediately after the fire.
which P6,307.33 belonged to the ACCFA. Upon by Act 4101 of the pre-Commonwealth Philippine - Their claim was denied on April 6, 1956. They
discovery of the loss, ACCFA immediately notified in Legislature, prescribing that: received notice on April 19, 1956.
writing the survey company on 10 October 1958, SEC. 61-A: A condition, stipulation or agreement - The Angs brought an action against the agent on
and presented the proof of loss within the period in any policy of insurance, limiting the time for May 11, 1956. The court denied the suit and the mfr
fixed in the bond; but despite repeated demands the commencing an action thereunder to a period of on Sept. 3 and 12, 1957.
surety company refused and failed to pay. less than one year from the time when the cause - The Angs filed against Fulton on May 26, 1958.
Whereupon, ACCFA filed suit against appellee on 30 of action accrues is void. - There was a clause in the policy:
May 1960. - Since a "cause of action" requires, as essential 13.If the claim be in any respect fraudulent, or if
- Defendant Alpha Insurance & Surety Co., Inc., elements, not only a legal right of the plaintiff and a any false declaration is made or used in support
(now appellee) moved to dismiss the complaint for correlative obligation of the defendant but also "an thereof, or if any fraudulent means or devices are
failure to state a cause of action, giving as reason act or omission of the defendant in violation of said used by the Insured or any one acting on his
that (1) the same was filed more than one year after legal right," the cause of action does not accrue until behalf to obtain any benefit under this Policy, or, if
plaintiff made claim for loss, contrary to the eighth the party obligated refuses, expressly or impliedly, to the loss or damage be occasioned by the wilful act
condition of the bond, providing as follows: comply with its duty (in this case, to pay the amount or with the connivance of the Insured, or, if the
EIGHT LIMITATION OF ACTION: No action, suit or of the bond). The year for instituting action in court claim be made and rejected and an action or suit
proceeding shall be had or maintained upon this must be reckoned, therefore, from the time of be not commenced within twelve months after
Bond unless the same be commenced within one appellee's refusal to comply with its bond; it can not such rejection or (in case of arbitration taking
year from the time of making claim for the loss upon be counted from the creditor's filing of the claim of place in pursuance of the 18th condition of this
which such action, suit or proceeding, is based, in loss, for that does not import that the surety Policy) within twelve months after the arbitrator or
accordance with the fourth section hereof. company will refuse to pay. In so far, therefore, as arbitrators or umpire shall have made their award
(2) the complaint failed to show that plaintiff had condition eight of the bond requires action to be filed all benefit under this Policy shall be forfeited."
filed civil or criminal action against Ladines, as within one year from the filing of the claim for loss,
required by conditions 4 and 11 of the bond; and (3) such stipulation contradicts the public policy ISSUE
that Ladines was a necessary and indispensable expressed in Section 61-A of the Philippine Insurance WON the suit against the agent tolled the
party but had not been joined as such. Act. prescription period, such that the filing against
- At first, the Court of First Instance denied - Condition eight of the bond, therefore, is null and Fulton was only 9 months after the claim was
dismissal; but, upon reconsideration, the court void, and the appellant is not bound to comply with rejected
reversed its original stand, and dismissed the its provisions. The discouraging of unnecessary
complaint on the ground that the action was filed litigation must be deemed a rule of public policy, HELD
beyond the contractual limitation period. Hence, this considering the unrelieved congestion in the courts. NO
appeal. As a consequence of the foregoing, action may be - The bringing of the action against the Paramount
Surety & Insurance Company, the agent of the
INSURANCE Page
106

defendant company, cannot have any legal effect thru payment to third persons, said third persons' Corporation. For the damage caused, ZENITH paid
except that of notifying the agent of the claim. recourse being thus limited to the insured alone. But MLA MAHOGANY P5,000 in amicable settlement. MLA
Beyond such notification, the filing of the action can in the case at bar, there was no contract shown. MAHOGANY's general manager executed a Release of
serve no other purpose. There is no law giving any What then was the basis of the RTC and the CA Claim, subrogating respondent company to all its
effect to such action upon the principal. Besides, to say that the insurance contract was a third- right to action against San Miguel Corporation.
there is no condition in the policy that the action party liability insurance policy? Consequently, - On 11 Dec 1972, ZENITH wrote Insurance
must be filed against the agent, and the Court can the trial court was confused as it did not distinguish Adjusters, Inc. to demand reimbursement from San
not by interpretation extend the clear scope of the between the private respondent's cause of action Miguel. Insurance Adjusters, Inc. refused
agreement beyond what is agreed upon by the against the owner and the driver of the Lady Love reimbursement, alleging that San Miguel had already
parties. taxicab and his cause of action against petitioner. paid petitioner P4,500, as evidenced by a cash
- Their contract is the law between the parties, and The former is based on torts and quasi-delicts while voucher and a Release of Claim executed by the
their agreement that an action on a claim denied by the latter is based on contract. General Manager of petitioner.
the insurer must be brought within one year from - Even assuming arguendo that there was such a - ZENITH thus demanded from petitioner
the denial, governs, not the rules on the prescription contract, private respondent's cause of action can reimbursement of the sum of P4,500 paid by San
of actions. not prevail because he failed to file the written Miguel.
Disposition The judgment appealed from is hereby claim mandated by the Insurance Code (before - City Court ordered petitioner to pay respondent
set aside and the case dismissed, with costs against it was amended-action must be brought within P4,500.
plaintiffs-appellees. six months from date of the accident (this is - CFI affirmed the City Court's decision in toto.
what’s applicable here) ; after amendment- - CA affirned CFI, with the modification that
TRAVELLERS INSURANCE & SURETY CORP. v. "action or suit for recovery of damage due to loss or petitioner was to pay the total amount of P5,000 it
CA (MENDOZA) injury must be brought in proper cases, with the had earlier received from ZENITH.
272 SCRA 536 Commissioner or the Courts within one year from Petitioner’s Claims
HERMOSISIMA, JR; May 22, 1997 denial of the claim, otherwise the claimant's right of > It is not bound to pay P4,500, and much more,
action shall prescribe" ). He is deemed, under this P5,000 to ZENITH as the subrogation in the Release
NATURE legal provision, to have waived his rights as against of Claim it executed in favor of respondent was
The petition herein seeks the review and reversal of petitioner-insurer. conditioned on recovery of the total amount of
the decision of respondent Court of Appeals affirming Disposition petition granted damages petitioner had sustained. Since total
in toto the judgment of the Regional Trial Court in an damages were valued by petitioner at P9,486.43 and
action for damages filed by private respondent SUN INSURANCE v. CA (supra p.57) only P5,000 was received by petitioner, MLA
Vicente Mendoza, Jr. as heir of his mother who was MAHOGANY argues that it was entitled to go after
killed in a vehicular accident. COASTWISE v. CA (supra p.70) San Miguel to claim the additional P4,500.
> It cites Art. 220711 and Art. 130412 of the Civil
FACTS CEBU SHIPYARD v. WILLIAM LINES (supra p.3) Code, and claims a preferred right to retain the
-an old lady was hit by a taxicab. The taxicab was amount coming from San Miguel, despite the
later identified and a case was filed against the MANILA MAHOGANY MANUFACTURING CORP v. subrogation in favor of ZENITH.
driver and owner. Later, an amendment was filed to CA (ZENITH INSURANCE CORP) Respondent’s Arguments
include the insurance company. RTC and CA ordered 154 SCRA 652 > There was no qualification to its right of
that the owner, driver as well as the insurance PADILLA; October 12, 1987 subrogation under the Release of Claim executed by
company be held solidarily liable. petitioner, the contents having expressed all intents
NATURE and purposes of the parties.
ISSUE Petition to review CA decision ordering Manila
WON RTC and CA erred Mahogany Manufacturing Corporation to pay Zenith ISSUE
HELD Insurance Corporation P5,000 with 6% annual WON the insurer may recover the sum of P5,000
YES interest, attorney's fees, and costs of suit
- Where the contract provides for indemnity against 11
Article 2207: “If the plaintiff's property has been insured, and he has received indemnity from
liability to third persons, then third persons to whom FACTS the insurance company for the injury or loss arising out of the wrong or breach of contract
complained of the insurance company shall be subrogated to the rights of the insured against the
the insured is liable can sue the insurer. Where the - From 6 March 1970 to 6 March 1971, MLA wrongdoer or the person who has violated the contract. If the amount paid by the insurance
contract is for indemnity against actual loss or MAHOGANY insured its Mercedes Benz 4-door sedan company does not fully cover the injury or loss the aggrieved party shall be entitled to recover the
payment, then third persons cannot proceed against with ZENITH. deficiency from the person causing the loss or injury.”
12
the insurer, the contract being solely to reimburse - On 4 May 1970, the insured vehicle was bumped Article 1305: “A creditor, to whom partial payment has been made, may exercise his right for
the remainder, and he shall be preferred to the person who has been subrogated in his place in
the insured for liability actually discharged by him and damaged by a truck owned by San Miguel virtue of the partial payment of the same credit.”
INSURANCE Page
107

Petitions for review on certiorari of a decision of the to litigation and for recovery of the sums of money
HELD CA they advanced to Lim for the purchase of the
YES aircrafts in question. (this constitutes the second
Ratio Since the insurer can be subrogated to only FACTS petition but will no longer be discussed because it is
such rights as the insured may have, should the - In 1965, Jacob S. Lim was engaged in the airline not relevant to the topic)
insured, after receiving payment from the insurer, business as owner-operator of Southern Air Lines - After trial on the merits, a decision was rendered
release the wrongdoer who caused the loss, the (SAL), a single proprietorship. holding Lim liable to pay Pioneer but dismissed
insurer loses his rights against the latter. But in such -On May 17, 1965, at Tokyo, Japan, Japan Domestic Pioneer's complaint against all other
a case, the insurer will be entitled to recover from Airlines (JDA) and Lim entered into and executed a defendants.
the insured whatever it has paid to the latter, unless sales contract for the sale and purchase of two (2) - CA modified the trial court's decision in that the
the release was made with the consent of the DC-3A Type aircrafts and one (1) set of necessary plaintiff’s complaint against all the defendants
insurer. spare parts for the total agreed price of US (including Lim) was dismissed.
Reasoning $109,000.00 to be paid in installments.
- Although petitioner’s right to file a deficiency claim - On May 22, 1965, Pioneer Insurance and Surety ISSUE
against San Miguel is with legal basis, without Corporation, as surety, executed and issued its WON the petition of Pioneer Insurance and Surety
prejudice to the insurer's right of subrogation, Surety Bond No. 6639 in favor of JDA, in behalf of its Corporation against all defendants was rightly
nevertheless, when Manila Mahogany executed principal, Lim, for the balance price of the aircrafts dismissed
another release claim discharging San Miguel from and spare parts.
"all actions, claims, demands and rights of action -Border Machinery and Heavy Equipment Company, HELD
that now exist or hereafter arising out of or as a Inc. (Bormaheco), Francisco and Modesto Cervantes YES
consequence of the accident" after the insurer had (Cervanteses) and Constancio Maglana contributed - Both the TC and CA made the finding that Pioneer
paid the proceeds of the policy - the compromise some funds used in the purchase of the above reinsured its risk of liability under the surety bond it
agreement of P5,000 being based on the insurance aircrafts and spare parts. They executed two (2) had executed in favor of JDA, collected the proceeds
policy - the insurer is entitled to recover from the separate indemnity agreements in favor of Pioneer, of such reinsurance in the sum of P295,000, and
insured the amount of insurance money paid. Since one signed by Maglana and the other jointly signed paid with the said amount the bulk of its alleged
petitioner by its own acts released San Miguel, by Lim for SAL, Bormaheco and the Cervanteses. liability to JDA under the said surety bond. The total
thereby defeating private respondent’s right of - On June 10, 1965, Lim doing business under the amount paid by Pioneer to JDA is P299,666.29. Since
subrogation, the right of action of petitioner against name and style of SAL executed in favor of Pioneer Pioneer has collected P295,000.00 from the
the insurer was also nullified. as deed of chattel mortgage as security for the reinsurers, the uninsured portion of what it paid to
- As held in Phil. Air Lines v. Heald Lumber Co., latter's suretyship in favor of the former. It was JDA is the difference between the two amounts, or
under Art. 2207, the real party in interest with stipulated therein that Lim transfer and convey to P3,666.28. This is the amount for which Pioneer may
regard to the portion of the indemnity paid is the the surety the two aircrafts. sue defendants, assuming that the indemnity
insurer and not the insured. - Lim defaulted on his subsequent installment agreement is still valid and effective. But since the
SUBROGATION: The right of subrogation can only payments prompting JDA to request payments from amount realized from the sale of the mortgaged
exist after the insurer has paid the insured, the surety. chattels are P35,000.00 for one of the airplanes and
otherwise the insured will be deprived of his right to - Pioneer paid a total sum of P298,626.12. P2,050.00 for a spare engine, or a total of
full indemnity. If the insurance proceeds are not - Pioneer then filed a petition for the extrajudicial P37,050.00, Pioneer is still overpaid by P33,383.72.
sufficient to cover the damages suffered by the foreclosure of the said chattel mortgage before the Therefore, Pioneer has no more claim against
insured, then he may sue the party responsible for Sheriff of Davao City. defendants.
the damage for the remainder. To the extent of the - The Cervanteses and Maglana, however, filed a - The payment to the petitioner made by the
amount he has already received from the insurer third party claim alleging that they are co-owners of reinsurers was not disputed. Considering this
enjoys the right of subrogation. the aircrafts, admitted payment, the only question was the effect
Disposition Petition DENIED. Judgment appealed - On July 19, 1966, Pioneer filed an action for judicial of payment made by the reinsurers to the petitioner
from is AFFIRMED with costs against petitioner. foreclosure with an application for a writ of - In general a reinsurer, on payment of a loss
preliminary attachment against Lim and acquires the same rights by subrogation as are
PIONEER INSURANCE v. CA ( BORDER respondents, the Cervanteses, Bormaheco and acquired in similar cases where the original insurer
MACHINERY & HEAVY EQUIPMENT INC) Maglana. pays a loss (Universal Ins. Co. v. Old Time Molasses
175 SCRA 668 **Maglana, Bormaheco and the Cervanteses filed Co.).
GUTIERREZ, JR.; July 28, 1989 cross-claims against Lim alleging that they were not - The rules of practice in actions on original
privies to the contracts signed by Lim and, by way of insurance policies are in general applicable to actions
NATURE counterclaim, sought for damages for being exposed
INSURANCE Page
108

or contracts of reinsurance (Delaware, Ins. Co. v. Corporation (Canlubang) under its motor vehicle which the latter may have against the third party
Pennsylvania Fire Ins. Co.). insurance policy. Among the provisions of the policy whose negligence or wrongful act caused the loss.
- Hence the applicable law is Article 2207 of the new was a “own-damage” clause whereby Panmalay The right of subrogation is not dependent upon, nor
Civil Code, to wit: Art. 2207. If the plaintiff’s agrees to indemnify Canlubang in cases of damage does it grow out of, any privity of contract or upon
property has been insured, and he has received caused by “accidental collision or overturning, or written assignment of claim. It accrues simply upon
indemnity from the insurance company for the injury collision or overturning consequent upon mechanical payment of the insurance claim by the insurer
or loss arising out of the wrong or breach of contract breakdown or consequent upon wear and tear”. Exceptions
complained of, the insurance company shall be - On 1985, the insured car was sideswept and (1) if the assured by his own act releases the
subrogated to the rights of the insured against the damaged by a car owned by Erlinda Fabie, driven by wrongdoer or third party liable for the loss or
wrongdoer or the person who has violated the an unknown driver who fled the scene. Panmalay, in damage, from liability, the insurer's right of
contract. If the amount paid by the insurance accordance with the policy, defrayed the cost of subrogation is defeated;
company does not fully cover the injury or loss, the repair of the insured car and was subrogated to the (2) where the insurer pays the assured the value of
aggrieved party shall be entitled to recover the rights of Canlubang against the driver and owner of the lost goods without notifying the carrier who has
deficiency from the person causing the loss or injury the pick-up. Panmalay then filed a complaint for in good faith settled the assured's claim for loss, the
- If a property is insured and the owner receives the damages with RTC Makati against Erlinda Fabie and settlement is binding on both the assured and the
indemnity from the insurer, it is provided in said her driver on the grounds of subrogation, with the insurer, and the latter cannot bring an action against
article that the insurer is deemed subrogated to the latter failing and refusing to pay their claim. Fabie the carrier on his right of subrogation;
rights of the insured against the wrongdoer and if filed a Motion for Bill of Particulars. (3) where the insurer pays the assured for a loss
the amount paid by the insurer does not fully cover - RTC: dismissed complaint for lack of cause of which is not a risk covered by the policy, thereby
the loss, then the aggrieved party is the one entitled action (payment by PANMALAY of CANLUBANG's effecting "voluntary payment", the former has no
to recover the deficiency. Evidently, under this legal claim under the "own damage" clause of the right of subrogation against the third party liable for
provision, the real party in interest with regard to insurance policy was an admission by the insurer the loss
the portion of the indemnity paid is the insurer and that the damage was caused by the assured and/or Reasoning
not the insured (. Air Lines, Inc. v. Heald Lumber its representatives) – Panmalay appealed - Both TC and CA are incorrect.
Co., and Manila Mahogany Manufacturing - CA: dismissed appeal, affirmed RTC (applying the ON TC: “Own damage” (not found in the insurance
Corporation v. Court of Appeals) ejusdem generis rule held that Section III-1 of the policy) simply meant that Panmalay had assumed to
- It is clear from the records that Pioneer sued in its policy, which was the basis for settlement of reimburse the cost for repairing the damage to the
own name and not as an attorney-in-fact of the CANLUBANG's claim, did not cover damage arising insured vehicle. It’s different from “Third Party
reinsurer. Accordingly, the appellate court did not from collision or overturning due to the negligence of Liability” coverage (liabilities arising from the death
commit a reversible error in dismissing the third parties as one of the insurable risk) of or bodily injuries suffered by 3 rd parties) and from
petitioner's complaint as against the respondents for “Property Damage” coverage (liabilities from damage
the reason that the petitioner was not the real party ISSUE caused by insured vehicle to properties of 3rd parties)
in interest in the complaint and, therefore, has no WON the insurer PANMALAY may institute an action ON CA: the terms of a contract are to be construed
cause of action against the respondents. to recover the amount it had paid its assured in according to the sense and meaning of the terms
Disposition Petitions dismissed. Questioned decision settlement of an insurance claim against private which the parties thereto have used. In the case of
of CA affirmed. respondents as the parties allegedly responsible for property insurance policies, the evident intention of
the damage caused to the insured vehicle, in the contracting parties, i.e., the insurer and the
accordance with A2207, NCC assured, determine the import of the various terms
PAN MALAYAN INSURANCE CORPORATION v. and provisions embodied in the policy. It is only
CA (FABIE, HER UNKNOWN DRIVER) HELD when the terms of the policy are ambiguous,
184 SCRA 54 YES equivocal or uncertain, such that the parties
CORTES, April 3, 1990 Ratio Article 2207 of the Civil Code is founded on themselves disagree about the meaning of particular
the well-settled principle of subrogation. If the provisions, that the courts will intervene. In such an
NATURE insured property is destroyed or damaged through event, the policy will be construed by the courts
PETITION to review the decision of the Court of the fault or negligence of a party other than the liberally in favor of the assured and strictly against
Appeals assured, then the insurer, upon payment to the the insurer
assured, will be subrogated to the rights of the - Both Panmalay and Canlubang had the same
FACTS assured to recover from the wrongdoer to the extent interpretation regarding the coverage of insured risk
- Pan Malayan Insurance Company (Panmalay) that the insurer has been obligated to pay. Payment regarding “accidental collision or overturning…” to
insured the Mitsubishi Colt Lancer car registered in by the insurer to the assured operates as an include damages caused by 3rd party to Canlubang so
the name of Canlubang Automotive Resources equitable assignment to the former of all remedies it was improper for CA to ascribe meaning contrary
INSURANCE Page
109

to the clear intention and understanding of the failed to pay the amount of the loss in spite of out of the wrong or breach of contract complained
parties. repeated demands. of, the insurance company shall be subrogated to the
- Court on several occasions defined “accident” or - Upon defendant's motions, the lower court rights of the insured against the wrongdoer or the
“accidental” as taking place “without one’s foresight dismissed the complaint as to Jamila on the ground person who has violated the contract".
or expectation, an event that proceeds from an that there was no allegation that it had consented to - The lower court denied plaintiff's motion. They
unknown cause, or is an unusual effect of a known the subrogation and, therefore, Fireman's Fund had filed a second MR, calling the lower court's attention
cause and, therefore, not expected” [Dela Cruz v. no cause of action against it. It also dismissed the to the fact that the issue of subrogation was of no
Capital Insurance & Surety Co.] The concept complaint as to First Quezon City on the ground of moment because Firestone, the subrogor, is a party-
"accident" is not necessarily synonymous with the res judicata. It appears that the same action was plaintiff and could sue directly Jamila in its own right.
concept of "no fault". It may be utilized simply to previously filed in a civil case which was dismissed Without resolving that contention, the lower court
distinguish intentional or malicious acts from because of the failure of the same plaintiffs and their denied plaintiffs' second MR.
negligent or careless acts of man. counsel to appear at the pre-trial.
- damage/loss to insured vehicle due to negligence - Upon an MR, the lower court set aside its order of ISSUE
of 3rd parties not listed as exceptions to coverage in dismissal and sustained plaintiff's contention that WON the complaint of Firestone and Fireman's Fund
the insurance policy there was no res judicata as to First Quezon City states a cause of action against Jamila
- Interpretation given by Panmalay is more in because the civil case was dismissed without
keeping with rationale behind rules on interpretation prejudice. However, due to inadvertence, the lower HELD
of insurance contracts in favor of assured or court did not state in its order why it set aside its YES
beneficiary: indemnity or payment prior order dismissing the complaint with respect to - Fireman's Fund's action against Jamila is squarely
- EVEN if voluntarily indemnified Canlubang, as Jamila. Jamilla had originally moved for the dismissal sanctioned by article 2207. As the insurer, Fireman's
interpreted by TC: the insurer who may have no of the complaint on the ground of lack of cause of Fund is entitled to go after the person or entity that
rights of subrogation due to "voluntary" payment action. Its basis for its contention were: (1) that the violated its contractual commitment to answer for
may never. theless recover from the third party complaint did not allege that Firestone, pursuant to the loss insured against.
responsible for the damage to the insured property the contractual stipulation quoted in the complaint, - The trial court erred in applying to this case the
under Article 1236 of the Civil Code. [Sveriges had investigated the loss and that Jamila was rules on novation. The plaintiffs in alleging in their
Angfartygs Assurans Forening v. Qua Chee Gan] represented in the investigation and (2) that Jamila complaint that Fireman's Fund "became a party in
Disposition the present petition is GRANTED. did not consent to the subrogation of Fireman's Fund interest in this case by virtue of a subrogation right
Petitioner's complaint for damages against private to Firestone's right to get reimbursement from given in its favor by" Firestone, were not relying on
respondents is hereby REINSTATED. Let the case be Jamila and its surety. The lower court in its order of the novation by change of creditors as contemplated
remanded to the lower court for trial on the merits. dismissal had sustained the second ground. in articles 1291 and 1300 to 1303 of the Civil Code
- Jamila in its MR invoked the first ground which but rather on article 2207.
FIREMAN'S FUND INSURANCE COMPANY v. had never been passed upon by the lower court. But - Article 2207 is a restatement of a settled principle
JAMILA & COMPANY, INC. the lower court in its order granting Jamila's motion of American jurisprudence. Subrogation has been
70 SCRA 323 for reconsideration, completely ignored that first referred to as the doctrine of substitution. It is an
AQUINO; April 1976 ground. It reverted to the second ground which was arm of equity that may guide or even force one to
relied upon in its order previous order. The lower pay a debt for which an obligation was incurred but
FACTS court reiterated its order, stating that Fireman's which was in whole or in part paid by another.
- Jamila & Co., Inc. or the Veterans Philippine Fund had no cause of action against Jamila because - Subrogation is founded on principles of justice
Scouts Security Agency contracted to supply security Jamila did not consent to the subrogation. The court and equity, and its operation is governed by
guards to Firestone. Jamila assumed responsibility did not mention Firestone, the co-plaintiff of principles of equity. It rests on the principle that
for the acts of its security guards. The First Quezon Fireman's Fund. substantial justice should be attained regardless of
City Insurance Co., Inc. executed a bond in the sum - Firestone and Fireman's Fund filed an MR on the form, that is, its basis is the doing of complete,
of P20,000 to guarantee Jamila's obligations under ground that Fireman's Fund was suing on the basis essential, and perfect justice between all the parties
that contract. of legal subrogation whereas the lower court without regard to form.
- On May 18, 1963 properties of Firestone valued at erroneously predicated its dismissal order on the - Subrogation is a normal incident of indemnity
P11,925 were lost allegedly due to the acts of its theory that there was no conventional subrogation insurance. Upon payment of the loss, the insurer is
employees who connived with Jamila's security because the debtor's consent was lacking. entitled to be subrogated pro tanto to any right of
guard. Fireman's Fund, as insurer, paid to Firestone - The plaintiffs cited article 2207 of the Civil Code action which the insured may have against the third
the amount of the loss. Fireman's Fund was which provides that "if the plaintiff's property has person whose negligence or wrongful act caused the
subrogated to Firestone's right to get reimbursement been insured, and he has received indemnity from loss. The right of subrogation is of the highest
from Jamila. Jamila and its surety, First Quezon City the insurance company for the injury or loss arising equity. The loss in the first instance is that of the
INSURANCE Page
110

insured but after reimbursement or compensation, it Front Shipping pay the damages suffered by it. The endeavor to establish that the loss, destruction or
becomes the loss of the insurer. demands were unheeded and the insurance deterioration was due to a fortuitous event; an
- Although many policies including policies in the companies were obliged to pay Republic Flour act/omission of the owner of the goods; the
standard form, now provide for subrogation, and P2,189,433 character of the goods or defects in their packing; or
thus determine the rights of the insurer in this - By virtue of the insurance companies’ payment, an order or act of a competent public authority.
respect, the equitable right of subrogation as the they were subrogated to the rights of Republic Flour. - However, Republic Flour is also found to be guilty
legal effect of payment inures to the insurer without Petitioners filed a complaint against North Front of contributory negligence for not immediately
any formal assignment or any express stipulation to Shipping, claiming the loss was exclusively staring the unloading operations and for providing no
that effect in the policy. Stated otherwise, when the attributable to the latter’s fault and negligence. explanation for the delay. As such, it should share at
insurance company pays for the loss, such payment Having surveyed the vessel, it was found that the least 40% of the loss.
operates as an equitable assignment to the insurer barge had cracks in its bodega. The hatches on the Disposition The decision of the CA is REVERSED
of the property and all remedies which the insured crates of grain were not sealed and the tarpaulins and SET ASIDE
may have for the recovery thereof. That right is not used in covering them were not new, contrary to
dependent upon, nor does it grow out of, any privity North Front Shipping’s claims. North Front Shipping PHILIPPINE AMERICAN LIFE INSURANCE
of contract, or upon written assignment of claim, and reiterated that the barge was inspected prior to COMPANY v. CA (ELIZA PULIDO)
payment to the insured makes the insurer an loading and found seaworthy and were issued a 344 SCRA 360
assignee in equity. permit to sail by the Coast Guard. They further GONZAGA-REYES; November 15, 2000
- On the other hand, Firestone is really a nominal averred that the grains were farm wet and not
party in this case. It had already been indemnified properly dried before loading. NATURE
for the loss which it had sustained. Obviously, it - The court dismissed the complaint, ruling that the This petition for review on certiorari seeks to reverse
joined as a party-plaintiff in order to help Fireman's contract entered into was a charter-party the Decision of the Special Second Division of the
Fund to recover the amount of the loss from Jamila agreement; as such, only ordinary diligence in the Court of Appeals
and First Quezon City. Firestone had tacitly assigned care of the goods was required of North Front
to Fireman's Fund its cause of action against Jamila Shipping. FACTS
for breach of contract. Sufficient ultimate facts are - On January 9, 1989, petitioner received from one
alleged in the complaint to sustain that cause of ISSUE Florence Pulido an application for life insurance,
action. WON defendant is required to observe extraordinary dated December 16, 1988, in the amount of
diligence in its vigilance over the goods it transports P100,000.00 which designated her sister, herein
TABACALERA v. NORTH FRONT SHIPPING private respondent, as its principal beneficiary.
272 SCRA 527 HELD Because the insurance applied for was non-medical,
BELLOSILLO; May 16, 1997 YES petitioner did not require a medical examination and
- As a corporation engaged in the business of issued a policy on the sole basis of the application on
FACTS transporting cargo offering its services February 11, 1989. On April 1992, petitioner
- 20,234 sacks of corn grains valued at P3.5M were indiscriminately to the public, it is without a doubt a received private respondent’s claim, which declared
shipped on board North Front 777, defendant’s common carrier. As such, it has the burden of that the insured, Florence Pulido, died of acute
vessel. The cargo was consigned to Republic Flour proving that it observed extraordinary diligence to pneumonia on September 10, 1991.
Mills Corp. under Bill of Lading No. 001 and insured avoid responsibility for the lost cargo. The clean bill - Petitioner withheld payment on the ground that the
with Tabacalera, Prudential Guarantee & Assurance, of lading it issued disprove the master of the vessel’s policy claimed under was void from the start for
and New Zealand Insurance. claim that the grains were farm wet when loaded. If having been procured in fraud. It is petitioner’s
- Republic Flour was advised of the vessel’s arrival in they were wet, the master of the vessel should have contention that even before they received private
Manila, but did not immediately commence the known that the grains would eventually deteriorate respondent’s claim for death benefits, their
unloading operations. Unloading was sometimes when sealed in hot compartments in hatches of a investigation concerning the subject policy yielded
stopped due to varying weather and sometimes for ship and should have undertaken precautionary the information that the insured, Florence Pulido,
no apparent reason. Unloading was only completed measures to avoid this. The arrival of the goods at died in 1988, before the application for insurance on
20 days after the arrival of the barge; by then, the the place of destination in bad order makes a prima her life was made. While this was communicated to
cargo was short 26.333 metric tons and the rest was facie case against the common carrier, which must private respondent in a letter, private respondent
already moldy and deteriorating. prove its non-liability. had already filed her claim earlier that month. In
- Analyses showed that the deterioration was caused - While petitioners presented evidence of the vessel’s another letter, however, petitioner confirmed to
by moisture content from salt water, which could be bad shape and a laboratory analysis revealing that private respondent receipt of the claim papers and
arrested by drying. However, Republic Flour rejected the grains were contaminated with salt water, assured her that her case was “being given
the entire cargo and demanded that defendant North defendants failed to rebut said arguments or even preferential attention and prompt action”.
INSURANCE Page
111

- Petitioner caused another investigation respecting and convincing evidence that is required to prove it. insured value of the lost and damagcd goods,
the subject policy. Pursuant to the findings of this A failure to do so would leave intact the presumption including other expenses in connection therewith, in
second investigation, petitioner stood by its initial of good faith and regularity in the performance of the total amount of $1,134.46 U.S. currency.
decision to treat the policy as void and not to honor public duties, which was the basis of both - As subrogee of the rights of' the shipper and/or
the claim. On November 9, 1992, private respondent court and the trial court in finding the consignee, the insurer, St. Paul Fire & Marine
respondent enlisted the services of counsel in date of Florence Pulido’s death to be as plaintiff- Insurance Co., instituted an action against the
reiterating her claim for death benefitsPetitioner still private respondent maintained. defendants for the recovery of said amount of
refused to make payment and thus, this action. - We cannot likewise give credence to petitioner’s $1,134.46, plus costs.
submission that the inconsistencies in the - The defendants resisted the action. However, for
ISSUE testimonies of the witnesses for plaintiff-private the purpose only of avoiding litigation without
WON lower court erred in holding that there was no respondent are in themselves evidence of fraud. admitting liability to the consignee, the defendants
fraud Such alleged inconsistencies are matters of offered to settle the latter’s claim in full by paying
credibility which had been ably passed upon by the the C.I.F. value of the damaged cargo, but this offer
HELD lower court. was declined by the plaintiff.
- The records bear out that since the onset of this Disposition the instant petition is DENIED - The LC rendered judgment ordering the
case, the main issue has always been whether there defendants to pay the plaintiff the sum of P300.00.
was fraud in the obtainment of the disputed policy, ST.PAUL FIRE & MARINE INSURANCE CO v. The plaintiff filed a MFR contending that it should
or put differently, whether the insured, Florence MACONDRAY & CO INC recover the amount of $1,134.46 or its equivalent in
Pulido, was in fact dead before the application for 70 SCRA 122 pesos at the rate of P3.90, instead of P2.00, but this
insurance on her life was made. This the lower ANTONIO; March 25, 1976 was denied. Hence, this appeal.
courts had effected ruled on, upon a preponderance
of the evidence duly received from both parties. We FACTS ISSUES
see no reversible error in the finding of both - Winthrop Products, Inc. shipped aboard the SS 1. WON in case of loss or damage, the liability of the
respondent court and the trial court in favor of the "Tai Ping", owned and operated by Wilhelm carrier to the consignee is limited to the C.I.F. value
correctness of the entries in Certificate of Death, Wilhelmsen, 218 cartons and drums of drugs and of the goods which were lost or damaged
duly registered with the Local Civil Registrar of medicine, with the freight prepaid, which were 2. WON the insurer who has paid the claim in dollars
Bagulin, La Union, which declared that Florence consigned to Winthrop-Stearns, Inc. Barber to the consignee should be reimbursed in its peso
Pulido died of acute pneumonia on September 10, Steamship Lines, Inc., agent of Wilhelm Wilhelmsen equivalent on the date of discharge of the cargo or
1991. Dr. Irineo Gutierrez, the Municipal Health issued Bill of Lading No. 34, in the name of Winthrop on the date of the decision
Officer of Bagulin, La Union whose signature Products, Inc. as shipper, with arrival notice in-
appeared in the death certificate, testified in addition Manila to consignee Winthrop-Stearns, Inc. The HELD
that he ministered to the ailing Florence Pulido for shipment was insured by the shipper against loss 1. YES
two days immediately prior to her death. This fact is and/or damage with the St. Paul Fire & Marine Ratio The purpose of the bill of lading is to provide
likewise noted in the death certificate. Insurance Company. for the rights and liabilities of the parties in reference
- Death certificates, and notes by a municipal health - The SS "Tai Ping" arrived at the Port of Manila and to the contract to carry. The stipulation in the bill of
officer prepared in the regular performance of his discharged its aforesaid shipment into the custody of lading limiting the common carrier's liability to the
duties, are prima facie evidence of facts therein Manila Port Service, the arrastre contractor for the value of the goods appearing in the bill, unless the
stated. A duly-registered death certificate is Port of Manila. The said shipment was discharged shipper or owner declares a greater value, is valid
considered a public document and the entries found complete and in good order with the exception of and binding. This limitation of the carrier's liability is
therein are presumed correct, unless the party who one (1) drum and several cartons which were in bad sanctioned by the freedom of the contracting parties
contests its accuracy can produce positive evidence order condition. Because consignee failed to receive to establish such stipulations, clauses, terms, or
establishing otherwise. Petitioner’s contention that the whole shipment and as several cartons of conditions as they may deem convenient, provided
the death certificate is suspect because Dr. Gutierrez medicine were received in bad order condition, the they are not contrary to law, morals, good customs
was not present when Florence Pulido died, and consignee filed the corresponding claim in the and public policy. A stipulation fixing or limiting the
knew of Florence’s death only through Ramon amount of P1,109.67 representing the C.I.F. value of sum that may be recovered from the carrier on the
Piganto, does not merit a conclusion of fraud. No the damaged drum and cartons of medicine with the loss or deterioration of the goods is valid, provided it
motive was imputed to Dr. Gutierrez for seeking to carrier and the Manila Port Service. However, both is (a) reasonable and just under the circumstances,
perpetuate a falsity in public records. Petitioner was refused to pay such claim. Consequently, the and (b) has been fairly and freely agreed upon. In
likewise unable to make out any clear motive as to consignee filed its claim with the insurer, St. Paul the case at bar, the liabilities of the defendants-
why Ramon Piganto would purposely lie. Mere Fire & Marine Insurance Co., the insurance company, appellees with respect to the lost or damaged
allegations of fraud could not substitute for the full on the basis of such claim, paid to the consignee the shipments are expressly limited to the C.I.F. value of
INSURANCE Page
112

the goods as per contract of sea carriage embodied P2,000.00, by way of compromise. The Insured "3. In terms of and subject to the limitations of
in the bill of lading. rejected it and made a counter-offer for P4,000.00, and for the purposes of this Section, the Company
- The plaintiff-appellant, as insurer, after paying the but the Company did not accept it. will indemnify any authorized Driver who is driving
claim of the insured for damages under the - On September 18, 1962, the Insured and Carlito's the Motor Vehicle . . . "
insurance, is subrogated merely to the rights of the parents filed a complaint against the Company to "Conditions
assured. As subrogee, it can recover only the collect the proceeds of the policy. In its answer, the "7. In the event of death of any person entitled
amount that is recoverable by the latter. Since the Company admitted the existence thereof, but to indemnify under this Policy, the Company will, in
right of the assured, in case of loss or damage to the pleaded lack of cause of action on the part of the respect of the liability incurred by such person,
goods, is limited or restricted by the provisions in the plaintiffs. indemnify his personal representatives in terms of
bill of lading, a suit by the insurer as subrogee - TC rendered a decision sentencing the Company to and subject to the limitations of this Policy, provided,
necessarily is subject to like limitations and pay to the plaintiffs the sum of P4,000.00 and the that such representatives shall, as though they were
restrictions. costs. Hence, this appeal by the Company, which the Insured, observe, fulfill and be subject to the
2. On the date of the discharge of the cargo. The contends that plaintiffs have no cause of action Terms of this Policy insofar as they can apply.
peso equivalent was based by the consignee on the because: 1) the Coquias have no contractual relation "8. The Company may, at its option, make
exchange rate of P2.015 to $1.00 which was the rate with the Company; and 2) the Insured has not indemnity payable directly to the claimants or heirs
existing at that time. complied with the provisions of the policy concerning of claimants, with or without securing the consent of
arbitration. or prior notification to the Insured, it being the true
PHILAM v. AUDITOR (supra p.59) intention of this Policy to protect, to the extent
ISSUES herein specified and subject always to the Terms of
FIELDMEN’S v. ASIAN SURETY (supra p.60) 1. WON there was contractual relations between the this Policy, the liabilities of the Insured towards the
Coquias and the Company passengers of the Motor Vehicle and the Public."
EQUITABLE v. RURAL INSURANCE (supra p.60) 2. WON the insured has not complied with the - Thus, the policy under consideration is typical of
provisions of the policy concerning arbitration contracts pour autrui, this character being made
COQUIA v. FIELDMEN'S INSURANCE CO. INC. more manifest by the fact that the deceased driver
26 SCRA 178 HELD paid fifty percent (50%) of the corresponding
CONCEPCION; November 29, 1968 1. Although, in general, only parties to a contract premiums, which were deducted from his weekly
may bring an action based thereon, this rule is commissions. Under these conditions, it is clear that
NATURE subject to exceptions, one of which is found in the the Coquias — who, admittedly, are the sole heirs of
Appeal from the decision of the CFI certified by CA Art 1311 CC, reading: the deceased — have a direct cause of action against
"If a contract should contain some stipulation in the Company, and, since they could have maintained
FACTS favor of a third person, he may demand its this action by themselves, without the assistance of
- December 1, 1961, appellant Fieldmen's Insurance fulfillment provided he communicated his acceptance the Insured, it goes without saying that they could
Company, Inc. issued, in favor of the Manila Yellow of the obligor before its revocation. A mere incidental and did properly join the latter in filing the complaint
Taxicab Co., Inc. a common carrier accident benefit or interest of a person is not sufficient. The herein.
insurance policy, covering the period from December contracting parties must have clearly and 2. Based upon Section 17 of the policy:
1, 1961 to December ,1962. It was stipulated in said deliberately conferred a favor upon a third person." "If any difference or dispute shall arise with
policy that: - Does the policy in question belong to such class of respect to the amount of the Company's liability
"The Company will, subject to the Limits of Liability contracts pour autrui? under this Policy, the same shall be referred to the
and under the Terms of this Policy, indemnify the In this connection, said policy provides, inter alia: decision of a single arbitrator to be agreed upon by
Insured in the event of accident caused by or arising "Section I — Liability to Passengers. 1. The both parties or failing such agreement of a single
out of the use of Motor Vehicle against all sums Company will, subject to the Limits of Liability and arbitrator, to the decision of two arbitrators, one
which the Insured will become legally liable to pay in under the Terms of this Policy, indemnify the Insured to be appointed in writing by each of the parties
respect of: Death or bodily injury to any fare-paying in the event of accident caused by or arising out of within one calendar month after having been
passenger including the Driver, Conductor and/or the use of Motor Vehicle against all sums which the required in writing so to do by either of the parties
Inspector who is riding in the Motor Vehicle insured Insured will become legally liable to pay in respect and in case of disagreement between the
at the time of accident or injury." of: Death or bodily injury to any fare-paying arbitrators, to the decision of an umpire who shall
- While the policy was in force, or on February 10, passenger including the Driver. . . who is riding in have been appointed in writing by the arbitrators
1962, a taxicab of the Insured, driven by Carlito the Motor Vehicle insured at the time of accident or before entering on the reference and the costs of
Coquia, met a vehicular accident to which he died. injury. and incidental to the reference shall be dealt with
The Insured filed therefor a claim for P5,000.00 to "Section II. — Liability to the Public in the Award. And it is hereby expressly stipulated
which the Company replied with an offer to pay and declared that it shall be a condition precedent
INSURANCE Page
113

to any right of action or suit upon this Policy that rebels who wanted to obtain canned goods, rice and preponderance of evidence. But petitioner failed to
the award by such arbitrator, arbitrators or umpire medicines as provisions for their comrades in the do so.
of the amount of the Company's liability hereunder forest, and that such loss was an excepted risk under - The petitioner relies on the Sworn Statements of
if disputed shall be first obtained." paragraph No. 6 of the policy conditions of Fire Jose Lomocso and Ernesto Urbiztondo as well as on
- The record shows that none of the parties to the Insurance Policy No. F-1397, which provides: the Spot Report of Pfc. Arturo V. Juarbal. A witness
contract invoked this section, or made any reference This insurance does not cover any loss or damage can testify only to those facts which he knows of his
to arbitration, during the negotiations preceding the occasioned by or through or in consequence, personal knowledge, which means those facts which
institution of the present case. In fact, counsel for directly or indirectly, of any of the following are derived from his perception. Consequently, a
both parties stipulated, in the trial court, that none occurrences, namely: witness may not testify as to what he merely learned
of them had, at any time during said negotiations, (d) Mutiny, riot, military or popular uprising, from others either because he was told or read or
even suggested the settlement of the issue between insurrection, rebellion, revolution, military or heard the same. Such testimony is considered
them by arbitration, as provided in said section. usurped power. hearsay and may not be received as proof of the
Their aforementioned acts or omissions had the Any loss or damage happening during the truth of what he has learned.
effect of a waiver of their respective right to demand existence of abnormal conditions (whether Disposition the appealed Decision is MODIFIED.
an arbitration. physical or otherwise) which are occasioned by or The rate of interest on the adjudged principal
Disposition The decision appealed from should be through or in consequence, directly or indirectly, amount of Two Hundred Thousand Pesos
as it is hereby affirmed in toto, with costs against the of any of said occurrences shall be deemed to be (P200,000.00) shall be six percent (6%) per annum
herein defendant-appellant, Fieldmen's Insurance loss or damage which is not covered by this computed from the date of filing of the Complaint in
Co., Inc. insurance, except to the extent that the Insured the trial court. The awards in the amounts of Fifty
shall prove that such loss or damage happened Thousand Pesos (P50,000.00) as actual damages,
COUNTRY BANKERS INSURANCE CORP v. independently of the existence of such abnormal Fifty Thousand Pesos (P50,000.00) as exemplary
LIANGA BAY conditions. damages, Five Thousand Pesos (P5,000.00) as
DE LEON; January 25, 2002 - Finding the denial of its claim unacceptable, Lianga litigation expenses, and Ten Thousand Pesos
Bay then instituted in the trial court the complaint (P10,000.00) as attorney?s fees are hereby
NATURE for recovery of "loss, damage or liability" against DELETED.
Petition for review on certiorari Country Bankers.
- RTC ruled in favor of the cooperative. CA affirmed. DBP POOL OF ACCREDITED INSURANCE v.
FACTS RADIO MINDANAO NETWORK
- Lianga Bay is a duly registered cooperative ISSUE 480 SCRA 314
judicially declared insolvent and is here represented WON the cause of the loss was an excepted risk MARTINEZ; January 27, 2006
by, Cornelio Jamero. Country Bankers Insurance under the terms of the fire insurance policy
and Lianga Bay entered into a contract of fire NATURE
insurance. Country Bankers insured the HELD Petition for certiorari
respondent’s stocks-in-trade against fire loss, - Where a risk is excepted by the terms of a policy
damage or liability during the period starting from which insures against other perils or hazards, loss FACTS
June 20, 1989 at 4:00 p.m. to June 20, 1990 at 4:00 from such a risk constitutes a defense which the - In the evening of July 27, 1988, the radio station of
p.m., for the sum of P200,000.00. insurer may urge, since it has not assumed that risk, Radio Mindanao Network located at the SSS Building
- On July 1, 1989, at or about 12:40 a.m., the and from this it follows that an insurer seeking to in Bacolod City was burned down causing damage in
respondent’s building at Barangay Diatagon, Lianga, defeat a claim because of an exception or limitation the amount of over one million pesos. Respondent
Surigao del Sur was gutted by fire, resulting in the in the policy has the burden of proving that the loss sought to recover under two insurance policies but
total loss of the respondent’s stocks-in-trade, pieces comes within the purview of the exception or the claims were denied on the basis that the case of
of furnitures and fixtures, equipments and records. limitation set up. If a proof is made of a loss the loss was an excepted risk under condition no. 6
- Due to the loss, the respondent filed an insurance apparently within a contract of insurance, the burden (c) and (d), to wit:
claim with the petitioner under its Fire Insurance is upon the insurer to prove that the loss arose from 6. This insurance does not cover any loss or damage
Policy, submitting: (a) the Spot Report of Pfc. Arturo a cause of loss which is excepted or for which it is occasioned by or through or in consequence, directly
V. Juarbal, INP Investigator, dated July 1, 1989; (b) not liable, or from a cause which limits its liability. or indirectly, of any of the following consequences,
the Sworn Statement of Jose Lomocso; and (c) the Stated else wise, since Country bank here is namely:
Sworn Statement of Ernesto Urbiztondo. defending on the ground of non-coverage and relying (c) War, invasion, act of foreign enemies, hostilities,
- The petitioner, however, denied the insurance upon an exemption or exception clause in the fire or warlike operations (whether war be declared or
claim on the ground that, based on the submitted insurance policy it has the burden of proving the not), civic war.
documents, the building was set on fire by 2 NPA facts upon which such excepted risk is based, by a
INSURANCE Page
114

(d) Mutiny, riot, military or popular uprising, board Judy VII, a barge leased by Lea Mer, the LOADSTAR SHIPPING CO INC v. PIONEER ASIA
insurrection, rebellion, revolution, military or vessel sank, resulting in the loss of the cargo. INSURANCE CORP
usurped power. GR No. 157481
- The insurers maintained that based on witnesses ISSUE QUISUMBING; January 24, 2006
and evidence gathered at the site, the fire was WON Lea Mer is liable for the loss of the cargo
caused by the members of the Communist Party of NATURE
the Philippines/New People’s Army. Hence the refusal HELD Review on certiorari (1) the Decision dated October
to honor their obligations. YES 15, 2002 and (2) the Resolution dated February 27,
- The trial court and the CA found in favor of the - Common carriers are bound to observe 2003 of CA
respondent. In its findings, both courts mentioned extraordinary diligence in their vigilance over the
the fact that there was no credible evidence goods and the safety of the passengers they FACTS
presented that the CCP/NPA did in fact cause the fire transport, as required by the nature of their business - June 6, 1984 - Petitioner Loadstar Shipping Co.,
that gutted the radio station in Bacolod. and for reasons of public policy. Extraordinary Inc. (LOADSTAR), registered owner and operator of
diligence requires rendering service with the greatest the vessel M/V Weasel, entered into a voyage-
ISSUE skill and foresight to avoid damage and destruction charter with Northern Mindanao Transport Company,
WON the insurance companies are liable to pay to the goods entrusted for carriage and delivery. Inc. for the carriage of 65,000 bags of cement from
Radio Mindanao Network under the insurance policies - Common carriers are presumed to have been at Iligan City to Manila. The shipper was Iligan Cement
fault or to have acted negligently for loss or damage Corporation, while the consignee in Manila was
HELD to the goods that they have transported. This Market Developers, Inc. (MARKET)
YES presumption can be rebutted only by proof that they - June 24, 1984 - 67,500 bags of cement were
- The Court will not disturb the factual findings of the observed extraordinary diligence, or that the loss or loaded on board M/V Weasel and stowed in the cargo
appellant and trial courts absent compelling reason. damage was occasioned by any of the following holds for delivery to the consignee. The shipment
Under this mode of review, the jurisdiction of the causes: was covered by petitioner’s Bill of Lading dated June
court is limited to reviewing only errors of law. “(1) Flood, storm, earthquake, lightning, or other 23, 1984.
- Particularly in cases of insurance disputes with natural disaster or calamity; - Prior to the voyage, the consignee insured the
regard to excepted risks, it is the insurance “(2) Act of the public enemy in war, whether shipment of cement with respondent Pioneer Asia
companies which have the burden to prove that the international or civil; Insurance Corporation (PIONEER) for P1,400,000, for
loss comes within the purview of the exception or “(3) Act or omission of the shipper or owner of the which it issued Marine Open Policy No. MOP-006
limitation set up. It is sufficient for the insured to goods; dated September 17, 1980, covering all shipments
prove the fact of damage or loss. Once the insured “(4) The character of the goods or defects in the made on or after September 30, 1980
makes out a prima facie case in its favor, the duty or packing or in the containers; - June 25, 1984 - Captain Montera of M/V Weasel
burden of evidence shifts to the insurer to controvert “(5) Order or act of competent public authority.” ordered the vessel to be forced aground which
said prima facie case. - To excuse the common carrier fully of any liability, rendered the entire shipment of cement as good as
Disposition Petition dismissed. Decision of the CA is the fortuitous event must have been the proximate gone due to exposure to sea water. LOASTAR thus
affirmed. and only cause of the loss. It should have exercised failed to deliver the goods to MARKET in Manila.
due diligence to prevent or minimize the loss before, - MARKET demanded from LOADSTAR full
LEA MER INDUSTRIES v. MALAYAN INSURANCE during and after the occurrence of the event. reimbursement of the cost of the lost shipment.
471 SCRA 698 - Petitioner bore the burden of proving that it had LOADSTAR refused to reimburse the MARKET despite
PANGANIBAN; September 30, 2005 exercised extraordinary diligence to avoid the loss, repeated demands.
or that the loss had been occasioned by a fortuitous - March 11, 1985 – PIONEER paid the MARKET
NATURE event -- an exempting circumstance. P1,400,000 plus an additional amount of P500,000,
Petition for Review - The evidence presented by petitioner in support of the value of the lost shipment of cement. In return,
its defense of fortuitous event was sorely the MARKET executed a Loss and Subrogation
FACTS insufficient. It was not enough for the common Receipt in favor of PIONEER concerning the latter’s
- Ilian Silica Mining entered into a contract of carrier to show that there was an unforeseen or subrogation rights against LOADSTAR.
carriage with Lea Mer Industries for the shipment of unexpected occurrence. - October 15, 1986 – PIONEER filed a complaint
900 metric tons of silica sand valued at P565,000. Disposition Petition is DENIED and the assailed against LOADSTAR with the RTC Manila alleging
Consigned to Vulcan Industrial and Mining that: (1) the M/V Weasel was not seaworthy at the
Corporation, the cargo was to be transported from Decision and Resolution are AFFIRMED. Costs commencement of the voyage; (2) the weather and
Palawan to Manila. The silica sand was placed on sea conditions then prevailing were usual and
against petitioner. expected for that time of the year and as such, was
INSURANCE Page
115

an ordinary peril of the voyage for which the M/V > Planters Products, Inc. v. CA - public carrier unexpected hazard. LOADSTAR has only itself to
Weasel should have been normally able to cope with; shall remain as such, notwithstanding the charter of blame for its misjudgment.
and (3) LOADSTAR was negligent in the selection the whole or portion of a vessel by one or more Disposition petition is DENIED
and supervision of its agents and employees then persons, provided the charter is limited to the ship
manning the M/V Weasel. only, as in the case of a time-charter or voyage-
- LOADSTAR alleged that no fault nor negligence charter. It is only when the charter includes both
could be attributed to it because it exercised due the vessel and its crew, as in a bareboat or demise
diligence to make the ship seaworthy, as well as that a common carrier becomes private, at least
properly manned and equipped and failure to deliver insofar as the particular voyage covering the charter-
was due to force majeure. party is concerned.
- February 15, 1993 - RTC decided in favor of 2. YES
PIONEER and that LOADSTAR , as a common carrier, - As a common carrier, LOADSTAR is required to
bears the burden of proving that it exercised observe extraordinary diligence in the vigilance over
extraordinary diligence in its vigilance over the goods the goods it transports. When the goods placed in its
it transported. The trial court explained that in case care are lost, LOADSTAR is presumed to have been
of loss or destruction of the goods, a statutory at fault or to have acted negligently. LOADSTAR has
presumption arises that the common carrier was the burden of proving that it observed extraordinary
negligent unless it could prove that it had observed diligence in order to avoid responsibility for the lost
extraordinary diligence. LOADSTAR’S defense of cargo.
force majeure was found bereft of factual basis as a - Compania Maritima V CA - It requires common
PAG-ASA report that at the time of the incident, carriers to render service with the greatest skill and
tropical storm “Asiang” had moved away from the foresight and “to use all reasonable means to
Philippines was presented. ascertain the nature and characteristics of goods
- October 15, 2002 – CA affirmed RTC Decision with tendered for shipment, and to exercise due care in
modification the handling and stowage, including such methods
as their nature requires.
ISSUES - A1734 CC enumerates the instances when a carrier
1. WON LOADSTAR is a common carrier under might be exempt from liability for the loss of the
Article 1732 CC goods.
2. Assuming it is a common carrier, WON proximate (1) Flood, storm, earthquake, lightning,
cause of the loss of cargo was not a fortuitous event or other natural disaster or calamity;
but was allegedly due to the failure of petitioner to (2) Act of the public enemy in war,
exercise extraordinary diligence whether international or civil;
(3) Act or omission of the shipper or
HELD owner of the goods;
1. YES (4) The character of the goods or defects
- A1732 CC defines a “common carrier” as in the packing or in the containers; and
follows: (5) Order or act of competent public
Common carriers are persons, corporations, firms authority
or associations engaged in the business of carrying - LOADSTAR claims that the loss of the goods was
or transporting passengers or goods or both, by due to a fortuitous event under paragraph 1. Yet, its
land, water, or air, for compensation, offering their claim is not substantiated. It is supported by
services to the public. evidence that the loss of the entire shipment of
- LOADSTAR is a corporation engaged in the cement was due to the gross negligence of
business of transporting cargo by water and for LOADSTAR
compensation, offering its services indiscriminately - Records show that in the evening of June 24, 1984,
to the public. Thus, without doubt, it is a common the sea and weather conditions in the vicinity of
carrier. Even if it entered into a voyage-charter Negros Occidental were calm. The records reveal
agreement with Northern Mindanao Transport that LOADSTAR took a shortcut route, instead of the
Company, Inc, it did not in any way convert the usual route, which exposed the voyage to
common carrier into a private carrier.
i

You might also like